UNION PUBLIC SERVICE COMMISSION

UNION PUBLIC SERVICE COMMISSION
HIGH COURT OF DELHI : NEW DELHI
Writ Petition (Civil) No.13451 of 2009
D.D. 13.01.2010
Hon’ble Mr. Justice Madan B.Lokur &
Hon’ble Mr. Justice Mukta Gupta
Dr. Vineet Relhan
Vs.
U.P.S.C. & Anr.
…
Petitioner
…
Respondents
Selection:
Whether failure to enclose essential documents like certificates of qualification,
reservation etc., to the application for the post entails rejection of the application? – Yes.
The petitioner was a candidate for the post of Specialist Grade-II (Dermatology) –
He did not enclose (i) Matriculation certificate (ii) M.B.B.S. certificate and (iii) M.D.
(Dermatology) degree certificate along with his application – Hence his application was
rejected – The petitioner filed original petition before CAT asserting that he had
produced all the documents – CAT on verification of the original documents found that
the above documents were not enclosed hence rejected the original application – In this
writ petition filed against CAT order the High Court after considering the decisions of the
Supreme Court dismissed the writ petition upholding the decision of CAT.
Held:
The petitioner being an educated person should have submitted the application
properly. He ought to have been clear about the fact that the requisite certificates must be
furnished along with the application. For his failure to do so the petitioner has only
himself to blame.
Decisions in Charles K.Skaria & Ors. v. Dr. C.Mathew & Ors (1980) 2 SCC 752
and Dolly Chhanda v. Chairman, JEE & Ors. (2005) 9 SCC 779, have been distinguished.
Cases referred:
1. (1980) 2 SCC 752 - Charles K.Skaria & Ors. v. Dr. C.Mathew & Ors
2. (2005) 9 SCC 779 - Dolly Chhanda v. Chairman, JEE & Ors.
ORDER
MADAN B. LOKUR, J.
The petitioner is aggrieved by an order dated 4th November, 2009 passed by the
Central Administrative Tribunal, Principal Bench in OA No.1914/2009.
2.
The petitioner made an application in response to an advertisement for
appointment to the post of Specialist Grade-II (Dermatology). When the petitioner was
not called for an interview, he filed an Original Application before the Tribunal on 16th
July, 2009.
3.
During the pendency of the Original Application, the petitioner received a letter
from the Respondents on 23rd July, 2009 informing him that since he had not annexed the
necessary certificates, his application form was not accepted. The certificates that the
petitioner did not submit with the application form were:(i)
Matriculation certificate as proof of date of birth
(ii)
MBBS degree certificate and
(iii)
M.D. (Dermatology) degree certificate
4.
The petitioner contended before the Tribunal that the certificate were in fact
submitted by him and the application was received by an official of the Union Public
Service Commission (UPSC) who spent a considerable amount of time in checking the
application. It is only thereafter that the petitioner was given a card for being called for
an interview.
5.
The UPSC contended before the Tribunal that the petitioner did not furnish the
requisite certificates along with the application form.
6.
The only way in which the issue could be sorted out by the Tribunal was to call
for the original application form, which it did. The Tribunal noted that the petitioner
submitted the following documents along with the application form:-
7.
(i)
published papers;
(ii)
clinical trials conducted;
(iii)
conferences attended and presentation done;
(iv)
other academic activities; and
(v)
experience certificate.
On a perusal of the original application form, the Tribunal was satisfied that the
documents mentioned in the letter dated 23rd July, 2009 were not submitted by the
petitioner and on this basis rejected the original application filed by him.
8.
Before us, learned counsel for the petitioner contended that the documents could
have been produced by him later on and his mere failure to submit the requisite
documents at the appropriate time ought not to have an adverse effect. In this regard,
learned counsel for the petitioner relied upon Charles K.Skaria and others v. Dr.
C.Mathew and others, (1980) 2 SCC 752 which was followed in Dolly Chhanda v.
Chairman, JEE and others, (2005) 9 SCC 779.
9.
We have perused the two decisions cited by learned counsel for the petitioner. It
is true that the Supreme Court held that a formalistic and ritualistic approach should not
be followed in such matters.
However, in Charles K.Skaria the relevant proof of
eligibility was in fact produced by the candidates before the selection was made, as
mentioned in paragraph 20 of the Report. In Dolly Chhanda the candidate belonged to
the reserved MI category and there was some error in the certificate issued to her as a
result of which her candidature was cancelled. This error was later rectified and under
these circumstances, the Supreme Court held that depending upon the facts of a case,
there can be some relaxation in the matter of submission of proof and it would not be
proper to apply any rigid principle as it pertains to the domain of procedure.
10.
In so far as the present case is concerned, there is nothing on record to suggest
that the petitioner submitted the requisite documents at the appropriate time. That apart,
it is not as if the petitioner is uneducated or could not have filled the form intelligibly.
He claims to hold a degree of M.D. (Dermatology) and he ought to have been clear about
the fact that the requisite certificates must be furnished along with the application form.
For his failure to do so, the petitioner has only himself to blame.
11.
From the counter affidavit filed before us, it appears that the candidature of some
other candidates were also rejected on the ground that they failed to submit the requisite
certificate of educational qualifications, experience, community etc. It has also been
mentioned in the counter affidavit that the UPSC had clearly mentioned in the
advertisement that no provisional claim would be accepted and the requisite certificates
must be filed along with the application form. Given these facts, we are not inclined to
reopen the selection process or interfere with the impugned order.
12.
Dismissed.
***
IN THE HIGH COURT OF JUDICATURE AT MADRAS
Writ Appeal No.159 of 2010
D.D. 09.02.2010
The Hon’ble Mr. H.L.Gokhale, Chief Justice &
The Hon’ble Mr. Justice K.K.Sasidharan
The Chairman, U.P.S.C. & Ors.
Vs.
N.Kolanchi & Anr.
…
Appellants
…
Respondents
Recruitment:
Whether applications received after the last date fixed for receipt of applications can be
entertained? No. Postal delay or delay on the part of the courier is no excuse.
Petitioner was a candidate for Civil Services Examination 2009 as per notification
dated 6.12.2008 fixing 5.1.2009 as the last date for receipt of applications - Petitioner’s
application reached the Commission office on 6.1.2009 – Hence, it was rejected as per
Clause-3 of the notification – Petitioner challenged it before the Single Judge – As per
interim orders he appeared for preliminary examination and then main examination –
Single Judge directed the Commission to publish the result of the main examination –
Commission challenged the said direction in this Appeal – Division Bench following the
decision of the Apex Court in similar matters allowed the Appeal and set aside the order
of Single Judge.
Held:
The Clauses in the notification make this amply clear inasmuch as the candidates
are put on notice that their applications will not be entertained if they did not reach the
Commission’s office before the date provided in that behalf. This is principally so
because a large number of candidates appear in the Civil Services Examination and if any
latitude is allowed in case of one candidate, there is no reason why it should not be
allowed in the case of other candidates.
Further held:
That Section 27 of the General Clauses Act cannot help to draw any interference
that the application reached on 5.1.2009 when in fact it has not.
JUDGMENT
Mr. H.L.Gokhale:
Heard Mr. A.L. Somayaji, learned senior counsel appearing on behalf of Mr. K.
Sridhar in support of this appeal. Mr. N. Thiagarajan, learned senior counsel appears on
behalf of Mr. J. Ponnudurai for the first respondent; and Mr. J. Ravindran, learned
Assistant Solicitor General appears with Ms. M. Christella, learned Central Government
Standing Counsel on behalf of the second respondent.
2.
The appellants before us are the Chairman, Secretary and Under Secretary of the
Union Public Service Commission. They seek to challenge the order dated 19.11.2009
passed by a learned single Judge of this Court on the writ petition filed by the first
respondent herein, by which order the learned single Judge directed the appellants herein
to publish the result of the main examination written by the first respondent for selection
in the Civil Services Examination, 2009.
3.
The short facts leading to this appeal are this-wise ::As stated above, the appellant herein was an aspirant for selection in the Civil
Services Examination, 2009. As per the Notification dated 6.12.2008 published by the
Union Public Service Commission, the last date for receipt of applications from
candidates was 5.1.2009. Under the caption "Important", Clause-3 titled "Last Date for
Receipt of Applications" stated as follows :
"All applications must reach the "Secretary, Union Public Service
Commission, Dholpur House, Shahjahan Road, New Delhi-110 069 either
by hand or by Post/Speed or by Courier, on or before 5th January, 2009".
Further, in Clause-6 of the detailed advertisement, it has been again repeated that the
completed application forms must reach the Commission's Office on or before 5.1.2009.
In the case of the applicants from far-off regions which have been specified in the
Notification, such as Assam, Meghalaya, Arunachal Pradesh, Mizoram, Manipur,
Nagaland, Tripura, Sikkim, Jammu & Kashmir, Lahaul and Spiti District and Pangi Sub
Division of Chamba District of Himachal Pradesh, Andaman and Nicobar Islands or
Lakshadweep or abroad, the last date for receipt of the applications is 12.1.2009 till 5.00
p.m. The benefit of the extended time is made available only to those applications which
are received by Post/Speed Post from the above mentioned areas/regions. Note-I to this
clause again reiterates that candidates should clearly note that the Commission will in no
case be responsible for non-receipt of their applications or any delay in receipt thereof on
any account whatsoever. It is also stated that no application received after the prescribed
last date will be entertained under any circumstances and all the late applications will be
summarily rejected. It is, therefore, emphasised that the candidates should ensure that
their application should reach the Commission's Office on or before the prescribed last
date.
Moreover, Clause-5 of Appendix-II to the Notification containing the
instructions/guidelines to the candidates makes it abundantly clear that the candidates are
advised in their own interest to ensure that the applications reach the Commission's
Office on or before the closing date and that applications received in the Commission's
Office after the closing date will not be considered.
4.
In the instant case, the admitted position is that the first respondent sent his
application from Hyderabad on 3.1.2009. According to the postal authorities, it has
reached the Office of Union Public Service Commission on 6.1.2009, whereas according
to the Union Public Service Commission, it has reached its Office on 7.1.2009. It is not
in dispute that as far as the application of the first respondent is concerned, it did not
reach the Commission's Office on or before 5.1.2009, which was the last date for
receiving the applications. The third respondent-Under Secretary, by his communication
dated 26.3.2009, informed the first respondent that the application of the first respondent
has been rejected for the reason that it was received late.
5.
This being the position, since the first respondent was not allowed to appear in the
examination, he filed Writ Petition No.8774 of 2009. The learned single Judge who
heard the matter allowed the first respondent to write the Preliminary Examination by
passing an interim order on 30.4.2009. Thereafter, the first respondent moved another
interim application for declaration of his result in the Preliminary Examination and also
to allow him to write the Main Examination. Another learned single Judge allowed that
application by passing the second interim order on 21.8.2009. While passing this order,
the learned single Judge made it clear in paragraph 4 of his order that his order was
passed without prejudice the contentions of the rival parties in the writ petition. In the
same paragraph, the learned single Judge also observed that the question of
maintainability as raised by the learned counsel for the appellants as well as by the first
respondent herein was to be decided later on. Further, the learned single Judge has
clarified in the last sentence of paragraph 3 of his order that because the result of the
preliminary examination was published and the first respondent was permitted to appear
in the Main Examination, that will not enure any right in favour of the first respondent in
the writ petition.
6.
The writ petition was subsequently heard by another learned single Judge, who
took the view that since this was the last chance, as contended by the first respondent, to
appear in this examination and since he had given the Main Examination also, the
aspirations of the first respondent should not be frustrated and therefore, he directed the
appellants herein to publish the first respondent's result in the Main Examination. The
writ petition was accordingly disposed of. Being aggrieved by this order, the present
appeal has been filed.
7.
Mr. A.L. Somayaji, learned senior counsel appearing for the appellants pointed
out that a large number of candidates appear for this examination. According to the
learned senior counsel, the notification for the examination was very clear, in that
specific provisions were made so as to avoid any controversy in this behalf and there is
no scope for any further interpretation to permit any such relaxation. Learned senior
counsel drew our attention to an order of the Apex Court passed in Civil Appeal No.2544
of 1998, wherein, in the case of late receipt of an application by the Public Service
Commission by courier service, the Apex Court had upheld the decision of the Public
Service Commission in rejecting such an application form. This order of the Apex Court
passed in Civil No.2544 of 1998 was cited before the learned single Judge, as can be seen
from paragraph 8 of the impugned order. However, the learned single Judge has not
followed the law laid down in this behalf by the Apex Court. Learned senior counsel,
therefore, submits that the appeal deserves to be allowed and the order of the learned
single Judge set aside.
8.
Mr. N. Thiagarajan, learned senior counsel appearing for the first respondent, on
the other hand, submitted that this was the last chance for the first respondent to appear in
the Civil Services Examination. The first respondent had sent his application before the
date by which the application was supposed to be received at the Commission's Office
and he had presumed that the application will reach the Commission's Office in time.
Learned senior counsel relied upon Section 27 of the General Clauses Act, 1897 to
submit that the application must be deemed to have been delivered in the ordinary course
of postal service. That apart, learned senior counsel submitted that with a view to do
complete justice, it would be desirable that the result of the first respondent is directed to
be declared, as has been done by the learned single Judge. This is because, the first
respondent had appeared in the Preliminary Examination as well the Main Examination
under the orders of this Court and therefore, this Court should not interfere with the order
passed by the learned single Judge. Section 27 of the General Clauses Act cannot help to
draw any inference that the application reached on 5.1.2009 when in fact it has not.
9.
We have noted the submissions made on either side.
The clauses of the
Examination Notification published by the Union Public Service Commission quoted
above are very clear and there is no ambiguity with regard to the same. The clauses make
it very clear that when an application is sent by post, either by ordinary post or by speed
post, it must reach the Union Public Service Commission on or before 5.1.2009. The
relaxation is given only in case of candidates from certain far certain flung territories.
The clauses quoted above also make this amply clear, inasmuch as the candidates are put
on notice that their applications will not be entertained if they did not reach the
Commission's Office before the date provided in that behalf. This is principally so
because a large number of candidates appear in the Civil Services Examination and if any
latitude is allowed in case of one candidate, there is no reason why it should not be
allowed in the case of other candidates. The first respondent should have been more
careful, knowing fully well that this was his last opportunity to appear in the said
examination. Further, the interim orders passed during the pendency of the writ petition
were sought by the first respondent and he cannot make any capital out of them. That
apart, the learned single Judge who granted the second interim order quoted above made
it clear that the order does not create any right in favour of the first respondent.
10.
Reliance is also placed on an order passed by a Division Bench of the Delhi High
Court, which is in the case of service of an application by registered post. That order was
passed in the case of C.W.P. No.3745 of 1999 decided on 10.1.2000, where, in a similar
situation, the Delhi High Court has also taken the view that the application should have
reached the Commission's Office before the stipulated date and if it does not so reach, the
same cannot be entertained.
11.
In view of what is stated above, the order passed by the learned single Judge can
no longer be retained. The writ appeal is, therefore, allowed. The order of the learned
single Judge is set aside. The writ petition filed by the first respondent will stand
dismissed. There shall be no order as to costs. Consequently, M.P. No.1 of 2010 is
closed. The appellants-Commission need not declare the result of the first respondent in
the examination in which he had appeared.
12.
Learned senior counsel appearing for the first respondent makes an oral
application for leave to appeal. The request is rejected.
***
2010 STPL (WEB) 345 SC
IN THE SUPREME COURT OF INDIA
CIVIL APPELLATE JURISDICTION
CIVIL APPEAL NOS.4310-4311 OF 2010
[Arising out of SLP (C) Nos.13571-72 of 2008]
D.D. 07.05.2010
(K.G.Balakrishnan, S.H.Kapadia, R.V.Raveendran, B.Sudershan Reddy & P.Sathasivam,
JJ.)
Union of India
Vs.
Ramesh Ram & Ors. etc.
... Appellant
... Respondents
With
CA Nos.4315-4316/2010 @ SLP (C) Nos.13297-98/2008
CA No.4319/2010 @ SLP (C) No. 13581 of 2008
CA Nos.4324-4328/2010 @ SLP (C) Nos. 14834-38 of 2008
And
WP(C) Nos. 297, 312, 336, 414, 416 & 539 of 2008
Selection:
Whether candidates belonging to reserved category selected against general/unreserved
vacancies on account of their merit (without the benefit of any relaxation/concession),
can opt for a higher choice of service earmarked for reserved category and thereby
migrate to reservation category? – Yes.
After conducting Civil Service Examination as per Civil Service Examinations
Rules U.P.S.C. sent the select list to DPO&T which in terms of Rule-16 recommended
candidates for appointment – Rule 16(2) enabled meritorious candidate of any of the
reservation categories to get a service of higher preference so that he may not be placed at
a disadvantageous position vis-à-vis other candidates of his category – Certain OBC
candidates in the reserve list (wait list) challenged Rule 16(2) before CAT contending
that adjustment of OBC merit candidates against OBC reservation vacancies was illegal –
CAT held that meritorious OBC candidates who were selected on merit must be adjusted
against general category – However, it ordered that Rule 16(2) may be applied in terms of
the decision in Anurag Patel’s case – Union of India and other aggrieved candidates
preferred writ petitions before the Madras High Court challenging CAT order – High
Court held that Rule 16(2) is unconstitutional and set aside the select list and directed the
Government and U.P.S.C. to re-do service allocation de hors Rule 16(2) – Against the
said order these SLPs are filed by Union of India & Ors. – Supreme Court has held that
Rule 16(2) is valid and not inconsistent with Rule 16(1) or Articles 14, 16(4) and 335 of
the Constitution and allowed the appeals and dismissed writ petitions challenging the
validity of Rule 16(2) - The Supreme Court has held as under:
i) MRC candidates who avail the benefit of Rule 16 (2) and adjusted in the
reserved category should be counted as part of the reserved pool for the
purpose of computing the aggregate reservation quotas. The seats vacated
by MRC candidates in the General Pool will be offered to General
category candidates.
ii) By operation of Rule 16 (2), the reserved status of an MRC candidate is
protected so that his/ her better performance does not deny him of the
chance to be allotted to a more preferred service.
iii) The amended Rule 16 (2) only seeks to recognize the inter se merit
between two classes of candidates i.e. a) meritorious reserved category
candidates b) relatively lower ranked reserved category candidates, for the
purpose of allocation to the various Civil Services with due regard for the
preferences indicated by them.
iv)
The reserved category candidates "belonging to OBC, SC/ ST
categories" who are selected on merit and placed in the list of
General/Unreserved category candidates can choose to migrate to the
respective reserved category at the time of allocation of services. Such
migration as envisaged by Rule 16 (2) is not inconsistent with Rule 16 (1)
or Articles 14, 16 (4) and 335 of the Constitution.
Cases Referred:
(1976) 2 SCC 310 - State of Kerala v. N.M. Thomas
(1992) Supp 3 SCC 217 - Indra Sawhney v. Union of India
(1995) 2 SCC 745 - R.K. Sabharwal v. State of Punjab
(1996) 3 SCC 253 - Ritesh R. Sah v. Dr. Y.L.Yamul and Others
(1996) 6 SCC 36 - State of Bihar v. M .Neeti Chandra
(1998) 4 SCC 1 - Post Graduate Institute of Medical Education and Research v. Faculty
Association,
(2005) 9 SCC 742 - Anurag Patel vs. U.P. Public Service Commission & Ors.
(2006) 4 SCC 550 - Union of India v. Satya Prakash
(2006) 8 SCC 212 - M. Nagaraj v. Union of India
JUDGMENT
K.G. BALAKRISHNAN, CJI
1.
Leave granted.
2.
The constitutional validity of sub-rules (2) to (5) of Rule 16 of the Civil Service
Examination Rules (hereinafter `Rules') relating to civil services examinations held by
the Union Public Service Commission in the years 2005 to 2007 is the subject-matter of
these appeals by special leave. A three Judge Bench of this Court, by order dated
14.5.2009 has referred these cases to the Constitution Bench as it raises an important
legal question as to whether candidates belonging to reserved category, who get
recommended against general/unreserved vacancies on account of their merit (without the
benefit of any relaxation/concession), can opt for a higher choice of service earmarked
for Reserved Category and thereby migrate to reservation category.
3.
Selection
to
three
All
India
Services
(Indian Administrative Service,
Indian Foreign Service and Indian Police Service) and fifteen Group `A' Services and
three Group `B' officers in various Government departments are made by the Union
Public Service Commission (hereinafter `UPSC'), by conducting Civil Service
Examinations periodically. Civil Service Examinations are held as per the Civil Service
Examinations Rules notified in regard to each examination. The Rules for the Civil
Service Examination which was to be held in 2005 by the UPSC were published by the
Department of Personnel and Training (hereinafter `DOP&T') vide Notification dated
4.12.2004.
4.
To appreciate the issue, it will be necessary to refer to the relevant rules. The
Preamble to the Rules enumerates 21 services. Rule 1 provides that the examination will
be conducted by the UPSC in the manner prescribed in Appendix-I to the Rules.
4.1)
Rule 2 of the Rules relates to preferences and is extracted below:
"2. A candidate shall be required to indicate in his/her application form for
the Main Examination his/her order of preferences for various
services/posts for which he/she would like to be considered for
appointment in case he/she is recommended for appointment by Union
Public Service Commission. A candidate who wishes to be considered for
IAS/IPS shall be required to indicate in his/her application if he/she would
like to be considered for allotment to the State to which he/she belongs in
case he/she is appointed to the IAS/IPS. Note.--The candidate is advised
to be very careful while indicating preferences for various services/posts.
In this connection, attention is also invited to rule 19 of the Rules. The
candidate is also advised to indicate all the services/posts in the order of
preference in his/her application form. In case he/she does
not give any
preference for any services/posts, it will be assumed that he/she has no
specific preference for those services. If he/she is not allotted to any one of
the services/posts for which he/she has indicated preference, he/she shall
be allotted to any of the remaining services/posts in which there are
vacancies after allocation of all the candidates who can be allocated to
services/posts in accordance with their preferences."
4.2)
Rule 3 relates to number of vacancies and provision for reservation and it reads as
follows:
"3. The number of vacancies to be filled on the result of the examination
will be specified in the Notice issued by the Commission. Reservation
will be made for candidates belonging to the Scheduled Castes, Scheduled
Tribes, Other Backward Classes and physically disabled categories in
respect of vacancies as may be fixed by the Government."
4.3)
Rule 15 provides for three examinations namely preliminary examination, main
written examination and interview test as follows:
"15. Candidates who obtained such minimum qualifying marks in the
Preliminary Examination as may be fixed by the Commission at their
discretion shall be admitted to the Main Examination; and candidates who
obtain such minimum qualifying marks in the Main Examination (written)
as may be fixed by the Commission at their discretion shall be summoned
by them for an interview for personality test: Provided that candidates
belonging to the Scheduled Castes or Scheduled Tribes or Other Backward
Classes may be summoned for an interview for a personality test by the
Commission by applying relaxed standards in the Preliminary
Examination as well as Main Examination (Written) if the Commission is
of the opinion that sufficient number of candidates from these
communities are not likely to be summoned for interview for a personality
test on the basis of the general standard in order to fill up vacancies
reserved for them."
4.4)
Rule 16 lays down the manner of selection, preparation of merit list and selection
of candidates. The said rule is extracted below:
"16.(1) After interview, the candidates will be arranged by the
Commission in the order of merit as disclosed by the aggregate marks
finally awarded to each candidate in the Main Examination. Thereafter,
the Commission shall, for the purpose of recommending candidates
against unreserved vacancies, fix a qualifying mark (hereinafter referred to
as general qualifying standard) with reference to the number of unreserved
vacancies to be filled up on the basis of the Main Examination. For the
purpose of recommending Reserved Category candidates belonging to
Scheduled Castes, Scheduled Tribes and Other Backward Classes against
reserved vacancies, the Commission may relax the general qualifying
standard with reference to number of reserved vacancies to be filled up in
each of these categories on the basis of the Main Examination: Provided
that the candidates belonging to the Scheduled Castes, Scheduled Tribes
and the Other Backward Classes who have not availed themselves of any
of the concessions or relaxations in the eligibility or the selection criteria,
at any stage of the examination and who after taking into account the
general qualifying standards are found fit for recommendation by the
Commission shall not be recommended against the vacancies reserved for
Scheduled Castes, Scheduled Tribes and the Other Backward Classes.
(2) While making service allocation, the candidates belonging to the
Scheduled Castes, the Scheduled Tribes or Other Backward Classes
recommended against unreserved vacancies may be adjusted against
reserved vacancies by the Govt. if by this process they get a service of
higher choice in the order of their preference.
(3) The Commission may further lower the qualifying standards to take
care of any shortfall of candidates for appointment against unreserved
vacancies and any surplus of candidates against reserved vacancies arising
out of the provisions of this rule, the Commission may make the
recommendations in the manner prescribed in sub-rules (4) and (5).
(4) While recommending the candidates, the Commission shall, in the first
instance, take into account the total number of vacancies in all categories.
This total number of recommended candidates shall be reduced by the
number of candidates belonging to the Scheduled Castes, the Scheduled
Tribes and Other Backward Classes who acquire the merit at or above the
fixed general qualifying standard without availing themselves of any
concession or relaxation in the eligibility or selection criteria in terms of
the proviso to sub-rule (1). Along with this list of recommended
candidates, the Commission shall also declare a consolidated reserve list
of candidates which will include candidates from general and reserved
categories ranking in order of merit below the last recommended
candidate under each category. The number of candidates in each of these
categories will be equal to the number of Reserved Category candidates
who were included in the first list without availing of any relaxation or
concession in eligibility or selection criteria as per proviso to sub-rule (1).
Amongst the reserved categories, the number of candidates from each of
the Scheduled Caste, the Scheduled Tribe and Other Backward Class
categories in the reserve list will be equal to the respective number of
vacancies reduced initially in each category.
(5) The candidates recommended in terms of the provisions of sub-rule
(4), shall be allocated by the Government to the services and where certain
vacancies still remain to be filled up, the Government may forward a
requisition to the Commission requiring it to recommend, in order of
merit, from the reserve list, the same number of candidates as
requisitioned for the purpose of filling up the unfilled vacancies in each
category."
4.5)
Rule 19 provides that due consideration will be given at the time of making
allocation on the results of the examination to the preferences expressed by a candidate
for various services at the time of his application and the appointment to various services
will also be governed by the Rules/Regulations in force, as applicable to the respective
Services at the time of appointment.
5.
The total vacancies notified by the participating services for the Civil Service
Examination, 2005 were 457 made up of General Category : 242, OBC category : 117,
Scheduled Castes : 166 and Scheduled Tribes : 32. As per Rule 16(1) and (4), UPSC
recommended 425 candidates in the first phase made up of the following: General -- 210,
OBC -- 117 (including 31 merit candidates); Scheduled Castes -- 66 (including 1 merit
candidate) and Scheduled Tribes -- 32. A consolidated Reserve list (wait-list) was also
prepared consisting of 64 candidates. The DOP&T after allocation of the candidates from
the first list, made a requisition for recommendation of candidates through the operation
of the reserve list. 26 Meritorious OBC candidates and one Meritorious Scheduled
Caste candidate recommended against unreserved vacancies, opted for reserved vacancies
as by that process, they got a service of higher choice in the order of preference. If the
said 27 meritorious reserved category candidates had been considered only for service
allocation against unreserved vacancies in competition with the General Category
candidates, they would have got a service of lower choice. Rule 16(2) enabled the
meritorious candidate of any of the reservation categories to get a service of higher
preference so that he may not be placed at a disadvantaged position vis a vis other
candidates of his category.
6.
The DOP&T could therefore adjust only 5 out of the 31 Meritorious Category
OBC candidates through their merit-cum-service preference option as General
Candidates. As a result, the UPSC recommended under Rule 16(5) of the Rules, 27
General Category candidates and 5 OBC candidates from the consolidated Reserve List.
7.
Certain OBC candidates in the Reserve (wait list) filed applications before the
Central Administrative Tribunal, Madras Bench, challenging Rule 16(2). It was
contended that adjustment of OBC merit candidates against OBC reservation vacancies
was illegal. According to them, such candidates should be adjusted against the general
(unreserved) vacancies, as that would have allowed more posts for OBC candidates and
would have allowed the lower ranked OBC candidates a better choice of service. They
contended that more meritorious OBC candidates should be satisfied with lower choice of
service as they became general (unreserved) candidates by reason of
their better
performance.
8.
The Tribunal, after interpreting amended Rule 16(2) in the light of the various
judgments of this Court, concluded that meritorious OBC candidates who were selected
on merit must be adjusted against the `General Category'. However, it ordered that Rule
16(2) may be applied in terms of decision of this Court in Anurag Patel vs. U.P. Public
Service Commission & Ors., (2005) 9 SCC 742, to ensure that allocation of service is in
accordance with rank-cum-preference with priority given to meritorious candidates for
service allocation.
9.
The Union of India and other aggrieved candidates preferred Writ Petitions before
the Madras High Court challenging
the
order
of
the
Central
Administrative
Tribunal. Some other aggrieved candidates got themselves impleaded in the said
proceedings. By the impugned order dated 20.3.2008, the High Court held Rule 16(2) as
unconstitutional. Consequently, the High Court set aside the select lists and directed the
Government of India and UPSC to redo service allocation de hors Rule 16(2).
10.
The first batch of civil appeals @ SLP [C] Nos.13571-13572 of 2008 is filed by
the Union of India against the said order dated 20.3.2008 in W.P. [C] Nos.1814 & 1815
of 2008. Other persons aggrieved by the said order have filed the remaining civil appeals.
Being aggrieved by the action of the Union Public Service Commission and the
Government of India by which candidates in Reserved Category selected in General
Category were given choice to opt for service of higher preference in terms of Rule 16(2)
of the Rules, some of the reservation category candidates have filed Writ Petition (C)
Nos.297, 312, 336 & 416 of 2008 under Art. 32 of the Constitution of India to declare
Rule 16(2),(3),(4) and (5) of the Civil Services Examination Rules, 2005 as ultra vires
being inconsistent with Rule 16(1) of the said Rules, as violative of Articles 14, 16(4)
and 335 of Constitution of India, consequential reliefs.
11.
We heard Mr. Gopal Subramanium, Learned Solicitor General of India, on behalf
of the Union of India. Ms. Indira Jaisingh, Learned ASG appeared in W.P.(C)
No.297/2008. Mr. P.P. Rao, Sr. Adv., Mr. P.S. Patwalia, Sr. Adv. and Mr. Anirudh
Sharma, Adv. represented the appellants in the other appeals. Mr. Raju Ramachandran,
Sr.
Adv., Mr. Nidheesh Gupta, Sr. Adv., Prof. Ravi Varma Kumar, Sr. Adv., Mr.
Santosh Paul, Adv., Mr. S.P. Sinha, Adv., Mr. Praveen Agarwal, Adv., and Mr. Shiv
Pujan Singh Adv., appeared on behalf of the writ petitioners and the respondents in the
writ appeals.
12.
The case of the contesting respondents is that the newly introduced system which
is different from the single list system followed earlier (prior to amendment of CSE
Rules) will undermine the rights of the Reserved Category candidates to get assigned to
services of higher preference (e.g. IAS, IPS or IRS). They also urged that this system will
reduce the aggregate number of reserved candidates who will be selected while
simultaneously increasing the number of general candidates. It also puts candidates who
come through the second list at a disadvantage in terms of seniority and promotions for
rest of their career in their respective services. By the impugned order, the High Court
had vindicated these grievances, particularly those raised by OBC candidates.
13.
In the light of the submissions made by the learned counsel appearing for
different appellants, the following questions arise for consideration:
I. Whether the Reserved Category candidates who were selected on merit
(i.e. MRCs) and placed in the list of General Category candidates could
be considered as Reserved Category candidates at the time of "service
allocation"?
II.
Whether Rule 16 (2), (3), (4) and (5) of the CSE Rules are
inconsistent with Rule 16 (1) and violative of Articles 14, 16 (4) and 335
of the Constitution of India?
III. Whether the order of the Central Administrative Tribunal was valid
to the extent that it relied on Anurag Patel v. Uttar Pradesh Public Service
Commission and Others, (2005) 9 SCC 742 (which in turn had referred to
the judgment in Ritesh R. Sah v. Dr. Y.L.Yamul and Others, (1996) 3
SCC 253, which dealt with reservations for the purpose of admission to
post graduate medical courses); and whether the principles followed for
reservations in admissions to educational institutions can be applied to
examine the constitutionality of a policy that deals with reservation in civil
services.
Re: Question I
14.
The relevant provision is Rule 16(2) of the Civil Services Examination Rules
which was amended by a notification dated 4.12.2004 issued by the Ministry of
Personnel, Public Grievances, and Pensions (DOP&T), New Delhi. The appellants'
contention is that the amended Rule 16 (2) intends to rectify an anomaly, as otherwise,
the interests of the Meritorious Reserved Category (hereinafter `MRC') candidates who
have toiled hard to qualify as per the general qualifying standard would be jeopardized.
Such candidates could find themselves in a position where Reserved Category candidates
who are less meritorious than them can possibly secure posts in a service of a higher
preference. The Union Government contends that the object of amending Rule 16 (2) is
to ensure that such an adverse incongruous position does not arise for more meritorious
candidates.
15.
Mr. Gopal Subramanium, the Learned Solicitor General of India, has brought
forth three implications and repercussions of the amended Rule 16 once it comes into
operation:
(i) It affords a Meritorious Reserved Candidate the benefit of
reservation insofar as Service Allocation is concerned. In other words, if
such a Meritorious Reserved Candidate – although entitled to a post in the
General list- is able to secure a better (or more preferred) post in the
Reserved List, Rule 16 (2) comes to his aid, and he is able to secure the
better post. This preserves and protects inter se merit amongst the
Reserved Candidates.
(ii) When Rule 16 (2) enables a Meritorious Reserved Candidate to
secure a post in the Reserved Category, that Candidate is to be treated as a
Reserved Candidate consistent with his Reserved Category status as per
the application form).
(iii) Once Rule 16 (2) is operated, the General post that would
otherwise have been available to the Meritorious Reserved Candidate is
now filled up by a (Wait Listed) General Candidate. The Respondents
have objected to the effect of Rule 16 (2) in so far as the second and third
aspects are concerned. They have no grievance with respect to the first
aspect. They contend that when an MRC candidate is entitled to a General
Merit slot, chooses to opt for a slot earmarked for a reservation category
the result should be a mutual exchange between the meritorious
reserved candidate and the reserved candidate. The MRC candidate will
carry the tag of a general candidate even when he occupies the reservation
post and the occupant of the reservation post will migrate to the general
merit slot vacated by the MRC candidate. If the MRC candidate migrating
to reservation category slot is counted as a reservation candidate, to that
extent there will be a reduction in the posts meant for reservation category
candidates.
16.
The Civil Services Examination conducted by Union Public Service
Commission (UPSC) has three stages: Preliminary Examination, Main Examination,
and Interview. The candidates appearing in the Examination have to render information
in the application form indicating their status as General, Other Backward Class (OBC),
Scheduled Castes (SC) or Scheduled Tribes (ST). Moreover, at a later stage the
candidates have to furnish their preferences of services in which they have to indicate
their choices in the event of qualification. This has been spelt out in Rule 2 of the CSE
Rules.
17.
In support of their contentions, the respondents have relied upon the following
observations of this Court in Union of India v. Satya Prakash, (2006) 4 SCC 550, (at
paras. 18, 19 and 20):
"18. By way of illustration, a Reserved Category candidate, recommended
by the Commission without resorting to relaxed standard (i.e. on merit)
did not get his own preference 'say IAS' in the merit/open category. For
that, he may opt a preference from the Reserved Category. But simply
because he opted a preference from the Reserved Category does not
exhaust quota of OBC category candidate selected under relaxed standard.
Such preference opted by the OBC candidate who has been recommended
by the Commission without resorting to the relaxed standard (i.e. on merit)
shall not be adjusted against the vacancies re-served for the Scheduled
Castes, Scheduled Tribes and other Backward Classes. This is the mandate
of proviso to Sub-rule 2 of Rule 16.
19.
In other words, while a Reserved Category candidate
recommended by the Commission without resorting to the relaxed
standard will have the option of preference from the Reserved Category
recommended by the Commission by re-sorting to relaxed standard, but
while computing the quota/percentage of reservation he/she will be
deemed to have been allotted seat as an open category candidate (i.e. on
merit) and not as a Re-served Category candidate recommended by the
Commission by resorting to relaxed standard.
20.
If a candidate of Scheduled Caste, Scheduled Tribe and other
Backward Class, who has been recommended by the Commission without
resorting to the relaxed standard could not get his/her own preference in
the merit list, he/she can opt a preference from the Reserved Category and
in such process the choice of preference of the Reserved Category
recommended by resorting to the relaxed standard will be pushed further
down but shall be allotted to any of the remaining services/posts in which
there are vacancies after allocation of all the candidates who can be
allocated to a service/post in accordance with their preference."
18.
The decision in Satya Prakash was rendered prior to the amendment of Rule 16(2)
and the learned judge had not contemplated the present version of the rule. Hence, this
decision is clearly distinguishable from the present case. Prior to the decision in Satya
Prakash's case (supra.), the practice had been that a single list of successful candidates
was released in respect of all the vacancies. At that time, MRC candidates were initially
treated as general candidates and had Rule 16(2) not been amended, a single list would
have been released for all 457 posts which were vacant in the year under consideration.
Accordingly, such a list would have contained 242 General candidates (including 32
MRC candidates). There would have been a separate list for 117 OBCs, 66 SCs and 32
STs (excluding MRC candidates). When the MRC Candidates were shifted from the
general list to the reserved list, there was an ouster of the relatively lower ranked
Reserved Category candidates who were initially selected as part of the reserved list. For
example when 27 MRC candidates (26 belonging to OBC and 1 SC) would have moved
from the General List to the Reserved List, 26 OBC and 1 SC candidates who were
ranked lower among the 117 OBC and 66 SC candidates initially selected in the Reserved
Category, would have been ousted.
19.
The unamended as well as amended Rule 16 (2) are as follows:-
Rule 16 (2) in the old Civil Rule 16 (2) in the current Service Examination
Rules Civil Service Examination Rules (vide notification dated 4.12.2004)
The candidates belonging to While making service any of the Scheduled
Castes allocation, the candidates or Scheduled Tribes or the belonging to
the Scheduled Other Backward Classes Castes, the Scheduled Tribes may,
to the extent of the or Other Backward Classes number of vacancies
recommended against reserved for the Scheduled unreserved vacancies
may be Castes and the Scheduled adjusted against reserved Tribes and
the Other vacancies by the Backward Classes be Government, if by this
recommended by the process, they get a service of Commission by a
relaxed higher choice in the order of standard, subject to the their
preference. fitness of these candidates for selection to services. Provided
that the candidates belonging to the Scheduled Castes and the Scheduled
Tribes and the Other Backward Classes who have been recommended
by the Commission without resorting to the relaxed standard referred to in
this sub-rule shall not be adjusted against the vacancies reserved for the
Scheduled Castes and the Scheduled Tribes and the Other Backward
Classes.
20.
The UPSC declares results in two stages and the same was done in the year 2006.
As per the final result of CSE 2005, out of 457 vacancies, 425 candidates
were
recommended for appointment which included 210 General, 117 OBC, 66 SC and 32 ST
candidates. The UPSC was maintaining a consolidated reserve list, i.e. a Wait List of 64
candidates (consisting of 32 general, 31 OBC and 1 SC candidate) ranking in order of
merit below the last recommended candidate under each of these categories as per Rule
16 (4) and (5) of the CSE Rules, 2005. Admittedly, 31 OBC category candidates who had
qualified in the General Merit List were not included in the General Category and instead
they were part of 117 OBC category candidates selected as part of the Reserved
Category. Hence, an equal number of OBC category candidates who were ranked lower
in the order of merit as part of the Reserved Category seats were initially ousted. The
purpose of including those OBC category candidates who had qualified in the General
Category was to give them a higher preferred service from the vacancies under the OBC
category. The CSE rules were accordingly amended to allow for such a migration.
21.
The Learned Solicitor General has described in detail how along with the list of
recommended candidates, the UPSC also prepares a Consolidated Reserve List. This
Consolidated Reserve List is a Wait List for filling the remaining 32 vacancies. It
contained two parallel sub-lists: Wait List A consisting of 32 General Candidates and
Wait List B consisting of 32 Reserved Candidates (31 OBCs and 1 SC) the 1 SC
candidate would be positioned in the Wait List at the same position in which the 1 SC
candidate was placed amongst the 32 MRC candidates. Two Wait Lists are prepared so
that depending on how the 32 MRCs are placed and in whatever contingency - whether
they are adjusted against General or Reserved Posts - there will remain a sufficient
number of candidates (both general and reserved) to be adjusted against the balance 32
posts in the second stage.
22.
When Department of Personnel and Training (DOP&T) received the Lists, the 32
MRC candidates were added to the list of 210 General candidates but at the same time
they were positioned in the reserved lists of 117 OBC candidates and 66 SC candidates as
well. The UPSC list counts the MRC candidates as part of the Reserved List for the
purpose of ascertaining the reservation quota in terms of percentage. The rationale cited
for this method is that for the purpose of service allocation, the DOP&T initially counts
the MRC candidates in both the General and the Reserved Lists. These candidates are
then placed against the better of the two services available to them under either of these
categories which is of course based on their order of preference. A Service is allocated by
moving downwards in the merit list in a serial manner, with each candidate in the merit
list getting the best available option as per his/her preference.
23.
The respondents have also placed strong reliance on this Court's decision in
Ritesh R. Sah v. Dr. Y.L.Yamul (1996) 3 SCC 253). The question in that case was
whether a Reserved Category candidate who is entitled to be selected for admission in
open competition on the basis of his/her own merit should be counted against the quota
meant for the Reserved Category or should he be treated as a general candidate. The
Court reached the conclusion that when a candidate is admitted to an educational
institution on his own merit, then such admission is not to be counted against the quota
reserved for Schedule Castes or any other Reserved Category. However, it is pertinent to
note that this decision was given in the context of admissions to medical colleges in
which G.B. Pattanaik J. (as His Lordship then was) had held:
"17. ...In view of the legal position enunciated by this Court in the
aforesaid cases the conclusion is irresistible that a student who is entitled
to be admitted on the basis of merit though belonging to a Reserved
Category cannot be considered to be admitted against seats reserved for
Reserved Category. But at the same time the provisions should be so made
that it will not work out to the disadvantage of such candidate and he may
not be placed at a more disadvantageous position than the other less
meritorious Reserved Category candidates. The aforesaid objective can be
achieved if after finding out the candidates from amongst the Reserved
Category who would otherwise come in the open merit list and then asking
their option for admission into the different colleges which have been kept
reserved for Reserved Category and thereafter the cases of less meritorious
Reserved Category candidates should be considered and they will be
allotted seats in whichever colleges the seats should be available. In other
words, while a Reserved Category candidate entitled to admission on the
basis of his merit will have the option of taking admission to the colleges
where a specified number of seats have been kept reserved for Reserved
Category but while computing the percentage of reservation he will be
deemed to have been admitted as an open category candidate and not as a
Reserved Category candidate..."
24.
There is an obvious distinction between qualifying through an entrance test for
securing admission in a medical college and qualifying in the UPSC examinations since
the latter examination is conducted for filling up vacancies in the various civil services.
In the former case, all the successful candidates receive the same benefit of securing
admission in an educational institution. However, in the latter case there are variations in
the benefits that accrue to successful candidates because
they are also competing
amongst themselves to secure the service of their choice. For example, most candidates
opt for at least one of the first three services [i.e. Indian Administrative Service (IAS),
Indian Foreign Service (IFS) and Indian Police Service (IPS)] when they are asked for
preferences. A majority of the candidates prefer IAS as the first option. In this respect, a
Reserved Category candidate who has qualified as part of the general list should not be
disadvantaged by being assigned to a lower service against the vacancies in the General
Category especially because if he had availed the benefit of his Reserved Category status,
he would have got a service of a higher preference. With the obvious intention of
preventing such an anomaly, Rule 16 (2) provides that an MRC candidate is at liberty to
choose between the general quota or the respective Reserved Category quota.
25.
Some factual examples can clarify the position. In 2005, an MRC (OBC)
candidate attained 21st Rank overall. With respect to his position in the General Merit
List, there were General Category IAS vacancies available, and he occupied the 17th out
of 45 General vacancies in the IAS. Thus, he did not need the assistance of Rule 16(2) to
get a post in a more preferred service since he was adjusted against the General List.
Accordingly, he opted out of the Reserved Category. This was in line with the
proposition that when a candidate is entitled to a certain post on his merit alone, he
should not be counted against the reserved quota. In contrast, another candidate who was
an MRC (OBC) candidate obtained 64th Rank overall in the CSE 2005. At his position in
the General List, he was entitled to a post in the IPS since the General Category IAS
vacancies had been exhausted by candidates above him in the General merit list.
However, IPS was his second preference while IAS was his first preference. If he were to
be considered against the vacancies in the Reserved Category, he would be entitled to a
post in the IAS because the 22 OBC IAS vacancies had not been exhausted at that point
of time. By the operation of Rule 16 (2), he was able to secure a post in the IAS, while
retaining his Reserved Status. Having availed of this benefit, he was adjusted against the
Reserved (OBC) category.
26.
Learned Counsel for respondent questioned the rationale of declaring the CSE
results in two phases in order to support the proposition that even if MRC candidates are
given a service of a higher preference, they should not oust lower-ranked Reserved
Category candidates. However, Rule 16(2) should not be interpreted in an isolated
manner since it was designed to protect the interests of MRC candidates.
MRC
candidates having indicated their status as SC/ST/OBC at the time of application, begin
their participation in the examination process as Reserved Candidates. Having qualified
as per the general qualifying standard, they have the additional option of opting out of the
Reserved Category and occupying a General Post. Where, however, they are able to
secure a better post in the Reserved List their placement in the General List should not
deprive them of the same. In that respect, the adjustment referred to in Rule 16(2) does
not, in fact, denote any change in the status of the MRC from General to Reserved. To the
contrary, it is an affirmation of the Reserved Status of the MRC candidate. Rule 16(2)
exists to protect this Reserved Status of the MRC candidates.
27.
We must also take note of the fact that when MRC candidates get adjusted against
the Reserved Category, the same creates corresponding vacancies in the General Merit
List (since MRC candidates are on both lists). These vacancies are of course filled up by
general candidates. Likewise, when MRC candidates are subsequently adjusted against
the General Category [i.e. without availing the benefit of Rule 16 (2)], the same will
result in vacancies in the Reserved Category which must in turn be filled up by Wait
Listed Reserved Candidates. Moreover, the operation of Rule 16 does not result in the
ouster of any of the candidates recommended in the first list. Many of the wait-listed
candidates are accommodated in the second stage, and the relatively lower ranked waitlisted candidates are excluded. It is pertinent to note that these excluded candidates never
had any absolute right to recruitment or even any expectation that they would be
recruited. Their chances depend on how the MRC candidates are adjusted.
28.
In the impugned judgment, the High Court had reasoned that allocation to a
particular post cannot be distinguished from allocation to a service for the purpose of
reservation. However, the High Court had not considered the fact that in the CSE
examination, the candidates are not competing for similar posts in one service but are
instead competing for posts in different services that correspond to varying preferences.
Furthermore, the impugned judgment did not appreciate the possibility that when an
SC/ST/OBC candidate qualifies on merit (i.e. without any relaxation/concession) there
can be a situation where a lower ranked OBC candidate gets allotted to a better service in
comparison to a higher ranked SC/ST/OBC candidate simply because the higher ranked
OBC candidate performed well enough to qualify in the General Category. Such a
situation is anomalous. As we have already discussed, the High Court's reliance on the
decision of this Court in Union of India v. Satya Prakash, (supra.), is not tenable since it
dealt with the effect of Rule 16 (2) as it existed prior to the amendment notified on
4.12.2004.
29.
A significant aspect which needs to be discussed is that the aggregate reservation
should not exceed 50% of all the available vacancies, in accordance with the decision of
this Court in Indra Sawhney v. Union of India, (1992) Supp 3 SCC 217. If the MRC
candidates are adjusted against the Reserved Category vacancies with respect to their
higher preferences and the seats vacated by them in the General Category are further
allotted to other Reserved Category candidates, the aggregate reservation could possibly
exceed 50 % of all of the available posts.
30.
In Post Graduate Institute of Medical Education and Research v. Faculty
Association, (1998) 4 SCC 1, G.N. Ray J. had clearly stated that the upper ceiling of 50%
reservations should not be breached:
"32. Articles 14, 15 and 16 including Articles 16(4), 16(4-A) must be
applied in such a manner so that the balance is struck in the matter of
appointments by creating reasonable opportunities for the reserved classes
and also for the other members of the community who do not belong to
reserved classes. Such a view has been indicated in the Constitution Bench
decision of this Court in Balaji case, Devadasan case and Sabharwal
case. Even in Indra Sawhney case the same view has been held by
indicating that only a limited reservation not exceeding 50% is
permissible. It is to be appreciated that Article 15 (4) is an enabling
provision like Article 16 (4) and the reservation under either provision
should not exceed legitimate limits. In making reservations for the
backward classes, the State cannot ignore the fundamental rights of the
rest of the citizens. The special provision under Article 15 (4) [sic 16 (4)]
must therefore strike a balance between several relevant considerations
and proceed objectively. In this connection reference may be made to the
decisions of this Court in State of A.P. v. U.S.V. Balram and C.A.
Rajendran v. Union of India. It has been indicated in Indra Sawhney that
clause (4) of Article 16 is not in the nature of an exception to clauses (1)
and (2) of Article 16 but an instance of classification permitted by clause
(1). It has also been indicated in the said decision that clause (4) of Article
16 does not cover the entire field covered by clauses (1) and (2) of Article
16. In Indra Sawhney case this Court has also indicated that in the
interests of the backward classes of citizens, the State cannot reserve all
the appointments under the State or even a majority of them. The doctrine
of equality of opportunity in clause (1) of Article 16 is to be reconciled in
such a manner that the latter while serving the cause of backward classes
shall not unreasonably encroach upon the field of equality."
31.
In State of Kerala v. N.M. Thomas, (1976) 2 SCC 310, the same proposition was
enunciated by A.N. Ray, C.J. who had held:
"26. The respondent contended that apart from Article 16 (4) members of
scheduled castes and scheduled tribes were not entitled to any favoured
treatment in regard to promotion. In T.Devadasan v. Union of India
reservation was made for backward classes. The number of reserved seats
which were not filled up was carried forward to the subsequent year. On
the basis of "carry forward" it was found that such reserved seats might
destroy equality. To illustrate, if 18 seats were reserved and for two
successive years the reserved seats were not filled and in the third year
there were 100 vacancies the result would be that 54 reserved seats would
be occupied out of 100 vacancies. This would destroy equality. On that
ground "carry forward" principle was not sustained in Devadasan's case
(supra). The same view was taken in the case of M.R.Balaji v. State
of
Mysore. It was said that not more than 50 percent should be reserved for
backward classes.
This ensures equality. Reservation is not a
constitutional compulsion but is discretionary according to the ruling of
this Court in Rajendran's case (supra)."
32.
Therefore, we are of the firm opinion that MRC candidates who avail the benefit
of Rule 16(2) and are eventually adjusted in the Reserved Category should be counted as
part of the reserved pool for the purpose of computing the aggregate reservation quotas.
The seats vacated by MRC candidates in the general pool will therefore be offered to
General Category candidates. This is the only viable solution since allotting these
General Category seats (vacated by MRC candidates) to relatively lower ranked Reserved
Category candidates would result in aggregate reservations exceeding 50% of the total
number of available seats. Hence, we see no hurdle to the migration of MRC candidates
to the Reserved Category.
Re: Question II
33.
We have extracted Rule 16 of the Civil Service Examination Rules, as per
notification dated 4.12.2004 issued by the Ministry of Personnel, Public Grievances and
Pensions (Department of Personnel and Training), New Delhi. A perusal of the rule
discloses the following: Rule 16(1) mandates that after the interview phase, the
candidates will be arranged in the order of merit on the basis of aggregate marks obtained
in the main examination. Later on, the UPSC shall fix a qualifying mark for
recommending the candidates for the unreserved vacancies. Proviso to sub-rule (1) lays
down that a candidate who belongs to the SC, ST & OBC categories and who has
qualified on his own in the merit list shall not be recommended against the vacancies
reserved for such classes if such candidate has not availed of any of the concessions or
relaxations in the eligibility or the selection criteria. The other sub-rules provide as to
how Meritorious Reserve Category candidates are to be adjusted and once they get
services of their preference after availing the benefit of their reserved status (as SC, ST,
OBC or any other applicable category), the candidates whose names are in the
consolidated reserve lists are to be subsequently adjusted. The consolidated wait list
includes the candidates from General Category and Reserved Category. If an MRC
candidate who belongs to OBC category has availed the benefit of his status for better
service allocation then the seat vacated by him will go to a General Category candidate. If
he chooses not to avail the benefits of special status then he would be counted in General
Category and the seat vacated by him in the Reserved Category will automatically go to a
candidate who belongs to the same Reserved Category.
34.
As per the submissions made before this Court, in the year 2005, 27 MRC
candidates were adjusted against Reserved Category and 5 MRC candidates were
adjusted in General Category. As already explained, the current process entails that a
Reserved Candidate, although having done well enough in the examination to have
qualified in the open category, does not automatically rescind his/her right to a post in the
Reserved Category. Furthermore, Rule 16(2) operates to recognize the inter se merit
amongst the Reserved Category Candidates. The two stage process is designed in a
manner that no person included in the first recommended list is subsequently eliminated.
However, since the wait list contains more candidates than available posts, it is inevitable
that some persons in the wait list will necessarily be excluded. Such exclusion is on the
basis of merit and the aggrieved parties were never promised a post.
35.
The following chart presented by the Learned Solicitor General explains how
service allocation has been done for the years 2005, 2006 and 2007:
Service Allocation in the Years 2005, 2006, 2007
Vacancy Position
Year
2005
2006
2007
General
Vacancies
242
273
382
OBC
Vacancies
117
144
190
SC
ST
Total
Vacancies Vacancies Vacancies
66
80
109
32
36
53
457
533
734
Candidates Recommended Against vacancies in the first case
Year
General
OBC
SC
ST
Total
Candidates Candidates Candidates Candidates Candidates
2005
210
117
(including (including
31 merit
1 merit
Candidates) Candidate)
2006
214
144
80
(including (including (including
41 merit
15 merit
2 merit
candidates) candidates) candidates)
2007
66
32
425
36
474
286
190
109
53
(including (including (including
76 merit
19 merit
1 merit
candidates) candidates) candidate)
638
However, we have been apprised that on account of the intervening order of the CAT
Chennai Bench (dated 17.09.07 in O.A. No. 690 and 775 of 2006), the Department of
Personnel & Training (DOP&T) has not been able to proceed with service allocation
against the second list. Similarly, for the years 2006 and 2007, the UPSC is maintaining
a Consolidated Reserve List of 116 and 192 candidates respectively, but DOP&T has not
sent any requisition for the second list as per Rule 16(5).
36.
In State of Bihar v. M .Neeti Chandra, (1996) 6 SCC 36, this Court was
confronted with broadly analogous issues. In that case, the Controller of Examinations,
Health Services, Government of Bihar, Patna had issued the prospectus for a competitive
examination for admission to post graduate courses in Patna Medical College (Patna),
Darbhanga Medical College (Laheria
Sarai), Rajendra Medical College (Ranchi) and
Mahatma Gandhi Medical College (Jamshedpur) for the year 1992. The prospectus
contained the following provisions with respect to reservations:
"The reservation of seats for various categories shall be as per the decision
of the government. There will be no economic criteria for the reservation.
Scheduled Caste 14% Scheduled Tribe 10% Extremely Backward Class
14% Backward Class 9% Ladies 3%. The Government of Bihar acting
through the Department of Personnel and Administrative Reforms
published a resolution dated 7-2-1992, bearing No. 11/K1-1022/91-K 20
[Hereinafter "Resolution No. 20"]. Paragraph 6 of the same is reproduced
below:
"As there is provision in direct appointment to the effect that the
candidates belonging to reserved classes, who are selected on the basis of
merit would not be adjusted against reserved seats, similarly maintaining
the same arrangement here also the candidates selected on the basis of
merit for admission into professional training institutes would not be
adjusted against the reserved quota for the candidates of the reserved
classes".
The High Court of Patna which considered the matter devised a method to remove the
anomalies. It initiated a process of allotment of seats by which the reserved seats were
offered first (i.e. before the general seats are filled first) to the candidates of the Reserved
Category on merit, and after all the reserved seats were so filled up, all other qualifying
candidates
of
the
Reserved
Category
were `adjusted' against open seats in the
General Category along with the general merit candidates and offered seats on meritcum-choice basis. Furthermore, the High Court made arrangement for the Reserved
Category of girls who could get seats under the reservation for girls or under those
reserved for SCs/STs etc., thereby retaining a choice between one of the two reservations.
The girls in excess of the reserved vacancies could seek admission on general merit. The
High Court held that by this procedure all the anomalies in the procedure for allotment of
seats could be removed. In the meantime, another resolution was passed which was
supposed to rectify the anomalies arising out of the operation of the previous Resolution.
The Resolution dated 22-3-1994 provided that casual vacancies occurring at a later stage
in the General Category or Reserved Category would be filled from amongst the
candidates of the respective category on merit and in that process no candidate would be
allotted a college/course below the choice of the college or course already allotted. The
High Court observed that the resolution takes care of the grievances of the candidates
who by reason of readjustment at the State for filling up subsequent vacancies often had
to lose the college/course of their choice but it did not address the anomaly that arises
when preparing the main merit list as per Resolution No. 20. State of Bihar moved this
Court in appeal against the judgment of the Patna High Court and the main ground was
that if the method suggested by the High Court was followed, all students of Reserved
Category who had secured the minimum marks would have to be admitted even though
there may not be adequate number of vacancies for them. A.M. Ahmadi, C.J. pronounced
this contention to be very genuine and laid down:
"10. Let us take a situation in which in a particular Reserved Category
there are x number of seats but the candidates qualifying according to
criteria fixed for that category are x+5 with the best among them also
qualifying on merit as general candidates. According to the arrangement
made by Circular No. 20, the first candidate gets a choice along with the
General Category candidate but being not high enough in the list, gets a
choice lesser than what he could secure in the Reserved Category to which
he was entitled. The x number of seats could then be filled up with the
four qualifying candidates being denied admission for want of seats. This
would have been harsh for the best candidate as well as violative of
Articles 14 and 16 of the Constitution. On the other hand, if the direction
of the High Court is followed, the first x number of candidates get seats
according to merit against the reserved seats but the remaining will also
have to be `adjusted' against the open seats for regular candidates. These
will be those who are not qualified according to general merit criteria and
so will necessarily displace 5 general candidates who would be entitled to
seats on merit.
11. In a particular year, the number of such candidates may be much
larger and thus the method evolved by the High Court may create much
hardship. The method will also not be in tune with the principles of
equality. Hence the method evolved by the High Court will have to be
struck down.
12. If however, the word `adjusted 'is read to mean considered along
with the general merit list candidates, it will lose much of its value. As per
the above illustration, the 5 candidates qualifying on Reserved Category
criteria having not secured enough marks according to general criteria,
cannot, at all be allotted any seat in the General Category.
13. At the same time, as pointed out above, all is not well with the
Government Circular No.20 as it operates against the very candidates for
whom the protective discrimination is devised. The intention of Circular
No. 20 is to give full benefit of reservation to the candidates of the
reserved. However, to the extent the meritorious among them are denied
the choice of college and subject which they could secure under the rule of
reservation, the circular cannot be sustained. The circular, therefore, can
be given effect only if the Reserved Category candidate qualifying on
merit with general candidates consents to being considered as a general
candidate on merit-cum-choice basis for allotment of college/institution
and subject."
37.
Learned Counsel on behalf of the petitioner in W.P.(C) No. 297 of 2008 has relied
upon the following observations of Jeevan Reddy J., in Indra Sawhney v. Union of India
(supra.) (para 811) :
"811. ...it is well to remember that the reservations under Article 16 (4)
do not operate
like a communal reservation. It may well happen that
some members belonging to, say, Scheduled Castes get selected in the
open competition field on the basis of their own merit; they will not be
counted against the quota reserved for Scheduled Castes; they will be
treated as open competition candidates."
The said observations are not of any assistance as no MRC candidate occupying a
General Category slot is being counted against the quota for the Reserved Category. For
example those MRC candidates belonging to the OBC category who cut across the
general qualifying standard and are appointed to general posts are not being counted
within the 27% quota earmarked for OBCs. However, MRC candidates who retain their
reserved status and avail of the benefit of Rule 16 (2) to occupy a reserved post are
counted against the reservation quota. When MRC candidates do not choose to accept the
General Category slot available to them on account of their merit, but opt to occupy a slot
reserved for reservation category candidates, because that post is more attractive, then
counting him/ her against reservation quota will not violate the law laid down in Indra
Sawhney (supra.).
38.
In M. Nagaraj v. Union of India (2006) 8 SCC 212, a Constitution Bench of this
Court held:
"102. ... Equality has two facets- "formal equality" and "proportional
equality". Proportional equality is equality "in fact" whereas "formal
equality" is equality "in law". Formal equality exists in the rule of law. In
the case of proportional equality the State is expected to take affirmative
steps in favour of disadvantaged sections of society within the framework
of liberal democracy. Egalitarian equality is proportional equality."
39.
Article 16 (4) of the Constitution provides that nothing in Article 16 shall prevent
the State from making any provision for the reservation of appointments or posts in
favour of any backward classes of citizens which, in the opinion of the State, is not
adequately represented in the services under the State. Article 16(4) empowers the State
to initiate measures in order to protect and promote the interests of backward classes
(OBC, SC & ST). The impugned measures in no way offend the equality clause since this
particular clause was inserted to safeguard the concerns of certain classes and shield their
legitimate claims in the domain of public employment. On behalf of the respondents in
the appeals, it was submitted Rules 16 (2), (3), (4) & (5) infringes Article 16(4). We do
not accept this proposition since Rule 16 (2) and the subsequent sub-rules merely
recognize and advance inter se merit among the Reserved Category candidates in the
manner that has been demonstrated before us by Learned Solicitor General.
40.
Therefore, Rule 16 protects the interests of a Reserved Category candidate
selected in the general (unreserved) category by giving him the option either to retain his
position in the open merit category or to be considered for a vacancy in the Reserved
Category, if it is more advantageous to him/her. The need for incorporating such a
provision is to arrest arbitrariness and to protect the interests of the Meritorious Reserved
Category candidates. If such rule is declared redundant and unconstitutional vis-à-vis
Article 14, 16 and 335 then the whole object of equality clause in the
would be
Constitution
frustrated and the MRC candidates selected as per the general qualifying
standard would be disadvantaged since the candidate of his/her category who is below
him/her in the merit list, may by availing the benefits of reservation attain a better service
when allocation of services is made. Rule 16 in essence and spirit protects the pledge
outlined in the Preamble of the Constitution which conceives of equality of status and
opportunity.
Re: Question III
41.
Central Administrative Tribunal, Chennai Bench in O.A. No. 690 of 2006 and 775
of 2006 had given the following directions -:
"(i) The impugned Rule 16 (2) is declared as valid so long as it is
confined to allocation of services and confirms to the ratio of Paras 4 to 6
of Anurag Patel order of the Hon'ble Apex Court.
(ii) The Supplementary List issued by the second respondent to the
first respondent dated 3.4.2007 is set aside. This would entail issue of a
fresh supplementary result from the reserved list of 64 in such a way that
adequate number of OBCs are announced in lieu of the OBCs who have
come on merit and brought under General Category. The respondents are
directed to rework the result in such a way the select list for all the 457
candidates are announced in one lot providing for 242-general, 117 OBC,
57 SC and 41 ST and also ensure that the candidates in OBC, SC & ST
who come on merit and without availing any reservation are treated as
general candidates and ensure that on equal number of such reserved
candidates who are of merit under General Category, are recruited for
OBC, SC & ST respectively and complete the select list for 457. Having
done this exercise, the respondents should apply Rule 16 (2) to ensure that
allocation of the service is in accordance with rank-cum-preference with
priority given to meritorious
reserved candidates for service allocation
by virtue of Rule 16 (2) which is as per para 5 of Anurag Patel order. The
entire exercise, as directed above, should be completed as per the order.
(iii) Applying the ratio of Anurag Patel decision of Hon'ble Apex Court
(Paras 6 & 7), if there is need for re-allocation of services, the respondents
will take appropriate measures to that extent and complete this process
also within two months from the date of receipt of a copy of this order."
The CAT had also issued the following direction as to how the results of the UPSC
examinations (2005) should have been announced:
"52. If the UPSC had followed the decision of the Hon'ble Apex Court cited supra and
released the select list in one go for all the 457 vacancies then it would have ensured that
the select list contained not only 117 OBCs but also an additional number of OBC
candidates by this number, in additional to 117 under 27% reservation, while
simultaneously be number of general candidates recruited will be less to the extent of
OBCs recruited on merit and included in the general list in the result of Civil Services
Examination, 2005. Once this order is met, the successful candidates list will include 242
candidates in the General Category which is inclusive of all those Reserved Category
candidates coming on merit plus 117 OBC, 57 SC and 41 ST exclusively from these
respective reserved categories by applying relaxed norms for them. If such a list is
subjected to Rule 16(2) of Civil Services Examination, 2005 in present form for making
service allocation only and then services are allotted based on Rule 16(2) in this context,
then the announcement of recruitment result and allocation services will be both in
accordance with law as per various judgments the Hon'ble Apex Court and in accordance
with the extent orders issued by the Respondent No.1 and also in keeping with spirit of
Rule 16 (2) so that, the meritorious reserved candidates get higher preference service as
compared to their lower ranked counter parts in OBC, ST,SC. In doing so, the
respondents also would notice that the steps taken by them in accordance with the
Rules 16 (3)(-)(5) are redundant once they issue the result of recruitment in one phase,
instead of two as they have become primary cause for the litigation and avoidable
confusion in the minds of the candidates seeking recruitment."
42.
We may refer to the brief facts in Anurag Patel v. Uttar
Service
Pradesh
Public
Commission, (supra.), referred to by the Tribunal. In the year 1990, the Uttar
Pradesh Public Service Commission [hereinafter `UPPSC') conducted a combined State
Services/Upper Subordinate Services examination for selection to various posts such as
Deputy Collectors in U.P. Civil (Executive) Services, Deputy Superintendent of Police in
U.P. Police Services, Treasury Officers/Account Officers in U.P. Finance and Accounts
Services, Sales Tax Officers, Assistant Regional Transport Officers, District Supply
Officers and various other posts. Pursuant to the notification issued by the UPPSC, a
large number of candidates appeared for selection. The UPPSC published the list of
selected candidates in August, 1992. Altogether 358 posts in various categories were
filled up.
The candidates belonging to the Backward Classes were entitled to get
reservation in selection in respect of 57 posts in various categories, out of a total number
of 358 posts. The posts in each category of service were filled up by choice of the
candidate and the person who secured higher position in the merit list opted for U.P. Civil
(Executive) Service and those who could not get the higher and important category of
service had to be satisfied with posts in services of lesser importance. In each category of
service, posts were reserved for SCs/STs, Backward Classes and handicapped persons
etc. The UPPSC treated the candidates belonging to SC/ST and Backward Classes who
got selection to the seats (posts) earmarked for general candidates as candidates
in the General Category and allotted them to various services depending upon the rank
secured by them in the select list. SC/ST and BC Candidates, who got lower rank in merit
lists of general category candidates got posting in lesser important services. However,
the SC/ST and BC Candidates who got selected to posts reserved in each category even
though they secured lesser rank in the whole list got appointed to reserved posts in each
category. This mode of appointments caused serious injustice to candidates who initially
applied in the Reserved Category, yet they got selected to the general seats (posts) as they
were meritorious and were entitled to get selected along with the general candidates.
However, their merit and ability did not pay any dividends as they got appointment only
to lesser important posts. This Court held:
"4. ... The authorities should have compared the candidates who are to be
appointed on general merit as also candidates who are to be appointed as
against the reserved vacancies and while making appointments the inter se
merit of the reserved candidates should have been considered and they
must have been given the option treating each service separately. As this
exercise was not followed, less meritorious candidates got appointment to
higher posts whereas more meritorious candidates had to be satisfied with
posts of lower category.
5. ...in the instant case, as noticed earlier, out of 8 petitioners in Writ
Petition No. 22753 of 1993, two of them who had secured Ranks 13 and
14 in the merit list, were appointed as Sales Tax Officer II, whereas the
persons who secured Ranks 38, 72 and 97, ranks lower to them, got
appointment as Deputy Collectors and the Division bench of the High
Court held that it is a clear injustice to the persons who are more
meritorious and directed that a list of all selected Backward Class
candidates shall be prepared separately including those candidates selected
in the General Category and their appointments to the posts shall be made
strictly in accordance with merit as per the select list and preference of a
person higher in the select list will be seen first and the appointment given
accordingly, while preference of a person lower in the list will be seen
only later. We do not think any error or illegality in the direction issued by
the Division Bench of the High Court.
6. If these candidates who got selection in the General Category are
allowed to exercise preference and then are appointed accordingly the
candidates who were appointed in the reserved categories would be
pushed down in their posts and the vacancies thus left by the General
Category candidates belonging to Backward Classes. There will not be
any change in the total number of posts filled up either by the General
Category candidates or by the Reserved Category candidates."
43.
The decision in Anurag Patel (supra.) rectified the anomaly which had occurred
since the U.P.P.S.C. had allotted services of lower preference to the candidates of
backward classes who were meritorious enough to qualify as per the criteria laid down
for General Category candidates. Such meritorious candidates were disadvantaged on
account of qualifying on merit which was patently offensive to the principles outlined in
Articles 14 and 16 of the Constitution. This Court had reached such conclusion to ensure
that allocation of service is in accordance with the rank-cum-preference basis with
priority given to meritorious candidates for service allocation.
44.
The decision in Anurag Patel (supra.) in turn referred to the earlier decision in
Ritesh R. Sah v. Dr. Y.L.Yamul and Others (supra.). However, we have already
distinguished the judgment in Ritesh R. Sah. That decision was given in relation to
reservation for admission to post-graduate medical courses and the same cannot be
readily applied in the present circumstances where we are dealing with the examinations
conducted by the UPSC. The ultimate aim of Civil Services aspirants is to qualify for the
most coveted services and each of the services have quotas for reserved classes, the
benefits of which are availed by MRC candidates for preferred service. As highlighted
earlier, the benefit accrued by different candidates who secure admission in a particular
educational institution is of a homogeneous nature. However, the benefits accruing from
successfully qualifying in the UPSC examination are of a varying nature since some
services are coveted more than others.
45.
The order of the CAT is valid to the extent that it relied on the ratio propounded
by this Court in Anurag Patel v. Uttar Pradesh Public Service Commission (supra.). Even
though that decision had in turn relied on the verdict of this Court in Ritesh R. Sah v. Dr.
Y.L.Yamul and Others, (supra.), the latter case is distinguishable from the present case
with respect to the facts in issue. However, we cannot approve of the conclusions arrived
at in the Central Administrative Tribunal order as it failed to take note of the unique
characteristics of the UPSC examinations.
46.
Reference was also made to R.K. Sabharwal v. State of Punjab, (1995) 2 SCC
745, this Court had declared that the State shall not count a Reserved Category candidate
selected in the open category against the vacancies in the Reserved Category. However,
by this it could not be inferred that if the candidate himself wishes to avail a vacancy in
the Reserved Category, he shall be prohibited from doing so. After
counsels'
considering the
submissions and deliberations among ourselves, we are of the view that the
ratio in that case is not applicable for the purpose of the present case. That case was
primarily concerned with the Punjab Service of Engineers in the Irrigation Department of
State of Punjab. The decision was rendered in the context of the posts earmarked for the
Scheduled Castes/ Scheduled Tribes and Backward Classes on the roster. It was noted
that once such posts are filled the reservation is complete. Roster cannot operate any
further and it should be stopped. Any post falling vacant in a cadre thereafter, is to be
filled from the category - reserved or general - due to retirement or removal of a person
belonging to the respective category. Unlike the examinations conducted by UPSC which
includes 21 different services this case pertains to a single service and therefore the same
cannot be compared with the examination conducted by UPSC.
The examination
conducted by UPSC is very prestigious and the top-most services of this nation are
included in this examination. In this respect, it is obvious that there is fierce competition
amongst the successful candidates as well to secure appointments in the most preferred
services. This judgment is strictly confined to the enabling provision of Article 16 (4) of
the Constitution under which the State Government has the sole power to decide whether
there is a requirement for reservations in favour of the backward class in the services
under the State Government. However, the present case deals with positions in the
various civil services under the Union Government that are filled through the
examination process conducted by the UPSC. Therefore, the fact-situation in R.K.
Sabharwal's case is clearly distinguishable.
47.
The proviso to Rule 16 (1) and Rule 16 (2) operate in different dimensions and it
is untenable to argue that these provisions are contradictory or inconsistent with each
other. As mentioned earlier, in the examination for the year 2005, 32 reserved candidates
(31 OBC candidates and 1 SC candidate) qualified as per the general qualifying standard
[Rule 16 (1)]. These MRC candidates did not avail of any of the concessions and
relaxations in the eligibility criteria at any stage of the examination, and further they
secured enough marks to place them above the general qualifying standard. MRC
candidates are entitled to one of the two posts – one depending on their performance in
the General list and other depending on their position in the Reserved List. When MRC
candidates are put in the General list on their own merit they do not automatically
relinquish their reserved status. By the operation of Rule 16 (2), the reserved status of an
MRC candidate is protected so that his/ her better performance does not deny such
candidate the chance to be allotted to a more preferred service. Where, however, an MRC
is able to obtain his preferred post by virtue of his/her ranking in the General List, he/ she
is not counted as a Reserved Candidate and is certainly not counted amongst the
respective reservation quota.
48.
We must also remember that affirmative action measures should be scrutinized as
per the standard of proportionality. This means that the criteria for any form of
differential treatment should bear a rational correlation with a legitimate governmental
objective. In this case a distinction has been made between Meritorious Reserved
Category candidates and relatively lower ranked Reserved Category candidates. The
amended Rule 16(2) only seeks to recognize the inter-se merit between these two classes
of candidates for the purpose of allocation to the various civil services with due regard
for the preferences indicated by the candidates.
49.
With regard to the specific characteristics of the UPSC examinations we hold that
Reserved Category candidates (belonging to OBC, SC or ST categories among others)
who are selected on merit and placed in the list of general/unreserved Category
candidates can choose to migrate to the respective reserved categories at the time of
allocation of services. Such migration is enabled by Rule 16 (2) of the Civil Services
Examination Rules, which is not inconsistent with Rule 16 (1) of the same or even the
content of Articles 14, 16 (4) and 335 of the Constitution of India.
50.
We sum up our answers-:
i) MRC candidates who avail the benefit of Rule 16 (2) and adjusted in the
reserved category should be counted as part of the reserved pool for the
purpose of computing the aggregate reservation quotas. The seats vacated
by MRC candidates in the General Pool will be offered to General
category candidates.
ii) By operation of Rule 16 (2), the reserved status of an MRC candidate is
protected so that his/ her better performance does not deny him of the
chance to be allotted to a more preferred service.
iii) The amended Rule 16 (2) only seeks to recognize the inter se merit
between two classes of candidates i.e. a) meritorious reserved category
candidates b) relatively lower ranked reserved category candidates, for the
purpose of allocation to the various Civil Services with due regard for the
preferences indicated by them.
iv)
The reserved category candidates "belonging to OBC, SC/ ST
categories" who are selected on merit and placed in the list of
General/Unreserved category candidates can choose to migrate to the
respective reserved category at the time of allocation of services. Such
migration as envisaged by Rule 16 (2) is not inconsistent with Rule 16 (1)
or Articles 14, 16 (4) and 335 of the Constitution.
51.
In view of the above, the civil appeals are allowed and the judgment of the
Madras High Court is set aside. The writ petitions challenging the validity of Rule 16(2)
are dismissed. The validity of Rule 16 of Civil Service Examination Rules 2005 (vide
notification dated 4.12.2004) is upheld. There will be no order as to costs.
***
CENTRAL ADMINISTRATIVE TRIBUNAL
PRINCIPAL BENCH
O.A. NO.2499/2009
M.A. NO.1683/2009
New Delhi, D.D. 18.05.2010
Hon’ble Mr. Shanker Raju, Member (J)
Hon’ble Dr. Ramesh Chandra Panda, Member (A)
Mr. Sebastian Augusthy & Ors.
Vs.
Govt of NCT of Delhi & Ors.
…
Applicants
…
Respondents
Promotion:
Applicants aggrieved by Note (i) in Column 12 of Recruitment Rules for the post
of Assistant Director/Principal ITIs/Principal Basic Training/Assistant Apprenticeship
Adviser notified on 05.05.2008 have challenged the promotion of respondents 4 and 5 in
terms of Note (i) in Column 12 of the Recruitment Rules – Tribunal after examining Note
(i) of the Recruitment Rules has held the same as valid and consequently held that
promotion of respondents 4 and 5 in terms of Note (i) is sustainable and not illegal and
dismissed the application.
Held:
That the educational qualification relaxation granted to the Departmental
candidates under Note (i) in Column 12 is across the board and not for any individual
alone. Hence, Note (i) in column 12 is not discriminatory but provides such relaxation
equally to all Departmental candidates.
Cases Referred:
JT 2008(3) SC 221 - Divisional Manager, Aravali Golf Course & Anr. vs. Chander Hass
2002(4) SCC 578 - P. Ramachandra Rao vs. State of Karnataka
ORDER
Dr. Ramesh Chandra Panda, Member (A) :
Three Applicants are aggrieved by the Note (i) in the Column No.12 of the
Recruitment Rules (RR) for the post of Assistant Director/Principal ITIs / Principal Basic
Training / Assistant Apprenticeship Adviser, notified on 05.05.2008 (pages 63-66). The
Applicants state that in July, 2009, Mr. Juwel Kujur (Respondent No.5) was promoted to
the post who otherwise was not having requisite educational qualification for the said
post but for the note added in the RR-2008 he got promotion. Mr. Juwel Kujur s
qualifications are Higher Secondary, ITI Carpenter when in fact the requisite
qualification for the post is Degree in Engineering. Further, in August, 2009 Ms. Shashi
Vasudeva was promoted to the post under the RR-2008 just because of the Note (i)
whereby the essential qualification is totally relaxed to accommodate candidates like her
who otherwise was not educationally qualified for the said post. She is holding a Degree
in Home Science. Being aggrieved by the said two promotions due to the Note (i) in
Column 12 of the RR-2008, the Applicants have approached the Tribunal under Section
19 of Administrative Tribunals Act, 1985 with following prayers :“(I)
Direct the Respondents to delete Note(i) in Column No.12 of
Recruitment Rules notified on 5.5.2008 for the post of Assistant
Director/Principal ITIs/Principal Basic Training/Assistant Apprenticeship
Adviser under the Respondent No.2 Department.
(II)
Direct the respondents to made promotions in accordance with
recruitment rules after the deletion of the note (i) in Column No.12 of
Recruitment Rules notified on 5.5.2008.
(III) Direct the Respondents to cancel the promotion order of
Respondent No.4 and 5 as they were not qualified as per recruitment rules
but for the note (i) in Column No.12 of Recruitment Rules notified on
5.5.2008.
(IV) Direct the respondent to pass any further orders or directions in
favour of the applicant as this Hon ble Tribunal may deem fit and proper
in the present facts and circumstances and in the interest of justice.”
2.
It is the case of the Applicant that earliest Recruitment Rules for the post of ITIs
Pusa, Arab Ki Sarai, Asstt. Apprenticeship Adviser was notified on 01.04.1967
(Annexure-D) which prescribed the essential qualification for the said post for promotion
as
Degree/Diploma
in
Mechanical,
Electrical
Engineering
of
recognized
University/Institution or equivalent. Further, the Recruitment Rules for the said posts
were for the second time notified on 02.11.1971 which envisaged the essential
qualification for the said post for direct recruitment, to be Degree in Engineering and for
promotees it was Diploma/certificate in Engineering or Technology from recognized
University/institution. For the 3rd time the Government issued a Notification dated
27.08.1981 (Annexure-E), where the Recruitment Rules for the said posts prescribed the
minimum qualification for promotion as Degree in Engineering in the subject concerned.
Once again the Recruitment Rules were issued on 26.07.1989
(Annexure-F) for the said
posts providing the essential qualification for the said posts and a note regarding
relaxation of qualification was added in addition to educational qualification existing as
per 1981 Recruitment Rules. In this Recruitment Rule there was no change in essential
qualifications from 1981 Rule, but the Note (i) Column 8 of the Recruitment Rules
provided relaxation to the educational qualification for direct recruitment which reads as
follows :“Note (i) qualifications are relaxable at the discretion of the UPSC in case
of candidate otherwise will qualified.”
In case of promotion, the Note (i) in the Column 12 was provided for the first time in the
said RR-1989, which reads as follows :“The requirement about the educational qualification shall not be
applicable in the case of the Departmental Candidates holding the feeder
posts on regular basis on the date of promulgation of these rules .”
Further, it is stated that the Writ Petition No.5930/2002 was filed by Ms. Sashi Vasudeva
(Respondent No.4) whereby she claimed promotion to the post of ITIs Pusa, Arab Ki
Sarai, Asst. Apprenticeship Adviser, Asst. Director Training, Principal Basic Training
Centre and the Writ was allowed by order dated 05.12.2003. The LPA No.501/2004 was
filed against the said order passed in CWP No.5930/2002 and the Division Bench of the
High Court of Delhi passed an order dated 29.08.2008 in the said LPA and allowed LPA .
As a result, Sashi Vasudeva (Respondent No.4 in the OA) was not considered for
promotion under Recruitment Rules of 1989. In the meantime, Recruitment Rules for the
post of ITIs Pusa, Arab Ki Sarai, Asst. Apprenticeship Adviser, Asst. Director Training,
Principal Basic Training Centre was published in Gazette of India on 05.05.2008 (a copy
of the RR-2008 is at Pages 63-66), where there was no change in essential qualification
from RRs of 1981 and 1989, but the Note (i) in the column 12 of the Recruitment Rules
introduced in 1989 with full relaxation in the educational qualification in case of the
internal candidates continued. Besides, Lady Principal became eligible in the feeder
category for promotion.
3.
Shri Ajit Singh, the learned counsel representing the Applicants, highlighting the
position of Recruitment Rules for the post right from 1967 to 2008, contended that the
Note (i) in Column 12 of the Recruitment Rules , 2008 was inserted by the Respondents
with the purpose and motive to benefit a few, as a result the Respondents 4 and 5 got
promoted. Thus, he terms the same as discriminatory and against the principles of natural
justice and also the same is violative of Article 14 and 16 of the Constitution of India.
Further, he contends that this Note (i) does not allow equality and equal opportunity.
Drawing our attention to the Rule 3.1.3 of Handbook of Recruitment Rules and the
general guidelines issued by the Department of Personnel and Training (DOPT), he
submitted that the Note (i) is in contravention of the said Rule 3.1.3.
He further
submitted that generally for the Scientific and Technical posts like the one being
considered, no relaxation of essential qualification used to be allowed.
He also
contended that the RRs are to be such that the best amongst the available feeder category
employees are to be chosen in the promotion and should not be opened to the unqualified
and undeserving.
Referring to the judgment of Hon ble High Court in the LPA
No.501/2004, he stated that the RR 2008 had been designed to overcome the judgment in
order to allow promotion to the Respondent No.4.
4.
On the other hand, refuting the contentions of Shri Ajit Singh, the learned counsel
for the Respondents Ms. Jyoti singh, submitted that there were 9 posts of Group A
Senior (Principals of ITIs / BTC / Assistant Director / Assistant Apprenticeship Advisor)
vacant and with the creation of one more post for ITI Vivek Vihar, the vacancy position
rose to 10. It was highlighted that similar Note (i) was existing in the RR 1989 and it was
continued in RR-2008. The Lady Principal was included as a feeder category in Column
12 of the RR-2008 which did not exist in the earlier RRs. Such inclusion, it was
contended, was the off shoot of the decision taken by the competent authority while
considering promotion of Respondent No.4. Further, it was submitted that the Applicants
joined between July, 2004 to August, 2006 and would be eligible for consideration for
promotion to such posts in the year 2012 to 2014, as they would fulfill the requisite
qualifying service in the feeder category in those years. In this back drop, it was
submitted that the Applicants are not affected due to Note (i) and also due to promotion
of both private Respondents.
5.
The Respondent No.3, Union Public Service Commission (UPSC) has filed its
reply affidavit on 06.11.2009 through the learned counsel Shri Rajinder Nischal. UPSC
has not added any new points in its reply.
6.
The Applicants filed a detailed rejoinder on 21.01.2010 enclosed copy of the
notings in the file dealing with framing of RR-2008 and the copy of communications
between the UPSC and Respondent Nos.1 and 2.
7.
Having heard the rival contentions, with the assistance of the counsel for the
parties, we perused the pleadings. Two issues come up for our consideration and
adjudication. Those are ; (i) whether the Note (i) in the Column 12 of the Recruitment
Rules dated 5.5.2008 is illegal and discriminatory to be deleted; and (ii) whether
promotion of Respondent No.4 and 5 is legally sustainable or not? Other issues are
consequential to these two main issues.
8.
We may consider the 1st issue now. It is noted that the Recruitment Rules have
been changed for the posts in question many times from 1967 to till date. Two RRs
relevant for this case relates to RR of 1989 and RR of 2008.
The educational
qualifications continue to be the same in both RRs and for both direct recruitment and
promotion. Same ratio of 33 1/3 % and 66 2/3 % between two categories for promotion
also continues. Note (i) in Column 12 of the RR 2008 is same as it was in RR 1989. The
Column 12 and the Note(i) in the Column 12 envisages the following :“Promotion :
1. For 33-1/3% of post : 5 years regular service in the post(s) of
Principal/Senior Surveyors/Training Evaluation Officer in the pay scale of
Rs.8000-13500/- in the Directorate of Training & Technical Education,
GNCT of Delhi and holding degree in Engg/Technology in the subject
concerned or its equivalent.
2. For 66-2/3% of post : 8 years regular service in the post(s) of the
Principals
&
Vice
Principals
ITIs/Assistant
inspector
of
Training/Industrial Liaison Officer-cum-Officer In charge/Training
Officer and Lady Principals of ITIs for Women in the pay scale of
Rs.6500-10500 in the Directorate of Training & Technical Education,
GNCT of Delhi and holding degree in Engg/Technology in the subject
concerned or its equivalent.
Note : (i) The requirement about the educational qualifications shall not be
applicable in case of departmental candidate holding the feeder post on the
date of promulgation of these rules.”
In Para 2 of this order, we have extracted the Note (i) in Column 12 of the RR of 1989. It
is apt to find that Note (i) in both the RRs are word by word same. We, therefore, find
that if the said note was good in 1989, it would continue to be good in 2008 as well. In
the RR 2008 Column 12, there is only one change i.e. addition of “Lady Principals of
ITIs for the Women” in the feeder categories for the promotion in the 66 2/3% of the
posts. This aspect, it is noted, has not been assailed by the Applicants in the present OA.
9.
In our opinion the prayers made by the Applicant to quash the Note (i) in Column
12 requires a careful examination. We can examine to find out whether such an entry in
the RR is unconstitutional and only then we can quash the illegal entry. Our power in this
regard is rather limited, and of course we cannot give directions of a legislative or
executive nature which can only be given by the legislature or executive. As held by the
Honourable Supreme Court in Divisional Manager, Aravali Golf Course & Anr. versus
Chander Hass (JT 2008(3) SC 221), the judiciary cannot encroach into the domain of the
legislature or executive. We are fully conscious of the fact that the seven Judge Bench of
the Apex Court in P. Ramachandra Rao versus State of Karnataka [2002(4) SCC 578] has
also taken the view that "giving directions of a legislative nature is not a legitimate
judicial function"
10.
This being the legal position on the powers of this Tribunal, we find that the RR
2008 has been framed under Article 309 of the Constitution of India and the RR 2008 is
statutory rule coming under the delegated legislative power of the Government. As
discussed within, the Note (i) in Column 12 existed even in the RR of 1989. The Counsel
for the Applicant advanced the argument that the said entry of Note (i) in Column 12 is
discriminatory as selected few in the Government would get benefit. It is noted that the
educational qualification relaxation granted to the Departmental candidates under the
Note (i) in Column 12 is across the board and not for any individual alone. Hence, the
Note (i) in Column 12 in our considered opinion is not discriminatory but provides such
relaxation equally to all Departmental candidates. Another contention was advanced by
the Counsel for the Applicants to say that the promotion posts are technical and as per
DOPT guidelines, educational qualifications being essential, educational qualification
relaxation granted would make the quality suffer and as such violative of the DOPT
guidelines and hence the Note (i) is unconstitutional. We have examined this contention.
It is noted that the DOPT guidelines are generic in nature and modifications are
permissible depending on the facts and circumstances of each case. In RR-2008, the
Official Respondents have taken conscious decision to continue the Note (i) in Column
12 of the RR-1989 in the RR of 2008. We do not consider such a provision in the RR
against the promotion of technically qualified persons for the post.
11.
In view of our above discussion on the 1st issue, we are of the opinion that the
Note (i) in Column 12 of the RR 2008 is vires of the Constitution.
12.
We may take up the 2nd issue namely whether the promotion of the Respondents
No. 4 and 5 is sustainable or illegal. Though the Respondents No. 4 and 5 were served
notice, neither they filed written reply nor their counsel argued the case to defend them.
We do not have the advantage to know their stand. But the pleadings available are
sufficient to adjudicate this issue.
13.
As per the extant Recruitment Rules 2008 for the said post, both private
Respondents are eligible. As the lady Principals of ITIs have been added as one of the
feeder categories Respondent No.4 got entitled for consideration. Such a provision was
not there in the earlier RR 1989. The Note (i) in Column 12 of the RR 2008 provided the
relaxation for the Departmental candidates. The Review / Regular Departmental
Promotion Committee (DPC) held on 12.5.2009 considered the candidates eligible for the
years 2004-05; 2006-07; 2007-08; and 2008-09. The Respondents No. 4 and 5 were
found eligible and fit for promotion by the DPC for the year 2008-09. We note that the
promotions for the year 2004-05; 2006-07; and 2007-08 were based on the RR of 1989
and for the year 2008-09 the RR of 2008 was the basis for promotion. Further,
procedurally, the competent authority has approved the recommendation of the DPC on
the basis of which the private Respondents have been promoted. It is noted that
Respondent No. 4 would attain the age of retirement in the year 2011 by which time none
of the Applicants would have completed even the required 8 years of residency in the
feeder category to be considered for promotion.
14.
Another point related to this issue raised during the hearing by the counsel for the
Respondents relates to the contention that the Applicants being not eligible for
consideration for promotion are not aggrieved to challenge promotion of Respondents
No. 4 and 5. We find there is force in such contentions. The Applicants dd not have
eligibility on the count of not fulfilling the prescribed 8 years residency in the feeder
category. We, therefore, find that neither the Applicants are eligible for promotion nor
they have any locus standi to challenge the promotion of the private Respondents. The
following table would support the point:Applicant No. Applicant's name
1.
2.
3.
15.
Date of joining
feeder category
Mr. Sebastian Augusthy
Mr. Anil Kumar
Mr. Mohammad Hussein
16.3.2005
15. 7.2004
28.6.2006
Completion of 8years
of residency
16.3.2013
15.7.2012
28.6.2014
In view of the above analysis we are of the considered opinion that the Applicants
have not been able convince us as to how the promotion of the Respondents No 4 and 5 is
not legally sustainable and on the other hand the Official Respondents have established
that the promotion of the private Respondents has been as per the extant Recruitment
Rules of 2008 and legally sustainable. Thus, our finding in the 2nd issue goes in favour of
the Respondents.
16.
Considering the facts and circumstances of the case and our findings on the
issues, we reach the conclusion that the Original Application has no merits and hence
dismissed. No costs.
***
IN THE SUPREME COURT OF INDIA
CIVIL APPELLATE JURISDICTION
SPECIAL LEAVE PETITION (C) NO.14889 OF 2009
D.D. 07.07.2010
Hon'ble Mr. Justice Altamas Kabir &
Hon'ble Mr. Justice Cyriac Joseph.
Govt. of India
through Secretary & Anr.
Vs.
Ravi Prakash Gupta & Anr.
..Petitioners
.. Respondents
Reservation:
Implementation of the Persons with Disabilities (Equal Opportunities, Protection, Rights
and Full Participation) Act, 1995. Whether Section 33 which provides for posts for
persons with disabilities is dependent on Section 32 of the Act which provides for
identification of posts in the establishment for appointment? – No.
Respondent No.1 who was at Sl.No.5 of the merit list prepared for visually
handicapped candidates in the Civil Services Examination conducted by U.P.S.C. in 2006
challenged his non-selection and approached Administrative Tribunal that vacancies from
the year 1996 from the date of coming into force of the Act were to be considered he
would be eligible for selection under disabled category – Administrative Tribunal
dismissed the application – 1st Respondent approached Delhi High Court which upheld
his contention and issued a mandamus to the Government by offering an appointment to
one of the reserved posts – In this S.L.P. filed by the Government of India against the
order of Delhi High Court, Supreme Court affirmed the decision of Delhi High Court and
dismissed the S.L.P.
Held:
The submission made on behalf of the Union of India regarding the
implementation of Section 33 of the Act only after identification of posts suitable for
appointment, under Section 32 of the Act runs counter to the legislative intent with which
the Act was enacted.
Further held:
It is only logical that as provided in Section 32 of the Act, posts have to be
identified for reservation for the purposes of Section 33, but such identification was
meant to be simultaneously undertaken with the coming into operation of the Act, to
give effect to Section 33.
Further held:
Reservation under Section 33 of the Act is not dependent on identification of the
posts.
JUDGMENT
ALTAMAS KABIR, J.
1.
The Government of India, through the Secretary, Ministry of Personnel & Public
Grievances, Department of Personnel and Training and through the Secretary, Ministry
of Social Justice and Empowerment, has filed this Special Leave Petition against the
judgment and order dated 25th February, 2009, passed by the Delhi High Court in Writ
Petition (Civil) No.5429 of 2008, allowing the Writ Petition and setting aside the order
dated 7th April, 2008, passed by the Central Administrative Tribunal, Principal
Bench, New Delhi, in O.A. No.1397 of 2007, filed by the Respondent No.1 herein, and
allowing the reliefs prayed for therein.
2.
The Respondent No.1 is a visually handicapped person who suffers from 100%
blindness. He appeared in the Civil Services Examination conducted by the Union Public
Service Commission in the year 2006. After clearing the preliminary examination, the
Respondent No.1 appeared for the main examination in October, 2006 and was declared
successful and was, thereafter, called for a personality test scheduled for 1st May,
2007. Pursuant to such interview, the names of 474 candidates who were selected were
released on 14th May, 2007. In the said list, the name of one other visually impaired
candidate also figured. The Respondent No.1 was at serial no.5 of the merit list prepared
for visually handicapped candidates, who had been declared successful in the
examination. According to the Respondent No.1, although there were more than 5
vacancies available in the visually handicapped category, only one post was offered
under the said category and he was, therefore, not given appointment despite the
vacancies available.
3.
Being aggrieved by the manner in which selections were made for
appointment in the visually handicapped category, the Respondent No.1 filed a Writ
Petition, being Writ Petition (Civil) No.5338 of 2007, before the Delhi High Court. The
same was subsequently withdrawn since it was the Central Administrative Tribunal
only which had jurisdiction to entertain such matters at the first instance. The
Respondent No.1, accordingly, withdrew the Writ Petition, with liberty to approach
the Central Administrative Tribunal. Thereafter, he filed an application under Section 19
of the Administrative Tribunals Act, 1985, which was registered as O.A. No.1397 of
2007, staking his claim for appointment under the reservation of vacancies for disabled
categories provided for under Section 33 of the Persons with Disabilities (Equal
Opportunities, Protection, Rights and Full Participation) Act, 1995, hereinafter referred to
as ‘the Disabilities Act, 1995'. The basic contention of the Respondent No.1 was that
since the aforesaid Act came into force in 1996 providing a statutory mandate for
reservation of 3% of the posts available for persons suffering from different kinds of
disabilities enumerated in Section 33 of the Disabilities Act, 1995, such reservation ought
to have been in force with effect from the date on which the Act came into force.
According to the Respondent No.1, if the vacancies were to be considered from the
year 1996, then instead of one vacancy being declared for the year in question, there
should have been at least 7 vacancies from the reserved categories of disabilities which
were interchangeable. It was, therefore, the case of the Respondent No.1 that having
regard to the number of appointments made with regard to the disabled categories
reserved under Section 33 of the Disabilities Act, 1995, since the Act came into force,
there were at least 7 posts which could be filled up in the year 2006. However, in that
year only one post from this category had been filled.
It was, therefore, the case of the
Respondent No.1 that being at serial no.5 of the list of successful candidates amongst the
physically impaired candidates, there were sufficient number of vacancies in which he
could have been appointed and that the authorities had acted contrary to the provisions of
the above Act upon the faulty reasoning that the vacancies in the reserved posts could not
be declared, without first identifying the same for the purposes of Sections 32 and 33 of
the Disabilities Act, 1995.
4.
The case of the Respondent No.1 having been negated by the Tribunal, the
Respondent No.1 as indicated hereinbefore, moved the High Court and the High Court,
upon accepting the Respondent No.1's case, set aside the order of the Central
Administrative Tribunal dated 7th April, 2008, and allowed the Respondent No.1's O.A.
No.1397 of 2007 filed before the Tribunal. While allowing the said application, the High
Court, upon observing that a clear vacancy was available to which the Respondent No.1
could be accommodated on the basis of his position in the merit list, issued a mandamus
to the Respondent No.1 to offer him an appointment to one of the reserved posts by
issuing an appropriate appointment letter, within six weeks from the date of the order.
Certain consequential orders were also passed together with cost of Rs.25,000/- to be
paid by the petitioner herein.
5.
On behalf of the Government of India, which is the Petitioner herein, learned
Additional Solicitor General, Ms. Indira Jaising, submitted that the submissions
advanced on behalf of the Respondent No.1 which had been accepted by the High Court,
were not tenable and that the Government of India had been actively involved in
complying with the provisions of the Disabilities Act, 1995, after it came into force.
The learned ASG contended that the Government of India had been making reservation
for physically handicapped persons in Group `C' and `D' posts from 1977 and in order to
consider the growing demand from the visually handicapped persons, a meeting for
identification of jobs in various Ministries/Departments was scheduled in 1985 and 416
such posts were identified in Group `A' and `B' posts. In 1986, an Office Memorandum
was issued by the Department of Personnel & Training (DoPT) providing for
preference to be given to handicapped person for these posts.
In 1988, another Office
Memorandum was issued by the Government of India indicating that the identification
done in the year 1986 would remain valid till the same was modified.
After the Act
came into force in 1996, a further Office Memorandum was issued, whereby
reservation of physically handicapped persons in identified Group `A' and Group `B'
posts/services was extended to posts which were to be filled up through direct
recruitment. Learned ASG submitted that in 1999 the Ministry of Social Justice &
Empowerment constituted an Expert Committee to identify/review posts in categories `A,
`B', `C' and `D', in which recommendations were made for identification of posts for the
visually handicapped persons.
The report of the Expert Committee was accepted by the
Ministry in 2001 and posts were duly identified for persons with disabilities.
Learned ASG, however, made it clear that the 416 posts, which had been identified in
1985, did not include All India Services and that for the first time in 2005, the posts of
the Indian Administrative Service were identified in compliance with the provisions
of Section 33 of the Disabilities Act, 1995 and pursuant to such identification, the posts
were reserved and filled up. Ms. Jaising also submitted that reservation upto 3% of
vacancies in the reserved posts were, accordingly, identified with effect from 2006 and
the claim of the Respondent No.1 for appointment on the basis of the argument that the
reservation should have taken effect from 1996 when the Act came into force, was liable
to be rejected.
6.
Appearing in-person, Mr. Ravi Prakash Gupta, the Respondent No.1 herein,
strongly defended the impugned judgment of the High Court and urged that the Special
Leave Petition filed by the Government of India was liable to be dismissed.
Mr. Gupta submitted that the fact that he was completely blind was known to the
Petitioners and their respective authorities from the very beginning, since he had annexed
his blindness certificate with his original application in the proforma provided by the
Union Public Service Commission (U.P.S.C.), which showed the percentage of his
blindness as 100%.
However, the
main
thrust
of
Mr. Gupta's submissions was
that when the Disabilities Act, 1995, came into force in 1996, it was the duty of the
concerned authorities to reserve 3% of the total vacancies available immediately
thereafter. The plea of non-identification of posts prior to the year 2006 was only an
attempt to justify the failure of the Petitioners to act in terms of the Disabilities Act, 1995.
Mr. Gupta submitted that the High Court had negated such contention made on behalf of
the Petitioners and rightly directed the Petitioners to calculate the number of vacancies in
terms of Section 33 of the above Act from 1996 when the said Act came into force.
7.
Mr. Gupta then submitted that in terms of the Department's OM No.3635/3/2004
dated 29th December, 2005, reservations have been earmarked and should have been
made available from 1996 itself and in the event
filled
the
vacancies
could
not
be
up owing to lack of candidates, the same could have been carried forward for two
years after which the same could have been treated as lapsed. Mr. Gupta submitted that
although the Petitioners were fully aware of the said Office Memorandum, they chose not
to act on the basis thereof and as admitted on behalf of the Government of India, the IAS
cadre was identified in 2006 for the purposes of Section 33 of the Disabilities Act, 1995.
In fact, the Act remained on paper as far as visually challenged candidates were
concerned and only after the judgments of the Delhi High Court in the case of Ravi
Kumar Arora and in the case of T.D. Dinakar were delivered, that the identification
process was started. Mr. Gupta submitted that it would be pertinent to mention that the
two above-mentioned candidates were appointed in the Civil Services without waiting
for identification of their respective services on the orders of the High Court.
8.
Mr. Gupta submitted that the plea of non-identification of posts in the IAS till
the year2006 could not absolve the petitioners of their statutory obligation to provide for
reservation in terms of Section 33 of the aforesaid Act.
9.
During the course of hearing, leave had been granted to one A.V. Prema Nath and
one Mr. Rajesh Singh to intervene in the proceedings. The submissions made by the
Respondent No.1 have been repeated and reiterated on behalf of the Intervenor No.1, Shri
A.V. Prema Nath by A. Sumathi, learned Advocate. His written submissions are
embellished with references to various decisions of this Court, including the decision in
Francis Coralie Mullin vs. Administrator, Union Territory of Delhi & Ors. [(1981) 1 SCC
608], regarding the right to life under Article 21 of the Constitution. The main thrust of
the submissions is with regard to the denial of rights to persons with disabilities under
Section 33 of the Disabilities Act, 1995, which prevent them from enjoying their
fundamental rights to equality and the right to live, by the State.
10.
More detailed submissions were made by Mr. S.K. Rungta, learned Advocate,
appearing on behalf of the Intervenor No.2, Mr. Rajesh Singh, and it was also sought to
be pointed out that the said intervenor was himself a candidate from amongst the visually
impaired candidates and had, in fact, been placed at serial no.3 in rank in the merit list for
visually impaired candidates in the Central Services Examinations, 2006, whereas the
Respondent No.1 had been placed at serial no.5. In other words, what was sought to be
projected was that Shri Rajesh Singh had a better claim for appointment from amongst
the visually impaired candidates over the Respondent No.1 and that if the vacancies in
the reserved category were to be calculated from 1996 and even from 2001, when
identification of posts in respect of Civil Services forming part of the IAS Cadre was
sought to be effected and a notification to that effect was issued, the Respondent No.1
could not have been appointed.
11.
It was further submitted that in the decision of this Court in The National
Federation of Blind vs. Union Public Service Commission & Ors. [(1993) 2 SCC 411],
the demand by blind candidates for being permitted to write the examination in Braille
script, or with the help of a Scribe, for posts in the IAS was duly accepted for recruitment
to the lowest posts in the service reserved for such persons.
It was also held that blind
and partially blind persons were eligible for appointment in Government posts. It was
submitted that the submissions made on behalf of the Petitioners that the
notification
in respect of the services in respect of the Group `A' and `B' services in the IAS in 2005
was not a fresh exercise, but only an attempt to consolidate and strengthen the
identification already available and that such an exercise could at best be said to be
enabling and supplementary action for the smooth implementation of the statutory
provisions containing the scheme of reservation for persons with disabilities, could not be
taken as an excuse to postpone the benefit which had already accrued to candidates
falling within 3% of the vacancies indicated in Section 33 of the Disabilities Act, 1995.
It was also urged that after the issuance of OM dated 29th December, 2005 and OM dated
26th April, 2006, there was hardly any room for the Government of India to deny the
benefit of reservation to persons with disabilities, including the blind, in Civil Services
encompassing the IAS from the year 1996 itself. Furthermore, since the Act itself did not
make any distinction between Group `A' and Group `B' services and Group `C' and
Group `D' services, it was not available to the Government of India to contend that since
identification had been done only for Group `C' and Group `D' services, prior to the year
2005, reservation in respect of Group `A' and `B' services, which include the IAS, for
which identification was commenced in 2005, would only be available thereafter.
12.
On behalf of the Intervenor No.2, it was submitted that the Special Leave
Petition
was liable to be dismissed with exemplary costs.
13.
We have examined the matter with great care having regard to the nature of the
issues involved in relation to the intention of the legislature to provide for integration
of persons with disabilities into the social main stream and to lay down a strategy for
comprehensive development and programmes and services and equalization of
opportunities for persons with disabilities and for their education, training,
employment and rehabilitation amongst other responsibilities. We have considered the
matter from the said angle to ensure that the object of the Disabilities Act, 1995, which is
to give effect to the proclamation on the full participation and equality of the people with
disabilities in the Asian and Pacific Region, is fulfilled.
14.
That the Respondent No.1 is eligible for appointment in the Civil Services after
having been declared successful and having been placed at serial no.5 in the disabled
category of visually impaired candidates, cannot be denied.
The only question which
is relevant for our purpose is whether on account of the failure of the Petitioners to
identify posts for persons falling within the ambit of Section 33 of the Disabilities Act,
1995, the Respondent No.1 should be deprived of the benefit of his selection purportedly
on the ground that there were no available vacancies in the said category.
The other
question which is connected with the first question and which also requires our
consideration is whether the reservation provided for in Section 33 of the Disabilities
Act, 1995, was dependent on identification of posts suitable for appointment in such
categories, as has been sought to be contended on behalf of the Government of India in
the instant case.
15.
Although, the Delhi High Court has dealt with the aforesaid questions, we wish
to add a few observations of our own in regard to the objects which the legislature
intended to achieve by enacting the aforesaid Act. The submission made on behalf of
the Union of India regarding the implementation of the provisions of Section 33 of the
Disabilities Act, 1995, only after identification of posts suitable for such appointment,
under Section 32 thereof, runs counter to the legislative intent with which the Act was
enacted. To accept such a submission would amount to accepting a situation where the
provisions of Section 33 of the aforesaid Act could be kept deferred indefinitely by
bureaucratic inaction. Such a stand taken by the petitioners before the High Court was
rightly rejected. Accordingly, the submission made on behalf of the Union of India that
identification of Grade `A' and `B' posts in the I.A.S. was undertaken after the year 2005
is not of much substance. As has been pointed out by the High Court, neither Section 32
nor Section 33 of the aforesaid Act makes any distinction with regard to Grade `A', `B',
`C' and `D' posts.
They only speak of identification and reservation of posts for
people with disabilities, though the proviso to Section 33 does empower the appropriate
Government to exempt any establishment from the provisions of the said Section, having
regard to the type of work carried on in any department or establishment. No such
exemption has been pleaded or brought to our notice on behalf of the petitioners.
16.
It is only logical that, as provided in Section 32 of the aforesaid Act, posts have
to be identified for reservation for the purposes of Section 33, but such identification was
meant to be simultaneously undertaken with the coming into operation of the Act, to
give effect to the provisions of Section 33. The legislature never intended the provisions
of Section 32 of the Act to be used as a tool to deny the benefits of Section 33 to these
categories of disabled persons indicated therein.
Such a submission strikes at the
foundation of the provisions relating to the duty cast upon the appropriate Government to
make appointments in every establishment (emphasis added). For the sake of reference,
Sections 32 and 33 of the Disabilities Act, 1995, are reproduced herein below:
"32. Identification of posts which can be reserved for persons with
disabilities.- Appropriate Governments shall (a) Identify posts, in the
establishments, which can be reserved for the persons with disability; (b)
At periodical intervals not exceeding three years, review the list of posts
identified and up-date the list taking into consideration the developments
in technology.
33.Reservation of posts.- Every appropriate Government shall
appoint in every establishment such percentage of vacancies not less
than three per cent for persons or class of persons with disability of
which one per cent each shall be reserved for persons suffering from(i) blindness or low vision;
(ii) hearing impairment;
(iii) locomotor disability or cerebral palsy, in the posts
identified for each disability: Provided, that the appropriate
Government may, having regard to the type of work carried on in
any department or establishment by notification subject to such
conditions, if any, as may be specified in such notification, exempt
any establishment from the provisions of this section."
17.
While it cannot be denied that unless posts are identified for the purposes of
Section 33 of the aforesaid Act, no appointments from the reserved categories contained
therein can be made, and that to such extent the provisions of Section 33 are dependent
on Section 32 of the Act, as submitted by the learned ASG, but the extent of such
dependence would be for the purpose of making appointments and not for the purpose
of making reservation.
In other words, reservation under Section 33 of the Act is not
dependent on identification, as urged on behalf of the Union of India, though a duty
has been cast upon the appropriate Government to make appointments in the number of
posts reserved for the three categories mentioned in Section 33 of the Act in respect of
persons suffering from the disabilities spelt out therein.
In fact, a situation has also
been noticed where on account of non-availability of candidates some of the reserved
posts could remain vacant in a given year.
For meeting such eventualities, provision
was made to carry forward such vacancies for two years after which they would lapse.
Since in the instant case such a situation did not arise and posts were not reserved under
Section 33 of the Disabilities Act, 1995, the question of carrying forward of vacancies
or lapse thereof, does not arise.
18.
The various decisions cited by A. Sumathi, learned Advocate for the first
intervenor, Shri A.V. Prema Nath, are not of assistance in the facts of this case, which
depends on its own facts and interpretation of Sections 32 and 33 of the Disabilities Act,
1995.
19.
We, therefore, see no reason to interfere with the judgment of the High Court
impugned in the Special Leave Petition which is, accordingly, dismissed with costs.
All interim orders are vacated.
The petitioners are given eight weeks' time from today
to give effect to the directions of the High Court.
20.
The petitioners shall pay the cost of these proceedings to the respondent No.1
assessed at Rs.20,000/-, within four weeks from date.
***
CENTRAL ADMINISTRATIVE TRIBUNAL, ALLAHABAD BENCH, ALLAHABAD
Original Application No.653/2010
D.D. 21.12.2010
Hon’ble Mr. S.N.Shukla, Member-A &
Hon’ble Mr.Sanjeev Kaushik, Member-J
Prem Chand Yadav
Vs.
Union of India & Anr.
…
Applicant
…
Respondents
Examination:
Whether a candidate can be debarred from appearing in Civil Services Examination for
furnishing wrong/misleading information in the application form? – Yes.
Shri Prem Chand Yadav was a candidate for the Civil Services (Prelim)
Examination, 2007 (Roll No.42475) - In his Detailed Application Form (DAF) for CS(M)
Exam.2007 he had suppressed the information of previous appearances in Civil Services
Examinations - In his DAF for Civil Services (Main) Exam 2007 he stated that number of
chances availed by him as 7 including this one - On scrutiny it was found that he has
already availed more chances than he is eligible for, thus furnishing wrong/misleading
information in his application form - He was issued show-cause notice on 26.2.2009 - In
his reply he admitted the allegations leveled in the applicant in the examinations to be
held in 2007 and also passed an order debarring him for 10 years w.e.f. 24.3.2008 from
all examinations to be conducted by the Commission as per Rules for the Civil Services
Examination 2007.
It was argued by the Ld. Counsel for the petitioner that there was no malafide
intention and the information furnished as an inadvertent error - Not convinced with the
explanation submitted by the petitioner, Hon’ble CAT upheld the Commission’s order
and dismissed the O.A.
Held:
The candidates who are going to appear in the Civil Services Examination are
expected to read the instructions carefully and to fill up application form carefully and
meticulously. In the instant case the applicant has already taken seven attempts. He
knows it well. For the reasons best known to him persuaded himself to take a chance to
bluff in a hope of not being caught. The luck conspired otherwise. We have found no
reason to interfere with the impugned order. O.A. stands dismissed.
Hon’ble Sanjeev Kaushik, Member-J:
1.
The applicant has filed the instant Original Application under Section 19 of the
Administrative Tribunal Act, 1985 challenging the order dated 28th March, 2008 passed
by Union Public Service Commission, New Delhi (hereinafter referred as UPSC)
(Annexure A-1).
The brief facts are that the applicant belongs to backward class
category submitted an application to the UPSC for Civil Services (Preliminary)
Examination 2007.
AT the time of verification of the application by the UPSC it
transpired that the applicant has not furnished correct information as asked in the
application form.
Accordingly, Respondent No.2 issued a Show Cause Notice dated
26.02.2008 incompliance of Rule 4 and 14 of the Civil Services (Main) Examination
2007, was granted 15 days time to submit reply. In compliance of the above stated notice
the applicant submitted his reply on 06.03.2008 wherein he has admitted the allegations
leveled in the show cause notice. After considering the reply the Respondent No.2
passed the impugned order dated 28.03.2008 (Annexure A-1). By impugned order the
Respondent No.2 cancelled the candidature of the applicant in the examination to be held
in 2007 and also passed an order debarring him 10 years w.e.f. 24.03.2008 from all
examination to be conducted by the Respondent No.2.
2.
We have heard the Ld. Counsel for the respective parties and with their able
assistance perused the paper books as well as the rules.
3.
The Ld. Counsel for the applicant Shri A.D.Singh has argued that the impugned
order is illegal as there was no malafide intention on the part of the applicant to mislead
the Respondent while furnishing the information in the application form. Further, that
the information submitted in the application form is an inadvertent error. Therefore,
penalty imposed by the respondents is disproportionately harsh.
4.
On the other hand the Ld. Counsel for the respondents Sri. K.C.Sinha argued
that the applicant knowingly furnished false information in the application form. He
further submitted that in the application form at column No.25(a) and (b) it is
categorically asked from a candidate the number of chances availed by him.
The
applicant disclosed seven chances without disclosing that he has already appeared in the
examination in the year 1999 and 2000 as well. While disclosing a limited information
that he appeared 2001 onwards. He therefore, knowingly mislead the respondents by
submitting false information of availing only seven chances including the present one
instead of the correct position of having utilized seven attempts before the current
attempt. The motivation for submitting false information will be clear from the reading
of rules reproduced below.
5.
We have perused the Rule for the Civil Services (Main) Examination, 2007,
notified on 16th December 2006 by Ministry of Personnel, Public Grievance and Pension
specifically Rule 4 and 14 which reads as under:
Rule 4:“Rule 4:Every candidate appearing at the examination who is
otherwise eligible, shall be permitted four attempts at the examination:
Provided that this restriction on the number of attempts will not apply in
the case of Scheduled Castes and Scheduled Tribes candidates who are
otherwise eligible.
Provided further that the number of attempts permissible to candidates
belonging to Other Backward Classes, who are otherwise eligible, shall be
seven. The relaxation will be available to the candidates who are eligible
to avail of reservation applicable to such candidates.
Note:(i)
An attempt at a Preliminary Examination shall be deemed to be an
attempt at the Examination.
(ii) If a candidate actually appears in any one paper in the Preliminary
Examination, he/she shall be deemed to have made an attempt at the
Examination.
(iii) Notwithstanding the disqualification/cancellation of candidature, the
fact of appearance.
Rule 14:“14.
A candidate who is or has been declared by the Commission to
be guilty of:-
(i)
Obtaining support for his candidature by the following means, namely:-
(a) offering illegal gratification to; or
(b) applying pressure on, or
(c) blackmailing or threatening to blackmail any person connected with the
conduct of the examination, or
(ii) impersonation; or
(iii) procuring impersonation by any person; or
(iv) submitting fabricated documents or documents which have been
tampered with; or
(v) making statements which are incorrect or false or suppressing material
information; or
(vi) resorting to the following means in connection with his candidature for
the examination namely:(a) obtaining copy of question papers through improper means;
(b) finding out the particulars of the persons connected with secret works
relating to the examination;
(c) influencing the examiners; or
(vii) using unfair means during the examination; or
(viii) writing obscene matter or drawing obscene sketches in the scripts; or
(ix) misbehaving in the examination hall including tearing of the scripts,
provoking fellow examinees to boycott examination, creating a
disorderly seen and the like; or
(x) harassing or doing bodily harm to the staff employed by the commission
for the conduct of their examination; or
(xi) being in possession of or using any mobile phone pager or any
electronic equipment or device or any other equipment capable of
being used as a communication device during the examination; or
(xii) violating any of the instructions issued to candidates along with their
admission certificates permitting them the examination; or
(xiii) attempting to commit or, as the case may be, abetting the commission of
all or any of the acts specified in the foregoing clauses;
may in addition to rendering himself liable to criminal prosecution, be
liable:(a) to be qualified by the Commission from the Examination for which he is a
candidate; and/or
(b) to be debarred either permanently or for a specified period:(i)
(ii)
by the Commission from any examination or selection held by them;
by the Central Government from any employment under them; and
(c) if he is already in service under Government to disciplinary action under
the appropriate rules:
provided that no penalty under this rule shall be imposed except after:(i)
(ii)
giving the candidate an opportunity of making such representation in
writing as he may wish to make in that behalf; and
taking the representation, if any, submitted by the candidate within the
period allowed to him into consideration.”
From the bare perusal of the above stated rule it is crystal clear that a candidates
belonging to Other Backward Classes, who are otherwise eligible, shall be entitled to
avail seven chances. Since, the applicant has already availed seven chances excluding
the present attempt, therefore, no infirmity can be found against the impugned order.
Rule 14 provides for punishment to be inflicted upon the candidates who have been
declared by the Commission to be guilty, rule 14 (xiii) (b) provides for either
permanently debarring the candidate from appearing in the examination conducted by the
U.P.S.C. or for a specified period.
6.
In view of the above stated facts, we are of the considered view that the
explanation submitted by the applicant is not convincing. The candidates who are going
to appear in the Civil Services Examination are expected to read the instructions carefully
and to fill up application form carefully and meticulously.
In the instant case the
applicant has already taken seven attempts. He knows it well. For the reasons best
known to him persuaded himself to take a chance to bluff in a hope of not being caught.
The luck conspired otherwise. We have found no reason to interfere with the impugned
order. O.A. stands dismissed.
***
CENTRAL ADMINISTRATIVE TRIBUNAL: PRINCIPAL BENCH
O.A. 115/2011
D.D. 19.01.2011
Hon’ble Mr. L.K.Joshi, Vice Chairman (A) &
Hon’ble Mrs. Meera Chhibber, Member (J)
Dr. Kanika Chandra …
Vs.
U.P.S.C.
…
Applicant
Respondent
Examination:
Whether an application for the post is liable to be rejected for non compliance with
instructions contained in the advertisement? – Yes.
Applicant was a candidate for Combined Medical Services Examination, 2011 –
Her application was rejected because of photo/signature not uploaded as per
specifications – Applicant contended she should have been given opportunity to carry out
correction – Rejecting her contention the Tribunal dismissed the application.
Held:
In the advertisement it was made clear that if the application is incomplete it
would not be entertained and therefore it is duty of the candidates who applied to ensure
that they apply in proper format and given the complete information. Since applications
of other candidates were rejected for similar defects if the application of the applicant is
allowed it would itself amount to discrimination because applications of other similarly
situated persons have been rejected. Simply because the applicant has approached the
Court she cannot have advantage over others.
ORDER
By Hon’ble Mrs. Meera Chhibber, Member (J)
Applicant has challenged order dated 20.12.2010 whereby she has been informed
that her application for Combined Medical Services Examination, 2011 has been rejected
because Photo/Signature not uploaded as per specifications (page 8).
2.
It is submitted by the applicant she is a medical graduate having MBBS degree
and has appeared in the post graduate (D.N.B.) examination held in December, 2010 after
completing the course at Sr. Ganga Ram Hospital.
3.
Respondents had advertised in the Employment News dated 11.9.2010 and had
also put an advertisement on its website inviting applications for the Combined Medical
Services Examination, 2011, to be held on 16.1.2011 all over the country from medical
graduates.
The applicant had applied online vide application form CMS/Form
No.1110008876/024698/13092010/031534/EX.III by uploading her photo and signatures
on 13.9.2010 for the Combined Medical Services Examination, 2011. However, her
application has been rejected on the ground that photo/signatures not uploaded as per
specifications.
4.
It is submitted by the counsel for the applicant that applicant should have been
given an opportunity to carry out correction in the application form if something was left
out instead of rejecting her application and further that her application could not have
been rejected at the last moment, prejudicing applicant s career and professional life.
Since respondents have stated that they would not entertain any other application, she had
no other option but to file the present OA. She has sought a direction to the respondents
to allow her to carry out the correction in the application form and allow her to take the
examination.
5.
We had given dasti notice to Ms. Jasmine Ahmed, Counsel for the respondents
with a direction to show us the application form submitted by the applicant. Today, when
the matter was called out, counsel for the respondents produced the application form
given by the applicant online which shows that applicant s photo was not even visible
and there were no signatures at all in the application form meaning thereby the
application was incomplete. It could not be termed as a complete application as it was not
even signed by the applicant. It is submitted by the counsel for the applicant that she
could be given an opportunity to correct the application. We are afraid, we cannot accept
the said contention because UPSC receives applications in thousands and thousands. In
the advertisement it was made clear that if application is incomplete, it would not be
entertained, therefore, it is the duty of the candidates who apply to ensure that they apply
in the proper format and give the complete application form. Counsel for the respondents
informed us that there are applications of number of other candidates which are
incomplete and they have rejected all such applications.
6.
If we allow the applicant to correct her application form, it would itself amount to
discrimination because applications of all other similarly situated persons have been
rejected. Simply because the applicant has approached the court, she cannot have
advantage over others. It was open to the applicant to apply offline in order to ensure that
the application form is given properly. Since the application form has not even been
signed by the applicant, the relief, as claimed by the applicant cannot be granted. The
OA is accordingly rejected at the admission stage itself. No costs.
***
CENTRAL ADMINISTRATIVE TRIBUNAL
PRINCIPAL BENCH, NEW DELHI
OA No. 1947/2010
D.D. 15.03.2011
Hon’ble Mr. L.K. Joshi, Vice Chairman (A)
Hon’ble Mrs. Meera Chhibber, Member (J)
Umesh Chandra
Vs.
Govt.of NCT of Delhi & Ors.
…
Applicant
…
Respondents
Recruitment:
Whether Government servants can challenge the authority of State to make amendments
or alterations in the Rules? – No.
As per 1985 Recruitment Rules, a candidate having degree in Electrical or
Mechanical Engineering was eligible for promotion to the post of Executive Engineer in
Delhi Jal Board which is governed by Delhi Water Board Act – Under the said Act UPSC
is empowered to make the regulations and in case of difference of opinion between Jal
Board and UPSC the matter has to be referred to Central Government – As per Amended
Recruitment Rules of 2009 eligibility condition for promotion from Assistant Engineer to
Executive Engineer was fixed at 8 years for degree holders and 6 years for diploma
holders by Delhi Jal Board as per recommendations of UPSC without taking concurrence
from the Government of NCT of Delhi – Applicant challenged the amendment Rules
mainly on the ground that the rules could not have been amended without taking
concurrence of the Government – Rejecting the contention of the applicant that degree
holder AEs have been degraded the Tribunal dismissed O.A. being devoid of merits.
Held:
It is not within the domain of courts to decide how Recruitment Rules will be
framed. It is the job of the Department or UPSC to decide how best Recruitment Rules
can serve the purpose of the department.
Further held:
Government servants have only right to safeguarding rights or benefits already
earned, acquired or accrued but they cannot challenge the authority of State to make such
amendments or alterations in the rules.
ORDER
Hon’ble Mrs. Meera Chhibber, Member (J)
Applicant has challenged Notification dated 21.12.2009 whereby Delhi Jal Board
Recruitment Regulation, 2009 for the post of Executive Engineer (Electrical &
Mechanical) have been notified.
2.
It is submitted by the applicant he joined Delhi Jal Board in 1983 as Junior
Engineer and was promoted as Assistant Engineer (Electrical & Mechanical) (hereinafter
referred to as E&M) in 2001 on current duty charge. He had completed 4 years degree
course in the Electrical Stream in 1994.
3.
As per 1985 Recruitment Rules, a candidate having degree in Electrical or
Mechanical was eligible for promotion to the post of Executive Engineer. However,
these rules have been amended in 2009 by the respondent No.2, i.e., Delhi Jal Board only
on the directions of UPSC without taking concurrence from Government of NCT of
Delhi even though Department of Urban Development is the administrative Ministry of
Delhi Jal Board, therefore, its approval was necessary.
4.
Being aggrieved, applicant had given representation at the stage of proposed
regulations itself but neither his representation was considered nor any reply was given to
the applicant before amending the rules, thus principles of natural justice have been
violated.
5.
It is submitted by the learned counsel for the applicant that in the meeting held on
7.10.2009 between Delhi Jal Board and Government of NCT of Delhi, Delhi Jal Board
was advised to recast the proposed RRs by incorporating ratio of 80%:20% between
Degree Holders and Diploma Holders and to effect certain changes in order to bring
uniformity amongst different agencies of the Government of Delhi. The changes were
sent also to the UPSC for seeking their concurrence. However, UPSC advised DJB to
first notify the approved RRs and then seek modification, if necessary. It was on the basis
of this advice that the RRs have been notified in 2009 without taking into account the
RRs of other agencies viz. CPWD and NDMC where there is definite quota rule between
Degree holders and Diploma holders for promotion to the post of Ex. Engineer (E&M).
He has thus prayed that the O.A. may be allowed.
6.
M.A.No.1586/2010 was filed by Diploma holders Asstt. Engineers for being
impleaded as party. They have stated that one Shri Subhash Chand, Diploma holder, A.E.
had filed O.A. No.1692/2009 seeking the following relief :“To direct the Respondent No.1 and 2 to constitute and notify after
seeking clearance from concerned authorities including UPSC and
Hon ble Lt. Governor of Delhi immediately the RR s for the post of E.E
(E&M) in such a fashion thereby according equal opportunities to A.E
(E&M) diploma holders as well as A.E (E&M) degree holders with
varying experience so as to accord promotional avenues to the applicant
and other employees who are diploma holders.”
7.
The said O.A. was disposed of with following directions :“Mr. R.R. Dua, counsel representing the applicant, at this stage confines
the relief, out of reliefs enumerated in the relevant paragraph, only with
regard to direction to be issued to the respondents to finalize amendment
in the rules, process of which has already started in June, 2007. It would
appear from the records of the case, as per averments made in the Original
Application, that the respondents are indeed considering amendment to the
rules. One of the proposals in amending the rules is to give some
percentage i.e. 33% promotion to the applicant and other similarly situate
persons holding the post of Assistant Engineer to the post of Executive
Engineer, and in the meanwhile, other persons, who are holding degree as
compared to the applicant holding diploma have been promoted. It is
urged that delay in finalizing the rules would be prejudicial to the
applicant inasmuch as, if the rules are amended to give some percentage in
promotion to the employees as the applicant, who are diploma holders,
many posts may not be left out for promotion to them.
2.
Considering the totality of facts and circumstances of the case, we
direct the Chief Executive Officer, Delhi Jal Board, Varunalya, Phase-II,
Karol Bagh, New Delhi, second respondent herein, to complete the
exercise of amending the rules in the manner deemed just and proper, as
expeditiously as possible and preferably within a period of three months
from the date of receipt of certified copy of this order. We are conscious
that without calling upon the respondents we would not have passed the
above directions, but in view of the directions given above, there would be
perhaps no need to have their reply as surely, the amendment in the rules
is in contemplation by the respondents and, as mentioned above, the
process of which has already started in June, 2007. However, since we
have passed the order aforesaid without calling upon the respondents to
show cause, they are at liberty to approach this Tribunal, if they so desire.”
8.
Since directions were not complied with, Shri Subhash Chand had filed CP
No.525/2009. It was during the pendency of the CP that the RRs were finally notified on
21.12.2009, as such CP was dropped on 23.12.2009.
9.
They have further stated that they are eligible as per the amended rules to be
considered whereas applicants in O.A. are not even eligible because they have been
regularised as A.E (E&M) only on 02.03.2009, therefore, they are interested parties.
Their interests would be adversely affected in case the OA is allowed. Moreover, they are
on the verge of retirement, therefore, their request for impleadment may be allowed.
Accordingly, MA 1586/2010 filed for impleadment was allowed.
10.
UPSC has clarified that one of the mandated functions of the Union Public
Service Commission (hereinafter referred to as “Commission”) is to be consulted in the
framing of the recruitment rules for various civil posts/services under the Central
Government/Union Territories as well as certain local bodies and organizations
(including Delhi Jal Board (hereinafter referred to as “DJB”) by virtue of power vested in
it under Article 320 and 321 of the Constitution of India. In terms of Section 54 (a) & (b)
of the Delhi Water Board Act, 1998, the DJB is required to frame the Recruitment Rules
for the posts of Executive Engineer (Electrical & Mechanical) under the Board in
consultation with the Commission. Section 54(2) of the said Act further provides that in
case of any difference of opinion between the Commission and the DJB on any matter,
the DJB shall refer the matter to the Central Government and the decision of the Central
Government thereon shall be final.
11.
The provisions in the recruitment rules are prescribed by the Commission as per
the requirements specified by the user Organisation and in conformity with the executive
instructions and guidelines/norms laid down by the Department of Personnel and
Training, Government of India. The Commission’s role is advisory in this respect as it
scrutinizes proposals received from Ministry/Departments and tenders its advice keeping
in view the extant guidelines.
12.
They have explained that on 30.01.2008, a proposal for amending the recruitment
rules for the post of Executive Engineer (Electrical & Mechanical) in the pre-revised
scale of pay of Rs.1000-15200/- was received by the respondent Commission from the
DJB. The DJB proposed to adopt the method of recruitment in vogue in their Civil
Engineering Department where Assistant Engineer (Civil) with 8 years of regular service
and possessing a Degree in Civil Engineering or with ten years of regular service and
holding a Diploma in Civil Engineering (Civil) were eligible for promotion to the post of
Executive Engineer (Civil). The recruitment rules of Executive Engineer (E&M) in force
in DJB at that time allowed promotion of only those Assistant Engineers who possessed a
Degree in Electrical or Mechanical Engineering.
13.
After examining the proposal made by DJB, the respondent Commission
concurred the proposal vide letter dated 09.05.2008 and Delhi Jal Board was requested to
notify the recruitment rules as approved by the Commission within 10 weeks as required
under the Ministry of Home Affairs (Department of
Personnel & Administrative
Reforms) OM No. AB-14017/12/87-Estt (RR) dated 18.3.1988.
However, without
notifying and giving effect to the recruitment rules approved by the Commission, the DJB
again approached the Commission vide their letter dated 5th March, 2009 with a request
to modify the approved version of the Recruitment Rules by fixing a quota of 66.67% for
promotion of Assistant Engineer (E&M) with 7 years of regular service and having a
Degree in Electrical/Mechanical Engineering and 33.33% for Assistant Engineer (E&M)
with 9 years of regular service and having a Diploma in Electrical/Mechanical
Engineering in view of the representations received from large number of Assistant
Engineers (E&M) conveying their grievance in respect to the amendments approved by
the Commission in May, 2008. No other reason, apart from the representations made by
the Assistant Engineers (E&M), was cited by the DJB while proposing further
modification in the Recruitment Rules for the post of Executive Engineer (E&M).
14.
While the proposal made by DJB again was being examined by the respondent
Commission, the DJB brought to its notice, vide their letter dated 20th July, 2009, the
following directions contained in the judgment order 1.7.209 of the Hon’ble CAT, PB,
New Delhi in OA No.1962/2009:“considering the totality of the facts and circumstances of the case,
we direct the Chief Executive Officer, Delhi Jal Bioard, Varunalaya
Phase-II, Karol Bagh, New Delhi, Second respondent herein to complete
the exercise of amending the recruitment rules in the manner deemed just
and proper as expeditiously as possible and preferably within a period of 3
months from the date of receipt of the certified copy of this order”.
15.
In view of above, the Commission requested the DJB to notify the approved rules
first and thereafter approach the Commission for further compliance with the Hon ble
CAT s directions to complete the process of amending the RRs of Executive Engineer
(E&M) within 3 months was an overwhelming priority for the DJB, it is preposterous on
the part of the applicant to say that the Notification dated 21.12.2009 was issued by the
DJB at the behest of the respondent Commission. They have thus prayed that the OA
may be dismissed.
16.
Delhi Jal Board in their counter affidavit have stated that respondents have right
to amend the service conditions as well as the RRs for better administration of the
organization and same cannot be challenged simply on the ground that it does not suit
some persons or it affects their promotional avenues.
17.
Moreover, similar OA wherein Notification dated 21.12.2009 was challenged and
was relied upon by the applicant herein has been dismissed as withdrawn on 13.8.2010,
therefore, this OA may also be dismissed.
18.
They have also explained that the expeditious amendment was necessitated in
view of the direction of this Hon’ble Tribunal vide order dated 1.7.2009 in OA
No.1692/2009 to complete the exercise of amending the rules within 3 months. Seniority
and promotion is strictly and mandatorily governed by the DOP&T guidelines. Earlier
the RRS was based on the 3rd Central Pay Commission which necessitated amendment
after the implementation of the 5th Central Pay Commission in view of DOP&T’s OM
dated 25.5.1998 which mandates that all the ministries/departments to take steps to
amend the existing Service Rules/Recruitment Rules to prescribe eligibility conditions
with reference to the revised pay scale. Accordingly the RRs was amended and the
eligibility condition for promotion from Assistant Engineer in the pay scale of Rs.650015200 to Executive Engineer in the pay scale of Rs.10,000-15200 was fixed at 8 years for
Degree Holders while 10 years for Diploma Holders. It is not binding to fix the quota
amongst the Diploma Holder and Degree Holder when both are discharging similar duties
and responsibilities. In any case, the ratio as advised by the respondent No.1 was
forwarded to the Respondent No.3/Commission who desired that the Delhi Jal Board
should first notify the RRs which are already approved. Therefore, in view of the
insistence of the Respondent No.3/UPSC to notify the RRs already approved coupled
with the fact that this Hon’ble Tribunal vide order dated 1.7.2009 in OA No. 1692/2009
had also directed to complete the exercise of amending the rules within 3 months, the
RRs for the post of Executive Engineer (E&M) was notified by the Department of Urban
Development, GNCTD in its present form. Answering respondent is fully empowered to
frame its Regulation under clause (m) of the Section 109 read with Sections 7 and 51 of
Delhi Water Board Act, 1998 (Delhi Act No.4 of 1998), being an autonomous body and
respondent No.3 is also fully empowered under Section 53-54 of the Delhi Water Board
Act, 1998 for conveying the concurrence to the answering respondent. There is no
provision to get the prior approval of the respondent No.1 before framing/amending its
regulation in terms of Delhi Water Board Act, 1998. Respondent No.1 is only nodal
agency to notify the RRs as approved by the Delhi Water Bard and concurred by the
UPSC.
19.
They have further stated that the representation of the applicant was examined and
due weightage has been given to the Degree holder Assistant Engineer (E&M) by
keeping their eligibility service at 8 years for promotion in comparison of 10 years for the
Diploma Holders. They have further submitted that the Recruitment Rules are statutory in
nature and are framed according to the requirement of job profile. There is no rule/policy
to follow the practice of other departments. Different Government departments frame
their RRs as per their own requirements.
20.
They have also explained that the applicant is very junior to the Diploma Holder
Assistant Engineer (E&M). He has been promoted as Assistant Engineer in the year 2009
only whereas the Diploma holders are working as Assistant Engineer (E&M) on regular
basis since 1989 onwards. They have thus prayed that the O.A. may be dismissed and
interim order may be vacated.
21.
We have heard all the parties and perused the pleadings also. On 11.6.2001 this
Tribunal had directed the respondents not to act on the impugned Notification dated
21.12.2009. Only two contentions were raised by the learned counsel for the applicant (a)
RR couldn’t have been amended by Delhi Jal Board without taking concurrence of Govt.
of NCT of Delhi and that applicant who is a Degree holder has been put to an inferior and
disadvantageous position as he has been brought at par with Diploma holder A.E.
22.
As far as 1st point is concerned, it would be relevant to refer to Section 53, 54 and
109 (2) (m) of the Delhi Water Board Act, 1998, which read as under:“53. Consultation with Union Public Service Commission - No
appointment to any post above the rank of Asst. Engineer shall be made
except in consultation with the Union Public Service Commission
(hereinafter referred to as “the Commission”) Provided that no such
consultation with the Commission shall be necessary in regard to the
selection for appointment of any person.
(a) In a temporary capacity for a period not-exceeding one year;
(b) To a post when at the time of such appointment a person to be
appointed thereto possesses the qualifications prescribed for such post and
is in the service of the central government or of the government in the
same or a higher scale of pay”.
54.
(1) The commission may make regulations for the following
matters namely: a. The procedure to be followed by the commission in advertising posts,
inviting applications, scrutinizing the same and selecting candidates for
interview.
b. The procedure to be followed by the commission for selecting
candidates for appointment and by the Board for consultation with the
commission.
c. Any other matter which is incidental to, or necessary for,
the purpose of consultation with the commission,
(2) In the case of any difference of opinion between the commission and
the Board on any matter, the Board shall refer the matter to the central
government and the decision of the central government thereon shall be
final.
109. (1)
The Board may make regulations concerning any matter
which is to be or may be prescribed by regulations under this Act or in
respect of which this Act makes no provision or makes insufficient
provision and such provision is, in the opinion of the Board, necessary for
the implementation of this Act.
(2)
Without prejudice to the generality of its power under sub-section
(1), the Board may make regulations regarding the following matters,
namely (m)
Terms and conditions of service of officers and employees of the
Board”.
23.
From above, it is clear that it is the UPS Commission which is empowered to
make the regulations and in case there is any difference of opinion between Delhi Jal
Board and the UPSC, the matter has to be referred to the Central Government.
24.
No rule could be produced by the counsel for the applicant in spite of opportunity
given that the rules could not have been amended without taking concurrence of the
Government of NCT of Delhi. On the contrary, respondents have specifically stated that
there is no provision to get the prior approval of Government of NCT of Delhi. They
have explained that Government of NCT of Delhi is only the Nodal Ministry for
notifying the RRs as approved by DJB and concurred by the UPSC
25.
Even otherwise, respondents have explained this amendment became necessary in
view of the recommendations made by the Vth CPC and O.M. dated 25.5.1998 issued by
DOP&T.
26.
They have further explained that there was already a direction issued by the
Tribunal in one of the cases to notify the amended RRs within the stipulated period,
therefore, no further time could have been spent in processing further amendments. It
was in view of the directions given by the Tribunal that UPSC had directed the DJB to
issue the amended RRs as already concurred. In view of the facts as explained above, we
do not find merit in the first contention. The same is accordingly rejected.
27.
Counsel for the applicant next contended that he has been degraded by comparing
himself with a Diploma Holder. This contention is also baseless because as far as Degree
Holders are concerned, they have to have 8 years regular service as AE for becoming
eligible for promotion the post of Executive Engineer whereas Diploma Holder AE
would become eligible only after 10 years of regular service so the Degree Holders are on
a better footing. It is thus wrong to suggest that they have been degraded. After all, if
respondents felt Diploma Holder AE should also be made eligible for further promotion,
Degree Holder AEs cannot have any valid objection to it. In any case it is not within the
domain of courts to decide how RRs should be framed. It is the job of the Department or
the UPSC to decide how best RRs can serve the purpose of the department.
28.
In P.U.Joshi and others Vs. Acountant General, Ahmedabad and others reported
in 2003 (2) Supreme Court Cases 632 the Hon ble Supreme Court has, inter alia,
observed that determination of conditions of service, alteration thereof by amending
rules, constitution, classification or abolition of posts, cadres or categories of service,
amalgamation, bifurcation of departments, reconstitution, restructuring of the pattern etc.
all pertain to executive policy and within exclusive discretion of the State, subject to
limitations and restrictions envisaged in the Constitution. Government servants have only
right to safeguarding rights or benefits already earned, acquired or accrued but they
cannot challenge the authority of State to make such amendments or alterations in rules.
29.
Challenge to RRs cannot be sustained in law simply because it does not suit one
individual. We, therefore, find no merit in the second contention also. No other point
was argued. The OA is accordingly dismissed being devoid of any merit. No costs.
***
CENTRAL ADMINISTRATIVE TRIBUNAL, PRINCIPAL BENCH, NEW DELHI
O.A. NO.1185/2011
D.D. 28.03.2011
Hon’ble Mr. Justice V.K.Bali, Chairman &
Hon’ble Mr. L.K.Joshi, Vice Chairman (A)
Rajesh Nigam
Vs.
Union of India & Ors.
…
Applicant
…
Respondents
Promotion:
Annual Confidential Reports:
In the seniority list of A.E.E. issued on 22.6.2001 the name of the applicant
appeared at Sl.No.61 whereas that of 4th respondent is at Sl.No.64 – In June 2004, DPC
for promotion to the post of Executive Engineer for the vacancy year 2004-05 the
applicant was considered but not promoted, whereas the 4th respondent was promoted –
The applicant made representation against his non promotion and in September 2005 the
applicant was considered against the vacancy for the year 2005-06 and was promoted but
was shown as junior to 4th respondent – In the seniority list issued on 14.3.2006 the
applicant was shown at Sl.No.50 whereas the 4th respondent was shown at Sl.No.38 –
After getting information regarding his ACRs the applicant came to know that he was
graded below benchmark for the period from April, 1999 to March, 2001 but the same
was not communicated to him – Hence applicant filed this application with
Miscellaneous Application seeking condonation of delay – Tribunal in view of the fact
that the applicant slept over the matter for a period of five years held that there was no
reason to condone the delay and hence dismissed the application as barred by time and
for delay and laches.
Held:
The cause of action accrued to the applicant not after the decision of the
Supreme Court in the matter of Dev Dutt v. Union of India dated 23.5.2008 but when his
immediate junior was promoted in June 2004. As the applicant slept over the matter for a
period of five years it was not a case for condoning the delay consequently dismissed the
application as barred by time.
ORDER
Justice V.K.Bali, Chairman:
Rajesh Nigam, Executive Engineer, applicant herein, has filed this Original
Application under Section 19 of the Administrative Tribunals Act, 1985, seeking
direction to be issued to the respondents to promote him for the vacancy year 2004-05
and quash and set aside the proceedings qua the applicant which have been made by
considering the ACRs for the period 04/1999 to 03/2000 and 07/2000 to 03/2001 and to
declare the downgrading of the report for the year 07/2000 to 03/2001 by reviewing
officer and countersigning officer without assigning any reason. The applicant also seeks
direction to be issued to the respondents to consider him by constituting a review DPC
for the vacancy year 2004-05 for the post of Executive Engineer (QS&C) without
considering his ACRs for the period 04/1999 to 03/2000 and gradings of the reviewing
officer and countersigning officer in ACRs for the period from 07/2000 to 03/2001, and
to grant all consequential benefits.
2.
The facts, as set out in the Application, would reveal that the applicant joined as
Assistant Surveyor of Works (ASW) re-designated as AEE (QS&C) on 29.06.1998
through UPSC Engineering Services Examination. Shailender Kumar, arrayed as fourth
respondent herein, joined as ASW on 09.03.1999 and was thus junior to the applicant. In
the seniority list issued on 22.06.2001, the name of the applicant appears at Sl.No.61
whereas the fourth respondent is at serial no.64. In June, 2004, a DPC for promotion to
the post of Executive Engineer was held for the vacancy year 2004-05 wherein the
applicant was considered but not promoted, whereas fourth respondent was promoted in
the panel dated 17.06.2004. On 14.05.2005, the applicant made representation against his
non-promotion but received no reply. In September, 2005, the applicant was considered
against the vacancy for the year 2005-06 for the post of Executive Engineer and was
promoted as well but was shown junior to the fourth respondent. In the seniority list that
came into being on 14.03.2006, the applicant was shown at serial no.50 whereas the
fourth respondent was shown at serial no.38.
It is the case of the applicant that
subsequent to judgment of Hon’ble Supreme Court in the matter of Dev Dutt v. Union of
India (CA No.7631/2002 decided on 23.05.2008), applicant sought information regarding
his ACRs and came to know that his ACRs for the period from April, 1999 to March,
2000 and July, 2000 to March, 2001 have been graded below benchmark though the same
were never communicated to him.
3.
Inasmuch as, present Original Application is barred by time, the applicant has
filed Miscellaneous Application bearing MA No.905/2011 under Section 21(3) of the
Administrative Tribunals Act, 1985 seeking condonation of delay. In the application
aforesaid, it has, inter alia, been pleaded that the applicant gave representations in May,
2004; September, 2004 and May, 2005, but the respondents did not reply to any of the
representations aforesaid, however, promoted him in September, 2005. It is the case of
the applicant that he was given the copies of his ACRs vide letter dated 04.08.2010 and
only then he could know the reasons for his non-promotion due to below benchmark
gradings. When the applicant came to know about the reasons for his non-promotion in
2010, he has filed this Original Application.
4.
We have heard learned counsel for the applicant and with his assistance
examined the records of the case.
5.
The applicant was ignored in the matter of promotion to the post of Executive
Engineer in June, 2004. He made some representations to the respondents in 2004 and
2005. Even though he was promoted in September, 2005, he would not take any further
action in the matter. It is only in July, 2010 that the applicant sought information under
Right to Information Act, 2005 from where he stated to have come to know the reasons
for his non-promotion in 2004. The applicant slept over the matter for a period of five
years as during this period he took no action in the matter. It appears to us that the
applicant was satisfied with the promotion which came about in September, 2005 and it is
only on getting a cue from the judgment of Hon’ble Supreme Court in case of Dev Dutt
(supra) that he thought of making an application under Right to Information Act, 2005 to
know the reasons of his non-promotion. During the course of arguments, counsel for the
applicant would contend that the cause of action accrued to the applicant when the
Hon’ble Supreme Court rendered its decision in the case of Dev Dutt (supra). It is not
possible to accept the contention raised by the counsel for the applicant as in fact the
cause of action accrued to the applicant when he was not promoted in June, 2004 and
terminus a quo for limitation would start from the date when his immediate junior was
promoted. It is for this reason the application is barred by time and so, even as thought
by the applicant, an application seeking condonation of delay has been filed. We do not
find sufficient reason to condone the delay. If the applicant, after making representations
in 2004 and 2005, was not told about the reason for his non-promotion in 2004, he could
well get to know about the same under Right to Information Act as the same came into
being in 2005. The applicant would do nothing for five years and seek information under
Right to Information Act, 2005 only after the judgment of Hon’ble Supreme Court in Dev
Dutt’s case (supra). It is for that reason, we have observed about that the applicant only
after the judgment of Hon’ble Supreme Court dated 23.05.2008 in Dev Dutt’s case
(supra) made an application under Right to Information Act, 2005. The OA is clearly
barred by time as prescribed under Section 21 of Administrative Tribunals Act, 1985 and
there is no sufficient reason to condone the delay.
The Original Application also
deserves to be dismissed for unexplained delay and laches. It is not permissible for the
applicant to re-open his ACRs, which were below benchmark and which pertain to the
year 1999 and 2000 at this distance of time.
6.
Original Application is dismissed being barred by time and for unexplained
delay and laches.
***
IN THE CENTRAL ADMINISTRATIVE TRIBUNAL: HYDERABAD BENCH:
HYDERABAD
ORIGINAL APPLICATION NO.210 OF 2010
D.D. 07.04.2011
Hon’ble Mr. M.Kanthaiah, Member (Judicial) &
Hon’ble Mr. Ranbir Singh, Member (Admn.)
Dr. Gourhari Kamila
…
Vs.
The Secretary, UPSC & Ors…
Applicant
Respondents
Examination:
Whether failure to produce copies of B.Sc. degree and marks sheet in support of having
Physics or Mathematics which is one of an essential educational qualifications for the
post entails rejection of the candidate’s application treating it as incomplete? – Yes.
The applicant was a candidate for the post of Deputy Director (Ballistics) –
Qualification prescribed for the post is Master’s Degree in Physics/Forensic
Science/Mathematics with Physics or Mathematics as one of the subjects at Bachelor of
Science level – The applicant failed to produce B.Sc. (Maths) certificate along with his
application – His application was rejected as incomplete in terms of condition in the
notification that if no copies are sent with the application, it is liable to be rejected –
Aggrieved by the same the applicant has filed this application – Tribunal rejected the
contention of the applicant earlier in 1992 when the applicant had applied for the post of
Junior Scientific Officer he was asked to furnish information/clarification similar
opportunity should have been given to him in the instant case and consequently dismissed
the application.
Held:
The furnishing of copies of certificates stated in the advertisement was a
necessary condition for consideration of the application. Admittedly, the applicant had
not furnished a copy of his B.Sc. Degree and marks sheet in support of having Physics or
Mathematics as one of the subject, which, as per the Recruitment Rules, was an essential
educational qualification for the post. Therefore, Respondent No.1 was right in rejecting
the applicant’s application treating it as incomplete.
Merely because in 1992, when the applicant had applied for the post of Junior
Scientific Officer (B), Respondent No.1 had issued him letter dated 25th November, 1992
asking him to furnish information/clarification, is no ground for undertaking a similar
exercise in the instant case. Each case has to be decided on the basis of its own facts and
circumstances. The Recruitment Rules in instant case did not provide for seeking any
such information/clarification. The Recruitment Rules as notified, had to be complied
with and we find that the applicant has not complied with the same and Respondent No.1
has complied with the same by rejecting the application treating it as incomplete. There
is nothing unfair or arbitrary in this action calling for our judicial interference.
Cases Referred:
(1980) 3 SCC 202 - Uma Shankar Sharma v. Union of India & Ors.
(1999) 2 SCC 189 - Dr. Ram Sevak Singh v. Dr. U.P. Singh & Ors.
2010 STPL (Web) 76 DEL Dr. Vineet Relhan v. Union Public Service Commission
ORDER
Hon’ble Mr. Ranbir Singh, Member (Admn.)
As per the OA, the applicant while working as a Senior Scientific Officer
(Ballistics) in the 3rd Respondent’s organization, had applied to the Union Public Service
Commission, through proper channel, for the post of Deputy Director (Ballistics) for
which, interviews were scheduled to be held on 16th March 2010. But, he had not
received interview call letter. The applicant states that Respondent No.3 had forwarded
his application to Respondent No.1 well within the time limit, which was acknowledged
by Respondent No.1’s office. He was allotted Roll No.26. He had made representation
dated 20th February 2010 to Respondent No.1 requesting to issue call letter for interview,
to which there was no response. Hence, the present OA seeking the following relief:
“(a)
Call for the records pertaining to the Notification No.13 issued
in Employment News dated 12-18 July, 2008 calling for applications from
the eligible candidates for selection to the post of Deputy Director
(Ballistics) and declare the inaction on the part of the 1st Respondent in not
issuing the call letter to the applicant for the scheduled interview on
16.03.2010, denying the participation in the selection for the post of
Deputy Director (Ballistics) declaring the same as arbitrary, illegal,
unwarranted, misconceived, for extraneous considerations, and in
violation of Articles 14 and 16 of the Constitution.
(b)
Direct the respondents to issue call letter for interview
scheduled to be held on 16.03.2010 at UPSC, New Delhi duly allowing
the applicant to participate in the said selection of the post of Deputy
Director (Ballistics) duly making available the required documents that are
necessary for such participation and to declare the results thereon along
with the juniors to the applicant. And,
(c)
All consequential benefits.”
The grounds taken in this OA are as follows:-
(i) The applicant holds the degree of M.Sc. (Mathematics) having studied
Bachelor’s Degree in Science with Physics, Chemistry and Mathematics (Hons.). He has
been performing his duties in the 3rd respondent’s organization with unblemished records
of service.
(ii) Having fulfilled all the conditions stipulated in the eligibility criteria, he is
entitled to be considered for the recruitment to the post of Deputy Director (Ballistics)
subject to selection to be conducted by the Union Public Service Commission.
Therefore, there is no reason for the 1st respondent to deny him participation in the
selection. The inaction on the part of the Respondent No.1 to dispatch call letter for the
interview to the applicant, is arbitrary, illegal and for extraneous and obvious reasons.
And,
(iii) His juniors in rank, age and experience have received call letters, and he
being the most eligible candidate as per the eligibility criteria, has been deprived of his
fair chances of getting selection for the said post. It is a clear case of violation of
principles of natural justice by calling his juniors who have less service and no
experience in the relevant field and lower in the cadre.
3.
A reply statement has been filed on behalf of Respondent No.1 wherein it is
submitted that the applicant had not submitted his B.Sc. degree and marks sheet in
support of having Physics or Mathematics as one of the subjects, which was a part of the
prescribed essential education qualifications required for the post as per the Recruitment
Rules.
Therefore, his application was rejected under the category of ‘Incomplete
Applications”. Finding no merit in the case, his representation was rejected.
4.
The applicant has filed a rejoinder reiterating the contentions made in the OA.
It is submitted that nowhere in the advertisement, it is mentioned that the lower
qualification i.e., B.Sc., (Maths) certificate had to be enclosed. He had produced a
certificate of higher qualification i.e., M.Sc. (Maths) certificate, in tune with the language
used in the Notification. The applicant states that he had substantially complied with the
condition laid down in the Advertisement, as the essential qualification prescribed for the
post is Master’s Degree with Mathematics. Even in an earlier recruitment to the post of
Junior Scientific Officer (Ballistics) by the same recruiting agency i.e, the Union Public
Service Commission, where the essential qualification was similar to that of the present
one for the selection to the post of Deputy Director (Ballistics), the Union Public Service
Commission had considered his application, and later corresponded with him to submit a
clear-cut experience certificate by specifying a due date for the same, which the applicant
had complied with, and was called for the interview. The above said clarification was
sought by the Union Public Service Commission, even though the applicant had
submitted an experience certificate along with that application, to specifically ascertain
whether he had experience in the relevant field i.e., analytical methods and research in
the field of Ballistics. After receiving the same, an interview call letter was issued to him
incorporating the condition that the interview was subject to the production of a clear-cut
proof that two years or more experience in the analytical methods and research. During
the interview, when the applicant had once again produced the same, he was selected.
There is no reason or logic now to have any doubt that the applicant had studied M.Sc.,
(Mathematics) without having Mathematics as a subject at the graduation level. There is
no stipulation in the advertisement that a graduation certificate also should be submitted
along with the application in addition to the Master’s Degree Certificate.
5.
We have heard the learned counsel for the parties and perused the pleadings.
6.
The learned counsel for the applicant has drawn our attention towards (1980) 3
SCC 202 Uma Shankar Sharma v. Union of India and others wherein it was held that
conditions of eligibility should not be viewed too technically. He has also drawn our
attention towards the judgment dated 7th October 2009 of the High Court of Punjab and
Haryana in the case of Union of India and another v. Bhanu Pratap Singh, in which it was
held that when the relevant information was furnished in a column other than the one
prescribed and an annexure was appended giving out the details of the experience of the
applicant, the application could not have been treated as incomplete. Our attention has
been drawn towards (1999) 2 SCC 189 Dr. Ram Sevak Singh v. Dr. U.P. Singh and
others, wherein it was held that the qualification of an M.Ed., satisfies the criterion of
Master’s Degree in one of the subjects taught in the college.
7.
The learned counsel for the respondents has drawn our attention towards 2010
STPL (Web) 76 DEL Dr. Vineet Relhan v. Union Public Service Commission, wherein,
vide judgment dated 13th January 2010, it was held that since it was rejected on the
ground that the applicant had not annexed the necessary certificates with his application
and the candidature of some other candidates was also similarly rejected, there was no
justification for judicial interference.
8.
The point for consideration is whether the decision of respondent No.1 in
rejecting the application of the applicant on the ground of its being incomplete is fair.
9.
The Advertisement No.13 published by the Union Public Service Commission
in the Employment News 12-18 July 2008, at Serial No.8, had invited applications for the
post of Deputy Director (Ballistics) in the Central Forensic Science Laboratory. The
essential educational qualification was stated as follows:
“Master’s Degree in Physics/Forensic Science/Mathematics with
Physics or Mathematics as one of the subjects as Bachelor of Science
level from a recognized or equivalent.”
10.
In the instructions and additional information to candidates for recruitment by
selection, at Serial No.7, a note was given regarding the certificate to be attached. It
stated inter alia as follows:
“Candidates should note that they should attach with their applications
attested/self certified copies of the following documents ---- ---(ii) Degree or Diploma certificate or other certificates in support of their
educational qualifications”.
Note III, below Serial No.7, states as follows:
“If no copies of the above certificates are sent with the application, it is
liable to be rejected and no appeal against its rejection will be
entertained.”
11.
In the advertisement, a format of application has been given which states on the
first page of the format, at Sl.No.3, as follows, in respect of the general instructions for
filling the application form:
“Copies of certificates should be attached in support of information
given in the form where necessary. Any information contained in the
attached certificates shall not be considered unless it is claimed in the
application form.”
12.
It would be clear from the aforesaid that the furnishing of copies of certificates
stated in the advertisement was a necessary condition for consideration of the application.
Admittedly, the applicant had not furnished the copy of his B.Sc. Degree and marks sheet
in support of having Physics or Mathematics as one of the subject, which, as per the
Recruitment Rules, was an essential educational qualification for the post. Therefore,
Respondent No.1 was right in rejecting the applicant’s application treating it as
incomplete.
13.
Merely because in 1992, when the applicant had applied for the post of Junior
Scientific Officer (B), Respondent No.1 had issued him letter dated 25th November 1992
asking him to furnish information/clarification, is no ground for undertaking a similar
exercise in the instant case. Each case has to be decided on the basis of its own facts and
circumstances. The Recruitment Rules in instant case did not provide for seeking any
such information/clarification. The Recruitment Rules as notified, had to be complied
with and we find that the applicant has not complied with the same and Respondent No.1
has complied with the same by rejecting the application treating it as incomplete. There is
nothing unfair, or arbitrary in this action calling for our judicial interference.
14.
Accordingly, finding no merit, this OA is dismissed. No costs.
***
IN THE SUPREME COURT OF INDIA
CIVIL APPELLATE JURISDICTION
Civil Appeal No.6349 of 2011
[Arising out of S.L.P. (C) No.11779 of 2011]
D.D. 05.08.2011
Hon’ble Mr. Justice Dr.Mukundakam Sharma &
Hon’ble Mr. Justice Anil R.Dave
Secretary, U.P.S.C. & Anr.
Vs.
S.Krishna Chaitanya
…
Appellants
…
Respondent
With
Interlocutory Application No.1
Recruitment:
Granting interim order which has the effect of granting the final relief is deprecated.
Respondent was a candidate for Civil Services Examination 2010 – He
submitted his application for Preliminary Examination of UPSC on 28.1.2010 by courier
– As Admission Certificate was not received he gave a representation in that behalf on
20.4.2010 – Respondent was informed that his application for Preliminary Examination
was not received – Respondent approached CAT, Hyderabad seeking interim relief to
furnish Admission Certificate for appearing for the examination – By interim order dated
12.5.2010 respondent was issued admission certificate on his furnishing copy of his
application subject to result of the original application and the respondent appeared for
the Preliminary Examination – After hearing the matter finally CAT by an order dated
1.9.2010 permitted the respondent to take the Main Examination if he was found
qualified – UPSC challenged the order of CAT by filing writ petition before the High
Court of Andhra Pradesh and the writ petition was dismissed – Petitioner was permitted
to appear for interview if he was qualified in the main examination – Result of interview
was kept in sealed cover - Being aggrieved by the said order this Appeal has been filed
by UPSC – Supreme Court in view of Clause-7 of the advertisement which prescribes the
procedure for submitting the application and obtaining acknowledgment card from UPSC
held that the respondent has failed to prove that he submitted his application through
courier and set aside the order of the High Court as well as the order of CAT.
Held:
We are of the view that the appellants cannot be directed to declare the final
result of the respondent, especially when his application form had not been received by
the appellants within the period prescribed. We ignore the second application form
which was submitted by him in pursuance of the direction given by the Tribunal.
We may add here that this Court has observed time and again that an interim
order should not be of such a nature that by virtue of which a petition or an application,
as the case may be, is finally allowed or granted even at an interim stage. We reiterate
that normally at an interlocutory stage no such relief should be granted that by virtue of
which the final relief, which is asked for and is available at the disposal of the matter is
granted. We, however, find that very often courts are becoming more sympathetic to the
students and by interim orders authorities are directed to permit the students to take an
examination without ascertaining whether the concerned candidate had a right to take the
examination.
For any special reason in an exceptional case, if such a direction is given, the
court must dispose of the case finally on merits before declaration of the result. In the
instant case, we have found that the respondent not only took the preliminary
examination but also took the main examination and also appeared for the interview by
virtue of interim orders though he had no right to take any of the examinations. In our
opinion, grant of such interim orders should be avoided as they not only increase work of
the institution which conducts examination but also give false hope to the candidates
approaching the court.
JUDGMENT
ANIL R. DAVE, J.
1.
Leave granted.
2.
Being aggrieved by the Judgment and Order dated 7.2.2001 passed in W.P.
No.33367 of 2010 by the High Court of Andhra Pradesh at Hyderabad, confirming the
Order dated 1st September, 2010, passed by the Central Administrative Tribunal,
Hyderabad Bench at Hyderabad, this appeal has been filed by the appellants – the
Secretary and the Joint Secretary of Union Public Service Commission (UPSC).
3.
According to the case of the respondent, being desirous of taking Civil Services
Examination, 2010, he had filled up his application form and had sent the same to UPSC
through DTDC Courier and Cargo Ltd. The respondent had handed over his application
form to the above named courier on 28th January, 2010, and the courier had intimated to
the respondent that the application form was delivered to UPSC on 29th January, 2010.
Thus, according to the respondent, his application form had been duly received by UPSC
and, therefore, he was expecting his admission certificate but as he had not received it
even in the month of April, 2010, he had made a representation to the appellants on 20th
April, 2010, making a grievance with regard to non-issuance of admission certificate to
him. In pursuance of the aforestated representation made by the respondent, a letter dated
23rd April, 2010, was addressed to the respondent whereby he was informed that his
application for Civil Services Examination (Preliminary), 2010 had not been received by
the appellants and the respondent was also requested to furnish acknowledgment card
duly stamped by UPSC to enable the appellants to take further action in the matter.
4.
As the respondent had not received any acknowledgement card from the
appellants, the respondent rushed to the Central Administrative Tribunal, Hyderabad, by
filing O.A. No.470 of 2010 praying inter alia for an interim relief to the effect that the
appellants be directed to furnish an admission certificate to the respondent so that the
respondent can take the examination. By an interim order dated 12th May, 2010, the
Central Administrative Tribunal directed the respondent to submit a copy of his
application form to the appellants and directed the appellants to issue an admission
certificate to the respondent so that the respondent can take the examination. It was
clarified that the admission certificate would be subject to the final result of the said
original application.
5.
In pursuance of the aforestated interim order passed by the Central Administrative
Tribunal (CAT), the respondent had filed another application form which was received
by the appellants around 17th May, 2010 and in pursuance of the said application form,
an admission certificate was issued to the respondent and he took the Civil Services
Examination (Preliminary).
6.
The aforestated original application was finally heard by the CAT and by an
Order dated 1st September, 2010, the application was allowed, whereby the appellants
were directed to declare result of the respondent and if he was found qualified, he should
be permitted to take the Civil Services Examination (Mains), 2010. While allowing the
application, the Tribunal had considered reply filed on behalf of the appellants. It was
stated in the reply filed on behalf of the appellants that no application form from the
respondent was received by the appellants. The respondent had specifically stated that his
application form bearing No.37573985 had been submitted through the courier named
hereinabove to the appellants on 29th January, 2010 at 4 p.m. The respondent had mainly
relied upon an acknowledgement given to him by the courier to the effect that his
application form had been delivered to the appellants on 29th January, 2010 at 4 p.m. and
an affidavit had also been filed in support of the said averment by Shri V.S. Kumar Raju,
Manager, Administration, Regional Office of DTDC, Hyderabad. The aforestated
averments of the respondent were specifically denied by the deponent of an affidavit filed
on behalf of the appellants. While passing the final order, the Tribunal had considered the
above facts and had also observed about two possibilities - either the application form of
the respondent was misplaced in the office of the appellants or the courier agency had
failed to deliver the application form of the respondent to the appellants. The Tribunal did
not come to the final conclusion that the application form of the respondent was delivered
to the appellants or the appellants in fact had received the application form of the
respondent. Though the Tribunal observed in its order that it was difficult to come to a
definite conclusion that the application form of the respondent was in fact received by the
appellants, the Tribunal gave a final direction to the appellants to declare the result of the
respondent and if he was found successful in the Civil Services Examination
(Preliminary), he should also be permitted to take the Civil Services Examination (Mains)
and should also be permitted to appear for interview. Thus, the application filed by the
respondent was allowed by the Tribunal by the order dated 1st September, 2010.
7.
The aforestated order of the Tribunal was challenged before the High Court by the
appellants by filing Writ Petition No.33367 of 2010.
After hearing the concerned
advocates and after considering the above facts, the High Court disposed of the petition
by observing that the respondent be permitted to take the Civil Services Examination
(Mains) and should also be permitted to appear for the interview, if he is qualified in the
Civil Services Examination (Mains). With the aforesaid observations, the petition was
disposed of by the High Court.
8.
It is pertinent to note that during the pendency of the aforesaid proceedings, the
respondent took the Civil Services Examination (Mains) and also appeared for the oral
interview. The final result has not been declared and it has been retained by the appellants
in a sealed cover. Interlocutory Application No.1 has been filed by the respondent before
this Court praying for directions to the appellants to declare the result of the respondent
and keep a post vacant in a particular cadre so as to enable him to join the service. The
said application is also pending for hearing.
9.
Mr. Parag P. Tripathi, learned Additional Solicitor General appearing for the
appellants submitted that the impugned order of the High Court confirming the order of
the Tribunal is absolutely unjust and improper especially in view of the fact that neither
the Tribunal nor the High Court had come to any final conclusion that the application
form of the respondent was in fact submitted to the appellants.
10.
The learned counsel apprised us of the procedure with regard to acceptance of
application forms and he had also kept the entire relevant record pertaining to the
application forms regarding the Civil Services Examination, 2010 in this Court. He
explained to us as to how an application form was being received by the appellants. He
submitted that as per normal practice of the appellants, whenever any application form
pertaining to the Civil Services Examination is sent by post, the candidate sending it by
post is supposed to enclose a self addressed acknowledgement card, with postal stamp
affixed, along with the application form. The said acknowledgement card is returned by
the appellants to the concerned candidate with a distinct numerical mark affixed thereon.
The acknowledgement card is sent by post to the concerned candidate. If any application
form is received by the appellants either through hand delivery or through a courier, the
person who hands over the application form to a representative of the appellants at a
particular counter, would be given an acknowledgement card after affixing a stamp
having a distinct numerical mark.
11.
He further stated that a facsimile of each stamp having distinct numerical mark is
also retained by affixing it in a register maintained by the appellants so that in an event of
any effort to forge the acknowledgement mark, fraud can be detected easily. The register
containing such marks and record pertaining to the applications received on each day was
placed before this Court for its perusal.
12.
According to the leaned Additional Solicitor General, in view of the aforestated
procedure, if the application form of the respondent bearing No.37573985 had been
received by the appellants, an acknowledgment card ought to have been received by the
courier’s representative, who had personally handed over the application form to a
representative of the appellants. He further submitted that according to the respondent,
his application form was submitted on 29th January, 2010 at 4 p.m. A list of all
applications, which had been received on 29th January, 2010, was shown to this Court
but in the said list, there was no reference to the application form bearing no.37573985,
belonging to the respondent. He, therefore, submitted that in fact the application form of
the respondent had not been received by the appellants.
13.
The learned counsel for the appellants further submitted that 100 application
forms and record pertaining thereto is retained in one separate packet and he also
explained the system whereby all application forms are received and processed by the
appellants. Even in the packets containing application forms received on 29th January,
2010, the respondent’s form was not found.
14.
The learned counsel further submitted that as the application form of the
respondent had never been received by the appellants, it would not be proper to declare
result of the respondent because as per the case of the appellants, the form of the
respondent was never submitted to the appellants. In such an event, declaration of the
result of the respondent would be absolutely unjust and would set a wrong precedent. He,
therefore, submitted that the appeal be allowed and the judgment of the High Court
confirming the order of the Tribunal be quashed and set aside.
15.
On the other hand, Mr. L. Nageshwara Rao, learned senior counsel appearing for
the respondent mainly submitted that the respondent had forwarded his application form
through DTDC Courier and Cargo Ltd. and the courier had delivered the form to the
appellants on 29th January, 2010. He also relied upon an affidavit filed by a responsible
officer of the above named courier agency stating that the respondent’s application form
was delivered to U.P.S.C. on 29th January, 2010.
16.
He further submitted that there was no reason for the respondent to make any
false averment with regard to submission of the application form because the respondent
was quite serious about the examination and in fact he had passed the Civil Services
Examination (Preliminary) and the respondent was quite hopeful of even succeeding in
the Civil Services Examination (Mains) and oral interview. He further submitted that
there was no reason for the courier agency not to deliver the application form of the
respondent and there was no reason for a responsible officer of the courier agency to file
a false affidavit supporting the respondent to the effect that his application form had been
submitted to the appellants.
17.
The learned counsel further submitted that by declaration of the result, there
would be no harm to anyone because if the respondent is not declared successful, he
would not get any benefit but if in fact he is found successful in the examination as well
as in the oral interview and if he is not given benefit of doubt, career of a bright young
person would be ruined. He, therefore, submitted that the judgment of the High Court
confirming the order of the Tribunal is just and legal and, therefore, the appeal should be
dismissed.
18.
We have heard the learned counsel at length and have also meticulously gone
through the relevant record produced before this Court by the learned Additional Solicitor
General.
19.
It is pertinent to note that the respondent, at no point of time, had adduced any
evidence before the Tribunal or even before this Court to the effect that the appellants
had received the application form of the respondent bearing no.37573985.
20.
Right from the beginning i.e. the stage at which an original application was filed
before the Tribunal, the respondent had relied upon an affidavit filed by the Manager
Administration, Regional Office of the DTDC Courier and Cargo Ltd., having its branch
office at Hyderabad. According to his affidavit, the respondent’s application form had
been delivered to the appellants on 29th January, 2010. The application form had not
been delivered by him personally but it was delivered by an employee of the above
named courier agency and so as to substantiate his say, he had relied upon the delivery
Run Sheet No.12878919 dated 29th January, 2010. The said run sheet is a part of the
record. Upon perusal of the run sheet, we do not find any acknowledgement given by any
of the officers of the appellants to the effect that an application form of the respondent
was received by the appellants. The said run sheet incorporates numbers of consignments
which had been addressed to UPSC, Shahjahan Road, New Delhi. Beyond numbers of
five different consignments and name of UPSC, to whom the consignments were to be
sent, there is no indication on the said run sheet that the said consignments were received
on behalf of UPSC.
21.
In our opinion, on the basis of the aforestated record, by no stretch of imagination
one can say that the respondent’s application form had been received by the appellants.
22.
As the case involves a career of a young man, who can turn out to be a good civil
servant, we had very meticulously gone through the record maintained by the appellants.
Looking to the system which is being followed by the appellants, we find that the said
system is very comprehensive and flawless. It is very clear that if the application form of
the respondent had been received by the appellants in the manner provided, it would have
been recorded somewhere. Even the eight digit number of the application form of the
respondent has not been recorded anywhere. Receipt of an application form through a
courier is treated as ‘hand delivery’ by the appellants. In case of receipt of an application
by hand delivery, on the spot, an acknowledgement card stamped with a distinct
numerical mark is handed over to the person who delivers the application form. If the
application form had been delivered by a representative of the courier agency to the
office of the appellants, there was no reason for the appellants not to give a duly stamped
acknowledgement card bearing a distinct numerical mark. No such acknowledgment
card, duly stamped, could be produced by the respondent or by the courier agency. Thus ,
on perusal of the record and looking the facts of the case, we come to a conclusion that
no proof could be submitted by the respondent that the application form was received by
the appellants.
23.
It is pertinent to note here that while passing the final order, even the Tribunal
was not sure whether the application form of the respondent was received by the
appellants. The Tribunal, in para 8 of its final order dated 1st September, 2010, has
observed as under:
“8. ………..It is quite possible that the applicant’s application had been
misplaced. It is also quite possible that the courier agency failed to deliver
the application form of the applicant at the respondent’s office……”.
Thus, even while giving final direction to the appellants with regard to permitting the
respondent to take the Civil Services Examination, the Tribunal had not come to a
definite finding and specific conclusion that the application form of the respondent was in
fact received by the appellants but the same had been misplaced by the appellants. In our
opinion, in such a set of circumstances, it would not be proper to direct the appellants to
permit the respondent to take the examination especially when there was nothing on
record to show that the respondent had submitted his application form to the appellants.
24.
We also record that there was some negligence on the part of the respondent. The
learned counsel appearing for the appellants had drawn our attention to the advertisement
given by UPSC inviting applications from the candidates who were desirous of joining
civil service and taking examination for that purpose. Clause 7 of the said advertisement
relating to acknowledgement of application is reproduced hereinbelow:
“7. Acknowledgment of applications:
Immediately on receipt of an application from a candidate, the
Acknowledgment Card submitted by him/her alongwith the Application
Form will be dispatched to him/her by the Commission’s Office duly
stamped in token of receipt of his/her Application. If a candidate does not
receive the Acknowledgement Card within 30 days, he/she should at once
contact the Commission by quoting his/her Application Form No.(8 digit)
and name and year of examination. Candidates delivering the Application
form in person at the Commission’s Counter will be issued
Acknowledgment Card at the Counter itself. The mere fact that a
candidate’s application has been acknowledged by the Commission does
not mean that his/her candidature for the examination has been accepted
by the Commission. Candidates will be informed at the earliest possible
about their admission to the examination or rejection of their application.”
25.
According to the respondent, he had forwarded his application form through the
aforestated courier on 28th January, 2010. If the respondent did not receive any
acknowledgment for a period of 30 days from the date on which he had forwarded his
application form, he ought to have made necessary enquiry in the office of the appellants.
Even according to the case of the respondent, for the first time on 20th April, 2010, he
made an enquiry about his application form as he had not received the acknowledgment
card from the appellants. As stated in the aforestated clause no.7, as a prudent candidate,
the respondent ought to have made enquiry latest by the end of February, 2010, but for
the reasons best known to the respondent, he waited upto 20th April, 2010 to make an
enquiry whether his application form was received by the opponents. In our opinion, no
vigilant student aspiring to become a responsible officer of the State would remain so
indifferent so as not to make any enquiry for more than two months. It is also pertinent to
note that the respondent was not taking the examination for the first time. According to
him, he had taken the examination earlier also but unfortunately he was not successful.
Thus, he was having experience about the way in which the application form is filled up,
how that is to be submitted and the way in which acknowledgement card is sent by the
appellants. In our opinion, this negligence on his part has resulted into his sufferance and
he himself is only to be blamed for the events.
26.
For the aforestated reasons, we are of the view that the appellants cannot be
directed to declare the final result of the respondent, especially when his application form
had not been received by the appellants within the period prescribed. We ignore the
second application form which was submitted by him in pursuance of the direction given
by the Tribunal.
27.
We may add here that this Court has observed time and again that an interim
order should not be of such a nature that by virtue of which a petition or an application,
as the case may be, is finally allowed or granted even at an interim stage. We reiterate
that normally at an interlocutory stage no such relief should be granted that by virtue of
which the final relief, which is asked for and is available at the disposal of the matter is
granted. We, however, find that very often courts are becoming more sympathetic to the
students and by interim orders authorities are directed to permit the students to take an
examination without ascertaining whether the concerned candidate had a right to take the
examination. For any special reason in an exceptional case, if such a direction is given,
the court must dispose of the case finally on merits before declaration of the result. In the
instant case, we have found that the respondent not only took the preliminary
examination but also took the main examination and also appeared for the interview by
virtue of interim orders though he had no right to take any of the examinations. In our
opinion, grant of such interim orders should be avoided as they not only increase work of
the institution which conducts examination but also give false hope to the candidates
approaching the court.
28.
For the reasons stated hereinabove, we allow the appeal by quashing and setting
aside the judgment delivered by the High Court as well as the order of the Tribunal with
no order as to costs. The Interlocutory Application filed by the respondent is also
rejected.
***
CENTRAL INFORMATION COMMISSION
Decision No.CIC/SG/A/2009/003055/6523
Appeal No.CIC/SG/A/2009/003055
D.D. 22.01.2010
Mr. Shailesh Gandhi, Information Commissioner
Mr.Kanhiya Lal
…
Vs.
Mrs.Shukla Malhotra, PIO & DDE …
Directorate of Education, New Delhi.
Appellant
Respondent
R.T.I. Act:
When information sought under R.T.I. Act is voluminous the applicant should be offered
inspection of the relevant files as provided under 7(9) R.T.I. Act.
The applicant through 100 queries sought information regarding admissions
released in Sarvodaya Govt. Co-Ed School, New Delhi and also sought various
procedures followed – The PIO gave reasons, rules and various clarifications – 1st appeal
filed was dismissed on the ground that the information has been provided – This 2nd
appeal before the Central Information Commission considering the volume of the
information sought CIC disposed of the appeal with a direction to offer inspection of the
relevant files as provided under Section 7(9) R.T.I. Act.
Held:
When such long RTI queries are sent to a public authority the PIO will be advised
to use the provisions of Section 7(9) RTI Act and offer inspection of the relevant files to
the appellant instead of trying to spend a lot of time and diverting the resources
significantly.
Information sought:
The appellant through 100 queries had sought information regarding admissions
released in Sarvodaya Govt. Co-Ed School, B-4, Paschim Vihar, New Delhi for the
session 2008-2009. Through his queries he had sought various procedures followed in
class-11 admissions etc.
PIO’s Reply:
The PIO in his point-wise reply gave reasons, rules and various clarifications.
First Appeal:
Incomplete information was provided.
Order of the FAA:
The FAA ordered that, “After going through the application, appeal, reply of he
PIO and statement of the authorized officials of Dist. W-A, I am of the view that relevant
information has been provided to the appellant. The contention of the appellant of not
being satisfied with the information provided by PIO cannot be fulfilled as there is no
barometer to define satisfaction level and no provisions exists in the Act to measure the
satisfaction level and the reply of the PIO is sustained.
Moreover, the appellant
continued to create ruckus and unruly scene in the office till 12.30 PM by disturbing the
officials working in the office.
As the information has been provided by the
PIO/DDE(W-B) and there is no locus standi in the appeal as such no further direction to
the PIO is required.”
Ground of the Second Appeal:
Incomplete information was provided by the PIO.
Relevant Facts emerging during Hearing:
The following were present
Appellant: Mr. Kanhiya Lal;
Respondent: Mrs. Shukla Malhotra, Public Information Officer & DDE (West-B Zone);
The appellant has used the RTI without a sense of responsible and has asked 100
queries stretching it to 16 pages. The Citizen’s has been given a right but he must use
this with some sense of responsibility and sending a 100 queries over 16 pages or more
does not display any sense of responsibility. A citizen must understand that it is in his
interest that the Government functions efficiently and it is not correct to try and over
burden or pulverize the Government’s functions. Inspite of this the PIO has tired to give
the information. The appellant was asked to identify what information had not been
provided to him. He is not able to give any instance of information with he has sought
which is not been provided. A citizen must realize that he is asking for the Government
to allocate some resource to provide the information to him. Hence he must use this with
respect or the fact that the Government is responsible to all citizens and not just to him.
Some restraint in the number of queries must be exercised by the citizens.
When such long RTI queries are sent to a public authority the PIO would be
advised to use the provisions of Section 7(9) RTI Act and offer an inspection of the
relevant files to the appellant instead of trying to spend a lot of time and diverting the
resources significantly.
Decision:
The appeal is disposed.
***
CENTRAL INFORMATION COMMISSION
Appeal No.CIC/WB/A/2008/01256 dated 18.7.2008
Right to Information Act 2005 – Section 19
D.D. 9.2.2010
Mr. Wajahat Habibullah, Chief Information Commissioner
Shri Wasi Ul Haque
Vs.
U.P.S.C.
…
Appellant
…
Respondent
R.T.I. Act:
Whether it is open to the applicant under R.T.I. Act to bundle a series of requests into one
application and seek information under Section 7(1) of R.T.I. Act? – No.
Applicant under R.T.I. Act sought information from U.P.S.C. contained in a
booklet comprised of 459 questions spread over 139 pages concerning every aspect of the
functioning of the UPSC including the condition of toilet, bath rooms, wash rooms and
the details of the individual health of every examiner engaged in the evaluation etc.C.P.I.O. of UPSC rejected the application – Appeal filed before the Appellate Authority
of the UPSC was also dismissed – In the Appeal filed before the Central Information
Commission (C.I.C), the C.I.C. while declining to impose penalty in filing frivolous
application against the applicant dismissed the appeal following its decision dated
19.6.2009 in Rajinder Singh vs. C.B.I.
Held:
As per Section 6(1) read with Section 7(1) of the Act, a request means that the
questions and the answers must share an embryonic relationship, the genus of the
application must be one and sub questions can constitute different species of the same
genus. Most of the questions of the applicant do not constitute ‘information’ as defined
in the law and because the remaining, even if they were to be so admitted, are without
focus so establishing genus, it is not possible to treat the application to be a valid request
admissible under RTI Act.
FACTS:
By an application of 6-2-2008 Shri Wasi Ul Haque of JNU, New Delhi applied to
CPIO Shri Prachish Khanna, DS (E-1) EA, UPSC seeking information contained in a
booklet comprised of 459 questions spread over 139 pages concerning every aspect of the
functioning of the UPSC including the condition of toilet, bath rooms, wash rooms and
the details of the individual health of every examiner engaged in the evaluation of CS
Main Examination 2006. The covering letter to this petition concludes as follows:
“You are suggested to reveal/ inform proactively in sync with the spirit of
this RTI petition in the welfare of the candidates and the general public.
Every query in this RTI petition serves some urgent public interest
manifestly or latently.”
To this CPIO Shri Prachish Khanna in his letter of 5-3-08 responded as follows:
“2. It may be noted that the information can be provided in the form in
which it is sought, unless it would disproportionately divert the resources
of the public authority, under section 7(9) of RTI Act, 2005. The present
application runs into 137 pages with 459 questions and thousands of subquestions which by own admission of the applicant is an elaborate RTI
query which may require more than normal time of 30 days mandated by
RTI Act, 2005.
3. While seeking a large quantity of data and information of different
types and nature, the bonafide public interest in seeking the information
has not been made clear. It is difficult to discern any public interest from
questions like; ‘How many members of the Commission were/ are
bespectacled?’
or
‘How many examiners fought among themselves while examining the
answer sheets for CSE, 2006?’ (Question No. 62)
or
‘Please briefly indicate all the English/ grammatical/typographical errors
in this RTI’ (question No. 406).’
The above are examples only. In addition many questions are interrogatory in nature
eliciting Commission’s explanation on certain events, which is not covered under the
definition of information under RTI Act.
4. Most of the useful information is clearly available on the UPSC
website: www.upsc.gov.in under Pro-active Disclosure (RTI) heading.
5. An information seeker should also keep in mind the cost effectiveness
of disclosure of information. It is therefore, requested to minimize and
prioritise the information needs, which can be provided without unduly
jeopardising the normal activities of the Commission.”
Shri Wasi Ul Haque then moved an appeal before Shri K.S. Bariar, JS (Exam.)
dated 19-3-08 pleading as follows:
“Sir, it is true that many questions in the said RTI petition are apparently
futile but there is a latent significance to all of them. This RTI was also
meant to be an IQ test and a test of the nature, temperament, disposition
and style of working of the Commission hence there were many
apparently irrelevant questions supplied deliberately. The way of your
denial to each individual question would also have revealed a lot about the
Commission. The concerned CPIO was supposed to separate the wheat
from the chaff but instead he has chosen to throw the baby out with bath
water. Mr. Khanna has definitely not even read the complete petition he
has read only words/ pages/ paragraphs. The Commission has failed the
first test here is your second and last chance.
Mr. Khanna has not noticed that there were many crucial questions raised
in the said petition. Are questions and allegations relating to
impersonation in the Civil Services examination not important? Are
queries relating to errors in the application forms to question papers to
your website not important? You make an error in the question paper and
nobody knows what you did to compensate the candidates. The coaching
institutes are making tall and fake claims about their achievements
because you are not revealing the data. This is merely a sample from the
said petition. Are such questions useless and will ‘disproportionately’
divert public resources’? Please read it carefully sir and you will know
what it is a wealth of information that you could use also to improve the
examination (that is if you really want to improve). IPSC must be held
responsible for the malaise that affects the CSE. You cannot shy away
from the responsibility. Many queries relate to your errors how can you
not read/ reply to them? If Mr. Khanna had read it, many queries in the
said petition should have given him sleepless nights (given the enormous
public interest issues raised therein).”
Shri Wasi Ul Haque also reduced the number of questions put by him to
approximately 110, citing the number of each in the appeal petition. By his order of
18.4.08 Shri K.S. Bariar upheld the response of CPIO, UPSC and dismissed the appeal as
below:
“I note that the reply given by the CPIO, UPSC with regard to his requests
in the RTI application is appropriate and justified. In view of the above, I
do not see any point to intercede to in the matter as solicited by the
appellant in his instant appeal.”
Shri Wasi Ul Haque has then moved a second appeal before us with the following
prayer:
“Keeping the above inexplicable behaviour of the Commission in view, I
seek release of information on my RTI petition. I want the Commission to
read each and every letter of the petition. This petition has a lot of ‘public
interest’ contained in it. A constitutional authority like the UPSC cannot
be so lazy and lackluster so as not be able to read and respond to 139
pages that deal directly with it’s functioning. This booklet has many issues
which when revealed to the public will cause a lot of embarrassment to the
Commission. Yet the Commission did not even read it. I also want to
know how they used my 150 Rupees that I had submitted as application
fee. I also want to know why they did not submit a soft copy of the replies
as requested.’
The appeal was heard on 3-2-2010. The following are present:
Respondents
Shri Prachish Khanna, Dy. Secretary & CPIO.
Shri V. D. Arora, Under Secretary, UPSC.
Shri Naresh Kaushik, Advocate.
Ms. Amita Kalkal Chaudhary, Advocate.
Ms. Aditi Gupta, Advocate.
Although informed of the date of hearing through our notice of 22-1-2010
appellant Shri Wasi Ul Haque has opted not to be present.
Learned counsel for respondents Shri Naresh Kaushik submitted that the
application was frivolous and vexatious and deserves not only to be dismissed on these
grounds but also subjected to costs. In support of the latter argument learned counsel Shri
Naresh Kaushik submitted the following documents:
1. Copy of the order of Supreme Court of India Presidential Election Petition No. 1 of
1997 dated 24.11.07 – Charan Lal Sahu & Anr. Vs. K. R. Narayanan & Anr. In this case
the Full Bench of the Hon’ble Supreme Court consisting of Justices S.C. Agarwal, G. N.
Ray, A.S. Anand, S.P. Bharucha and S. Rajendra Babu, JJ arrived at the following
conclusion :
“In Charan Lal Sahu vs. Giani Zail Singh [Supra] this Court, while referring to the
Election petition fled by petitioner No.1. had observed:
"It is regrettable that election petition challenging the election to the high
office of the President of India should be filed in a fashion as cavalier as
the one which characterises these two petitions. The petitions have an
extempore appearance and not even a second look, leave alone a second
thought appears to have given to the manner of drafting these petitions or
to the contentions raised therein. In order to discourage the filing of such
petitions, we would have been justified in passing a heavy order of costs
against the two petitions, we would have been justified in passing a heavy
order of costs against the two petitioners. But that is likely to create a
needless misconception that this Court, which has been constituted by the
Act as the exclusive forum for deciding election petition whereby a
presidential or vice-presidential election is challenged, is loathe to
entertain such petitions. It is of the essence of the functioning of a
democracy that election to public offices must be open to the scrutiny of
an independent tribunal. A heavy order of costs in these two petitions,
howsoever justified on their own facts should not result in nipping in the
bud a well-founded claim on a future occasion. Therefore, we refrain from
passing any order of costs and, instead, express our disapproval of the
lighthearted and indifferent manner in which these two petitions are
drafted and filed."[1.7]
In Mithilesh Kumar vs. Sri R. Venkataraman & Ors.[supra], this Court had
observed:
"While we expect every conscientious citizen eligible to file an election
petition to question an election on the grounds prescribed by the Act, we
do not wish that any petitioner should make use of this Court as a forum to
file a petition without giving adequate thought to its contents and also the
provisions of law governing the case merely to seek some chap publicity.
We regret to say that seeing one's name in newspapers everyday has lately
become the worst intoxicant and the number of people who have become
victims of it is increasing day by day." [.537]
In Mithilesh Kumar Sinha. vs. Returning Officer for presidential Election this
Court observed it as follows:
".......... Experience has shown that the solemnity and significance
attaching to such petitions has been reduced to a farce by the cavalier
fashion in which resort is had to this remedy. The mere fact that the entire
gamut of both these petitions is fully covered by several earlier decisions
of this Court to some of which these very petitioners were parties shows
that the existing provisions are inadequate to prevent such abuse of the
process of law." [p.698]
We find that these observations have had no effect. This election petition, which
has been jointly filed by the two petitioners, shows no improvement. It suffers
from the same defects as the earlier petitions filed by the petitioners. It seems that
the petitioners are obsessed with a desire that they should find a place in some
Book of Records. They find the temptation to file an election petition after the
Presidential election too difficult to resist. It is a matter of regret the petitioner
No.1, who happens to be an advocate himself, has been persisting in this past time
knowing well that such conduct on his part amounts to an abuse of the process of
law. This Court has so far refrained from imposing costs in the election petitions
that were filed by the petitioners earlier. It is high time that the petitioners who
have persisted in filling this petition in spite of the law laid down authoritatively
by this Court in the earlier decisions are saddled with costs.
2. Mary Angel and Ors. vs. State of Tamilnadu, CRLA No. 570/1990 announced
on 30.5.99. In this case the Division Bench of the Supreme Court consisting of
Justices K. Thomas & M. S. Shah, JJ, after detailed examination on the issue of
whether the High Court has jurisdiction to impose exemplary cost of Rs. 10,000/to be paid by each of the appellant by rejecting a frivolous and factious petition,
the learned Justices have arrived at the following conclusion :
“In the result we hold that while exercising inherent jurisdiction under
section 482, Court has power to pass ‘such orders’ (not inconsistent with
any provision of the Code) including the order for costs in appropriate cases,
(i) to give effect to any order passed under the Code or (ii) to prevent abuse
of the process of any Court or (iii) otherwise to secure the ends of justice.
As stated above, this extraordinary power is to be used in extraordinary
circumstances and in a judicious manner. Costs may be to meet the litigation
expenses or can be exemplary to achieve the aforesaid purposes.”
.
DECISION NOTICE
In our decision announced on 19.6.’09 in Rajinder Singh Vs. CBI – Complaint
No. CIC/WB/C/2007/00967 we have decided as follows:
“The issue hinges around the application required to be made for obtaining
information u/s 7 (1). Under this clause a CPIO, on receipt of ‘a request’ is
expected to deal with it expeditiously when with accompanied with a fee.
It is, therefore not open to the applicant under the RTI Act to bundle a
series of requests into one application unless these requests are treated
separately and paid for accordingly.
In our experience in disposing of appeals that in fact many such have been
treated as one application even though they contain a multiplicity of
requests. However, we concede that a request may be comprised of a
question with several clarificatory or supporting questions stemming from
the information sought. Such an application will indeed be treated as a
single request and charged for accordingly.”
On examination of the case references submitted by learned counsel for respondent in the
present case our view is that neither of these would have a bearing on this specific case
before us, and indeed on this Commission, because although these deal with inherent
powers, whereas the Presidential Election Petition No. 1 of 1997 deals with the inherent
powers of the Supreme Court of India, the case CRLA No. 570 of 1990 deals specifically
with the question involved “under sec. 482 of the Cr.P.C”. Sec. 482 of the Cr.P.C. reads
as follows:
“Sec. 482 CrPC Saving of inherent power of High Court
Nothing in this Code shall be deemed to limit or affect the inherent powers
of the High Court to make such orders as may be necessary to give effect
to any order this Code, or to prevent abuse of the process of any court or
otherwise to secure the ends of justice.”
From this it would be obvious that such a power cannot be exercised by this
Commission, which has no criminal powers whatsoever, much less the powers of a High
Court, and at best is a quasi judicial authority empowered to function as a Civil Court in
certain circumstances. However, in the decision of this Commission, in the file quoted
above, that as per section 6 (1) read with section 7 (1) of the Act a request means that the
questions and the answers must share an embryonic relationship, the genus of the
application must be one and sub questions can constitute different species of the same
genus. Given that for reasons already intimated to appellant Shri Wasi Ul Haque by the
CPIO, most of his questions do not constitute ‘information’ as defined in the law and
because the remaining, even if they were to be so admitted, are without focus so
establishing genus, it is not possible to treat this application despite the painstaking
efforts demonstrably put into compiling it, to be a valid request admissible under the RTI
Act. For this reason this appeal is dismissed.
Reserved in the hearing, this decision is announced after examination of the
rulings submitted by learned counsel on this ninth day of February, 2010. Notice of this
decision be given free of cost to the parties.
***
CENTRAL INFORMATION COMMISSION
Appeal No.CIC.WB/A/.2009/00275 dated 14.3.2009
Right to Information Act 2005 – Section – 19
D.D. 02.08.2010
Hon’ble Mr. Wajahat Habibullah, Chief Information Commissioner
Shri A.Pandian
Vs.
U.P.S.C.
…
Appellant
…
Respondent
R.T.I.:
Whether opinion/views of the public authority can be sought under R.T.I. Act: - No.
The appellant in his initial application under R.T.I. Act has sought reasons on the
basis of which the Selection Committee recommended the name of Thiru K.Raghubathi
for the post of Junior Director of Rural Development/Additional Director of Rural
Development – The said information was provided by the Central Pubic Information
Officer – Aggrieved by the information furnished this appeal was filed before the Central
Information Commission (CIC) - CIC holding that the information sought by the
applicant in his initial application has been fully provided by CPIO giving the reasons on
the basis of which the Selection Committee recommended the name of Thiru
K.Raghubathi, has dismissed the appeal
Held:
Under Section 2(f) of R.T.I. Act an information seeker should clearly specify the
required information which should be available in any material form. He is not expected
to elicit views of the CPIO through any form of query. Even where the information
provided to expose wrongdoing, the remedy for that will not lie with the information
Commission but with the appropriate court of law.
Facts:
By an application of 30.10.2008 Shri A.Pandian of Dindigul, Tamil Nadu
applied to the APIO, UPSC seeking the following information:
1) Since Thiru K. Ragubathi’s Promotion to JDRD/ADRD has been ordered
through rule 39(a)(i) which is clearly repugnant to Special rule 2(a)(b), in
the eye of law the promotion of Thiru K.Raghubathi to Joint Director of
Rural Development/Additional Director of Rural Development is invalid.
Special rule 2(b) prevails over General rule 39(a)(i). Therefore, Annual
Confidential reporting written for Joint Director and Additional Directors
posts and which have been taken into consideration for his selection
(2007-08) into I.A.S. have also become invalid. So I request you kindly
state the reasons for selecting ineligible candidate Thiru K.Ragubathi into
I.A.S. (2007-08)?
2) Your kind attention is invited to General rule 4 explanation III which
clearly states that no temporary panel shall be prepared in respect of the
post for which consultation of the Tamil Nadu Public Service Commission
is not required and the list of name prepared if any, shall be a regular one.
(Copy of the rule is enclosed) whereas Thiru K.Ragubathi has been
promoted as Joint Director and Additional Director only temporarily as
per rule 39(a)(i) As these posts do not come under Tamil Nadu Public
Service Commission Purview Promotion under Rule 39(9)(i) being
temporary ought not have been made. Therefore the ACRS (Selfassessment report) written during his service of JDRD/ADRDS will not be
valid and consequently the ACRs written (self-assessment report) during
these period have also been invalid. Please explain why this above fact
has not been considered? And the reasons for selecting ineligible
candidate Thiru K.Ragubathi may kindly be stated?
3) Thiru K.Ragubathi has been appointed as Divisional Development Officer
in the year August 1994 and has been temporarily promoted as Joint
Director of Rural Development Govt. of Tamil Nadu in Sept. 1997. As his
temporary promotion to JDRD Subsequently to ADRD is as per rule
39(a)(i) and such promotions have become invalid due to rule 4
explanation III and prevalence of special rule 2(b) over rule 39 a (i) his
length service beyond the period of Sept 1997 has also become invalid.
So he is left with the valid service from August 1994 to Sept 1997 to
roughly 3 years and one month in adequate and he does not fulfil the
requirement of Regulation 4(iii) of I.S.S. (Appointment by Selection)
(1997) which stipulates 8 years continuous service. Why this aspect has
not been considered?”
To this Shri A. Pandian received a response from CPIO Ms. Richa Mishra, DS
(AIS) dated 7.11.08 informing him as follows:
“It is informed that forwarding of names of eligible Non State Civil
Service Officers, keeping in view the specific provision of the IAS
(Appointment by Selection) Regulations, 1997, is in the purview of the
State Govt. Further, the promotion of State Service Offices in the State
Service is guided by the relevant Service Rules and is in the domain of
the State Govt. concerned. In the instant case, while forwarding the
proposal for convening the SCM for selection of Non-SCS officers to
the IAS of Tamil Nadu cadre for the year 2007, the Govt. of Tamil Nadu
vide their letter dated 28.10.2007 informed that Shri K.Raghubathi was
continuously holding the post declared equivalent to the post of Deputy
Collector in the SCS from 17.08.1994. On scrutiny of the service
records of Shri Raghubathi which were certified as valid by the State
Govt., and personal interview, the Selection Committee recommended
the name of the officer for inclusion in the Select List of 2007.
In this regard attention is also invited to the decision of the CIC in
Appeal No.1404/IC (A)/2007 (F.No.CIC/MA/A//2007/00585) dated
02.11.2007 wherein it has been decided that “Under Section 2(f) of the
Act, an information seeker should clearly specify the required
information which should be available in any material form. He is not
expected to elicit views of the CPIO through any form of query, as
attempted by the appellant.”
Aggrieved Shri Pandian moved an appeal dated 19.11.08 received in UPSC on
24-11-08 contesting the appointment of Thiru K.Raghubathi relying on Court Rulings as
follows:
“The Principle quoted by Hon’ble Supreme Court in AIR 2003 SC 3935
shall apply to this case also.
AIR 2003 SC 3935.
Dr. Yadav & R.K.Singh.
Constitution of India Art 309-Proviso-UP. Govt. Servants. Seniority
Rules (1991) R-3-Rules framed under Art 309 - Are for transitory period
– must give way to Special Rules once framed – However, if special rules
are already operating field – General rules framed under Art 309 – would
not apply – Doctrine of Generalia Specialties non-derogant shall apply.
The Fundamental rules appendix I, annexure VII (statutory service Rules)
4. “Promotions made other wise than in accordance with the provisions of
the statutory service rules are illegal. Such promotion, if already made,
should be set aside and the persons promoted should be reverted with
effect from the dates on which they had been promoted and the excess pay
drawn by them should be recovered (unless the amount is waived by
Government).
Government do not, however, consider that the
consequential vacancies should be filled with retrospective effect from the
dates of reversion of those who had been promoted otherwise than in
accordance with rules. They have, therefore, decided that these vacancies
should be filled from the dates on which the promotions are actually made.
(G.O. Pres No.847 dt: 27.5.1935).
As per Art 320(i) of the Constitution of India, “It shall be the duty of the
Union Public Service Commission to conduct examinations for
appointments to the services of the Union.
Therefore scrutiny of the eligibility of the candidate is a constitutional
duty thrust upon the UPSC.”
Upon this Shri Rajesh, Jt. Secretary (AIS), UPSC in his order of 19.12.08 has
held as follows:
“In this regard, it is mentioned that the factual positions of the matter was
conveyed to him by the CPIO vide letter dated 7.11.2008. Further, the
attention of the appellant is also drawn to the decision of the CIC
(No.1404/ICA/2007 dated 2.11.2007) in case of Prakash Khemchandan,
wherein it has been decided that “Under Section 2(6) of the Act, an
information seeker should clearly specify the required information, which
should be available in any material form. He is not expected to elicit
views of the CPIU through any form of query, as attempted by the
appellant”. Further the queries raised by Shri Pandian have already been
replied by the CPIO, as mentioned above.
This has brought Shri A.Pandian before us in second appeal with the following
prayer:
“The Hon’ble CIC may please to issue direction to UPSC to chancel the
selection of Thiru K.Ragubathi I.A.S.”
Appellant has based this plea on the following reasoning:
“The preamble of RTI act says “whereas democracy requires an
informed citizenry and transparency of information which are vital to its
functioning and also to contain and to hold Government and there
instrumentalities accountable to the governed”.
At the time of assuming office the Hon’ble Central Information
Commission and members are administered oath of office which includes
the phrase “I will uphold the constitution and the laws”. This is as per
section 13(3) of the RTI act. (The first schedule). Hon’ble CIC is
entrusted with the task of upholding the constitution and the laws.
Therefore Hon’ble CIC may direct the UPSC to cancel the said selection.
So that case law quoted in first appeal and rules quoted may be upheld.”
In her response to the appeal notice of the Commission, present CPIO Ms.
Rashmi Sinha, DS, UPSC has after detailed examination of the processing of this
application concluded as follows:
“It is submitted that vide the decision in appeal No.1404/IC(A)/2007
(F.No.CIC/MA/A/2007/00585) dated 02.11.2007 this Hon’ble CIC has
held that “under Section 2(f) of the Act, an information seeker should
clearly specify the required information which should be available in any
material form. He is not expected to elicit views of the CPIO through any
form of query, as attempted by the appellant”. However, the factual
position of the matter has already been informed to the appellant. Further,
forwarding of names of eligible Non State Civil Service officers, keeping
in view the specific provisions of the IAS (Appointment by Selection)
Regulations, 1997, is in the exclusive purview of the State Govt. and the
promotion of State Service officers in the State Service is guided by the
relevant Service Rules and is in the domain of the State Govt. concerned.
Therefore, no further information in this regard could be furnished to him
on part of the U.P.S.C.
In view of the submissions made above it is humbly requested that this
Hon’ble CIC may be pleased to dismiss the instant appeal as devoid of
merit.”
The appeal was heard through videoconference on 2.8.2010. The following are
present.
Appellant: (at NIC Studio, Dindigul)
Shri A.Pandian
Respondents (at CIC chambers, Delhi)
Shri Sunil Kumar, JS, UPSC & AA
Ms. Rashmi Sinha, DS, CPIO, UPSC
The attention of the appellant Shri Pandian was invited to Section 2 (j) of the
RTI Act 2005. As per that clause only information that can be accessed under the RTI
Act can be disclosed. Even where the information provided to expose wrongdoing, the
remedy for that will not lie with the Information Commission but with the appropriate
court of law. Appellant Shri Pandian agreed that he will now agitate the matter before
the appropriate court of law.
DECISION NOTICE
The information sought by appellant in his initial application has been fully
provided in the response of CPIO Ms. Richa Mishra, who has also specifically described
the reasons on the basis of which the selection committee recommended the name of
Thiru. Raghubathi. Under the circumstances, and in light of the acceptance by appellant
Shri K. Pandian in the hearing as described above, we find this appeal without merit
under the RTI Act and it is hereby dismissed.
Announced in the hearing. Notice of this decision be given free of cost to the
parties.
***
CENTRAL INFORMATION COMMISSION
Appeal No.CIC/AT/A/2010/000757
D.D. 12.11.2010
Hon’ble Mr. A.N.Tiwari, Chief Information Commissioner
Shri D.P.Bhatia
Vs.
Customs and Central Excise, New Delhi
…
Appellant
…
Respondent
R.T.I. Act:
Whether file notings in vigilance files are exempt from disclosure under Section 8(1)(g)
RTI Act? – Yes.
Shri D.P.Bhatia had filed a complaint against one Addl. Commissioner of
Customs and Central Excise, New Delhi (Public Authority) – Through RTI Application
he wanted to know the action taken on his complaint along with photocopies of entire
correspondence/note sheets – CPIO and Appellate Authority of the public authority
denied him copies of correspondence and note sheets under Section 8(1)(g) of the RTI
Act – Respondent reiterated that disclosure of the details of the file noting was potentially
risky for the officers who made noting in the file in a matter as sensitive as the complaint
of one officer of the public authority against another, argued that since the officers
making the file notings were recording their notes in confidence in a confidential file,
they answered the description of those who were the source of an information for
decision making purposes and must be provided the protection under Section 8(1)(g) of
the RTI Act.
CIC decided that protection needs to be provided to the officers of sensitive
Department such as the vigilance in performing their duties from the probing eyes of
outsiders hence no disclosure of file noting in Vigilance file is to be made.
ORDER
This matter came for hearing on 03.11.2010 pursuant to Commission’s notice
dated 13.10.2010.
Appellant was present in person, while the respondents were
represented by Dr. Shobhit Jain, Deputy Commissioner & CPIO and Shri Sanjeev Yadav,
Vigilance Officer.
2.
Appellant had filed a complaint dated 31.10.2009 against one, Shri Sansar
Chand, Additional Commissioner before the Vigilance Department of the public
authority and now through his RTI-application dated 19.02.2010, desired to know the
action taken on his complaint from the date of receipt till 19.02.2010 – date-wise.
Additionally, he wanted to receive photocopies of the entire correspondence made in this
regard as well as copies of the note-sheets.
3.
In the second-appeal, it is the appellant’s plea that the CPIO and the Appellate
Authority, through their respective communications/orders dated 18.03.2010 and
22.04.2010, had denied him copies of correspondence and note-sheets, on the ground that
disclosing the names of the offices dealing with the matter were barred under Section
8(1)(g) of the RTI Act.
4.
Appellant had called the reasons spelt-out by the respondents as ‘baseless’. He
claims that he had a right to know as to what action was taken by the public authority
against the Additional Commissioner of Central Excise and Customs in respect of whom
appellant had filed a complaint.
5.
Respondents, while reiterating their point that disclosure of the details of the
file-noting was potentially risky for the officers who made notings in the file in a matter
as sensitive as the complaint of one officer of the public authority against another, argued
that since the officers making the file-notings were recording their notes in confidence in
a confidential file, they answered the description of those who were the source of an
information for decision-making purposes and must be provided the protection under
Section 8(1)(g) of the RTI Act.
6.
Respondents further pointed out that the original complaint-petition of the
appellant having been initially processed by the Vigilance Department was transmitted on
04.2.2010 to the HRD Department of the public authority, where all action in such
matters usually lie. They, therefore, claimed that there was no further action at their end
in the matter of this complaint of the appellant.
7.
The short-point for decision is whether file-notings in vigilance files should be
allowed to be disclosed.
8.
This matter is covered by the ratio of Commission’s decision in K.G.Bablani
Vs. DG Vigilance, Customs & Central Excise; Appeal No.CIC/AT/A/2009/0617; Date of
Decision: 16.09.2009, wherein it was held that the file-notings in vigilance files cannot be
authorized to be disclosed as these amounted to information confidentially held by the
public authority and thereby came within the scope of Section 11(1) read with Section
2(n) of the RTI Act. The operating portion of that order read as follows:6.
This is not the first case in which employees of a public
authority have demanded disclosure of file-notings in matters of
vigilance and disciplinary enquiries held against them. In most cases,
the purpose is to find out the identity of those officers who had taken
favourable and those who had taken unfavourable view of the conduct of
such employees in recording the file-notes. The employees are aware
that it is these notes, which eventually lead to decisions for, or against
them by the competent authority and want, for their own different
purposes, to gain access to the identities of those recording the notes as
well as the notes recorded to pursue their agendas about, or against the
officers recording those notes. It has happened in a few cases that even
bona-fide comments made in such sensitive files by officers when
disclosed to the person in respect of whom such comments were made,
brought retribution to the officer recording the notes in the shape of a
court preceding, a notice for damages and so on. In some cases, even
intimidation was resorted to. Frequently, offices recording such notes
were juniors to those in respect of whom the case was being processed.
Naturally no officer recording the note wanted his identity to be
disclosed lest he became the victim himself later at the hands of the
senior person, whose conduct it became his duty to examine at some
stage. Confidentiality of note-files, therefore, is an entirely wholesome
principle conducive to good governance. Any compromise with
objectivity in processing matters extant in the file, is potentially
damaging to governance by exposing those entrusted with the charge of
processing the matter to, undue, and sometimes, intimidating, scrutiny
by interested parties.
7.
It is my belief, therefore, that a public authority which is
authorized to hold file-notings in sensitive giles, such as vigilance and
disciplinary matters, confidential under the provisions of Section 124 of
the Indian Evidence Act can also hold such documents (file-notings)
confidential under Section 11(1) of RTI Act read with Section 2(n), on
satisfying certain conditions. Section 2(n) of RTI Act, which states that
………… renders a public authority holding the information a “thirdparty” in respect of the confidential information it holds. Since the
information satisfies the requirement of being a third-party information,
it being confidential as well as it comes within the scope of Section
11(1) ……………..
8.
Since the matter comes squarely within the purview of Section
11(1) being a confidential third-party information, the reason why it can
be disclosed is that it is in public interest and, not otherwise. It needs to
be proved that public interest supersedes the protected interest if such
information were to be disclosed……………..
10.
I do not think that applying Section 10(1) and hiding the name
of the authors of the file-notings will serve any purpose. As has been
rightly pointed out by the respondents, even without the authors’ names,
the identity of the authors of the notes could be disclosed by reference to
the hierarchies through which the file passed as well as the handwriting
in which the notes were recorded.”
9.
It was also pointed out in that order that protection needs to be provided to the
officers of sensitive Department such as the vigilance in performing their duties from the
probing eyes of outsiders.
10.
Consistent with the above, it is held that there shall be no disclosure in respect
of the above query.
11.
Matter disposed of with these directions.
12.
Copy of this direction be sent to the parties.
***
CENTRAL INFORMATION COMMISSION
Appeal No.CIC/SG/C/2010/001124
D.D. 16.11.2010
Hon’ble Mr. Shailesh Gandhi, Chief Information Commissioner
Mr. M.Kumar
…
Vs.
PIO, Centralized RTI Cell, New Delhi…
Complainant
Respondent
R.T.I. Act:
Whether IPO of higher denomination sent against application fee of Rs.10/- can be
returned to the complainant? – Yes.
Facts and the background of the case is that Shri M.Kumar filed two RTI
applications with the PIO, NDMC enclosing therewith IPO of Rs.100/- and Rs.50/respectively as RTI application fee for getting some information - The PIO returned the
original IPO of Rs.100/- and Rs.50/- to the complainant - Aggrieved by the same, the
complainant filed complaint under Section 18 of RTI with the CIC.
CIC in its above decisions observed that in the instant cases the complainant
was required to deposit an application fee of Rs.10/- only - Where the complainant
deposited an application fee exceeding Rs.10/- for covering the expenses incurred in
providing the information, the PIO was not bound to accept the same - As per the
applicable law, the PIO was required to accept IPO of Rs.10/- as application fee and IPO
of any other sum was liable to be returned - Further, such practice on the part of RTI
applicants cannot be encouraged as PIOs cannot be asked to maintain ledgers for
additional fees received from the applicants and refund them from time to time - Hence
the complaint is dismissed.
Facts arising from the Complaint:
The Complainant filed a RTI application with the PIO & Joint Director (EstateI) on 23.08.2010 asking for certain information. The Central APIO, Centralized RTI
Cell, P.R. Department, vide letter dated 26.08.2010, observed that IPO of Rs.100 was
paid as RTI application fee and therefore, the original IPO of Rs.100 was returned to the
Complainant. Aggrieved by the same the complainant filed a complaint under Section 18
of the RTI Act with the Commission.
Observations:
Section 6(1) of the RTI Act stipulates that a person who desires to obtain any
information from a public authority shall make a request in writing along with the
prescribed fees. Rule 3 of the Right to Information (Regulation of Fee and Cost) Rules,
2005 (the “RTI Rules”) lays down that a request for information under Section 6(1) of the
RTI Act shall be accompanied by an application fee of Rs.10 by way of cash against
proper receipt or by demand draft or banker’s cheque or IPO payable to the Accounts
Officer of the public authority.
In the instant case, the complainant was required to deposit an application fee of
Rs.10 only. Where the complainant deposited an application fee exceeding Rs.10 for
covering the expenses incurred in providing the information, the PIO was not bound to
accept the same. As per the applicable law, the PIO was required to accept IPO of Rs.10
as application fee and IPO of any other sum was liable to be returned. Further, such
practice on the part of the RTI applicants cannot be encouraged as PIOs cannot be asked
to maintain ledgers for additional fees received from the applicants and refund them from
time to time.
Decision:
The Complaint is dismissed.
Notice to this decision be given free of cost to the parties.
***
CENTRAL INFORMATION COMMISSION
Decision No.CIC/ST/A/2011/000704/12914
Appeal No.CIC/SG/A/2011/000704
D.D. 17.06.2011
Hon’ble Mr. Shailesh Gandhi, Information Commissioner
Mr. Alex K.A.
Vs.
Ms Kavit Jorge, PIO & RPFC-II
…
Appellant
…
Respondent
R.T.I. Act:
Whether unsigned application filed under RTI Act seeking information can be considered
as a valid application? – No.
The appellant filed an unsigned RTI application to the respondent seeking some
information on the working of EPF Organisation – No information was provided by the
CPIO as the application was not signed by the appellant – The 1st Appellate Authority
also held that an unsigned application cannot be treated as a valid application under RTI
Act – Appellant filed 2nd appeal before the Central Information Commission (CIC) - CIC
upheld the action of CPIO by holding that an application under RTI Act is not signed by
the appellant it cannot be considered as a valid application.
Information sought:
Details of Information sought
A. What was the procedure to EPF membership? Forms, enclosures required to be
submitted by the member and employer, Age limit and quitting of membership.
B. What was the responsibility from an employer’s side to give membership, remittance
of subscription percentage, contribution percentage any other amount required to be
remitted by the employer and its order number with date.
C. On retirement, resignation, death the name/number forms enclosure to be submitted by
the member/nominee/employer in detail.
D. Any intimation is given by the EPF office regarding members pension dates, forms to
be submitted, enclosures required etc to the member or employer. Any such orders are
then. Any citizen’s charter is applicable in the EPF office.
E. Any criterion fixed for timely payment of PF amount and the pension on receipt of
application. Give its order no. with date. Give date of receipt of F-19 and EPF Pension
application and sanction of pension with PPo np. With date for the last one year to assess
the time required for getting pension. Also give the name of retire/resigned employees of
KR 2640 from F-10 submitted by the employee for the above period.
F. What is the procedure to get status of application submitted by the members Any
acknowledgment is given by the EPF officer. Any redressal mechanism. Give Order No.
with details.
G. What is the procedure to process all PF application received from employer/member?
Any acknowledgement is given to employer/member/Nominee. What rule prevent such a
system? Given order no and date. Any training is arranged by the EPF office amount the
changing rules to the employer/members. Any circulars issued in regard to them. If so
how many trainings have been conducted and how many orders have been issued.
H. How many registered letters received from employers in the last one year? How
many disposed and how many pending? How many registered letters received from
employer code KR/TVM/2640 with no. of applications enclosed, title name of
application, name of applicant and its present status etc.
I. How is the Scheme certificate prepared? An application is necessary for the same?
How many scheme certificates delivered by ordinary post/UCP or Regd. Post. any
certificate scheme is issued without submitting application in employer code
KR/TVM/2640. Any scheme certificate in the above code lost/returned unclaimed after
pension scheme is implemented.
J. Any communication is given to the employer on issue of every PPO or scheme
certificate to the member. If not, what is the order with date. How many different kinds
of EPG application pending at the Trivandrum office in relation to KR/TVM/2640.
K. How many pension application lost from the Trivandum office with respect to
KR/TVM/26040 w.e.f. January 2009.
L. How many e-mail complaints received regarding non receipt of pension even after
submitting application w.e.f. January 2010 in relation to KR/TVM/2640. Name of email
complainant, content, its present position and any reply given to them.
M. How many scheme certificate application received and certificate issued to the
members of KR/TVM/2640. What is the method of delivery scheme certificate by
ordinary post, UCP or registered post. Give Order No. with date.
N. Any amount is remitted by the employer to EPF office towards administrative charges
of its members account dealings. What is the rule, amount with order No. and date. Any
right is there from management aide to give better EPF service. How is pension papers
returned to the employer. Any directions. Give order no. with date.
O. Name address, phone no of appellate authority.
P. (not a query – general observation made).
Q. /2640 how many pensions pension has been stopped? How many not certificate not
submitted. How many mobile connection issued to staff of EPF Trivandum office. If
supplied from office give its numbers, e-mails id of EPF office Trivandum, Chochin and
Delhi.
R. The details of unanswered above question with reasons. Any unclear part may be
linked with KR/TVM/2640 and period not specified may be related w.e.f. January. The
RTI Act is applicable to EPF Organisations or not?
Reply of the Public Information Officer (PIO):
No information was provided as the original RTI Application was not signed by the
appellant.
Grounds for the First Appeal:
The appellant did not receive any information. The grounds for rejection of the RTI
Application were not justified.
Order of the First Appellate Authority (FAA):
An unsigned application cannot be treated as proper under the RTI Act. The applicant
had been informed of this anomaly. The proof of postage exist with the PIO. Appeal
disposed.
Ground of the Second Appeal:
The applicant submitted the RTI Application to the EPF office Trivandrum, through the
office of the Prime Minister. The PMO transferred the application to the CPIO, Ministry
of Labour and Employment on 17.09.2010. In the meanwhile (04/10/2010), the applicant
requested the EPF, Trivandrum to add his e-mail id and mobile no. to the original
application. No reply was received. On 24.01.2011, he received intimation of his RTI
application being rejected and returned on account of unsigned application.
The
appellant claims this to be false.
Relevant facts emerging during hearing:
The following were present:
Appellant: Mr. Alex K.A. on video conference from NIC-Pathanamthitta Kerala studio;
Respondent: Ms Kavit Jorge, PIO & RPFC-II on video conference from NIC-Kerala
State Center Studio;
The appellant has sent an unsigned RTI application which has been rejected by the PIO.
He claims to have sent email and another application later. Since the RTI application was
not signed by him it cannot be considered a valid RTI application. Also the Commission
feels that sending RTI application to the PM’s office when the information is sought
about PF is not right and a citizen should try and file RTI application where he believes
the information exist.
Decision
The Appeal is disposed.
***
IN THE SUPREME COURT OF INDIA
CIVIL APPELALTE JURISDICTION
CIVIL APPEAL NO.6454 OF 2011
[Arising out of SLP [C] No.7526/2009]
D.D. 09.08.2011
Hon’ble Mr. Justice R.V.Raveendran &
Hon’ble Mr. Justice A.K.Patnaik
Central Board of Secondary Education & Anr. … Appellants
Vs.
Aditya Bandopadhyay & Ors.
… Respondents
With
CA No. 6456 of 2011 (@ SLP (C) No.9755 of 2009)
CA Nos.6457-6458 of 2011 (@ SLP (C) Nos.11162-11163 of 2009)
CA No.6461 of 2011 (@ SLP (C) No.11670 of 2009)
CA Nos.6462 of 2011 (@ SLP (C) No.13673 of 2009)
CA Nos.6464 of 2011 (@ SLP (C) No.17409 of 2009)
CA Nos. 6459 of 2011 (@ SLP (C) No.9776 of 2010)
CA Nos.6465-6468 of 2011 (@ SLP (C) Nos.30858-30861 of 2009)
R.T.I. Act:
Whether answer scripts are exempt from disclosure under Section 8(1)(e) & (g) RTI Act?
– No.
Whether in view of the decision of the Supreme Court in Maharashtra State Board of
Secondary Education - 1984 (4) SCC 27 and other cases on the issue an examinee can
seek inspection of his answer books or certified copies thereof? – No.
Whether the information namely seeking inspection of answer books or certified copies
thereof is exempt from disclosure under Section 8(1)(e) & (g) of RTI Act? – No.
Examinees in respect of academic examinations conducted by School Boards etc.,
and in respect of recruitment examinations conducted by PSCs sought copies of their
answer scripts under RTI Act - On being denied they approached the respective
Information Commissions which directed to furnish copies - Against the said directions
writ petitions were filed by the Examination Conducting Bodies before respective High
Courts which also ruled in favour of examinees - Aggrieved by the same, Examination
Conducting Bodies approached the Supreme Court in the above cases – The Supreme
Court after examining the relevant provisions of RTI Act, Section 8(1)(e) & (g) in
particular under which the request of examinees for copies of answer scripts was rejected
held that there is no fiduciary relationship between the examination conducting bodies
and evaluators and as such examination conducting bodies cannot refuse copies of answer
scripts - As far as examinees are concerned even if it is held that there is fiduciary
relationship between them and the examination conducting bodies since the examinees
are seeking inspection or copies of their own evaluated answer scripts the same cannot be
refused - Regarding Section 8(1)(g) which provides for exemption from disclosure of
information which endangers the life or physical safety of examiners etc., the Supreme
Court held that such information can be given after masking the identity of examiners etc.
– Consequently, the appeals were disposed of
Held:
Section 22 of RTI Act has overriding effect and will prevail over the provisions of
the bye-laws/rules of the examining bodies in regard to examinations. As a result, unless
the examining body is able to demonstrate that the answer-books fall under the exempted
category of information described in clause (e) of section 8(1) of RTI Act, the examining
body will be bound to provide access to an examinee to inspect and take copies of his
evaluated answer-books, even if such inspection or taking copies is barred under the
rules/bye-laws of the examining body governing the examinations. Therefore, the
decision of this Court in Maharashtra State Board and subsequent decisions following the
same, will not affect or interfere with the right of the examinee seeking inspection of
answer-books or taking certified copies thereof.
Further held:
Even though there is fiduciary relationship between examination conducting body
and the examinee there is no question of the fiduciary (examination conducting body)
withholding information relating to the beneficiary (examinee), from the beneficiary
himself.
Further held:
As far as the examiner is concerned it cannot be said that the examining body
holds the evaluated answer books in a fiduciary relationship qua the examiner.
Therefore, the examining body does not hold the evaluated answer books in a fiduciary
relationship. The examining body entrusts the answer-books to an examiner for
evaluation and pays the examiner for his expert service. The work of evaluation and
marking the answer-book is an assignment given by the examining body to the examiner
which he discharges for a consideration. Therefore, the examining body is not in the
position of a fiduciary with reference to the examiner. On the other hand, when an
answer-book is entrusted to the examiner for the purpose of evaluation, for the period the
answer-book is in his custody, the examiner is in the position of a fiduciary with
reference to the examining body and is barred from disclosing the contents of the answerbook or the result of evaluation to anyone other than the examining body. The examiner
does not have any copy-right or proprietary right or confidentiality right in regard to the
evaluation.
Further held:
The right to access information can not extend beyond the period during which
the examining body is expected to retain the answer-books.
Further held:
R.T.I. Act provides access to all information that is available and existing. A
public authority is not required to furnish information which requires drawing of
inferences and/or making of assumptions. It is also not required to provide ‘advice’ or
‘opinion’ to an applicant, nor required to obtain and furnish any ‘opinion’ or ‘advice’ to
an applicant.
Further held:
The Supreme Court has sounded a word of caution against misuse and abuse of
RTI Act in the name of transparency and accountability in the following observation:
“37. ………….
Indiscriminate and impractical demands or
directions under RTI Act for disclosure of all and sundry information
(unrelated to transparency and accountability in the functioning of public
authorities and eradication of corruption) would be counter-productive as
it will adversely affect the efficiency of the administration and result in the
executive getting bogged down with the non-productive work of collecting
and furnishing information. The Act should not be allowed to be misused
or abused, to become a tool to obstruct the national development and
integration, or to destroy the peace, tranquility and harmony among its
citizens. Nor should it be converted into a tool of oppression or
intimidation of honest officials striving to do their duty. The nation does
not want a scenario where 75% of the staff of public authorities spends
75% of their time in collecting and furnishing information to applicants
instead of discharging their regular duties. The threat of penalties under
the RTI Act and the pressure of the authorities under the RTI Act should
not lead to employees of a public authorities prioritizing ‘information
furnishing’, at the cost of their normal and regular duties.”
Cases Referred:
(1975) 4 SCC 428 - State of Uttar Pradesh v. Raj Narain
1984 (4) SCC 27 - Maharashtra State Board of Secondary Education vs. Paritosh B.
Sheth
(1997) 4 SCC 306 – Dinesh Trivedi v. Union of India
(2004) 2 SCC 476 – People’s Union for Civil Liberties v. Union of India
2004 (6) SCC 714 - Parmod Kumar Srivastava vs. Chairman, Bihar PAC
2004 (13) SCC 383 - Board of Secondary Education vs. Pavan Ranjan P
2007 (1) SCC 603 - Board of Secondary Education vs. S
2007 (8) SCC 242 - Secretary, West Bengal Council of Higher Secondary Education vs. I
Dass
(2009) 8 SCC 483 – Bihar School Examination Board vs. Suresh Prasad Sinha
JUDGMENT
R.V.RAVEENDRAN, J.
Leave granted. For convenience, we will refer to the facts of the first case.
2.
The first respondent appeared for the Secondary School Examination, 2008
conducted by the Central Board of Secondary Education (for short ‘CBSE’ or the
‘appellant’). When he got the mark sheet he was disappointed with his marks. He thought
that he had done well in the examination but his answer-books were not properly valued
and that improper valuation had resulted in low marks. Therefore he made an application
for inspection and re-evaluation of his answer-books. CBSE rejected the said request by
letter dated 12.7.2008. The reasons for rejection were:
(i)
The information sought was exempted under Section 8(1)(e) of RTI Act
since CBSE shared fiduciary relationship with its evaluators and maintain
confidentiality of both manner and method of evaluation.
(ii)
The Examination Bye-laws of the Board provided that no candidate shall
claim or is entitled to re-evaluation of his answers or disclosure or
inspection of answer book(s) or other documents.
(iii) The larger public interest does not warrant the disclosure of such
information sought.
(iv) The Central Information Commission, by its order dated 23.4.2007 in
appeal no. ICPB/A-3/CIC/2006 dated 10.2.2006 had ruled out such
disclosure.”
3.
Feeling aggrieved the first respondent filed W.P. No.18189(W)/2008 before the
Calcutta High Court and sought the following reliefs : (a) for a declaration that the action
of CBSE in excluding the provision of reevaluation of answer-sheets, in regard to the
examinations held by it was illegal, unreasonable and violative of the provisions of the
Constitution of India; (b) for a direction to CBSE to appoint an independent examiner for
reevaluating his answer-books and issue a fresh marks card on the basis of reevaluation;
(c) for a direction to CBSE to produce his answer-books in regard to the 2008 Secondary
School Examination so that they could be properly reviewed and fresh marks card can be
issued with re-evaluation marks; (d) for quashing the communication of CBSE dated
12.7.2008 and for a direction to produce the answer-books into court for inspection by the
first respondent. The respondent contended that section 8(1)(e) of Right to Information
Act, 2005 (‘RTI Act’ for short) relied upon by CBSE was not applicable and relied upon
the provisions of the RTI Act to claim inspection.
4.
CBSE resisted the petition. It contended that as per its Bye-laws, reevaluation and
inspection of answer-books were impermissible and what was permissible was only
verification of marks. They relied upon the CBSE Examination Bye-law No.61, relevant
portions of which are extracted below:
“61. Verification of marks obtained by a Candidate in a subject
(i) A candidate who has appeared at an examination conducted by the
Board may apply to the concerned Regional Officer of the Board for
verification of marks in any particular subject. The verification will be
restricted to checking whether all the answer's have been evaluated and
that there has been no mistake in the totalling of marks for each question
in that subject and that the marks have been transferred correctly on the
title page of the answer book and to the award list and whether the
supplementary answer book(s) attached with the answer book mentioned
by the candidate are intact. No revaluation of the answer book or
supplementary answer book(s) shall be done.
(ii) Such an application must be made by the candidate within 21 days
from the date of the declaration of result for Main Examination and 15
days for Compartment Examination.
(iii) All such applications must be accompanied by payment of fee as
prescribed by the Board from time to time.
(iv) No candidate shall claim, or be entitled to, revaluation of his/her
answers or disclosure or inspection of the answer book(s) or other
documents.
Xxxx
(vi) In no case the verification of marks shall be done in the presence of
the candidate or anyone else on his/her behalf, nor will the answer books
be shown to him/her or his/her representative.
(vii) Verification of marks obtained by a candidate will be done by the
officials appointed by or with the approval of the Chairman.
(viii) The marks, on verification will be revised upward or downward, as
per the actual marks obtained by the candidate in his/her answer book.
Xxxx
62. Maintenance of Answer Books
The answer books shall be maintained for a period of three months and
shall thereafter be disposed of in the manner as decided by the Chairman
from time to time.”
(emphasis supplied)
CBSE submitted that 12 to 13 lakhs candidates from about 9000 affiliated schools across
the country appear in class X and class XII examinations conducted by it and this
generates as many as 60 to 65 lakhs of answer books; that as per Examination Bye-law
No.62, it maintains the answer books only for a period of three months after which they
are disposed of. It was submitted that if candidates were to be permitted to seek reevaluation of answer books or inspection thereof, it will create confusion and chaos,
subjecting its elaborate system of examinations to delay and disarray. It was stated that
apart from class X and class XII examinations, CBSE also conducts several other
examinations (including the All India Pre-Medical Test, All India Engineering Entrance
Examination and Jawahar Navodaya Vidyalaya’s Selection Test). If CBSE was required
to re-evaluate the answer-books or grant inspection of answer-books or grant certified
copies thereof, it would interfere with its effective and efficient functioning, and will also
require huge additional staff and infrastructure. It was submitted that the entire
examination system and evaluation by CBSE is done in a scientific and systemic manner
designed to ensure and safeguard the high academic standards and at each level utmost
care was taken to achieve the object of excellence, keeping in view the interests of the
students. CBSE referred to the following elaborate procedure for evaluation adopted by
it:
“The examination papers are set by the teachers with at least 20 years of
teaching experience and proven integrity. Paper setters are normally
appointed from amongst academicians recommended by then Committee
of courses of the Board. Every paper setter is asked to set more than one
set of question papers which are moderated by a team of moderators who
are appointed from the academicians of the University or from amongst
the Senior Principals. The function of the moderation team is to ensure
correctness and consistency of different sets of question papers with the
curriculum and to assess the difficulty level to cater to the students of
different schools in different categories. After assessing the papers from
every point of view, the team of moderators gives a declaration whether
the whole syllabus is covered by a set of question papers, whether the
distribution of difficulty level of all the sets is parallel and various other
aspects to ensure uniform standard. The Board also issues detailed
instructions for the guidance of the moderators in order to ensure uniform
criteria for assessment.
The evaluation system on the whole is well organized and fool-proof. All
the candidates are examined through question papers set by the same
paper setters. Their answer books are marked with fictitious roll numbers
so as to conceal their identity. The work of allotment of fictitious roll
number is carried out by a team working under a Chief Secrecy Officer
having full autonomy. The Chief Secrecy Officer and his team of
assistants are academicians drawn from the Universities and other
autonomous educational bodies not connected with the Board. The Chief
Secrecy Officer himself is usually a person of the rank of a University
professor. No official of the Board at the Central or Regional level is
associated with him in performance of the task assigned to him. The codes
of fictitious roll numbers and their sequences are generated by the Chief
Secrecy Officer himself on the basis of mathematical formula which
randomize the real roll numbers and are known only to him and his team.
This ensures complete secrecy about the identification of the answer book
so much so, that even the Chairman, of the Board and the Controller of
Examination of the Board do not have any information regarding the
fictitious roll numbers granted by the Chief Secrecy Officer and their real
counterpart numbers.
At the evaluation stage, the Board ensures complete fairness and
uniformity by providing a marking scheme which is uniformity applicable
to all the examiners in order to eliminate the chances of subjectivity.
These marking schemes are jointly prepared at the Headquarters of the
Board in Delhi by the Subject Experts of all the regions. The main purpose
of the marking scheme is to maintain uniformity in the evaluation of the
answer books.
The evaluation of the answer books in all major subjects including
mathematics, science subjects is done in centralized “on the spot”
evaluation centers where the examiners get answer book in interrupted
serial orders. Also, the answer books are jumbled together as a result of
which the examiners, say in Bangalore may be marking the answer book
of a candidate who had his examination in Pondicherry, Goa, Andaman
and Nicobar islands, Kerala, Andhra Pradesh, Tamil Nadu or Karnataka
itself but he has no way of knowing exactly which answer book he is
examining. The answer books having been marked with fictitious roll
numbers give no clue to any examiner about the state or territory it
belongs to. It cannot give any clue about the candidate’s school or centre
of examination. The examiner cannot have any inclination to do any
favour to a candidate because he is unable to decodify his roll number or
to know as to which school, place or state or territory he belongs to.
The examiners check all the questions in the papers thoroughly under the
supervision of head examiner and award marks to the sub parts
individually not collectively. They take full precautions and due attention
is given while assessing an answer book to do justice to the candidate.
Reevaluation is administratively impossible to be allowed in a Board
where lakhs of students take examination in multiple subjects.
There are strict instructions to the additional head examiners not to allow
any shoddy work in evaluation and not to issue more than 20-25 answer
books for evaluation to an examiner on a single day. The examiners are
practicing teachers who guard the interest of the candidates. There is no
ground to believe that they do unjust marking and deny the candidates
their due. It is true that in some cases totaling errors have been detected at
the stage of scrutiny or verification of marks. In order to minimize such
errors and to further strengthen and to improve its system, from 1993
checking of totals and other aspects of the answers has been trebled in
order to detect and eliminate all lurking errors.
The results of all the candidates are reviewed by the Results Committee
functioning at the Head Quarters. The Regional Officers are not the
number of this Committee. This Committee reviews the results of all the
regions and in case it decides to standardize the results in view of the
results shown by the regions over the previous years, it adopts a uniform
policy for the candidates of all the regions. No special policy is adopted
for any region, unless there are some special reasons. This practice of
awarding standardized marks in order to moderate the overall results is a
practice common to most of the Boards of Secondary Education. The
exact number of marks awarded for the purpose of standardization in
different subjects varies from year to year. The system is extremely
impersonalized and has no room for collusion infringement. It is in a word
a scientific system.”
CBSE submitted that the procedure evolved and adopted by it ensures fairness and
accuracy in evaluation of answer-books and made the entire process as foolproof as
possible and therefore denial of re-evaluation or inspection or grant of copies cannot be
considered to be denial of fair play or unreasonable restriction on the rights of the
students.
5.
A Division Bench of the High Court heard and disposed of the said writ petition
along with the connected writ petitions (relied by West Bengal Board of Secondary
Education and others) by a common judgment dated 5.2.2009. The High Court held that
the evaluated answer-books of an examinee writing a public examination conducted by
statutory bodies like CBSE or any University or Board of Secondary Education, being a
‘document, manuscript record, and opinion’ fell within the definition of “information” as
defined in section 2(f) of the RTI Act. It held that the provisions of the RTI Act should be
interpreted in a manner which would lead towards dissemination of information rather
than withholding the same; and in view of the right to information, the examining bodies
were bound to provide inspection of evaluated answer books to the examinees.
Consequently it directed CBSE to grant inspection of the answer books to the examinees
who sought information. The High Court however rejected the prayer made by the
examinees for re-evaluation of the answer-books, as that was not a relief that was
available under RTI Act. RTI Act only provided a right to access information, but not for
any consequential reliefs. Feeling aggrieved by the direction to grant inspection, CBSE
has filed this appeal by special leave.
6.
Before us the CBSE contended that the High Court erred in (i) directing CBSE to
permit inspection of the evaluated answer books, as that would amount to requiring
CBSE to disobey its Examination Bye-law 61(4), which provided that no candidate shall
claim or be entitled to re-evaluation of answer books or disclosure/inspection of answer
books; (ii) holding that Bye-law 61(4) was not binding upon the examinees, in view of
the overriding effect of the provisions of the RTI Act, even though the validity of that
bye-law had not been challenged; (iii) not following the decisions of this court in
Maharashtra State Board of Secondary Education vs. Paritosh B. Sheth [1984 (4) SCC
27], Parmod Kumar Srivastava vs. Chairman, Bihar PAC [2004 (6) SCC 714], Board of
Secondary Education vs. Pavan Ranjan P [2004 (13) SCC 383], Board of Secondary
Education vs. S [2007 (1) SCC 603] and Secretary, West Bengal Council of Higher
Secondary Education vs. I Dass [2007 (8) SCC 242]; and (iv) holding that the examinee
had a right to inspect his answer book under section 3 of the RTI Act and the examining
bodies like CBSE were not exempted from disclosure of information under section
8(1)(e) of the RTI Act. The appellants contended that they were holding the
“information” (in this case, the evaluated answer books) in a fiduciary relationship and
therefore exempted under section 8(1)(e) of the RTI Act.
7.
The examinees and the Central Information Commission contended that the object
of the RTI Act is to ensure maximum disclosure of information and minimum
exemptions from disclosure; that an examining body does not hold the evaluated answer
books, in any fiduciary relationship either with the student or the examiner; and that the
information sought by any examinee by way of inspection of his answer books, will not
fall under any of the exempted categories of information enumerated in section 8 of the
RTI Act. It was submitted that an examining body being a public authority holding the
‘information’, that is, the evaluated answer-books, and the inspection of answer-books
sought by the examinee being exercise of ‘right to information’ as defined under the Act,
the examinee as a citizen has the right to inspect the answer-books and take certified
copies thereof. It was also submitted that having regard to section 22 of the RTI Act, the
provisions of the said Act will have effect notwithstanding anything inconsistent in any
law and will prevail over any rule, regulation or bye law of the examining body barring
or prohibiting inspection of answer books.
8.
On the contentions urged, the following questions arise for our consideration :
(i)
Whether an examinee’s right to information under the RTI Act
includes a right to inspect his evaluated answer books in a public
examination or taking certified copies thereof?
(ii)
Whether the decisions of this court in Maharashtra State Board of
Secondary Education [1984 (4) SCC 27] and other cases referred to
above, in any way affect or interfere with the right of an examinee
seeking inspection of his answer books or seeking certified copies
thereof?
(iii)
Whether an examining body holds the evaluated answer books “in a
fiduciary relationship” and consequently has no obligation to give
inspection of the evaluated answer books under section 8 (1)(e) of RTI
Act?
(iv)
If the examinee is entitled to inspection of the evaluated answer books
or seek certified copies thereof, whether such right is subject to any
limitations, conditions or safeguards?
Relevant Legal Provisions
9.
To consider these questions, it is necessary to refer to the statement of objects and
reasons, the preamble and the relevant provisions of the RTI Act. RTI Act was enacted in
order to ensure smoother, greater and more effective access to information and provide an
effective framework for effectuating the right of information recognized under article 19
of the Constitution. The preamble to the Act declares the object sought to be achieved by
the RTI Act thus:
“An Act to provide for setting out the practical regime of right to
information for citizens to secure access to information under the control
of public authorities, in order to promote transparency and accountability
in the working of every public authority, the constitution of a Central
Information Commission and State Information Commissions and for
matters connected therewith or incidental thereto.
Whereas the Constitution of India has established democratic Republic;
And whereas democracy requires an informed citizenry and transparency
of information which are vital to its functioning and also to contain
corruption and to hold Governments and their instrumentalities
accountable to the governed;
And whereas revelation of information in actual practice is likely to
conflict with other public interests including efficient operations of the
Governments, optimum use of limited fiscal resources and the
preservation of confidentiality of sensitive information;
And whereas it is necessary to harmonise these conflicting interests while
preserving the paramountcy of the democratic ideal.”
Chapter II of the Act containing sections 3 to 11 deals with right to information and
obligations of public authorities. Section 3 provides for right to information and reads
thus: “Subject to the provisions of this Act, all citizens shall have the right to
information.” This section makes it clear that the RTI Act gives a right to a citizen to only
access information, but not seek any consequential relief based on such information.
Section 4 deals with obligations of public authorities to maintain the records in the
manner provided and publish and disseminate the information in the manner provided.
Section 6 deals with requests for obtaining information. It provides that applicant making
a request for information shall not be required to give any reason for requesting the
information or any personal details except those that may be necessary for contacting
him. Section 8 deals with exemption from disclosure of information and is extracted in its
entirety:
“8. Exemption from disclosure of information -- (1) Notwithstanding
anything contained in this Act, there shall be no obligation to give any
citizen,(a) information, disclosure of which would prejudicially affect the
sovereignty and integrity of India, the security, strategic, scientific or
economic interests of the State, relation with foreign State or lead to
incitement of an offence;
(b) information which has been expressly forbidden to be published by
any court of law or tribunal or the disclosure of which may constitute
contempt of court;
(c) information, the disclosure of which would cause a breach of privilege
of Parliament or the State Legislature;
(d) information including commercial confidence, trade secrets or
intellectual property, the disclosure of which would harm the competitive
position of a third party, unless the competent authority is satisfied that
larger public interest warrants the disclosure of such information;
(e) information available to a person in his fiduciary relationship,
unless the competent authority is satisfied that the larger public
interest warrants the disclosure of such information;
(f) information received in confidence from foreign Government;
(g) information, the disclosure of which would endanger the life or
physical safety of any person or identify the source of information or
assistance given in confidence for law enforcement or security purposes;
(h) information which would impede the process of investigation or
apprehension or prosecution of offenders;
(i) cabinet papers including records of deliberations of the Council of
Ministers, Secretaries and other officers:
Provided that the decisions of Council of Ministers, the reasons thereof,
and the material on the basis of which the decisions were taken shall be
made public after the decision has been taken, and the matter is complete,
or over:
Provided further that those matters which come under the exemptions
specified in this section shall not be disclosed;
(j) information which relates to personal information the disclosure of
which has no relationship to any public activity or interest, or which
would cause unwarranted invasion of the privacy of the individual unless
the Central Public Information Officer or the State
Public Information Officer or the appellate authority, as the case may be,
is satisfied that the larger public interest justifies the disclosure of such
information:
Provided that the information which cannot be denied to the Parliament or
a State Legislature shall not be denied to any person.
(2)
Notwithstanding anything in the Official Secrets Act, 1923 (19 of
1923) nor any of the exemptions permissible in accordance with subsection (1), a public authority may allow access to information, if public
interest in disclosure outweighs the harm to the protected interests.
(3)
Subject to the provisions of clauses (a), (c) and (i) of sub-section
(1), any information relating to any occurrence, event or matter which has
taken place, occurred or happened twenty years before the date on which
any request is made under section 6 shall be provided to any person
making a request under that section:
Provided that where any question arises as to the date from which the said
period of twenty years has to be computed, the decision of the Central
Government shall be final, subject to the usual appeals provided for in this
Act.”
(emphasis supplied)
Section 9 provides that without prejudice to the provisions of section 8, a request for
information may be rejected if such a request for providing access would involve an
infringement of copyright. Section 10 deals with severability of exempted information
and sub-section (1) thereof is extracted below:
“(1) Where a request for access to information is rejected on the ground
that it is in relation to information which is exempt from disclosure, then,
notwithstanding anything contained in this Act, access may be provided to
that part of the record which does not contain any information which is
exempt from disclosure under this Act and which can reasonably be
severed from any part that contains exempt information.”
Section 11 deals with third party information and sub-section (1) thereof is extracted
below:
“(1) Where a Central Public Information Officer or a State Public
Information Officer, as the case may be, intends to disclose any
information or record, or part thereof on a request made under this Act,
which relates to or has been supplied by a third party and has been treated
as confidential by that third party, the Central Public Information Officer
or State Public Information Officer, as the case may be, shall, within five
days from the receipt of the request, give a written notice to such third
party of the request and of the fact that the Central Public Information
Officer or State Public Information Officer, as the case may be, intends to
disclose the information or record, or part thereof, and invite the third
party to make a submission in writing or orally, regarding whether the
information should be disclosed, and such submission of the third party
shall be kept in view while taking a decision about disclosure of
information:
Provided that except in the case of trade or commercial secrets protected
by law, disclosure may be allowed if the public interest in disclosure
outweighs in importance any possible harm or injury to the interests of
such third party.”
The definitions of information, public authority, record and right to information in
clauses (f), (h), (i) and (j) of section 2 of the RTI Act are extracted below:
“(f) "information" means any material in any form, including records,
documents, memos, e-mails, opinions, advices, press releases, circulars, orders,
logbooks, contracts, reports, papers, samples, models, data material held in any
electronic form and information relating to any private body which can be
accessed by a public authority under any other law for the time being in force;
(h) "public authority" means any authority or body or institution of selfgovernment established or constituted(a) by or under the Constitution;
(b) by any other law made by Parliament;
(c) by any other law made by State Legislature;
(d) by notification issued or order made by the appropriate Government, and
includes any(i) body owned, controlled or substantially financed;
(ii) non-Government organisation substantially financed, directly or indirectly by
funds provided by the appropriate Government;
(i) "record" includes(a) any document, manuscript and file;
(b) any microfilm, microfiche and facsimile copy of a document;
(c) any reproduction of image or images embodied in such microfilm (whether
enlarged or not); and
(d) any other material produced by a computer or any other device;
(j) "right to information" means the right to information accessible under this Act
which is held by or under the control of any public authority and includes the right to(i)
inspection of work, documents, records;
(ii)
taking notes, extracts or certified copies of documents or records;
(iii)
taking certified samples of material;
(iv)
obtaining information in the form of diskettes, floppies, tapes, video cassettes
or in any other electronic mode or through printouts where such information is
stored in a computer or in any other device;
Section 22 provides for the Act to have overriding effect and is extracted below:
“The provisions of this Act shall have effect notwithstanding anything
inconsistent therewith contained in the Official Secrets Act, 1923 (19 of
1923), and any other law for the time being in force or in any instrument
having effect by virtue of any law other than this Act.”
10.
It will also be useful to refer to a few decisions of this Court which considered the
importance and scope of the right to information. In State of Uttar Pradesh v. Raj Narain
- (1975) 4 SCC 428, this Court observed:
“In a government of responsibility like ours, where all the agents of the
public must be responsible for their conduct, there can but few secrets. The
people of this country have a right to know every public act, everything,
that is done in a public way, by their public functionaries. They are
entitled to know the particulars of every public transaction in all its
bearing. The right to know, which is derived from the concept of freedom
of speech, though not absolute, is a factor which should make one wary,
when secrecy is claimed for transactions which can, at any rate, have no
repercussion on public security.”
(emphasis supplied)
In Dinesh Trivedi v. Union of India – (1997) 4 SCC 306, this Court held:
“In modern constitutional democracies, it is axiomatic that citizens have a
right to know about the affairs of the Government which, having been
elected by them, seeks to formulate sound policies of governance aimed at
their welfare. However, like all other rights, even this right has recognized
limitations; it is, by no means, absolute. ………………Implicit in this
assertion is the proposition that in transaction which have serious
repercussions on public security, secrecy can legitimately be claimed
because it would then be in the public interest that such matters are not
publicly disclosed or disseminated.
To ensure the continued participation of the people in the democratic
process, they must be kept informed of the vital decisions taken by the
Government and the basis thereof. Democracy, therefore, expects
openness and openness is a concomitant of a free society. Sunlight is the
best disinfectant. But it is equally important to be alive to the dangers that
lie ahead. It is important to realise that undue popular pressure brought to
bear on decision-makers is Government can have frightening side-effects.
If every action taken by the political or executive functionary is
transformed into a public controversy and made subject to an enquiry to
soothe popular sentiments, it will undoubtedly have a chilling effect on the
independence of the decision-maker who may find it safer not to take any
decision. It will paralyse the entire system and bring it to a grinding halt.
So we have two conflicting situations almost enigmatic and we think the
answer is to maintain a fine balance which would serve public interest.”
In People’s Union for Civil Liberties v. Union of India - (2004) 2 SCC 476, this Court
held that right of information is a facet of the freedom of “speech and expression” as
contained in Article 19(1)(a) of the Constitution of India and such a right is subject to any
reasonable restriction in the interest of the security of the state and subject to exemptions
and exceptions.
Re : Question (i)
11.
The definition of ‘information’ in section 2(f) of the RTI Act refers to any
material in any form which includes records, documents, opinions, papers among several
other enumerated items. The term ‘record’ is defined in section 2(i) of the said Act as
including any document, manuscript or file among others. When a candidate participates
in an examination and writes his answers in an answer-book and submits it to the
examining body for evaluation and declaration of the result, the answer-book is a
document or record. When the answer-book is evaluated by an examiner appointed by the
examining body, the evaluated answer-book becomes a record containing the ‘opinion’ of
the examiner. Therefore the evaluated answer-book is also an ‘information’ under the
RTI Act.
12.
Section 3 of RTI Act provides that subject to the provisions of this Act all citizens
shall have the right to information. The term ‘right to information’ is defined in section
2(j) as the right to information accessible under the Act which is held by or under the
control of any public authority. Having regard to section 3, the citizens have the right to
access to all information held by or under the control of any public authority except those
excluded or exempted under the Act. The object of the Act is to empower the citizens to
fight against corruption and hold the Government and their instrumentalities accountable
to the citizens, by providing them access to information regarding functioning of every
public authority. Certain safeguards have been built into the Act so that the revelation of
information will not conflict with other public interests which include efficient operation
of the governments, optimum use of limited fiscal resources and preservation of
confidential and sensitive information. The RTI Act provides access to information held
by or under the control of public authorities and not in regard to information held by any
private person. The Act provides the following exclusions by way of exemptions and
exceptions (under sections 8, 9 and 24) in regard to information held by public
authorities:
(i)
Exclusion of the Act in entirety under section 24 to intelligence and
security organizations specified in the Second Schedule even though
they may be “public authorities”, (except in regard to information with
reference to allegations of corruption and human rights violations).
(ii)
Exemption of the several categories of information enumerated in
section 8(1) of the Act which no public authority is under an obligation
to give to any citizen, notwithstanding anything contained in the Act
[however, in regard to the information exempted under clauses (d) and
(e), the competent authority, and in regard to the information excluded
under clause (j), Central Public Information Officer/State Public
Information Officer/the Appellate Authority, may direct disclosure of
information, if larger public interest warrants or justifies the
disclosure].
(iii)
If any request for providing access to information involves an
infringement of a copyright subsisting in a person other than the State,
the Central/State Public Information Officer may reject the request
under section 9 of RTI Act.
Having regard to the scheme of the RTI Act, the right of the citizens to access any
information held or under the control of any public authority, should be read in harmony
with the exclusions/exemptions in the Act.
13.
The examining bodies (Universities, Examination Boards, CBSC etc.) are neither
security nor intelligence organisations and therefore the exemption under section 24 will
not apply to them. The disclosure of information with reference to answer-books does not
also involve infringement of any copyright and therefore section 9 will not apply.
Resultantly, unless the examining bodies are able to demonstrate that the evaluated
answer-books fall under any of the categories of exempted ‘information’ enumerated in
clauses (a) to (j) of sub-section (1) section 8, they will be bound to provide access to the
information and any applicant can either inspect the document/record, take notes, extracts
or obtain certified copies thereof.
14.
The examining bodies contend that the evaluated answer-books are exempted
from disclosure under section 8(1)(e) of the RTI Act, as they are ‘information’ held in its
fiduciary relationship. They fairly conceded that evaluated answer-books will not fall
under any other exemptions in subsection (1) of section 8. Every examinee will have the
right to access his evaluated answer-books, by either inspecting them or take certified
copies thereof, unless the evaluated answer-books are found to be exempted under
section 8(1)(e) of the RTI Act.
Re : Question (ii)
15.
In Maharashtra State Board, this Court was considering whether denial of re-
evaluation of answer-books or denial of disclosure by way of inspection of answer books,
to an examinee, under Rule 104(1) and (3) of the Maharashtra Secondary and Higher
Secondary Board Rules, 1977 was violative of principles of natural justice and violative
of Articles 14 and 19 of the Constitution of India. Rule 104(1) provided that no reevaluation of the answer books shall be done and on an application of any candidate
verification will be restricted to checking whether all the answers have been examined
and that there is no mistake in the totalling of marks for each question in that subject and
transferring marks correctly on the first cover page of the answer book. Rule 104(3)
provided that no candidate shall claim or be entitled to re-evaluation of his answer-books
or inspection of answerbooks as they were treated as confidential. This Court while
upholding the validity of Rule 104(3) held as under :
“…. the “process of evaluation of answer papers or of subsequent
verification of marks” under Clause (3) of Regulation 104 does not attract
the principles of natural justice since no decision making process which
brings about adverse civil consequences to the examinees in involved. The
principles of natural justice cannot be extended beyond reasonable and
rational limits and cannot be carried to such absurd lengths as to make it
necessary that candidates who have taken a public examination should be
allowed to participate in the process of evaluation of their performances or
to verify the correctness of the evaluation made by the examiners by
themselves conducting an inspection of the answer-books and determining
whether there has been a proper and fair valuation of the answers by the
examiners."
So long as the body entrusted with the task of framing the rules or
regulations acts within the scope of the authority conferred on it, in the
sense that the rules or regulations made by it have a rational nexus with
the object and purpose of the statute, the court should not concern itself
with the wisdom or efficaciousness of such rules or regulations…. The
Legislature and its delegate are the sole repositories of the power to decide
what policy should be pursued in relation to matters covered by the Act
…and there is no scope for interference by the Court unless the particular
provision impugned before it can be said to suffer from any legal
infirmity, in the sense of its being wholly beyond the scope of the
regulation making power or its being inconsistent with any of the
provisions of the parent enactment or in violation of any of the limitations
imposed by the Constitution.
It was perfectly within the competence of the Board, rather it was its plain
duty, to apply its mind and decide as a matter of policy relating to the
conduct of the examination as to whether disclosure and inspection of the
answer books should be allowed to the candidates, whether and to what
extent verification of the result should be permitted after the results have
already been announced and whether any right to claim revaluation of the
answer books should be recognised or provided for. All these are
undoubtedly matters which have an intimate nexus with the objects and
purposes of the enactment and are, therefore, with in the ambit of the
general power to make regulations….”
This Court held that Regulation 104(3) cannot be held to be unreasonable merely because
in certain stray instances, errors or irregularities had gone unnoticed even after
verification of the concerned answer books according to the existing procedure and it was
only after further scrutiny made either on orders of the court or in the wake of contentions
raised in the petitions filed before a court, that such errors or irregularities were
ultimately discovered. This court reiterated the view that “the test of reasonableness is not
applied in vacuum but in the context of life’s realities” and concluded that realistically
and practically, providing all the candidates inspection of their answer books or reevaluation of the answer books in the presence of the candidates would not be feasible.
Dealing with the contention that every student is entitled to fair play in examination and
receive marks matching his performance, this court held :
“What constitutes fair play depends upon the facts and circumstances
relating to each particular given situation. If it is found that every possible
precaution has been taken and all necessary safeguards provided to ensure
that the answer books inclusive of supplements are kept in safe custody so
as to eliminate the danger of their being tampered with and that the
evaluation is done by the examiners applying uniform standards with
checks and crosschecks at different stages and that measures for detection
of malpractice, etc. have also been effectively adopted, in such cases it
will not be correct on the part of the Courts to strike down, the provision
prohibiting revaluation on the ground that it violates the rules of fair play.
It appears that the procedure evolved by the Board for ensuring fairness
and accuracy in evaluation of the answer books has made the system as
fool proof as can be possible and is entirely satisfactory. The Board is a
very responsible body. The candidates have taken the examination with
full awareness of the provisions contained in the Regulations and in the
declaration made in the form of application for admission to the
examination they have solemnly stated that they fully agree to abide by the
regulations issued by the Board. In the circumstances, when we find that
all safeguards against errors and malpractices have been provided for,
there cannot be said to be any denial of fair play to the examinees by
reason of the prohibition against asking for revaluation…. “
This Court concluded that if inspection and verification in the presence of the candidates,
or revaluation, have to be allowed as of right, it may lead to gross and indefinite
uncertainty, particularly in regard to the relative ranking etc. of the candidate, besides
leading to utter confusion on account of the enormity of the labour and time involved in
the process. This court concluded :
“… the Court should be extremely reluctant to substitute its own views as
to what is wise, prudent and proper in relation to academic matters in
preference to those formulated by professional men possessing technical
expertise and rich experience of actual day-to-day working of educational
institutions and the departments controlling them. It will be wholly wrong
for the court to make a pedantic and purely idealistic approach to the
problems of this nature, isolated from the actual realities and grass root
problems involved in the working of the system and unmindful of the
consequences which would emanate if a purely idealistic view as opposed
to a pragmatic one were to be propounded.”
16.
The above principles laid down in Maharashtra State Board have been followed
and reiterated in several decisions of this Court, some of which are referred to in para (6)
above. But the principles laid down in decisions such as Maharashtra State Board
depend upon the provisions of the rules and regulations of the examining body. If the
rules and regulations of the examining body provide for re-evaluation, inspection or
disclosure of the answer-books, then none of the principles in Maharashtra State Board
or other decisions following it, will apply or be relevant. There has been a gradual change
in trend with several examining bodies permitting inspection and disclosure of the
answer-books.
17.
It is thus now well settled that a provision barring inspection or disclosure of the
answer-books or re-evaluation of the answer-books and restricting the remedy of the
candidates only to re-totalling is valid and binding on the examinee. In the case of CBSE,
the provisions barring re-evaluation and inspection contained in Bye-law No.61, are akin
to Rule 104 considered in Maharashtra State Board. As a consequence if an examination
is governed only by the rules and regulations of the examining body which bar
inspection, disclosure or re-evaluation, the examinee will be entitled only for re-totalling
by checking whether all the answers have been evaluated and further checking whether
there is no mistake in totaling of marks for each question and marks have been
transferred correctly to the title (abstract) page. The position may however be different, if
there is a superior statutory right entitling the examinee, as a citizen to seek access to the
answer books, as information.
18.
In these cases, the High Court has rightly denied the prayer for reevaluation of
answer-books sought by the candidates in view of the bar contained in the rules and
regulations of the examining bodies. It is also not a relief available under the RTI Act.
Therefore the question whether reevaluation should be permitted or not, does not arise for
our consideration. What arises for consideration is the question whether the examinee is
entitled to inspect his evaluated answer-books or take certified copies thereof. This right
is claimed by the students, not with reference to the rules or bye-laws of examining
bodies, but under the RTI Act which enables them and entitles them to have access to the
answer-books as ‘information’ and inspect them and take certified copies thereof. Section
22 of RTI Act provides that the provisions of the said Act will have effect,
notwithstanding anything inconsistent therewith contained in any other law for the time
being in force. Therefore the provisions of the RTI Act will prevail over the provisions of
the bye-laws/rules of the examining bodies in regard to examinations. As a result, unless
the examining body is able to demonstrate that the answer-books fall under the exempted
category of information described in clause (e) of section 8(1) of RTI Act, the examining
body will be bound to provide access to an examinee to inspect and take copies of his
evaluated answer-books, even if such inspection or taking copies is barred under the
rules/bye-laws of the examining body governing the examinations.
Therefore, the
decision of this Court in Maharashtra State Board (supra) and the subsequent decisions
following the same, will not affect or interfere with the right of the examinee seeking
inspection of answer-books or taking certified copies thereof.
Re : Question (iii)
19.
Section 8(1) enumerates the categories of information which are exempted from
disclosure under the provisions of the RTI Act. The examining bodies rely upon clause
(e) of section 8(1) which provides that there shall be no obligation on any public
authority to give any citizen, information available to it in its fiduciary relationship. This
exemption is subject to the condition that if the competent authority (as defined in section
2(e) of RTI Act) is satisfied that the larger public interest warrants the disclosure of such
information, the information will have to be disclosed. Therefore the question is whether
the examining body holds the evaluated answer-books in its fiduciary relationship.
20.
The term ‘fiduciary’ and ‘fiduciary relationship’ refer to different capacities and
relationship, involving a common duty or obligation.
20.1) Black’s Law Dictionary (7th Edition, Page 640) defines ‘fiduciary relationship’
thus:
“A relationship in which one person is under a duty to act for the benefit
of the other on matters within the scope of the relationship. Fiduciary
relationships – such as trustee-beneficiary, guardian-ward, agent-principal,
and attorney-client – require the highest duty of care. Fiduciary
relationships usually arise in one of four situations : (1) when one person
places trust in the faithful integrity of another, who as a result gains
superiority or influence over the first, (2) when one person assumes
control and responsibility over another, (3) when one person has a duty to
act for or give advice to another on matters falling within the scope of the
relationship, or (4) when there is a specific relationship that has
traditionally been recognized as involving fiduciary duties, as with a
lawyer and a client or a stockbroker and a customer.”
20.2) The American Restatements (Trusts and Agency) define ‘fiduciary’ as one whose
intention is to act for the benefit of another as to matters relevant to the relation between
them. The Corpus Juris Secundum (Vol. 36A page 381) attempts to define fiduciary thus:
“A general definition of the word which is sufficiently comprehensive to
embrace all cases cannot well be given. The term is derived from the civil,
or Roman, law. It connotes the idea of trust or confidence, contemplates
good faith, rather than legal obligation, as the basis of the transaction,
refers to the integrity, the fidelity, of the party trusted, rather than his
credit or ability, and has been held to apply to all persons who occupy a
position of peculiar confidence toward others, and to include those
informal relations which exist whenever one party trusts and relies on
another, as well as technical fiduciary relations.
The word ‘fiduciary,’ as a noun, means one who holds a thing in trust for
another, a trustee, a person holding the character of a trustee, or a
character analogous to that of a trustee, with respect to the trust and
confidence involved in it and the scrupulous good faith and candor which
it requires; a person having the duty, created by his undertaking, to act
primarily for another’s benefit in matters connected with such
undertaking. Also more specifically, in a statute, a guardian, trustee,
executor, administrator, receiver, conservator, or any person acting in any
fiduciary capacity for any person, trust, or estate. Some examples of what,
in particular connections, the term has been held to include and not to
include are set out in the note.”
20.3) Words and Phrases, Permanent Edition (Vol. 16A, Page 41) defines ‘fiducial
relation’ thus :
“There is a technical distinction between a ‘fiducial relation’ which is
more correctly applicable to legal relationships between parties, such as
guardian and ward, administrator and heirs, and other similar
relationships, and ‘confidential relation’ which includes the legal
relationships, and also every other relationship wherein confidence is
rightly reposed and is exercised.
Generally, the term ‘fiduciary’ applies to any person who occupies a
position of peculiar confidence towards another. It refers to integrity and
fidelity. It contemplates fair dealing and good faith, rather than legal
obligation, as the basis of the transaction. The term includes those
informal relations which exist whenever one party trusts and relies upon
another, as well as technical fiduciary relations.”
20.4) In Bristol and West Building Society vs. Mothew [1998 Ch. 1] the term fiduciary
was defined thus :
“A fiduciary is someone who has undertaken to act for and on behalf of
another in a particular matter in circumstances which give rise to a
relationship of trust and confidence. The distinguishing obligation of a
fiduciary is the obligation of loyalty….. A fiduciary must act in good faith;
he must not make a profit out of his trust; he must not place himself in a
position where his duty and his interest may conflict; he may not act for
his own benefit or the benefit of a third person without the informed
consent of his principal.”
20.5) In Wolf vs. Superior Court [2003 (107) California Appeals, 4th 25] the California
Court of Appeals defined fiduciary relationship as under :
“any relationship existing between the parties to the transaction where one
of the parties is duty bound to act with utmost good faith for the benefit of
the other party. Such a relationship ordinarily arises where confidence is
reposed by one person in the integrity of another, and in such a relation the
party in whom the confidence is reposed, if he voluntarily accepts or
assumes to accept the confidence, can take no advantage from his acts
relating to the interests of the other party without the latter’s knowledge
and consent.”
21.
The term ‘fiduciary’ refers to a person having a duty to act for the benefit of
another, showing good faith and condour, where such other person reposes trust and
special confidence in the person owing or discharging the duty. The term ‘fiduciary
relationship’ is used to describe a situation or transaction where one person (beneficiary)
places complete confidence in another person (fiduciary) in regard to his affairs, business
or transaction/s. The term also refers to a person who holds a thing in trust for another
(beneficiary). The fiduciary is expected to act in confidence and for the benefit and
advantage of the beneficiary, and use good faith and fairness in dealing with the
beneficiary or the things belonging to the beneficiary. If the beneficiary has entrusted
anything to the fiduciary, to hold the thing in trust or to execute certain acts in regard to
or with reference to the entrusted thing, the fiduciary has to act in confidence and
expected not to disclose the thing or information to any third party. There are also certain
relationships where both the parties have to act in a fiduciary capacity treating the other
as the beneficiary. Examples of these are : a partner vis-à-vis another partner and an
employer vis-à-vis employee. An employee who comes into possession of business or
trade secrets or confidential information relating to the employer in the course of his
employment, is expected to act as a fiduciary and cannot disclose it to others. Similarly, if
on the request of the employer or official superior or the head of a department, an
employee furnishes his personal details and information, to be retained in confidence, the
employer, the official superior or departmental head is expected to hold such personal
information in confidence as a fiduciary, to be made use of or disclosed only if the
employee’s conduct or acts are found to be prejudicial to the employer.
22.
In a philosophical and very wide sense, examining bodies can be said to act in a
fiduciary capacity, with reference to students who participate in an examination, as a
government does while governing its citizens or as the present generation does with
reference to the future generation while preserving the environment. But the words
‘information available to a person in his fiduciary relationship’ are used in section 8(1)(e)
of RTI Act in its normal and well recognized sense, that is to refer to persons who act in a
fiduciary capacity, with reference to a specific beneficiary or beneficiaries who are to be
expected to be protected or benefited by the actions of the fiduciary – a trustee with
reference to the beneficiary of the trust, a guardian with reference to a
minor/physically/infirm/mentally challenged, a parent with reference to a child, a lawyer
or a chartered accountant with reference to a client, a doctor or nurse with reference to a
patient, an agent with reference to a principal, a partner with reference to another partner,
a director of a company with reference to a share-holder, an executor with reference to a
legatee, a receiver with reference to the parties to a lis, an employer with reference to the
confidential information relating to the employee, and an employee with reference to
business dealings/transaction of the employer. We do not find that kind of fiduciary
relationship between the examining body and the examinee, with reference to the
evaluated answer-books, that come into the custody of the examining body.
23.
The duty of examining bodies is to subject the candidates who have completed a
course of study or a period of training in accordance with its curricula, to a process of
verification/examination/testing of their knowledge, ability or skill, or to ascertain
whether they can be said to have successfully completed or passed the course of study or
training. Other specialized Examining Bodies may simply subject candidates to a process
of verification by an examination, to find out whether such person is suitable for a
particular post, job or assignment. An examining body, if it is a public authority entrusted
with public functions, is required to act fairly, reasonably, uniformly and consistently for
public good and in public interest. This Court has explained the role of an examining
body in regard to the process of holding examination in the context of examining whether
it amounts to ‘service’ to a consumer, in Bihar School Examination Board vs. Suresh
Prasad Sinha – (2009) 8 SCC 483, in the following manner:
“The process of holding examinations, evaluating answer scripts,
declaring results and issuing certificates are different stages of a single
statutory non-commercial function. It is not possible to divide this
function as partly statutory and partly administrative. When the
Examination Board conducts an examination in discharge of its statutory
function, it does not offer its "services" to any candidate. Nor does a
student who participates in the examination conducted by the Board, hires
or avails of any service from the Board for a consideration. On the other
hand, a candidate who participates in the examination conducted by the
Board, is a person who has undergone a course of study and who requests
the Board to test him as to whether he has imbibed sufficient knowledge to
be fit to be declared as having successfully completed the said course of
education; and if so, determine his position or rank or competence vis-avis
other examinees. The process is not therefore availment of a service by a
student, but participation in a general examination conducted by the Board
to ascertain whether he is eligible and fit to be considered as having
successfully completed the secondary education course. The examination
fee paid by the student is not the consideration for availment of any
service, but the charge paid for the privilege of participation in the
examination.……… The fact that in the course of conduct of the
examination, or evaluation of answer-scripts, or furnishing of mark-books
or certificates, there may be some negligence, omission or deficiency,
does not convert the Board into a service-provider for a consideration, nor
convert the examinee into a consumer ………”
It cannot therefore be said that the examining body is in a fiduciary relationship either
with reference to the examinee who participates in the examination and whose answerbooks are evaluated by the examining body.
24.
We may next consider whether an examining body would be entitled to claim
exemption under section 8(1)(e) of the RTI Act, even assuming that it is in a fiduciary
relationship with the examinee. That section provides that notwithstanding anything
contained in the Act, there shall be no obligation to give any citizen information
available to a person in his fiduciary relationship. This would only mean that even if the
relationship is fiduciary, the exemption would operate in regard to giving access to the
information held in fiduciary relationship, to third parties. There is no question of the
fiduciary withholding information relating to the beneficiary, from the beneficiary
himself. One of the duties of the fiduciary is to make thorough disclosure of all relevant
facts of all transactions between them to the beneficiary, in a fiduciary relationship. By
that logic, the examining body, if it is in a fiduciary relationship with an examinee, will
be liable to make a full disclosure of the evaluated answer-books to the examinee and at
the same time, owe a duty to the examinee not to disclose the answer-books to anyone
else. If A entrusts a document or an article to B to be processed, on completion of
processing, B is not expected to give the document or article to anyone else but is bound
to give the same to A who entrusted the document or article to B for processing.
Therefore, if a relationship of fiduciary and beneficiary is assumed between the
examining body and the examinee with reference to the answer-book, section 8(1)(e)
would operate as an exemption to prevent access to any third party and will not operate as
a bar for the very person who wrote the answer-book, seeking inspection or disclosure of
it.
25.
An evaluated answer book of an examinee is a combination of two different
‘informations’. The first is the answers written by the examinee and second is the
marks/assessment by the examiner. When an examinee seeks inspection of his evaluated
answer-books or seeks a certified copy of the evaluated answer-book, the information
sought by him is not really the answers he has written in the answer-books (which he
already knows), nor the total marks assigned for the answers (which has been declared).
What he really seeks is the information relating to the break-up of marks, that is, the
specific marks assigned to each of his answers. When an examinee seeks ‘information’
by inspection/certified copies of his answer-books, he knows the contents thereof being
the author thereof. When an examinee is permitted to examine an answer-book or obtain
a certified copy, the examining body is not really giving him some information which is
held by it in trust or confidence, but is only giving him an opportunity to read what he
had written at the time of examination or to have a copy of his answers. Therefore, in
furnishing the copy of an answer-book, there is no question of breach of confidentiality,
privacy, secrecy or trust. The real issue therefore is not in regard to the answer-book but
in regard to the marks awarded on evaluation of the answer-book. Even here the total
marks given to the examinee in regard to his answer-book are already declared and
known to the examinee. What the examinee actually wants to know is the break-up of
marks given to him, that is how many marks were given by the examiner to each of his
answers so that he can assess how is performance has been evaluated and whether the
evaluation is proper as per his hopes and expectations. Therefore, the test for finding out
whether the information is exempted or not, is not in regard to the answer book but in
regard to the evaluation by the examiner.
26.
This takes us to the crucial issue of evaluation by the examiner. The examining
body engages or employs hundreds of examiners to do the evaluation of thousands of
answer books. The question is whether the information relating to the ‘evaluation’ (that is
assigning of marks) is held by the examining body in a fiduciary relationship. The
examining bodies contend that even if fiduciary relationship does not exist with reference
to the examinee, it exists with reference to the examiner who evaluates the answer-books.
On a careful examination we find that this contention has no merit. The examining body
entrusts the answer-books to an examiner for evaluation and pays the examiner for his
expert service. The work of evaluation and marking the answer-book is an assignment
given by the examining body to the examiner which he discharges for a consideration.
Sometimes, an examiner may assess answer-books, in the course of his employment, as a
part of his duties without any specific or special remuneration. In other words the
examining body is the ‘principal’ and the examiner is the agent entrusted with the work,
that is, evaluation of answerbooks. Therefore, the examining body is not in the position
of a fiduciary with reference to the examiner. On the other hand, when an answer-book is
entrusted to the examiner for the purpose of evaluation, for the period the answer-book is
in his custody and to the extent of the discharge of his functions relating to evaluation,
the examiner is in the position of a fiduciary with reference to the examining body and he
is barred from disclosing the contents of the answer-book or the result of evaluation of
the answer-book to anyone other than the examining body. Once the examiner has
evaluated the answer books, he ceases to have any interest in the evaluation done by him.
He does not have any copy-right or proprietary right, or confidentiality right in regard to
the evaluation. Therefore it cannot be said that the examining body holds the evaluated
answer books in a fiduciary relationship, qua the examiner.
27.
We, therefore, hold that an examining body does not hold the evaluated answer-
books in a fiduciary relationship. Not being information available to an examining body
in its fiduciary relationship, the exemption under section 8(1)(e) is not available to the
examining bodies with reference to evaluated answer-books. As no other exemption
under section 8 is available in respect of evaluated answer books, the examining bodies
will have to permit inspection sought by the examinees.
Re : Question (iv)
28.
When an examining body engages the services of an examiner to evaluate the
answer-books, the examining body expects the examiner not to disclose the information
regarding evaluation to anyone other than the examining body. Similarly the examiner
also expects that his name and particulars would not be disclosed to the candidates whose
answer-books are evaluated by him. In the event of such information being made known,
a disgruntled examinee who is not satisfied with the evaluation of the answer books, may
act to the prejudice of the examiner by attempting to endanger his physical safety.
Further, any apprehension on the part of the examiner that there may be danger to his
physical safety, if his identity becomes known to the examinees, may come in the way of
effective discharge of his duties. The above applies not only to the examiner, but also to
the scrutiniser, co-ordinator, and head-examiner who deal with the answer book. The
answer book usually contains not only the signature and code number of the examiner,
but also the signatures and code number of the scrutiniser/coordinator/head examiner.
The
information
as
to
the
names
or
particulars
of
the
examiners/co-
ordinators/scrutinisers/head examiners are therefore exempted from disclosure under
section 8(1)(g) of RTI Act, on the ground that if such information is disclosed, it may
endanger their physical safety. Therefore, if the examinees are to be given access to
evaluated answerbooks either by permitting inspection or by granting certified copies,
such access will have to be given only to that part of the answer-book which does not
contain any information or signature of the examiners/coordinators/scrutinisers/head
examiners, exempted from disclosure under section 8(1)(g) of RTI Act. Those portions of
the
answer-books
which
contain
information
regarding
the
examiners/co-
ordinators/scrutinisers/head examiners or which may disclose their identity with
reference to signature or initials, shall have to be removed, covered, or otherwise severed
from the non-exempted part of the answer-books, under section 10 of RTI Act.
29.
The right to access information does not extend beyond the period during which
the examining body is expected to retain the answer-books. In the case of CBSE, the
answer-books are required to be maintained for a period of three months and thereafter
they are liable to be disposed of/destroyed. Some other examining bodies are required to
keep the answerbooks for a period of six months. The fact that right to information is
available in regard to answer-books does not mean that answer-books will have to be
maintained for any longer period than required under the rules and regulations of the
public authority. The obligation under the RTI Act is to make available or give access to
existing information or information which is expected to be preserved or maintained. If
the rules and regulations governing the functioning of the respective public authority
require preservation of the information for only a limited period, the applicant for
information will be entitled to such information only if he seeks the information when it
is available with the public authority. For example, with reference to answer-books, if an
examinee makes an application to CBSE for inspection or grant of certified copies
beyond three months (or six months or such other period prescribed for preservation of
the records in regard to other examining bodies) from the date of declaration of results,
the application could be rejected on the ground that such information is not available. The
power of the Information Commission under section 19(8) of the RTI Act to require a
public authority to take any such steps as may be necessary to secure compliance with the
provision of the Act, does not include a power to direct the public authority to preserve
the information, for any period larger than what is provided under the rules and
regulations of the public authority.
30.
On behalf of the respondents/examinees, it was contended that having regard to
sub-section (3) of section 8 of RTI Act, there is an implied duty on the part of every
public authority to maintain the information for a minimum period of twenty years and
make it available whenever an application was made in that behalf. This contention is
based on a complete misreading and misunderstanding of section 8(3). The said subsection nowhere provides that records or information have to be maintained for a period
of twenty years. The period for which any particular records or information has to be
maintained would depend upon the relevant statutory rule or regulation of the public
authority relating to the preservation of records. Section 8(3) provides that information
relating to any occurrence, event or matters which has taken place and occurred or
happened twenty years before the date on which any request is made under section 6,
shall be provided to any person making a request. This means that where any information
required to be maintained and preserved for a period beyond twenty years under the rules
of the public authority, is exempted from disclosure under any of the provisions of
section 8(1) of RTI Act, then, notwithstanding such exemption, access to such
information shall have to be provided by disclosure thereof, after a period of twenty years
except where they relate to information falling under clauses (a), (c) and (i) of section
8(1). In other words, section 8(3) provides that any protection against disclosure that may
be available, under clauses (b), (d) to (h) and (j) of section 8(1) will cease to be available
after twenty years in regard to records which are required to be preserved for more than
twenty years. Where any record or information is required to be destroyed under the rules
and regulations of a public authority prior to twenty years, section 8(3) will not prevent
destruction in accordance with the Rules. Section 8(3) of RTI Act is not therefore a
provision requiring all ‘information’ to be preserved and maintained for twenty years or
more, nor does it override any rules or regulations governing the period for which the
record, document or information is required to be preserved by any public authority.
31.
The effect of the provisions and scheme of the RTI Act is to divide ‘information’
into the three categories. They are :
(i)
Information which promotes transparency and accountability in the
working of every public authority, disclosure of which may also help
in containing or discouraging corruption (enumerated in clauses (b)
and (c) of section 4(1) of RTI Act).
(ii)
Other information held by public authority (that is all information
other than those falling under clauses (b) and (c) of section 4(1) of RTI
Act).
(iii)
Information which is not held by or under the control of any public
authority and which cannot be accessed by a public authority under
any law for the time being in force.
Information under the third category does not fall within the scope of RTI Act. Section 3
of RTI Act gives every citizen, the right to ‘information’ held by or under the control of a
public authority, which falls either under the first or second category. In regard to the
information falling under the first category, there is also a special responsibility upon
public authorities to suo moto publish and disseminate such information so that they will
be easily and readily accessible to the public without any need to access them by having
recourse to section 6 of RTI Act. There is no such obligation to publish and disseminate
the other information which falls under the second category.
32.
The information falling under the first category, enumerated in sections 4(1)(b) &
(c) of RTI Act are extracted below :
“4. Obligations of public authorities.-(1) Every public authority shall-(a) xxxxxx
(b) publish within one hundred and twenty days from the enactment of this
Act,-(i) the particulars of its organisation, functions and duties;
(ii) the powers and duties of its officers and employees;
(iii) the procedure followed in the decision making process, including
channels of supervision and accountability;
(iv) the norms set by it for the discharge of its functions;
(v) the rules, regulations, instructions, manuals and records, held by it or
under its control or used by its employees for discharging its functions;
(vi) a statement of the categories of documents that are held by it or under
its control;
(vii) the particulars of any arrangement that exists for consultation with, or
representation by, the members of the public in relation to the formulation
of its policy or implementation thereof;
(viii) a statement of the boards, councils, committees and other bodies
consisting of two or more persons constituted as its part or for the purpose
of its advice, and as to whether meetings of those boards, councils,
committees and other bodies are open to the public, or the minutes of such
meetings are accessible for public;
(ix) a directory of its officers and employees;
(x) the monthly remuneration received by each of its officers and
employees, including the system of compensation as provided in its
regulations;
(xi) the budget allocated to each of its agency, indicating the
particulars of all plans, proposed expenditures and reports on
disbursements made;
(xii) the manner of execution of subsidy programmes, including the
amounts allocated and the details of beneficiaries of such
programmes;
(xiii) particulars of recipients
authorisations granted by it;
of
concessions,
permits
or
(xiv) details in respect of the information, available to or held by it,
reduced in an electronic form;
(xv) the particulars of facilities available to citizens for obtaining
information, including the working hours of a library or reading room, if
maintained for public use;
(xvi) the names, designations and other particulars of the Public
Information Officers;
(xvii) such other information as may be prescribed; and
thereafter update these publications every year;
(c) publish all relevant facts while formulating important policies or
announcing the decisions which affect public;
(emphasis supplied)
Sub-sections (2), (3) and (4) of section 4 relating to dissemination of information
enumerated in sections 4(1)(b) & (c) are extracted below:
“(2) It shall be a constant endeavour of every public authority to take steps
in accordance with the requirements of clause (b) of sub-section (1) to
provide as much information suo motu to the public at regular
intervals through various means of communications, including
internet, so that the public have minimum resort to the use of this Act
to obtain information.
(3) For the purposes of sub-section (1), every information shall be
disseminated widely and in such form and manner which is easily
accessible to the public.
(4) All materials shall be disseminated taking into consideration the cost
effectiveness, local language and the most effective method of
communication in that local area and the information should be easily
accessible, to the extent possible in electronic format with the Central
Public Information Officer or State Public Information Officer, as the case
may be, available free or at such cost of the medium or the print cost price
as may be prescribed.
Explanation.--For the purposes of sub-sections (3) and (4), "disseminated"
means making known or communicated the information to the public
through notice boards, newspapers, public announcements, media
broadcasts, the internet or any other means, including inspection of offices
of any public authority.”
(emphasis supplied)
33.
Some High Courts have held that section 8 of RTI Act is in the nature of an
exception to section 3 which empowers the citizens with the right to information, which
is a derivative from the freedom of speech; and that therefore section 8 should be
construed strictly, literally and narrowly. This may not be the correct approach. The Act
seeks to bring about a balance between two conflicting interests, as harmony between
them is essential for preserving democracy. One is to bring about transparency and
accountability by providing access to information under the control of public authorities.
The other is to ensure that the revelation of information, in actual practice, does not
conflict with other public interests which include efficient operation of the governments,
optimum use of limited fiscal resources and preservation of confidentiality of sensitive
information. The preamble to the Act specifically states that the object of the Act is to
harmonise these two conflicting interests. While sections 3 and 4 seek to achieve the first
objective, sections 8, 9, 10 and 11 seek to achieve the second objective. Therefore when
section 8 exempts certain information from being disclosed, it should not be considered
to be a fetter on the right to information, but as an equally important provision protecting
other public interests essential for the fulfilment and preservation of democratic ideals.
34.
When trying to ensure that the right to information does not conflict with several
other public interests (which includes efficient operations of the governments,
preservation of confidentiality of sensitive information, optimum use of limited fiscal
resources, etc.), it is difficult to visualise and enumerate all types of information which
require to be exempted from disclosure in public interest. The legislature has however
made an attempt to do so. The enumeration of exemptions is more exhaustive than the
enumeration of exemptions attempted in the earlier Act that is section 8 of Freedom to
Information Act, 2002. The Courts and Information Commissions enforcing the
provisions of RTI Act have to adopt a purposive construction, involving a reasonable and
balanced approach which harmonises the two objects of the Act, while interpreting
section 8 and the other provisions of the Act.
35.
At this juncture, it is necessary to clear some misconceptions about the RTI Act.
The RTI Act provides access to all information that is available and existing. This is
clear from a combined reading of section 3 and the definitions of ‘information’ and ‘right
to information’ under clauses (f) and (j) of section 2 of the Act. If a public authority has
any information in the form of data or analysed data, or abstracts, or statistics, an
applicant may access such information, subject to the exemptions in section 8 of the Act.
But where the information sought is not a part of the record of a public authority, and
where such information is not required to be maintained under any law or the rules or
regulations of the public authority, the Act does not cast an obligation upon the public
authority, to collect or collate such nonavailable information and then furnish it to an
applicant. A public authority is also not required to furnish information which require
drawing of inferences and/or making of assumptions. It is also not required to provide
‘advice’ or ‘opinion’ to an applicant, nor required to obtain and furnish any ‘opinion’ or
‘advice’ to an applicant. The reference to ‘opinion’ or ‘advice’ in the definition of
‘information’ in section 2(f) of the Act, only refers to such material available in the
records of the public authority. Many public authorities have, as a public relation
exercise, provide advice, guidance and opinion to the citizens. But that is purely
voluntary and should not be confused with any obligation under the RTI Act.
36.
Section 19(8) of RTI Act has entrusted the Central/State Information
Commissions, with the power to require any public authority to take any such steps as
may be necessary to secure the compliance with the provisions of the Act. Apart from the
generality of the said power, clause (a) of section 19(8) refers to six specific powers, to
implement the provision of the Act. Sub-clause (i) empowers a Commission to require
the public authority to provide access to information if so requested in a particular ‘form’
(that is either as a document, micro film, compact disc, pendrive, etc.). This is to secure
compliance with section 7(9) of the Act. Sub-clause (ii) empowers a Commission to
require the public authority to appoint a Central Public Information Officer or State
Public Information Officer. This is to secure compliance with section 5 of the Act. Subclause (iii) empowers the Commission to require a public authority to publish certain
information or categories of information. This is to secure compliance with section 4(1)
and (2) of RTI Act. Sub-clause (iv) empowers a Commission to require a public authority
to make necessary changes to its practices relating to the maintenance, management and
destruction of the records. This is to secure compliance with clause (a) of section 4(1) of
the Act. Sub-clause (v) empowers a Commission to require the public authority to
increase the training for its officials on the right to information. This is to secure
compliance with sections 5, 6 and 7 of the Act. Sub-clause (vi) empowers a Commission
to require the public authority to provide annual reports in regard to the compliance with
clause (b) of section 4(1). This is to ensure compliance with the provisions of clause (b)
of section 4(1) of the Act. The power under section 19(8) of the Act however does not
extend to requiring a public authority to take any steps which are not required or
contemplated to secure compliance with the provisions of the Act or to issue directions
beyond the provisions of the Act. The power under section 19(8) of the Act is intended to
be used by the Commissions to ensure compliance with the Act, in particular ensure that
every public authority maintains its records duly catalogued and indexed in the manner
and in the form which facilitates the right to information and ensure that the records are
computerized, as required under clause (a) of section 4(1) of the Act; and to ensure that
the information enumerated in clauses (b) and (c) of sections 4(1) of the Act are
Published and disseminated, and are periodically updated as provided in sub-sections (3)
and (4) of section 4 of the Act. If the ‘information’ enumerated in clause (b) of section
4(1) of the Act are effectively disseminated (by publications in print and on websites and
other effective means), apart from providing transparency and accountability, citizens
will be able to access relevant information and avoid unnecessary applications for
information under the Act.
37.
The right to information is a cherished right. Information and right to information
are intended to be formidable tools in the hands of responsible citizens to fight corruption
and to bring in transparency and accountability. The provisions of RTI Act should be
enforced strictly and all efforts should be made to bring to light the necessary information
under clause (b) of section 4(1) of the Act which relates to securing transparency and
accountability in the working of public authorities and in discouraging corruption. But in
regard to other information,(that is information other than those enumerated in section
4(1)(b) and (c) of the Act), equal importance and emphasis are given to other public
interests (like confidentiality of sensitive information, fidelity and fiduciary relationships,
efficient operation of governments, etc.). Indiscriminate and impractical demands or
directions under RTI Act for disclosure of all and sundry information (unrelated to
transparency and accountability in the functioning of public authorities and 52
eradication of corruption) would be counter-productive as it will adversely affect the
efficiency of the administration and result in the executive getting bogged down with the
non-productive work of collecting and furnishing information. The Act should not be
allowed to be misused or abused, to become a tool to obstruct the national development
and integration, or to destroy the peace, tranquility and harmony among its citizens. Nor
should it be converted into a tool of oppression or intimidation of honest officials striving
to do their duty. The nation does not want a scenario where 75% of the staff of public
authorities spends 75% of their time in collecting and furnishing information to
applicants instead of discharging their regular duties. The threat of penalties under the
RTI Act and the pressure of the authorities under the RTI Act should not lead to
employees of a public authorities prioritising ‘information furnishing’, at the cost of their
normal and regular duties.
Conclusion
38.
In view of the foregoing, the order of the High Court directing the examining
bodies to permit examinees to have inspection of their answer books is affirmed, subject
to the clarifications regarding the scope of the RTI Act and the safeguards and conditions
subject to which ‘information’ should be furnished. The appeals are disposed of
accordingly.
***
ANDHRA PRADESH PUBLIC SERVICE COMMISSION
IN THE SUPREME COURT OF INDIA
CIVIL APPELLATE JURISDICTION
CIVIL APPEAL NO.2244 OF 2009
[Arising out of SLP (Civil) No.18308 of 2008]
D.D. 08.04.2009
Hon’ble Mr. Justice S.B.Sinha &
Hon’ble Mr. Justice Cyriac Joseph
A.P. Public Service Commission
Vs.
Baloji Badhavath & Ors.
…
Appellant
…
Respondents
Recruitment:
Whether short-listing of candidates in the ratio of 1:50 to the total number of vacancies
available at the material time irrespective of community for Group-I Services as per
G.O.Ms. No.570 dated 31.12.1997 is contrary to principles of natural justice and Articles
14 and 16 of the Constitution? – No.
As per notification dated 27.12.2007, 196 vacancies were notified for recruitment
for Group-I Services direct recruitment – 1,68,000 candidates applied – On the basis of
the marks secured in the Preliminary Examination 9,800 candidates were short-listed in
the ratio of 1:50 for the main examination – Respondents who appeared for Preliminary
Examination were not short-listed – They approached State Administrative Tribunal by
filing original application which was dismissed – Writ Petition filed by the respondents
was allowed with a direction to the Commission to prepare a statement showing the ratio
as also categorywise data of candidates permitted to appear for main examination –
Aggrieved by the same this appeal was filed by P.S.C. before the Supreme Court
contending among others that Article 16(4) of the Constitution providing for reservation
is an enabling provision and as such no writ of mandamus could be issued, the impugned
judgment would be detrimental to the interests of the meritorious candidates belonging to
the reserved category; respondents (writ petitioners) having failed to qualify in the
preliminary examination are estopped from questioning the validity GOMs No.570 or
notification dated 27.12.2007 etc. – On behalf of the respondents it was contended that a
candidate only by appearing in an examination cannot waive his fundamental or statutory
right and the State having not preferred any S.L.P. Public service Commission cannot be
said to have any locus standi to maintain this appeal – The Supreme Court after elaborate
discussion with reference to the several constitutional provisions allowed the appeal and
set aside the judgment of the High Court.
Held:
The provisions for holding the preliminary examination are for the purpose of
maintaining a basic standard. If categorywise statement is prepared as directed by the
High Court it may be detrimental to the interest of meritorious candidates belonging to
reserved category. The reserved category candidates have two options. If they are
meritorious enough to compete with the open category candidates, they are recruited in
that category. The candidates below them would be considered for appointment in the
reserved categories as laid down in Union of India & Anr. V. Satya Prakash & Ors.
(2006) 4 SCC 550, Ritesh R. Shah v. Dr. Y.L. Yamul (1996) 2 SCR 695 and R.K. Daria
v. Rajasthan Public Service Commission, (2007) 8 SCC 785.
Further held:
Reservation of women, handicapped etc. is on a horizontal basis. Reservation to
the category of candidates has to be given irrespective of the class or category of
candidates. If such a procedure, as directed by the High Court is followed the same would
give rise to a complexity.
Further held:
So far as the question of locus standi of the Public Service Commission to file
S.L.P. is concerned, it is held that the Commission has locus standi as the High Court not
only has set aside the GOMs dated 31.12.1997 but also set aside the notification dated
27.12.2007. (The decision in A.P. Public Service Commission v. P. Chandra
Mouleesware Reddy (2006) 8 SCC 330 relied upon by the respondents has been
distinguished)
Cases Referred:
(1955) 2 SCR 1 - Sangram Singh v. Election Tribunal, Kotah, Bhurey Lal Baya
(1968) 1 SCR 721 at 731-733 - C.A. Rajendran v. Union of India & Others
(1976) 2 SCC 310 - State of Kerala and Another v. N.M. Thomas & Ors.
(1980) 4 SCC 62 : (1981) 1 SCR 73 - Thammanna v. K. Veera Reddy Ors.
(1981) 1 SCC 246 - Akhil Bharatiya Soshit Karamchari Sangh (Railway) v. Union of
India & Ors.
1985 (2) APLJ 380 - S. Jafeer Saheb v. State of Andhra Pradesh
1992 Supp (3) SCC 217 - Indra Sawhney and Others v. Union of India & Ors.
(1996) 2 SCR 695 at 700-701 - Ritesh R. Shah v. Dr. Y.L. Yamul
(1996) 11 SCC 742 - Chattar Singh v. State of Rajasthan
(1999) 7 SCC 209 - Ajit Singh and Others (II) v. State of Punjab & Ors.
(2003) 11 SCC 559 - State of Punjab & Ors. v. Manjit Singh & Ors.
(2006) 4 SCC 550 - Union of India and Anr. V. Satya Prakash & Ors.
(2006) 8 SCC 330 - A.P. Public Service Commission v. P. Chandra Mouleesware Reddy
(2006) 10 SCC 261 - Pitta Naveen Kumar and Others v. Raja Narasaiah Zangiti & Ors.
(2007) 8 SCC 785 - R.K. Daria v. Rajasthan Public Service Commission
(2008) 6 SCC 1 - Ashoka Kumar Thakur v. Union of India & Ors.
ORDER
S.B. SINHA, J :
1.
2.
Leave granted.
Andhra Pradesh Public Service Commission is before us aggrieved by and
dissatisfied with a judgment and order dated 23.07.2008 passed by a Division Bench of
the Andhra Pradesh High Court in Writ Petition No. 16029 of 2008.
3.
The Government of Andhra Pradesh used to issue orders laying down norms to be
adopted for filling up of vacancies in Group - I services in the State comprising of Deputy
Collectors, Commercial Tax Officers, Assistant Prohibition and Excise Superintendents,
Assistant Commissioner of Labour, Deputy Superintendent of Police Category-2,
Divisional Fire Officers, District Registrars, Assistant Audit Officer and Assistant
Treasury Officer/ Assistant Accounts Officer. The selection process takes place in two
phases; the first being holding of an examination for the purpose of shortlisting of the
candidates and the second being holding of the main examination followed by interview.
4.
Both for preliminary examination as also the main examination, two criteria used
to be adopted; one for the general category candidates and other for the reserved category
candidates.
5.
Validity of the said procedure came up for consideration before a Division Bench
of the Andhra Pradesh High Court as far back in the year 1984 in S. Jafeer Saheb v. State
of Andhra Pradesh [1985 (2) APLJ 380]. Indisputably, a similar question came up for
consideration again in G. Raju v. Government of A.P. rep. by its Secretary [Writ Petition
No. 24247 of 2004 decided on 31.12.2004]
In S. Jafeer Saheb (supra), the contention of the State was that the reservation of
posts used to be made while admitting the candidates for examination itself and not in the
final selection in the ratio of 1:15. The question which, thus, posed, was as to whether
admission of candidates for the main examination by applying compensatory preference
even at the stage of admission in the main examination is violative of Articles 14 and 16
of the Constitution of India. Taking note of the provisions contained in Articles 14, 16
and 335 of the Constitution of India, the High Court held:
"11.
The purpose of holding a screening test is to ensure the basic
standard of eligibility of the candidates and even at the stage of admission
to the main examination the rule of reservation of posts cannot be applied.
Reservation for applicants is not permissible under Art. 16(4).
12.
The learned Advocate-General submits that if reservation rule is
not followed even at the stage of admitting candidates for the main
examination, a reserved vacancy is likely to remain unfilled. It is nobody's
case that as many candidates as there are reserved vacancies have not been
qualified for the main examination. Is there any rule of relaxation based on
reservation for a pass in the H.S.C. Examination or Intermediate
Examination or B.A. Examination? There can be no relaxation or waiver
of a basic standard of performance. There can be no compromise with the
maintenance of administrative efficiency which is barred by Art. 335 of
the Constitution."
It was furthermore held:
"14. Time is now ripe for Courts to lay down the limits to the lowering
of standards for the purpose of compensatory preference. The intensity of
compensatory preference cannot be at the expense of even-handedness and
merit and cannot proliferate to such an extent as to prove fatal to the basic
proficiency and efficiency. The intensity must vary depending on the
nature of the compensatory discrimination whether it is primarily for
individual benefit or whether the quality of public service is directly
affected. Krishna Iyer, J., observed in State of Kerala v. N.M. Thomas that
"to relax basic qualification is to compromise with the minimum
administrative efficiency and is presumably barred by Art. 335".
Lowering of standards for the purpose of compensatory discrimination is
limited to competing commitments to efficient administration.
Public interest demands concern for quality and prohibits waiver
or abandonment of quality. In Janki Prasad v. State of Jammu & Kashmir,
the Supreme Court observed that the setting of absurdly low minimal
scores made it a "travesty of selection".
***
***
***
16...When a candidate is ineligible or does not come up to a basic
standard, no relaxation can be granted. As already pointed out such
concession of preference based on reservation is not granted for a pass in
the H.S.C. Examination or Intermediate Examinat6ion or BA Degree
Examination, because they are eligibility test and not proficiency test."
6.
Indisputably, pursuant to or in furtherance of the said judgment of the High Court
dated 28.12.1984, the State of Andhra Pradesh issued fresh G.O.Ms. No. 570 dated
31.12.1997, providing that the candidates who had applied for Group - I services would
be shortlisted based on a preliminary examination (Screening Test) in the ratio of 1:50 to
the total number of vacancies available at the material time irrespective of community,
the relevant portions whereof read as under:
"...The number of candidates to be admitted to the Written examination
(Convention Type) would be 50 (fifty) times to the total number of
vacancies available at material time irrespective of communities.
3.
The papers except paper 2, i.e., General English may be answered in
English or Telugu or Urdu chosen by the candidates.
However, a
candidate is not permitted to write part of the paper in English and part of
it in Telugu.
4.
The paper on General English is a qualifying one and the standard
of this paper is that of S.S.C. The minimum qualifying marks in this paper
are 40% for O.Cs. 35% for B.Cs. and 30% for SC/STs and P.H. These
marks are not counted for ranking.
5.
In the event of the S.C. and S.T. candidates not coming up for
selection with the existing minimum prescribed for the selection in the
competitive examination conducted by the commission, their selection
shall be considered on the basis of rank with reference to their
performance in the written and Oral competitive examination."
7.
Indisputably, when in terms of the said GOMs, a notification in Advertisement No.
21 of 2003 calling applications for Group - I Services was issued in the year 2003,
another writ application came to be filed by G. Raju and seven others questioning the
legality thereof.
The Andhra Pradesh High Court by a judgment an order dated 31.12.2004 passed in
Writ petition No. 24247 of 2004, opined:
"13.
The contention of the learned counsel for the petitioners is that at
least the ratio of 1:50 should be maintained in respect of each post
reserved for community reservation, in such an event, it will enable the
reserved candidates to effectively participate in the selection and
candidates from out of them would be selected within the reservation
category, but this contention though appears to be appealing, cannot be
accepted. There cannot be any upper limit. If this has to be taken into
consideration, then 1:50 ratio should be considered to be appropriate and
reasonable, and inasmuch as it is assured by the A.P.P.S.C. that there will
not be any carry forward vacancies, we are not inclined to interfere with
the order passed by the Tribunal."
However, in that writ petition, the validity of GOMs dated 31.12.1997 was not in
question.
8.
A notification bearing No. 31 of 2007 was issued for Group - I Services Direct
Recruitment (General) on 27.12.2007 notifying 196 vacancies; inter alia laying down:
"3.
Caste & Community: Community Certificate issued by the
competent authority in terms of G.O.Ms No. 58, SW(J) Dept., dt: 12/5/97
should be submitted at appropriate time. As per General Rules for State
and Subordinate Service Rules, Rule 2(28) Explanation: - No person who
professes a religion different from Hinduism shall be
deemed a
member of scheduled caste. BCs, SCs & STs belonging to other States are
not entitled for reservation, candidates belonging to other States shall pay
the prescribed fee of Rs. 120/- (One hundred and Twenty only) through
I.P.O. Otherwise such applications will not be considered and no
correspondence on this will be entertained.
4.
Reservation for local candidates is not applicable as per concerned
Departmental Special Rules, except Post Code - 8, i.e., AAO in AP State
Audit Service.
5.
Reservation and eligibility in terms of General Rule 22 & 22-A of
A.P. State and Subordinate Service Rules are applicable.
***
***
***
10.
The Reservation to Women will apply as per General Rules. For
P.C. No. 066, women candidates are not eligible."
Relaxation of age was granted for the SC/ST and BCs candidates by five years. The
reserved categories of candidates were exempted from payment of fees.
9.
Pursuant to or in furtherance of the said notification, 1,68,000/- candidates applied.
A preliminary examination was held for all the candidates. Having regard to the ratio of
the number of candidates to be admitted for main examination being 1: 50, 9,800
candidates were shortlisted to take the main examination.
10.
Respondents herein appeared at the said preliminary examination without any
demur whatsoever. They, however, having not been shortlisted filed an original
application before the Andhra Pradesh State Administrative Tribunal. The said original
application was dismissed.
11.
Aggrieved by and dissatisfied therewith, they filed a writ petition before the
Andhra Pradesh High Court which by reason of the impugned judgment has been
allowed. The Commission was impleaded as a party in the writ petition.
The High Court, however, directed the Commission to prepare a statement showing
the ratio as also category-wise data of the candidates permitted to appear for the Main
examination as per the Commission. Despite noticing the ratio laid down, in regard to
certain category of candidates, as for example, OC, BC(C), BC(E), women and
physically handicapped candidates, in its earlier decisions in S. Jafeer Saheb (supra) as
also in G. Raju (supra), it was held that in the said case as the ratio of the candidates in
respect of those categories fell much short of 1:15 ratio, the said GOMs dated 31.12.1997
as also the notification dated 27.12.2007 were held to be ultra vires Articles 14 and 16 of
the Constitution of India.
It was stated that both in S. Jafeer Saheb (supra) as also in G. Raju (supra), the
GOMs No. 570 dated 31.12.1997 was not challenged at all.
The notification dated 27.12.2007 was said to be contrary to the principles of
natural justice as also Articles 14 and 16 of the Constitution of India on the following
premise:
(i)
Non-implementation of community-wise reservation attracts the
wrath of Article 16 of the Constitution of India.
(ii)
The right of reservation must be recognized at all levels.
Although S.Jafeer Saheb (supra) as also G. Raju (supra) lay down good
law but as community - wise reservation did not fall for consideration
therein, the said decisions were not binding upon it.
(iii) Non-fixation of a cut-off mark for each category of community
would also be violative of Articles 14 and 16 of the Constitution of India.
12.
Mr. L. Nageshwara Rao, learned senior counsel appearing on behalf of the
appellant and Mr. R. Sundaravaradhan, learned senior counsel appearing on behalf of the
respondent - State, would contend:
(i)
As Article 16(4) of the Constitution of India provides for an
enabling provision, no writ of mandamus could be issued.
(ii) Right of the candidates being only to be considered for selection
in terms of the extant rules, the High Court committed an error in passing
the impugned judgment.
(iii) The impugned judgment would be detrimental to the interests of
the meritorious candidates belonging to the reserved category.
(iv) The writ petitioners - respondents having failed to qualify in the
preliminary examination, are estopped and precluded from questioning the
validity of GOMs No. 570 or the notification dated 27.12.2007 issued by
the appellant - Commission.
(v)
Reservation for women and physically handicapped persons
being an incident of horizontal reservation and not a vertical reservation,
the impugned judgment cannot be sustained.
13.
Mr. P.P. Rao, learned senior counsel appearing on behalf of the respondents,
however, while conceding that the writ petitioners - respondents cannot claim any right of
reservation and no writ of mandamus can be issued, contended:
(i)
As the Constitution contemplates upliftment of weaker sections
by providing a percentage of seats for employment in the State and having
regard to the fact that both clauses (1) and (4) of Article 16 of the
Constitution of India provide for valid classification, the impugned
judgment should not be interfered with.
(ii)
Judging of the merit of the candidates having regard to the
provisions of Article 335 of the Constitution of India per se should not
allow the State and the Commission to stop all the candidates at the first
gate and then prevent them from appearing at the main examination as
thereby constitutional scheme to provide reservation would be frustrated
to a great extent.
(iii) The means to achieve the constitutional object and the goals
should not be defeated by inserting procedural provisions as a result
whereof what is being given by one hand should not be permitted to be
taken away by the other.
(iv) Reservation made in favour of women, physically handicapped,
etc, although pertains to horizontal reservation, the candidates of the said
categories cannot be treated differently.
(v) A candidate only by appearing in an examination cannot waive
his fundamental or a statutory right.
(vi) The State having provided for reservation in terms of Rules 22
and 22-A of the Andhra Pradesh State and Subordinate Service Rules,
1996 and furthermore, such a right having also been created in terms of
Regulation 14-A of the Andhra Pradesh Public Service Commission
Regulations, 1963, the writ petitioners - respondents obtained an
indefeasible right for consideration of their candidature so as to enable
them to compete with other candidates for appointment in the said post
which cannot be permitted to be taken away by reason of the impugned
GOMs No. 570 and the notification dated 27.12.2007.
(vii) The expression "irrespective of communities" used in GOMs No.
570 even otherwise cannot be read in such a manner so as to violate the
constitutional scheme, as reservation is not based on any religion or race,
particularly in view of the fact that the makers of the Constitution of India
thought of a casteless and classless society.
(viii) The High Court having declared the GOMs No. 570 as
unconstitutional and the State having not preferred any special leave
petition thereagainst, the Andhra Pradesh Public Service Commission
cannot be said to have any locus standi to maintain this appeal.
14.
The vacancies which were to be filled up by the State pertained to Group - I
services. The State indisputably subject to the constitutional limitations having regard to
its power contained in the proviso appended to Article 309 of the Constitution of India is
entitled to frame rules laying down the mode and manner in which vacancies are to be
filled up.
15.
If the State has the legislative competence to frame rules, indisputably, it can
issue governmental orders in exercise of its power under Article 162 of the Constitution
of India. It adopted one procedure. It was held to be ultra vires by the Andhra Pradesh
High Court in S. Jafeer Saheb (supra). It attained finality. The State amended the
procedure in the light of the said decision by GOMs No. 570 dated 31.12.1997. No new
policy was laid down which can be said to be contrary to or inconsistent with the decision
of the Andhra Pradesh High Court in S. Jafeer Saheb (supra). A notification containing
similar provisions issued by the Andhra Pradesh Public Service Commission in 2003 was
questioned. It may be true that the validity of the GOMs itself was not questioned but if
the terms of the notification were held to be unconstitutional, GOMs could have also
been declared as such.
16.
The Division Bench of the High Court indisputably was bound by the said
decision. It, however, proceeded to examine the constitutionality of the GOMs dated
31.12.1997 and the notification dated 27.12.2007 inter alia on the premise that the
validity of the said GOM and the notification was not tested on the touchstone of Articles
14 and 16 of the Constitution of India.
17.
The Constitution of India lays down provisions both for protective discrimination
as also affirmative action. Reservation of posts for the disadvantaged class of people as
also seats in educational institutions are provided for by reason of Articles 15 and 16 of
the Constitution of India. Reservation made for the members of the Scheduled Castes,
Scheduled Tribes and other Backward Classes would, however, is subject to Article 335
of the Constitution of India. Concededly, no citizen of India can claim reservation as a
matter of right. The provisions contained in Articles 15 and 16 of the Constitution of
India are merely enabling provisions. No writ of or in the nature of mandamus, thus,
could be issued. [See C.A. Rajendran v. Union of India & Others, (1968) 1 SCR 721 at
731-733, Indra Sawhney and Others v. Union of India and Others [1992 Supp (3) SCC
217, para 165 to 169, 428 to 432, 741 and 742, Ajit Singh and Others (II) v. State of
Punjab and Others (1999) 7 SCC 209, para 32 to 39, State of Punjab and Others v. Manjit
Singh and Others (2003) 11 SCC 559, para 7 and 12]
18.
The State, however, have made provisions for reservation. Indisputably, the
appellant - Commission has made regulations known as Andhra Pradesh Public Service
Commission Regulations, 1963, Regulation 14-A whereof reads as under:
"It shall be necessary for the Commission in the matter of recruitment
to the posts and services to strictly adhere to wherever applicable the
provisions contained:
(i)
in the General Rule 22 and 22-A;"
19.
The rules of procedure for holding the said examination have also been prescribed
known as the Andhra Pradesh Public Service Commission Rules of Procedure; Rule 4
whereof reads as under:
"4.
Where a direct recruitment is to be made by selection, i.e.,
after interview only, and where the number of qualified petitioners is
unduly large having regard to the actual number of vacancies available,
the Commission may restrict the number of candidates to be called for
interview to such extent as it may deem fit. Such shortlisting may be done
by the Commission either by holding a written test or on the basis of
preferential or higher qualifications and experience and after taking into
account the requirements with reference to the Rules 22, 22-A of the
General Rules for State and Subordinate Services and the Rule of
reservation in favour of local candidates where they are applicable."
20.
The State of Andhra Pradesh in exercise of its power conferred upon it by the
proviso appended to Article 309 of the Constitution of India framed rules known as the
Andhra Pradesh State and Subordinate Service Rules, 1996. Rules 22 and 22-A thereof
indisputably provide for reservation for appointments to a service, class or category in
favour of Scheduled Castes, Scheduled Tribes, Backward Classes, women, physically
handicapped, meritorious sportsmen, ex-servicemen, etc. Special rules and adhoc rules
have also been made for reservation of women by way of Rule 22-A thereof, stating:
"22-A. Women reservation in appointment: Notwithstanding anything
contained in these rules or special rules or Adhoc Rules:
(1)
In the matter of direct recruitment to posts for which women
are better suited than men, preference shall be given to women:
Provided that such absolute preference to women shall not
result in total exclusion of men in any category of posts.
(2)
In the matter of direct recruitment to posts for which women
and men are equally suited, there shall be reservation to women to an
extent to 33 1/3% of the posts in each category of Open Competition,
Backward Classes (Group - A), Backward Classes (Group - B), Backward
Classes (Group - C), Backward Classes (Group - D), Scheduled Castes,
Scheduled Tribes and Physically Handicapped and Ex-servicemen
quota:..."
21.
Appellant - Commission which has been constituted in terms of Article 315 of the
Constitution of India is bound to conduct examination for appointment to the services of
the State in terms of the Rules framed by the State. It is, however, free to evolve
procedure for conduct of examination.
While conducting the examination in a fair and
transparent manner as also following known principles of fair play, it cannot completely
shut its eyes to the constitutional requirements of Article 335 of the Constitution of India,
which reads as under:
"335 - Claims of Scheduled Castes and Scheduled Tribes to services
and posts
The claims of the members of the Scheduled Castes and
Scheduled Tribes shall be taken into consideration, consistently with
maintenance of efficiency of administration, in the making
appointments to services and posts in connection with the affairs of
Union or of a State:
the
the
of
the
Provided that nothing in this article shall prevent in making of any
provision in favour of the members of the Scheduled Castes and the
Scheduled Tribes for relaxation in qualifying marks in any examination or
lowering the standards of evaluation, for reservation in matters of
promotion to any class or classes of services or posts in connection with
the affairs of the Union or of a State."
In State of Kerala and Another v. N.M. Thomas and Others [(1976) 2 SCC 310],
this Court opined:
"41. Article 335 of the Constitution states that claims of members of
the Scheduled Castes and scheduled tribes shall be taken into
consideration in the making of appointments to the services and posts in
connection with affairs of the State consistent with the maintenance of
efficiency of administration. The impugned rule and the impugned orders
are related to this constitutional mandate. Without providing for relaxation
of special tests for a temporary period it would not have been possible to
give adequate promotion to the lower division clerks belonging to
Scheduled Castes and scheduled tribes to the posts of upper division
clerks. Only those lower division clerks who were senior in service will
get the benefit of the relaxation contemplated by Rule 13-AA and the
impeached orders. Proportion to upper division from lower division is
governed by the rule of seniority subject only to passing of the qualified
test. The temporary relaxation of test qualification made in favour of
Scheduled Castes and scheduled tribes is warranted by their inadequate
representation in the services and their overall backwardness. The
classification of the members of the Scheduled Castes and scheduled tribes
already in service made under Rule 13-AA and the challenged orders for
exempting them for a temporary period from passing special tests are
within the purview of constitutional mandate under Article 335 in
consideration of they claims to redress imbalance in public service and to
bring about parity in all communities in public services."
22.
How the Commission would judge the merit of the candidates is its function.
Unless the procedure adopted by it is held to be arbitrary or against the known principles
of fair play, the superior courts would not ordinarily interfere therewith. The State framed
Rules in the light of the decision of the High Court in S. Jafeer Saheb (supra). Per se, it
did not commit any illegality. The correctness of the said decision, as noticed
hereinbefore, is not in question having attained finality. The matter, however, would be
different if the said rules per se are found to be violative of Article 16 of the Constitution
of India. Nobody has any fundamental right to be appointed in terms of Article 16 of the
Constitution of India. It merely provides for a right to be considered therefor. A
procedure evolved for laying down the mode and manner for consideration of such a right
can be interfered with only when it is arbitrary, discriminatory or wholly unfair.
23.
We may notice that in Chattar Singh v. State of Rajasthan [(1996) 11 SCC 742],
Rule 13 of the Rajasthan State and Subordinate Services (Direct Recruitment by
Combined Competitive Examinations) Rules, 1962 prescribing the mode of conducting
preliminary as also main examination had been interpreted, opining :
"...What requires to be done is that the Public Service Commission
has to consider the number of vacancies notified or likely to be filled in
the year of recruitment for which notification was published. Then
candidates who had appeared for the Preliminary Examination and
qualified for Main Examination are to be screened by the test. The object
is to eliminate unduly long list of candidates so that opportunity to sit for
Main Examination should be given to candidates numbering 15 times the
notified posts/vacancies in various services; in other words for every one
post/vacancy there should be 15 candidates. There would be wider scope
to get best of the talent by way of competition in the examination. The
ultimate object is to get at least three candidates or as is prescribed, who
may be called for viva voce. Therefore, the lowest range of aggregate
marks as cut-off for general candidates should be so worked out as to get
the required number of candidates including OBCs, Scheduled Castes and
Scheduled Tribes. The lowest range would, therefore, be worked out in
such a way that candidates numbering 15 times the notified
posts/vacancies would be secured so as to afford an opportunity to the
candidates to compete in the Main Examination.
15. Under the proviso, if that range has not been reached by the
candidates belonging to the SCs or the STs, there may be 5% further cutoff from the last range worked out for the general candidates so as to
declare them as qualified for appearing in the Main Examination. In other
words, where candidates belonging to the SCs and STs numbering 15
times the total vacancies reserved for them are not available then the
Service Commission has to go down further and cut off 5% of the marks
from the lowest of the range prescribed for general candidates and then
declare as eligible the SC and ST candidates who secured 5% less than the
lowest range fixed by PSC for general candidates so as to enable them to
appear for the Main Examination. The candidates who thus obtain
qualifying marks are eligible to appear and write the Main Examination.
The respective proportion of 1:3 or as may be prescribed and candidates
who qualified in the Main Examination will be called by the Commission,
in their discretion, for interview. The Commission shall award marks to
each candidate interviewed by them, having regard to their character,
personality, address, physique and knowledge of Rajasthani culture as is in
vogue as per rules. However, for selection to the Rajasthan Police Service,
candidates having `C' Certificate of NCC will be given preference. The
marks so awarded shall be added to the marks obtained in the Main
Examination by each such candidate.
16. In working out this procedure, if the minimum of 15 times of the
candidates are identified and results declared, it would not be necessary to
pick up more General/Reserved candidates. It would not be necessary to
declare the result of more than 15 times the total notified vacancies/posts
so as to enable them to compete in the Main Examination. The object of
screening test is to eliminate unduly long number of persons to appear for
Main Examination. If more candidates are called by declaring their result
in Preliminary Examination, the object of Rule 13 would be frustrated."
24.
The Commission contends that in all the past examinations held, the vacancies
pertaining to the reserved categories had been filled up. The vacancies were not required
to be carried forward as sufficient numbers of candidates belonging to the reserved
category were available. It is in the aforementioned context, a statement was made in G.
Raju (supra) that the Commission shall not carry forward the vacancies.
In Pitta Naveen Kumar and Others v. Raja Narasaiah Zangiti and Others [(2006) 10
SCC 261], this Court held:
"52. The authority of the State to frame rules is not in question. The
purport and object for which the said notifications were issued also cannot
be said to be wholly arbitrary so as to attract the wrath of Article 14 of the
Constitution of India. The appellants herein no doubt had a right to be
considered but their right to be considered along with other candidates had
not been taken away. Both the groups appeared in the preliminary
examination. Those who had succeeded in the preliminary examination
were, however, allowed to sit in the main examination and the candidature
of those had been taken into consideration for the purpose of viva voce
test who had passed the written examination."
25.
Indisputably, the preliminary examination is not a part of the main examination.
The merit of the candidate is not judged thereby. Only an eligibility criterion is fixed. The
papers for holding the examination comprise of General Studies and Mental Ability. Such
a test must be held to be necessary for the purpose of judging the basic eligibility of the
candidates to hold the tests. How and in what manner the State as also the Commission
would comply with the constitutional requirements of Article 335 of the Constitution of
India should ordinarily not be allowed to be questioned.
The proviso appended to Article 335 of the Constitution, to which our attention has
been drawn by Mr. Rao, cannot be said to have any application whatsoever in this case.
Lowering of marks for the candidates belonging to the reserved candidates is not a
constitutional mandate at the threshold.
promotion.
It is permissible only for the purpose of
Those who possess the basic eligibility would be entitled to appear at the
main examination. While doing so, in regard to General English whereas the minimum
qualifying marks are 40% for OCs, it would be 35% for BCs and 30% for SC/STs and
physically handicapped persons. However, those marks were not to be counted for
ranking. We have noticed hereinbefore, that candidates belonging to the reserved
categories as specified in the notification are not required to pay any fee. Their age is
relaxed upto five years. It is, therefore, not correct to contend that what is given by one
hand is sought to be taken by another. They can, thus, appear in the examination for a
number of times. Indisputably, the right conferred upon the writ-petitioners - respondents
in terms of Rules 22 and 22-A of the Andhra Pradesh State and Subordinate Service
Rules, 1996 was to be protected. The extent of relaxation has been recognized. By reason
of such a provision, the right to be considered has not been taken away.
26.
Judging of merit may be at several tiers. It may undergo several filtrations.
Ultimately, the constitutional scheme is to have the candidates who would be able to
serve the society and discharge the functions attached to the office. Vacancies are not
filled up by way of charity. Emphasis has all along been made, times without number, to
select candidates and/ or students based upon their merit in each category. The
disadvantaged group or the socially backward people may not be able to compete with
the open category people but that would not mean that they would not be able to pass the
basic minimum criteria laid down therefor.
27.
We have been informed at the bar that 12 handicapped people have appeared at the
main examination and, thus, it would be possible to fill up the posts even from that
category of candidates.
28.
Submission of Mr. Rao that by reason of such a process, only the forward sections
of backward classes who have the advantage of undergoing coaching classes, etc. would
be given preference may not be correct. No statistical data had been placed before the
High Court or before us. We have not been furnished any information as to on what basis,
such a contention could be raised. All the candidates are highly educated. Merit is not
the monopoly of people living in urban areas. The State must adopt some criteria. It
having regard to its limited resources cannot allow lakhs and lakhs of candidates to
appear at the examination as also at the interview. It is bound to devise some procedure to
shortlist the candidates. Rule 4 of the Andhra Pradesh Public Service Commission Rules
of Procedure which refers to Rules 22 and 22-A of the Andhra Pradesh State and
Subordinate Service Rules, 1996 would apply only where shortlisting is done. The first
part of the said Rule empowers the Commission to restrict the number of candidates to be
called for interview to such extent as it may deem fit. While shortlisting, however, it may
hold a written test or provide for a preferential or higher qualification and experience and
only for that purpose it is required to take into account the requirements with reference to
Rules 22 and 22A of the Andhra Pradesh State and Subordinate Service Rules, 1996 and
the rule of reservation in favour of local candidates.
29.
Strong reliance has been placed by Mr. Rao on Sangram Singh v. Election
Tribunal, Kotah, Bhurey Lal Baya [(1955) 2 SCR 1] wherein Vivian Bose. J., stated as
under:
"Now a code of procedure must be regarded as such. It is procedure,
something designed to facilitate justice and further its ends: not a penal
enactment for punishment and penalties; not a thing designed to trip
people up. Too technical a construction of sections that leaves no room for
reasonable elasticity of interpretation should therefore be guarded against
(provided always that justice is done to both sides) lest the very means
designed for the furtherance of justice be used to frustrate it."
The said observation was made keeping in view the concept of justice. By reason of
providing for a preliminary examination, the right of the reserved category candidates has
not been taken away. The means cannot be allowed to defeat the ends which the
constitutional scheme seeks to achieve.
30.
Reliance has also been placed by Mr. Rao on Akhil Bharatiya Soshit Karamchari
Sangh (Railway) v. Union of India and Others [(1981) 1 SCC 246] wherein this Court
held:
"76. Proceeding on this footing, the fundamental right of equality of
opportunity has to be read as justifying the categorisation of SCs & STs
separately for the purpose of "adequate representation" in the services
under the State. The object is constitutionally sanctioned in terms, as
Articles 16 (4) and 46 specificate. The classification is just and reasonable.
We may, however, have to test whether the means used to reach the end
are reasonable and do not outrun the purposes of the classification. Thus
the scope of the case is narrowed down."
31.
We have proceeded on the same assumption. What was in question in that case
was as to whether in matter of promotions reservation of posts for SC/ST candidates is
unconstitutional. It is now a settled law that it is not. The said observation inter alia was
in tone with the opinion of Ray, CJ in N.M. Thomas (supra).
The said observations, in our opinion, have no application to the fact of the
present case.
32.
Submission of Mr. Rao is that the expression "irrespective of communities"
has wrongly been used. In support of the said contention reliance has been placed on the
expression "Irrespective of Race" in the Words and Phrases, Permanent Edition, Volume
22A, page 506, which is in the following terms:
"By Act May 2, 1890, c. 182 § 31, 26 Stat. 94, Sections 566, 567,
Mansf. Dig., was extended over Indian Territory with a proviso excepting
Indians and their estates. By act June 7, 1897, C.3, § 1, 30 Stat. 83, such
laws were made to apply to all persons in the territory "irrespective of
race" and by the Curtis Act June 28, 1898, C. 517, § 26, 30 Stat. 504, it
was provided that the laws of the Indian tribes should not longer be
enforced. Held that by virtue of such provisions, a noncitizen husband of a
Creek allottee who died after the birth of a child of the marriage was
entitled by the curtesy to a life estate in her allotted lands. Armstrong v.
Wood, 195 F. 137, 141"
33.
The word "community", however, is understood in different senses, keeping in
view the purport and object for which the said term is used. It may be given a natural
meaning. It may have to be read along with the words `caste' and `religion' in which event
it will have to have a narrower meaning than the dictionary meaning of a body of people
having same common interest. [See Advanced Law Lexicon, 3rd edition, 2005, page 907]
34.
Indisputably, when the said words were used, it took its clue from S. Jafeer Saheb
(supra).
The expression might have been loosely used but its purport and object had been
understood by candidates including the writ petitioners. The provisions for holding the
preliminary examination were for the purpose of maintaining a basic standard. The High
Court has directed deletion of the said words.
35.
Mr. P.P. Rao has relied upon the following paragraph of the decision of this Court
in N.M. Thomas (supra):
"43. Scheduled Castes and scheduled tribes are not a caste within the
ordinary meaning of caste. In Bhaiyalal v. Harikishan Singh12 this Court
held that an enquiry whether the appellant there belonged to the Dohar
caste which was not recognised as a scheduled caste and his declaration
that he belonged to the Chamar caste which was a scheduled caste could
not be permitted because of the provisions contained in Article 341. No
court can come to a finding that any caste or any tribe is a scheduled caste
or scheduled tribe. Scheduled caste is a caste as notified under Article
366(25). A notification is issued by the President under Article 341 as a
result of an elaborate enquiry. The object of Article 341 is to provide
protection to the members of Scheduled Castes having regard to the
economic and educational backwardness from which they suffer."
36.
For the views, we have taken earlier, it is not necessary for us to go into the above
mentioned larger question particularly having regard to the recent decision of this Court
in Ashoka Kumar Thakur v. Union of India and Others [(2008) 6 SCC 1]
37.
One other aspect of the matter must be kept in mind.
If category-wise statement is prepared, as has been directed by the High Court, it
may be detrimental to the interest of the meritorious candidates belonging to the reserved
categories. The reserved category candidates have two options. If they are meritorious
enough to compete with the open category candidates, they are recruited in that category.
The candidates below them would be considered for appointment in the reserved
categories. This is now a well-settled principle of law as has been laid down by this Court
in several decisions. [See for example, Union of India and Anr. V. Satya Prakash and
Ors. (2006) 4 SCC 550, para 18 to 20, Ritesh R. Shah v. Dr. Y.L. Yamul (1996) 2 SCR
695 at 700-701, R.K. Daria v. Rajasthan Public Service Commission, (2007) 8 SCC 785,
para 9]
38.
As we have entered into the merit of the matter, in our opinion, it is not necessary
to determine the question as to whether the writ petitioners - respondents having appeared
in the examination were estopped and precluded from filing the writ petition.
39.
Reservation of women, handicapped etc. is on a horizontal basis. Reservation to
the category of candidates has to be given irrespective of the class or category of
candidates. A final selection has to be made. If such a procedure, as directed by the High
Court, is to be taken recourse to, the same would give rise to a complexity.
40.
So far as the question of locus standi of the appellant to file this Special Leave
Petition is concerned, we are of the opinion that it has the locus standi. The High Court
not only has set aside the GOMs dated 31.12.1997 but it has also set aside the notification
dated 27.12.2007. If the High Court's judgment is to be implemented, a fresh selection
procedure has to be undertaken by the appellant. Furthermore, in terms of Order 41, Rule
4 of the Code of Civil Procedure, the appellate court, in the event, finds merit in the
appeal at the instance of one of the respondents may set aside the entire judgment
although another respondent had not appealed thereagainst.
41.
The Commission had undertaken the task of holding preliminary examination. It
had followed the procedure laid down in its notification issued in this behalf and the
GOMs issued by the State. It, therefore, could maintain a writ petition.
42.
Mr. Rao, however, submits that in order to maintain an appeal, the Commission
should have been a `person aggrieved'. Reliance in this behalf has been placed on
Thammanna v. K. Veera Reddy Ors. [(1980) 4 SCC 62 : (1981) 1 SCR 73] wherein in the
context of the provisions of Section 116C of the Representation of People Act, 1951, it
was held:
"14...As a general proposition, therefore, it may safely be stated that
before a person is entitled to maintain an appeal under Section 116-C, all
the conditions mentioned below, must be satisfied:
(1)
that the subject-matter of the appeal is a conclusive
determination by the High Court of the rights with regard to all or any of
the matters in controversy, between the parties in the election petition,
(2)
that the person seeking to appeal has been a party in the
election petition, and
(3)
that he is a "person aggrieved", that is a party who has
been adversely affected by the determination..."
No exception to the rule can be taken. We are not, however, dealing with the
election petition. We are dealing with a rule laid down by a constitutional authority as
also the State.
43.
Mr. Rao also relied upon the following paragraph of a decision of this Court in
A.P. Public Service Commission v. P. Chandra Mouleesware Reddy [(2006) 8 SCC 330]:
"13. The State of Andhra Pradesh, we may notice, did not question the
order of the Tribunal. The Commission was required to carry out fresh
exercise in compliance with the directions of the Tribunal. For the said
purpose, no fresh selection process was to be undertaken. If the State did
not have any objection to fill up the said posts realising the mistake
committed by it; we fail to see any reason as to why the Commission
should have felt aggrieved by the order of the Tribunal."
In that case, the State had accepted the judgment of the High Court. A mistake on
the part of the State to issue the impugned direction was in question therein. It was in that
context the aforementioned observations had been made.
Therein 19 posts were to be filled up whereas a direction was issued only to fill up
ten posts. The Tribunal directed the State to fill up all 19 posts. The State of Andhra
Pradesh did not question the order of the Tribunal. Even the Commission was not
required to carry out any fresh exercise to comply with the direction of the Tribunal. As
the order of the Tribunal was not found to be unjustified, the High Court refused to
interfere therewith.
44.
The observations were made only in the aforementioned context.
For the reasons aforementioned, the impugned judgment cannot be sustained,
which is set aside accordingly. The appeal is allowed. However, in the facts and
circumstances of the case, there shall be no order as to costs.
***
IN THE ANDHRA PRADESH ADMINISTRATIVE TRIBUNAL AT HYDERABAD
O.A. NO.11030/2008 & O.A. NO.11196/2008
D.D. 23.07.2009
Hon’ble Mr. T.Madan Mohan Reddy, Member (Judicial)
M.Chandrasekhar Goud & Anr.…
Vs.
The Govt. of A.P. & Ors.
…
Applicants
Respondents
Seniority:
Whether seniority list can be prepared according to roster points and not according to
merit secured by the candidates in the selection? – No.
Seniority list has to be prepared as per ranking prepared by the Commission in the
selection process – The applicants who secured 706 marks and 660 marks were selected
to the post of R.T.O. and allotted to Multi Zone II – The 1st respondent published final
seniority list of R.T.Os of Multi zones I and II fixing seniority on the basis of roster
points - The applicants objected to the seniority list on the ground that the seniority list
was prepared on the basis of roster points without following the ranking assigned by the
P.S.C. – Applicants further contended that their names were shown below the persons
who are less meritorious and who have secured lesser marks – After considering the
respective contentions the Tribunal held that respondents should not have split up the list
into two units as Multi Zones I & II and should have furnished the seniority list as per
merit secured in the competitive examination – Consequently allowed the applications
and directed the respondents to prepare seniority list of R.T.O. strictly on the basis of the
State wide selection and State wide merit as per the Notification and also as per
provisions of Rule 33(b) of the A.P. State and Subordinate Service Rules and not
according to the roster points.
Held:
Roster points are only to calculate the quantum of reservation and cannot be made
the basis to grant accelerated seniority.
Further held:
If the selection is made in one selection and the names are arranged according to
merit then person higher in merit cannot be lowered down in seniority.
ORDER
As the issue involved in both the O.As., is one and the same, therefore, both the
O.As., are disposed of by a common order.
The applicant in O.A. 11196/2008 is working as Regional Transport Officer in the
Office of R.T.A., Khairatabad, Hyderabad. The applicant in O.A. No.11030/2008 is
working as Regional Transport Officer, O/o R.T.A., Ibrahimpatnam, Ranga Reddy
District. They are challenging the final seniority list of Regional Transport Officer of
Multi-zones I and II in R.No.4301/C4/2008 dated 5.11.2008 issued by the Transport
Commissioner, Hyderabad stating that the list was prepared contrary to the rules and is
liable to be set aside and also for a direction to the respondents to prepare the seniority
list of Regional Transport Officers strictly on the basis of rank assigned by the Public
Service Commission in view of judgment of the Apex Court and also the judgment in
O.A.No.4991/2005 and batch and also as per Rule 33 (b) of A.P., State and Subordinate
Service Rules.
According to the applicants, the third respondent issued Notification No.21/2003
and 6/2004 for recruitment to the posts included in Group-I Services including Regional
Transport Officers. In pursuance of the above notification, they participated in the
selection process and the applicant in O.A.No.11030/2008 secured 706 marks and the
applicant in O.A.No.11196/2008 secured 660 marks. They were selected for the post of
Regional Transport Officer and they were allotted to Multi Zone II and they joined in the
service and since then they are continuing in posts of Regional Transport Officers.
According to the applicants, the select list was published by the third respondent i.e.,
APPSC. The applicant in O.A.No.11030/2008 was selected against O.C., quota and the
applicant in O.A.No.11196/2008 selected against O.C. (Women) quota, in view of their
merit, while the persons who secured lesser marks and lesser ranking were selected
against other reserved quotas. While so, the first respondent communicated tentative
provisional inter-se-seniority list of Regional Transport Officers of Multi-zones I and II
in Memo No.4301/C4/2008 dated 14.7.2008 wherein the name of the applicant was
included at Sl.No.124.
The first respondent called for objections against the said
seniority list within ten days from the date of receipt of the Memo. According to the
applicant, the said seniority list was prepared according to roster points but not according
to merit secured by the candidates in the selection process conducted by the third
respondent.
So, they submitted their objections to the provisional seniority list on
24.7.2008 explaining the rule position and requesting to prepare the seniority list as per
ranking prepared by the Public Service Commission in stead of roster points.
While the matter stood thus, the first respondent published the impugned final
seniority list of Regional Transport Officers of Multi-zones I and II in
R.No.4301/C4/2008 dated 5.11.2008 fixing the seniority of the applicant at Sl.No.124 on
the basis of roster points without considering any of the objections raised by them which
is clearly illegal and arbitrary and is opposed to Rule 33(b) of the State and Sub-ordinate
Service Rules. According to the applicants, the above seniority list was prepared for
promotion to the next higher post on the basis of roster points, without following the
ranking assigned by the APPSC, which is mandatory, according to the State and
Subordinate Service Rules. According to the applicants, their names were shown below
the persons who are less meritorious and who belong to reserved categories and who
secured lesser marks like the unofficial respondents.
According to the applicants the unofficial respondents, though secured lesser
marks and having less merit were shown above the applicants in the seniority list fixing
the seniority contrary to the A.P. State and Subordinate Service Rules and also the law
established.
The applicants further contended that the impugned seniority list was
prepared as per roster points and not on the basis of rank assigned by the APPSC, which
resulted in persons like the applicants who secured higher ranking were unjustly placed
below the persons who secured lesser ranking.
Further para 6 of the Notification
No.21/2003, the posts were organized into Multi-Zones and the selection will be made on
state wide merit. According to this condition, the APPSC made selection to the post of
R.T.O. State-wide and allotted the candidates to Multi-zones according to preference
given by the candidates. Therefore, fixing the seniority of the applicants below the
unofficial respondents is highly unjustified. Therefore, they filed this O.A. for setting
aside the impugned seniority list dated 5.11.2008 and for directing the respondents to
prepare the seniority list strictly on the basis of rank assigned by the Public Service
Commission.
The Respondents 1 and 2 filed counter affidavit stating that seniority list among
Regional Transport Officers selected by the APPSC by direct recruitment will be fixed by
the APPSC multi zone wise.
Accordingly, A.P. Public Service Commission
communicated the seniority list of both the multi zones in letter No.1432/RS-29/2003
dated 20.12.2007. As per Rule 33(b) of the A.P., State and Subordinate Service Rules,
the order of merit or order of preference indicated in a list of selected candidates prepared
by the Public Service Commission or other selecting authority, shall not be disturbed
inter-se with reference to the candidates position in such list or panel while determining
the seniority in accordance with this rule and notional dates of commencement of
probation to the extent necessary, shall be assigned to the persons concerned, with
reference to the order of merit or order of preference assigned to them in the said list. So,
according to the respondents at the time of appointment, the seniority shall be determined
accordingly.
The respondents further stated about the situation of integrating the
seniority between both the multi zones i.e., Multi Zone-I and Multi Zone-II. The multi
zone seniority has to be finalized without disturbing the seniority of the individuals in the
respective multi zone. Hence, while preparing the integrated seniority between Multi
Zone-I and Multi Zone-II, they followed the procedure stating that the 1st candidate
among Multi Zone-I (Smt. Ch.Sridevi) has to be compared with 1st candidate of Multi
Zone-II (Sri. G.C.Raja Ratnam) and the seniority of direct Recruit Regional Transport
Officers is prepared in accordance with the A.P. State and Subordinate Service Rules,
1996 and the objections raised by the applicant are not tenable and O.A., are liable to be
dismissed.
Heard both sides.
It is not disputed by the respondents that the applicants and unofficial respondents
are selected for the post of Regional Transport Officers in pursuance of selection
Notification dated 21/03 and 6/04 by the third respondent APPSC. Now according to the
applicants, they are more meritorious than the unofficial respondents and the respondents
1 to 3 erroneously shown them below the unofficial respondents giving a go bye to the
merit and following the roster which is illegal. On perusal of the notification issued, it is
clear that the post of R.T.O., is at post code No.4 and as per para 6 of the Notification
with regard to rule of reservation to the local candidates, it is stated that rule of
reservation to the local candidates is not applicable and for the post code 4, 5 and 6 only
posts are organized into multi zones and selection was made on State wide merit. Thus,
this clause of Notification makes it clear that there is no reservation for local candidates
though the posts organized into Multi Zones for administrative convenience. So, the
contentions of the respondents that the APPSC communicated the seniority list of both
the Multi Zones I & II in letter No.1432/RS-29/2003 dated 20.12.2007 and they followed
the same and as per Rule 33 (b) of A.P., State and Subordinate Service Rules it is clear
that the order communicated by the selecting authority, shall not be disturbed and the
contention that they followed the same inter se preparing the seniority list, cannot be
accepted. In fact, in the Notification, it is clearly stated that rule of reservation for the
local candidates is not applicable for the post code No.4, though the posts are organized
into multi zone and selection will be made only on state wide merit. Thus it is clear that
the selection was made only on State wide merit. It is not disputed by the respondents
that the applicant i.e., M.Chandrasekhar Goud in O.A.No.11030/2008, secured 706 marks
while the applicant i.e., Kum.Swapna Devi Reddy in O.A.No.11196/2008 secured 660
marks and it is also an admitted fact that the unofficial respondents in both the O.A., got
lesser marks and it is clear that they are less meritorious than the applicants in these
O.As.
In similar circumstances, C.W.P.No.14465/2004 was filed before the Hon’ble
High Court of Punjab and Haryana in a case between Pardaman Singh Vs. State of
Punjab and others and their Lordships by judgment dated 14.7.2004 held that roster
points are only to calculate the quantum of reservation and cannot be made the basis to
grant accelerated seniority. A roster point therefore, is not a point against which seniority
can be given to the reserved candidate. Against the said judgment, an appeal was filed
before the Apex Court in Civil Appeal No.4455/2008 in the case of Raj Karan Singh Vs.
State of Punjab and Others, their Lordships Justice Sri. A.K.Mathur and Justice Sri.
Dalveer Bhandari in their judgment dated 16.7.2008 held that if the selection is made in
one selection and the names are arranged according to merit then person higher in merit
cannot be lowered down in seniority. In that case one Pardaman Singh (Respondent No.3
therein) filed a Writ Petition, challenging the order dated 14.7.2004 by which he was
granted seniority above Raj Karan Singh, (appellant therein) in the cadre of Vaidyas and
the Government issued a notification stating that both the candidates are selected for the
post of Vaidyas in 1987. As per the recommendation of the selection committee dated
4.6.1987, Pardaman Singh (R-3) was placed at No.9 and Raj Karan Singh, the Appellant
before their Lordships, was placed at Sl.No.10.
But because of the Government’s
instructions dated 5.5.1975 with regard to appointment of the scheduled castes and
scheduled tribes to the effect that out of 50% of the post of Scheduled castes shall be
reserved for Mazhbi Sikhs/Balmikis and in that Raj Karan Singh who belongs to Balmiki
was given appointment against the post of Scheduled caste. Pardaman Singh was given
the next post. Raj Karan Singh made a representation against Pardaman Singh for
assignment of higher seniority and claimed seniority against Pardaman Singh on the
ground that he has been appointed against the first vacancy of scheduled caste Mazbi
Sikhs/Blamikis and therefore, he should be given seniority over Pardaman Singh. That
representation was acceded by the Government and seniority was changed and Raj Karan
Singh was given seniority above Pardaman Singh in the cadre of Scheduled Caste.
Pardaman Singh filed a W.P. and in that W.P. the Division Bench of the High Court
allowed the W.P. and held that as per the selection Committee’s recommendation,
Pardaman Singh has been placed at Sl.No.9 and Raj Karan Singh has been placed at
Sl.No.10 in the merit list. Therefore, the Division Bench held that Pardaman Singh who
was placed higher in merit by selection committee, should be given seniority above Raj
Karan Singh notwithstanding that Raj Karan Singh has been appointed in the first
Scheduled Caste vacancy. So, their Lordships held s mentioned above.
In these circumstances, it is clear that the merit is criteria and the roster point is
only to calculate the quantum of reservation. In the present case, the respondents stated
that they have prepared the seniority list as per roster point.
Thus, the procedure
followed by the respondents is clearly illegal and not correct. Even as per Rule 33 (b) of
A.P., State and Subordinate Service Rules, the order of merit or order of preference
indicated in a list of selected candidates prepared by the Public Service Commission or
other selecting authority, shall not be disturbed inter-se with reference to the candidates
position in such list or panel while determining the seniority in accordance with this rule
and notional dates of commencement of probation to the extent necessary, shall be
assigned to the persons concerned, with reference to the order of merit or order of
preference assigned to them in the said list. But as per letter dated 20.12.2007, the
Secretary of Public service Commission i.e., the third respondent forwarded the list of
selected candidates to the post of R.T.O., for Multi Zones I & II. In the Special Rules, it
is clearly stated that the rule of reservation cannot be applied and the post of R.T.O., is a
State Wide post and the selection will be made on State Wide merit.
The third
respondent should not have split up the list into two units as Multi Zones I & II. They
should have furnished the list as per merit secured in the competitive examination.
In the above facts and circumstances of the case, the impugned seniority list in
R.No.4301/C4/2008 dated 5.11.2008 and consequential errata published on 1.12.2008 are
liable to be set aside and they are accordingly set aside. The respondents are directed to
prepare the seniority list of R.T.O. strictly on the basis of the State wise selection and
state wide merit as per the Notification and also as per provisions of Rule 33(b) of the
A.P., State and Subordinate Service Rules and not according to the roster points, within a
period of eight weeks from the date of receipt of a copy of this order by following the
procedure prescribed in the rules.
The O.As., are disposed of accordingly.
***
ANDHRA PRADESH INFORMATION COMMISSION
HACA Bhavan, Hyderabad
Appeal No.3409/CIC/2009, dated 23.01.2010
Hon’ble Mr. C.D.Arha, Chief Information Commissioner
Sri. P.Sunand
…
Vs.
The Addl.Secretary/
PIO, A.P.P.S.C. & Anr.…
Appellant
Public Authority
R.T.I. Act:
Whether information related to selection process like names of the candidates,
reservation, marks in the written examination, marks in the interview, service allotted
sought in a format can be refused on the ground that it is third party information exempt
from disclosure under R.T.I. Act? – No.
Whether information can be refused on the ground that providing such information would
disproportionately divert the resources of the Public Authority or would be detrimental to
safety or preservation of record as provided under Section 7(9) R.T.I. Act? – Yes.
Appellant sought information regarding Group-I Services (Direct) General &
Limited Recruitments pursuant to notifications in a particular format it was rejected by
the Public Information Officer – Appeal filed before the 1st Appellate Authority was
dismissed – In this appeal before the Information Commission the ground urged for
rejection of the application that the information pertains to third party was rejected –
However, upholding the contention of the Public Service Commission that if the
information is required to be furnished in the format given by the appellant it would mean
substantial diversion of men, material and resources, Information Commission directed to
furnish the information i.e., total marks and in the bracket marks obtained in the
interview along with the names of the candidates to the appellant – Information
Commission also directed the Public Service Commission to put the information on the
website after the publication of the final select list – Information Commission also
directed the Public Service Commission to conduct an enquiry and fix responsibility for
the delay in giving reply to the appellant within 30 days of the application.
ORDER
Sri. P. Sunand, 28, Bal Reddy Nagar, Toli Chowki, Hyderabad 500 008
(PH:9703673308) has filed 2nd appeal dated 07.04.2009 which was received by this
Commission on 08.04.2009 for not getting the information sought by him from the PIO
and Appellate Authority/A.P. Public Service Commission, Hyderabad.
The brief facts of the case as per the appeal and other records received along with
it are that the appellant herein filed an application dated 08.12.2008 before the PIO
seeking the following information regarding Group-I Services (Direct) General &
Limited Recruitments made under the Notification No.21/2003 and its Supplemental
Notification No.6/2004 in the form of a hard copy (Prints or Xerox copies) in the format
suggested below:
1) All the candidates interviewed by the Interview Board under the Chairmanship of Mr.
Y.Venkatarami Reddy grouped into one list and arranged Community and Zone-wise:
Chaired by Mr. Y.Venkatarami Reddy, names of other members in the Interview Board:
Marks obtained
HT No.
Full Name Community Zone
In Written Interview Service
Allotted
if any
2) Similarly, the candidates interviewed by Chairpersons and members of all the other
boards arranged Community and Zone-wise.
3) Also furnish the information regarding the Group-I (General & Limited) recruitment
under the Notification No.10/2004 in the format as suggested above.
As the appellant did not get information from the PIO, filed 1st appeal dated
17.01.2009 before the Appellate Authority/Secretary, APPSC, Hyderabad.
The appellant has earlier preferred 2nd appeal before the Commission which was
returned vide order dated 26.03.2009 in Appeal No.2836/CIC/2009 for the following
defects:
1. The Appellant did not attest the enclosures as required under Rule 4(1) of the
Appeal Procedure Rules issued in GOMs.No.66, GA(I&PR-II) Dept., dated
25.02.2006.
2. The Appellant has not enclosed any proof (speed post/courier receipt) to show
that the 1st appeal has been filed before the 1st Appellate Authority.
Hindsight, he failed to collect an acknowledgment; however he can identify the person to
whom he had handed over the appeal. The Appellant further stated that he received an
order vide Memo No.140/RS-29/09 dated 19.03.2009 from APPSC i.e., 109 days after he
had made the RTI Application and 69 days of filing the 1st appeal from the APPSC
informing that the information he had sought comes under provisions of the RTI Act,
2005 which concerned 3rd party information and thus cannot be revealed; that the PIO
took an incredible 101 days jut to aver that it concerns 3rd party information.
The Appellant further stated that if the Commission finds the lack of an
acknowledgement as untenable, to guide him as to what steps he should take to see that
his application for information reaches the logical conclusion; to direct him whether he
should, by way of abundant caution, file a fresh appeal against the order dated 19.03.2009
with the first Appellate Authority, APPSC.
After examining the material papers available on record, the appeal was taken on
file and notices were sent to the parties concerned directing them to appear before this
Commission for hearing on 22.12.2009.
On 22.12.2009 the case was called. The Appellant was absent and through his
letter dated Nil received in the APIC on 22.12.2009 he has stated that as he is out of town
he cannot attend the hearing. The Respondent/Sri R.Shiva Shankar, Addl. Secretary,
APPSC was present. Keeping in view the sensitivity of the case, it would be prudent to
have a short adjournment enabling the Appellant also to be present and accordingly the
case was adjourned to 19.01.2010.
On 19.01.2010 the case was called. The Appellant and the Respondents/Sri.
M.G.Gopal, Secretary, APPSC and Sri. R.Shiv Shankar, Addl. Secretary were present.
Arguments concluded. The reply given to the Appelant by APPSC through their letter
dated 19.3.2009 has no legs to stand and is quashed. However, the point made by the
Secretary, APPSC that if the format given by the Appellant has to be adhered, it would
mean substantial diversion of men, material and resources as the total number of
candidates are 1218 for Group-II services and 1139 for Group I services. This volume of
work would attract the provision of Sec.7(9). This plea of the Secretary, APPSC is
sustained. However, the appellant desires a list of selected candidates which in the
proforma maintained by the APPSC has details of Sl.No., Regd. No., Community,
Gender, Ph., Status and Total Marks. The Appellant desires that he would like to have of
the total marks break up also indicating the marks obtained in the interview. This extra
effort can be put in by the APPSC for achieving greater transparency and credibility. The
respondents are directed to supply this information i.e., total marks and in the bracket
marks obtained in the interview can be indicated for each of the selected candidates along
with their names. Further, the APPSC is requested to put on their website immediately
once the final selection has been made so as to achieve great transparency and credibility.
It is also noted that the date of the application was 8.12.2008 and the reply sent to the
Appellant by the APPSC was 19.03.2009 i.e., after delay of nearly 100 days. The
Secretary, APPSC is directed to conduct an enquiry and fix responsibility as to why this
delay occurred when under law the respondent is duty bound to send a reply within 30
calendar days and to send copy of the enquiry report conducted by the Secretary to this
Court within 120 days of receipt of this order.
With the above directions, the appeal is closed.
***
ARUNACHAL PRADESH PUBLIC SERVICE COMMISSION
IN THE GAUHATI HIGH COURT
ITANAGAR BENCH
W.P. (C) NO.78 (AP) OF 2009
D.D. 24.06.2009
Hon’ble Mr. Justice B.D.Agarwal
Shri Ojing Siram
Vs.
The State of A.P. & Ors.
…
Petitioner
…
Respondents
Reservation to disabled persons:
Whether a candidate claiming PH reservation is required to produce disability certificate
in the prescribed form? – Yes.
In the recruitment to the post of Sub Treasury Officer petitioner and respondent
No.3 were candidates for the post reserved under PH category – After the written
examination in the merit list of PH candidates respondent No.3 was shown at Sl.No.1 and
petitioner was shown at Sl.No.8 and respondent No.3 was finally selected on the basis of
identity card issued by Deputy Commissioner certifying that he was physically
handicapped person – Aggrieved the petitioner filed this writ petition on the ground that
respondent No.3 was selected without furnishing requisite certificate – As per the notice
issued by the Commission PH persons were required to produce part-B certificate
regarding their disability issued by the competent authority on the basis of Part-A
medical certificate issued by the State Medical Board – Respondent No.3 furnished only
identify card issued by the Deputy Commissioner regarding disability whereas petitioner
produced both part-A and part-B certificates – In view of Office Memorandum dated
29.12.2005 containing guidelines for issuing part-A certificate by Medical Board
consisting of at least three members, out of which, at least one should be a Specialist in
the particular field - District Medical Board which issued Part-B Certificate did not have
any Orthopaedic Surgeon or Specialist, as its members, to assess the orthopaedic
disability of Respondent No.3 - Hence, High Court allowed the writ petition with a
direction to respondent to appear before the Medical Board including Orthopaedic
Surgeon/Specialist and direction to Medical Board to send its report/certificate directly
to the Commission and direction to Commission to reconsider the candidature of
respondent No.3 for selection after receipt of certificate.
Held:
Medical Board should consist of at least three members, out of which, one should
be a Specialist in the particular field to assess whether a persons is physically disabled or
not as defined under Section 2(o) of the Persons with Disabilities (Equal Opportunities,
Protection of Rights and Full Participation) Act, 1995.
Note:
The 3rd respondent aggrieved by the order filed W.A.No.12 (AP) 2009 and the
Appeal was disposed of as per order dated 18.12.2009 reserving liberty to the 3rd
respondent to take recourse to such provision of law as may be permissible, if the result
of the medical examination goes against him.
ORDER
Arunachal Pradesh Public Service Commission (in brief ‘the APPSC’) notified
certain vacancies vide advertisement dated 25.7.2006 inter alia, to the post of Sub-
Treasury Officer.
Both the writ petitioner and Respondent No.3 appeared in the
Combined Competitive Examination for the said jobs under the reserved quota for
physically disabled persons. On the basis of the written examination a separate merit list
of physically handicapped candidates was prepared wherein name of Respondent No.3
was shown at serial No.1 and the writ petitioner’s name was shown at serial No.8. In the
said list, the private Respondent No.3, Shri Rima Taipodia was shown to be
Orthopaedically Handicapped person with 50% disabilities, whereas, the writ petitioner,
Shri Ojing Siram was also shown to be Orthopaedically Handicapped person with 75%
disabilities. The Respondent No.3 was finally selected to the post of Sub-Treasury
Officer, on the basis of identity card issued by the Deputy Commissioner certifying that
the said person was physically handicapped person. On the other hand, the writ petitioner
furnished four documents in support of his disability and the documents included Part-A
and Part-B certificates issued by the Deputy Commissioner and medical Board.
2.
Being aggrieved with the selection of Respondent No.3 in the reserved quota, one
of the unsuccessful candidate i.e. the writ petitioner has challenged Respondent No.3’s
selection and has filed this writ petition basically to quash and set aside the Notification
dated 18.1.2009 (Annexure-IV) whereby the Respondent No.3 has been declared selected
for the post of Sub-Treasury Officer.
3.
I have heard Shri P Taffo, learned counsel for the writ petitioner as well as Shri R
H Nabam, learned Senior Government Advocate, for respondent No.1.
APPSC
(Respondent No.2) was represented by Shri N Tagia, learned counsel, whereas the private
Respondent No.3 was represented by Sri. K Ete, learned counsel. I have also perused the
pleadings, counter pleadings and documents submitted by the parties.
4.
Basically the Act, namely, the Persons with Disabilities (Equal Opportunities,
Protection of Rights and Full Participation) Act, 1995, has been enacted to give equal
opportunities to the physically challenged persons in public employment and also with
the objective that such persons may not be discriminated in public employment, if they
are otherwise suitable for certain specified posts. As per the mandate of the statute, the
APPSC reserved 3% posts for physically disabled persons and there is no dispute to the
fact that the aforesaid law has been followed.
5.
The only dispute is that the APPSC has selected Respondent No.3 without
furnishing requisite certificate by him and also without ascertaining about his physical
disability. From the documents filed along with the writ petition and the additional
affidavit, it appears to me that APPSC had informed the intending candidates by way of a
Notice published in a local newspaper that persons seeking employment, under the
reserved category for physically handicapped persons should submit Part-B Certificate of
such disability, issued by the competent authority on the basis of the Part-A, Medical
Certificate issued by the State Medical Board. However, Respondent No.3 furnished
only the Identity Card issued by the Deputy Commissioner, West Siang as is revealed
from the merit list of handicapped persons. However, according to the learned counsel
for the private respondent, along with Identity Card he had also furnished Part-A
Certificate as well as Passbook and in this way sufficient evidence of his physical
disablement was furnished before the competent authority. Shri K Ete, learned counsel
has also submitted that in view of the order dated 1.8.1998 issued by the Chief Secretary
to the Government of Arunachal Pradesh, the Deputy Commissioners are competent to
issue Identity Card and Disability Certificate on the basis of Part-A Certificate issued by
the DMO or CMO to such persons. Shri Ete, learned counsel further submitted that
Identity Card is virtually one and the same and is at par with the Part-B Certificate and as
such, there was no infirmity with the selection of the Respondent No.3 to the post of SubTreasury Officer.
6.
Admittedly, Part-B Certificate was not furnished by the Respondent No.3 before
APPSC. At the same time, Part-A Certificate of Respondent No.3, which has been
annexed with its affidavit, the District Medical Board did not include any Orthopaedic
Specialist.
7.
The Ministry of Personnel, Public Grievances and Pensions, Government of India,
has issued an Office Memorandum dated 29.12.2005 giving certain guidelines in the
matter of reservation for the persons with disabilities. Under clause 10 of this Office
Memorandum, it has been advised that Disability Certificate shall be issued by a Medical
Board consisting of at least three members, out of which, at least one should be a
Specialist in the particular field. However, as noted earlier, Part-B Certificate issued by
the District Medical Board, which formed the basis of issuing the Identity Card by the
Deputy Commissioner and eventually accepted by APPSC, did not have any Orthopaedic
Surgeon or Specialist, as its members, to assess the orthopaedic disability of Respondent
No.3.
Hence, it appears to me that Respondent No.3 has been selected in the reserved
quota without strict adherence to the guidelines in this regard.
8.
Situated thus, it would be just and proper to direct Respondent No.3 to appear
before the State Medical Board of Arunachal Pradesh within a period of 4 (four) weeks
from today and on such appearance, the said Medical Board would include one
Orthopaedic Surgeon/Specialist to certify whether Respondent No.3, namely, Shri Rima
Taipodia, is a physically disabled person or not, as defined under Section 2(o) of the
Persons with Disabilities (Equal Opportunities, Protection of Rights and Full
Participation) Act, 1995. It is further made clear that the State Medical Board shall send
its report/certificate directly to the APPSC and on receipt of such report/certificate, the
APPSC shall reconsider the candidature of the Respondent No.3 for his selection, to the
post of Sub-Treasury Officer, under the reserved quota.
9.
With the aforesaid directions, the writ petition stands disposed of.
***
IN THE GAUHATI HIGH COURT
ITANAGAR BENCH
W.P. (C) NO.100 (AP) OF 2009
D.D. 25.06.2009
Hon’ble Mr. Justice B.D.Agarwal
Shri Abu Taba
Vs.
The State of A.P. & Ors.
…
Petitioner
…
Respondents
Reservation to disabled persons:
Whether a person suffering from two disabilities each disability less than 40% but
together more than 40% could be selected against a post reserved for either disability? –
No.
In the recruitment for filling up 50 posts in different categories including the post
of Sub Treasury Officer, which was reserved for orthopaedically disabled persons – A
separate merit list for disabled persons was issued wherein details of disability percentage
etc., were mentioned - As per the said list petitioner was shown to be suffering from
Partial Blindness, whereas respondent No.3 was shown to be suffering from
Orthopaedically handicapped and also having speech disability – In the subsequent
certificate issued by Medical Board of General Hospital, Naharlagun on 22.10.2008 the
3rd respondent was selected for the post of Sub Treasury Officer – His selection was
challenged on the ground that he could not have been recommended for the post of Sub
Treasury Officer as the said post was reserved for orthopaedically disabled person
whereas the 3rd respondent was selected under visually disabled quota, respondent No.3
did not suffer from 40% disability which is the minimum requirement of disability under
the Act and that the Medical Certificate showing the applicant as disabled person due to
low vision was challenged on the ground that it is not a genuine one – On examination of
the certificates produced by 3rd respondent it was found that the Medical Board has
calculated orthopaedical disability at 30% and visual disability at 10% and taking the
aggregate the respondent No.3 was certified suffering from 40% disability – After
excluding the percentage of disability under locomotor disability visual disability
suffered by the 3rd respondent being less than 40% the High Court allowed the writ
petition filed by the petitioner and quashed the selection of the 3rd respondent against the
post of Sub Treasury Officer with a direction to the 3rd respondent to appear before the
State Medical Board within 4 weeks with a direction to the State Medical Board to
conduct another vision test of the respondent No.3 and issue appropriate certificate
clearly indicating the percentage of disability due to loss of vision if any reserving liberty
to the Commission to consider the case of the petitioner if he fulfils all the criteria for
recommendation in case respondent No.3 is found not eligible.
Held:
In view of the fact that in the notification one post of Sub Treasury Officer has
been reserved for orthopaedically and partially deaf persons, the 3rd respondent has been
appointed as Sub Treasury Officer on the ground he is visually disabled person against
the reserved quota for orthopaedically disabled person the Court held that the
Commission has committed gross illegality in recommending the name of respondent
No.3 against reserved quota for Orthopaedically handicapped person and imposed cost of
Rs.25,000/- to be paid by the Commission to the writ petitioner.
Note:
After Respondent No.3 was found not eligible for selection under PH category
(visually challenged) the petitioner filed one more writ Petition (W.P. No.4247
(AP)/2009) seeking a direction to the Commission to consider his case for selection in the
low vision category and the High Court as per order dated 1.6.2010 disposed of the writ
petition with a direction to the Commission to consider the candidature of the petitioner
for selection in the post of Entry Grade Cadre in the low vision category.
ORDER
The facts of the case would reflect the poor state of affairs in the office of the
Arunachal Pradesh Public Service Commission (in brief ‘APPSC’) and the facts being
narrated herein below would also reveal that an expert body like APPSC is selecting
candidates by way of superficial examination of documents. It is made clear that I am
not making any observation with regard to the quality of written test and interview being
conducted by the APPSC.
2.
Heard Shri M.Batt, learned counsel for the writ petitioner as well as Shri
R.H.Nabam, learned Senior Government Advocate for respondent No.1; the APPSC
(Respondent No.2) was represented by Shri N Tagia, learned counsel, whereas the private
Respondent No.3 was represented by Sri. A.Appang, learned counsel.
I have also
perused the pleadings as well as the documents for the parties.
3.
At the out set, I would like to mention here that yesterday also this Court had an
occasion to examine the legality of appointment to one Shri Rima Talipodia as SubTreasury Officer against the reserved quota for physically handicapped person. Finding
sufficient materials to doubt about the genuineness of the medical certificate of the said
person this Court had also directed the said Shri Rima Talipodia to undergo fresh medical
test before the State Medical Board of Arunachal Pradesh vide order dated 24.6.2009 in
WP(C) No.78 (AP) 2009.
4.
The writ petitioner of this case is challenging the recommendation of respondent
No.3, Shri Okan Sitek for his appointment as Sub-Treasury Officer against the reserved
quota for physically handicapped person. The APPSC started the process of recruiting
suitable persons for filling up 50 posts in different categories in the month of July, 2006.
After competitive examination a merit list was published on 14.10.2008 and thereafter
final list was published on 17.1.2009. The APPSC also reserved 3% posts for physically
disabled persons under the provisions of the “Persons with Disabilities (Equal
Opportunities, Protection of Rights and Full Participation) Act, 1995 (hereinafter referred
to as the ‘Disabilities Act’). As a matter of precautions the APPSC issued a pubic notice
on 20.3.2007 directing that physically handicapped persons are required to submit Part-B
Certificate of Disability/Handicapped Certificate issued by the competent Medical Board.
On the basis of such certificates submitted by the candidates, a separate merit list for the
disabled persons was issued, wherein details of nature of disability, percentage of
disability and documents furnished by the candidates in support of their disability were
also mentioned. As per this list the petitioner was shown to be suffering from Partial
Blindness, whereas respondent No.3 was shown to be suffering from Orthopaedically
handicapped and also having speech disability. However, respondent no.3 subsequently
submitted another certificate issued by a Medical Board of General Hospital, Naharlagun
on 22.10.2008. The earlier disability certificate was issued by a Medical Board of
Yingkiong, District Hospital on 30.4.2007. In other words, the Medical Certificate of
Yingkiong hospital was obtained after the vacancies were notified by the APPSC and in
this certificate there was no mention of any low vision of the respondent No.3. However,
relying upon the second certificate, the respondent No.3 has been recommended by the
APPSC for his appointment as Sub-Treasury Officer by the impugned notification dated
17.1.2009. This recommendation of respondent No.3 has been basically challenged on
the following grounds: i) That the respondent No.3 could not have been recommended for
the post of Sub-Treasury Officer as only one of such posts was reserved for
Orthopaedically disabled persons, whereas the appointment has been made against
visually disabled quota, ii) The respondent No.3 did not suffer from 40% disability,
which is the minimum requirement of disability U/s 2(t) of the Disability Act, iii) The
recommendation/appointment of respondent No.3 has also been challenged on the ground
that Medical Certificate showing him as a disabled person due to low vision is not a
genuine one.
5.
On the other hand, Sri. A.Apang, learned counsel for the respondent No.3 took a
preliminary objection about the maintainability of the writ petition on the ground that the
writ petitioner himself is not qualified to be recommended or appointed for any post, in as
much as, he did not secure 33% marks in as many as three papers. With regard to
Medical Certificates Sri Apang has submitted that since the respondent hailed from
Upper Siang District, of which Yingkiong is the Head Quarter, he approached District
Hospital for obtaining Disability Certificate and since there was no Eye Specialist in the
hospital, the vision of the petitioner could not be examined. The respondent No.3 has
furnished a document from the District hospital authority certifying that there was no eye
specialist in the hospital. Be that as it may, if the respondent No.3 was suffering from
low vision the Medical Board at Yingkiong was under an obligation to refer respondent
No.3 to any other Govt. Hospital for undergoing requisite medical test for his low vision.
According to the learned counsel, the respondent No.3 is still ready and willing to
undergo fresh medical test, if so directed by the court.
6.
So far as the APPSC is concerned, it was the submission of Sri. N.Tagia, learned
Standing counsel that APPSC acted upon the latest Medical Certificate furnished by the
respondent no.3 and as such the recommendation was not made on any extraneous
consideration, nor was it a malafide action.
7.
Sri. R.H.Nabam, learned Senior Govt. Advocate has submitted that the State
Govt. had no role in the selection of respondent No.3 or non-selection of the petitioner
since the APPSC is the competent authority for the purpose, and the said responsibility is
vested upon the APPSC.
8.
As required under the law the State Govt. has identified various posts, wherein
physically disabled persons can be considered for appointment.
A copy of the
notification dated 21.5.2007 has been submitted by the writ petitioner.
Under this
notification only Orthopaedically and partially deaf persons can be appointed as SubTreasury Officer. However, the respondent No.3 has been appointed as Sub-Treasury
Officer on the ground that he is a visually disabled person. In other words the respondent
No.3 was not recommended against the reserved quota for orthopaedically disabled
persons. After going through the documents, filed by the petitioner, it is difficult on my
part to ignore the gross mistake committed by the APPSC, which is certainly an expert
body for this purpose. It may also be mentioned here that only one post of Sub-Treasury
Officer was reserved and the APPSC had already recommended the name of one Shri
Rima Taipodia for the said post against reserved quota of Orthopaedically handicapped
persons. Hence, there was no scope to recommend the name of respondent No.3 for the
post of Sub-Treasury Officer. By doing so the APPSC has demonstrated as to how
irresponsibly it acts.
9.
The aforesaid illegality in recommending the name of respondent No.3 for his
appointment as Sub-Treasury Officer against reserved quota smacks foul play and this
aspect cannot be totally ruled out in the given circumstances. I say so, because when the
writ petitioner obtained opinion from the APPSC, exercising its right under Right to
Information Act, the APPSC initially intimated the petitioner that respondent No.3, Shri
Okan Sitek, along with Shri Rima Taipodia, was selected and recommended for the post
of Sub-Treasury Officer as handicapped person. In this reply letter dated 9.2.2009 it was
categorically stated that respondent No.3 was selected under reserved quota since he was
suffering from speech disability (Dumb). The APPSC also informed the petitioner that
the quota or Blind/Low Vision post was still unfilled. After the aforesaid reply the writ
petitioner submitted a representation claiming consideration of his recommendation
against the post earmarked for Blind/Low Vision persons. Only thereafter the APPSC
issued a corrigendum dated 4.3.2009 clarifying that earlier information was incorrect and
respondent No.3 was in fact suffering from Low Vision Disability. Even after realizing
the mistake the APPSC failed to take any corrective measure to cancel the appointment of
respondent No.3 as Sub-Treasury Officer, the post which was already filled up by way of
recommending the name of Shri Rima Taipodia. In my considered opinion, such
illegality, gross negligence and error by an expert body like APPSC cannot be condoned
and the authority has to be made accountable for such illegality.
10.
Section 2(b) and 2(o) defines ‘blindness’ and ‘locomotor disability’ under the
Disabilities Act. For ready reference, the aforesaid definitions are quoted below:
“(b) “blindness” refers to a condition where a person suffers from any of
the following conditions, namely:(i) total absence of sight; or
(ii) visual acuity not exceeding 6/60 or 20/200 (snellen) in the better eye
with correcting lenses; or
(iii) limitation of the field of vision subtending an angle of 20 degree or
worse;”
“(o) “locomotor disability” means disability of the bones, joints or
muscles leading to substantial restriction of the movement of the
limbs or any form of cerebral palsy;”
Similarly Section 2(t) of the Disabilities Act has laid down the minimum
percentage of disabilities for appointment against any identified post to physically
disabled persons. Section 2(t) is also quoted below:
“(t) “person with disability” means a person suffering from not less than
forty percent of any disability as certified by a medical authority.”
11.
Broadly speaking physical disability is confined to 3 (three) categories- (i)
Orthopaedically Disabled, (ii) Visually Disabled and (iii) Hearing Impaired. Hence, in
my considered view a person suffering from locomotor disability would also fall within
the category of ‘Orthopaedically handicapped’ persons. In the present case, the second
Disability Certificate furnished by the respondent No.3, certifying that Oken Sitak was
suffering from 40% disability, was based on his locomotor disability and low vision. The
doctors of the Medical Board has calculated 30% and 10% disability from each
disablement and in aggregate the respondent No.3 has been certified suffering from 40%
disability. According to the Medical Board, the respondent No.3 was suffering from
amputation of wrist as well as visual disability, which resulted in 40% disability. Hence,
it was the duty of the APPSC to exclude percentage of disability which fell in the
category of ‘locomotor disability’. In other words, the APPSC was entitled to take into
consideration only the percentage of disability which related to ‘Low Vision’ and not the
cumulative disability.
12.
Since it was a case of furnishing two disability certificates from two separate
Medical Boards, the APPSC should have scrutinised the candidature of respondent No.3
more carefully. Instead, the name of respondent No.3 appears to have been cleared even
without superficial examination of the documents. It is also mentioned here that in the
Disability Certificate the signature of the victim is also required to be obtained. However
in the second certificate there does not appear to be any signature of the candidate. This
aspect also did not draw the attention of the APPSC, which is really shocking. These
lapses also indicate that respondent No.3 was totally not questioned about his disability
before recommending his name for appointment as Sub-Treasury Officer, the post which
was already filled up by another candidate.
13.
For the aforesaid reasons, I find no hesitation or difficulty to quash the
recommendation of the name of the respondent No.3 against the post of Sub-Treasury
Officer. However, in the interest of justice, the respondent No.3, namely, Shri Okan
Sitek is directed to appear before State Medical Board of Arunachal Pradesh within a
period of 4 (four) weeks from today and on such appearance, the said Medical Board
would conduct another vision test of the respondent No.3 and issue appropriate
certificate, clearly indicating the percentage of disability due to loss of vision, if any. It
would be appropriate that the hospital authority shall constitute a fresh Medical Board
excluding the doctors who had earlier examined respondent No.3. The Medical Board is
further directed to send the Disability Certificate directly to APPSC under sealed cover
and on receipt of such report/certificate of Respondent No.3 the APPSC shall consider
the candidature of respondent No.3. In the event APPSC finds that Respondent No.3 is
not eligible for fresh recommendation as a visually challenged/disabled person, they shall
be at liberty to consider the case of the writ petitioner, if the petitioner fulfills all the
criteria for his recommendation and also taking a decision about his eligibility on the
basis of marks obtained by him in the written test.
14.
With the aforesaid directions, the writ petition stands allowed. Since, I hold that
the APPSC has committed gross illegality in recommending the name of respondent No.3
against reserved quota for Orthopaedically handicapped person, and also furnished wrong
information to the petitioner under RTI Act, I hereby, impose costs of Rs.25,000/(Twenty five thousand). Costs shall be paid by APPSC to the writ petitioner within a
period of 4 (four) weeks from today, failing which the amount shall carry interest @ 8%
per annum.
***
IN THE GAUHATI HIGH COURT
ITANAGAR BENCH
W.P. (C) NO.83 (AP) OF 2009
D.D. 15.09.2009
Hon’ble Mr. Justice P.K.Musahary
Shri Kipa Raja & Ors.
Vs.
The State of A.P. & Ors.
…
Petitioner
…
Respondents
Recruitment:
In the combined competitive examination pursuant to advertisement dt: 25.7.2006
the petitioners qualified in the written as well as viva-voce tests and in the final select list
dated 18.1.2009 the names of the petitioners appeared at Sl.Nos.7, 43, 47 and 49 – But
they were disqualified in the physical endurance test and in their place respondents 3 to 7
who were included in the selection list at Sl. Nos. 58, 61, 69, 70 and 89 were selected –
Petitioners challenged the selection of respondents 3 to 7 on the ground that in the
advertisement it was mentioned that candidates opting for Deputy Superintendent of
Police (DSP) post must possess the physical and medical standard i.e. physical fitness There is no mention about physical endurance test which was prescribed subsequently by
a notice dated 16.10.2008 - As the combined competitive examination Rules, 2001,
provide for physical endurance test for candidates who opted for the post of DSP, the
High Court upheld the action of the Commission in holding the physical endurance test
even though the same was not advertised in the advertisement particularly in view of the
fact that the petitioners participated in the selection process without challenging the
physical test prescribed as per notice dated 16.10.2008 and dismissed the writ petitions.
Held:
The petitioners could have challenged the notice prescribing physical test before
they participated in the said test but they have challenged the same only after they
participated and found to be unfit in the said physical test.
ORDER
Heard Mr. P.Taffo, learned counsel for the petitioners and Mr. N.Tagia, learned
Standing Counsel for the Respondent No.2/APPSC. Also heard Mr. R.H.Nabam, learned
Senior Govt. Advocate, appearing on behalf of the Respondent No.1 and Mr. K.Ete,
learned counsel for the Respondent Nos.3 to 7.
2.
The case of the petitioners is that they appeared in the combined competitive
examination conducted by the Arunachal Pradesh Public Service Commission in 2006
and they were found qualified in the written as well as viva-voce tests. In the final
selection list published by the Arunachal Pradesh Public Service Commission (hereinafter
referred to as ‘the Commission’ only) on 18.01.2009, the names of the petitioners
appeared at serial Nos.7, 42, 47 and 49. It is claimed that the petitioners were physically
and medically fit but they were disqualified in the physical endurance test and in place of
the petitioners, the private respondent Nos.3 to 7 were selected, although, in the select list
published by the Commission, their names appeared at serial Nos.58, 61, 69, 70 & 89.
The petitioners are aggrieved by the selection of the aforesaid private respondents against
the posts of Deputy Superintendent of Police (DSP).
3..
Mr. Taffo, learned counsel for the petitioners submits that in the advertisement
dated 25.07.2006 (Annexure-1 to the writ petition) published by the Commission, it is
provided that the candidates must be physically fit to tour/travel in remote places in
Arunachal Pradesh on foot. Candidates opting for Deputy Superintendent of Police
(APPS) post must possess the physical and medical standard i.e. physical fitness. There
is no mention about physical endurance test in the said advertisement but subsequently by
a notice dated 16.10.2008 (Annexure-II to the writ petition), the Commission provided
that the candidates who opted for the post of Dy. Superintendent of Police would have to
undergo standard physical test as envisaged in the Combined Competitive Examination
Rules, 2001.
It has also been provided that the candidates who have opted for DSP post,
will have to qualify in physical efficiency test, which includes 1600 metres (1 mile) race t
be completed in 7 minutes, long jump of 335.28 cm (3 chances to be given); and Obstacle
crossing etc. According to Mr. Taffo, since the original advertisement published by the
Commission did not indicate conducting of such test, the method taken by the
Commission in conducting the physical endurance test is unauthorized and illegal. It is
also submitted that such endurance test is meant for recruitment to the post of constable
etc. and it is not meant for the high ranking police officials like Deputy Superintendent of
Police. It is enough if the candidates who have opted for DSP post are found qualified in
the medical test and physical standard. Because of the wrong procedure followed by the
Commission in conducting the medical endurance test, which is not provided in the
original advertisement, the petitioners have been deprived of being selected in the posts
of DSP giving way for the private respondents who are in the much below position in the
merit list published by the Commission.
4.
Mr. Tagia, learned Standing Counsel for the Commission submits that although,
in he original advertisement published by the Commission, no mention was made about
the physical endurance test in respect of the candidates for the DSP posts, it was notified
subsequently by the Commission in its notice dated 16.10.2008 as per provision under the
Arunachal Pradesh Combined Competitive Examination Rules, 2001.
He refers to
appendix-III of Rule 11, para 4 of the said Rules which has been annexed as annexure-A
to the affidavit-in-opposition filed by the Commission, which provides amongst others
that “the physical fitness test must precede the personality test. This physical fitness shall
be conducted by Inspector General of Police in presence of one of the Members of the
Arunachal Pradesh Public Service Commission”. The candidate will be put through
physical efficiency test as indicated below:
“For Male candidates:
1.
1600 metres (1 mile) race to be completed in 7 minutes.
2.
Long jump of 335.28 cm (3 chances to be given)
3.
Obstacle crossing of any six of the following:
(i)
(ii)
182.88 cm x 25.40 cm pickets over which the candidates have to jump and
cross.
Clearing the wall of 152.40 cm x 213.36 cm x 60.96 cm.
(iii)
Jump by holding the rope from an obstacle of 243.84 cm.
(iv)
Tarzan swing of 304.80 cm long.
(v)
Parallel rope of 914.40 cm long
(vi)
Climbing the vertical rope of 487.68 cm long.
(vii)
Crossing 182.88 cm wall
For female candidates:
1. 100 metres race in 16 seconds
2. 800 metres race in 4 minutes.
3. Long jump of 304.80 cm (3 chances to be given)
4. High jump of 91.44 cm (3 chances to be given)”
5.
It has been specifically stated in para 6 of the counter affidavit that the petitioners
failed to qualify in the 1600 metres (1 mile) race in 7 minutes which is a vital event of the
candidates opting for the post of DSP. Further in para 7 of the aforesaid affidavit, it is
clearly stated that no recruitment criteria was altered at any point of time in the course of
recruitment process. It has further been stated in para 13 of the counter affidavit that
although, the petitioners had opted or preferred for DSP, they have been recommended
against their second preference and this was done because they could not qualify in the
1600 metres race in 7 minutes. Mr. Tagia submits that no injustice or discrimination has
been done in so far as the selection/recommendation was made by the Commission in
respect of the posts of DSP and no injustice has been done to the petitioners inasmuch as
they have been recommended for other posts as per their second choice.
6.
Mr. K.Ete, learned counsel appearing for the private respondent Nos.3 to 7 adopts
the submissions made by the learned Standing Counsel for the Commission. However, in
addition to the above submissions, Mr. Ete, submits that before the Arunachal Pradesh
Combined Competitive Examination Rules, 2001 were framed, the Govt. of Arunachal
Pradesh framed the Arunachal Pradesh Police Service Rules, 1989 wherein, in the case of
direct recruitment police service including DSP provisions were made for physical fitness
test, which must precede the personal test. Under rule 7 of the aforesaid Rules, it is
specifically provided that the candidates will be put through physical efficiency test,
which includes in case of male candidates, 1600 metres (1 mile) race to be complete in 7
minutes. The Arunachal Pradesh Combined Competitive Examination Rules, 2001 were
framed in the line of the Arunachal Pradesh Police Service Rules, 1989 in so far the
direct recruitment to the post of DSP is concerned inasmuch as some criteria have been
laid down for the physical fitness test.
7.
I have carefully gone through the provisions made in the Arunachal Pradesh
Combined Competitive Examination Rules, 2001.
Rule 11 of the aforesaid Rules
provides that the candidates applying for the post of Arunachal Pradesh Police Service
and called to the interview shall be required to undergo physical standard test as
prescribed in Appendix-III. For a male candidate, it is specifically provided that a male
candidate must complete 1600 metres (1 mile) race in 7 minutes. Since the Rules provide
for such physical efficiency test, the submissions made by the learned counsel for the
petitioners to the effect that the Commission of its own conducted the said physical test
without indicating anything about the same in the original advertisement, cannot be
accepted. The Commission, in my considered view, is bound to follow the aforesaid
2001 Rules in the matter of selection of the candidates because of the nature of duty and
responsibility attached to the post of DSP. The Rules have specifically provided for such
physical test and when the Commission has conducted such physical test on the
candidates including the present petitioners, no fault can be found in it. The present
petitioners have been found not qualified in the said physical test.
8.
It is to be noted that the Commission published the notice dated 16.10.2008
requiring the candidates who have opted for the posts of DSP to undergo standard
physical endurance test.
The petitioners participated in the said test before the
Commission for the aforesaid standard physical test and they were found to be not
successful in the said test.
Thereafter, they filed the present writ petition only on
27.02.2009 challenging the aforesaid physical test. The petitioners could have challenged
the aforesaid notice dated 16.10.2008 before they participated in the physical test but they
have challenged the same only after they participated and found to be unfit in the said
physical test. In the case of Nikhilesh Das Vs. State of Tripura and Others, reported in
2007 (2) GLT 754, this Court in para 15, held thus:
“15. I have carefully gone through the above decisions cited by the
learned senior counsel for the respondent Nos.2 & 3, the ratio of the above
decisions is that if a candidate takes a calculated chance and appears at the
interview without any protest and when he found that he has become
unsuccessful, he cannot turn round and questioned the legality of the same
by filing petition under Section 226 of the Constitution of India.
Admittedly, the petitioner, in the present case, did not raise any objection
against the advertisement mode of selection adopted by the TPSC rather
he appeared at the interview held for selection of the suitable candidates
for the post of Assistant Professor in the Commerce Stream. The
petitioner filed this present writ petition after the result of the said
selection test has been published. In the above factual matrix and
applying the principles laid down by the Hon’ble Apex Court, this Court
has no hesitation in holding that the petitioner is barred by the principle of
estoppel from challenging the result of the selection test held on
29.12.2001 and consequent appointments of the Respondent Nos.4 to 8 to
the posts of Assistant Professor in Commerce stream.”
The aforesaid decision squarely covers the present case.
9.
Because of the aforesaid position, I do not find any merit in this petition and
accordingly, the same is liable to be dismissed, which I do hereby.
10.
There shall be no order as to costs.
***
IN THE GAUHATI HIGH COURT
ITANAGAR BENCH
W.P. (C) NO.136 (AP) OF 2009
D.D. 15.09.2009
Hon’ble Mr. Justice Ranjan Gogoi
Shri Kirli Padu
Vs.
The State of A.P. & Ors.
Recruitment -
…
Petitioner
…
Respondents
Relative merit and overall merit:
When preference has to be exercised it is the relative merit and not overall merit, overall
merit has to be ignored.
Petitioner was a candidate for 42 posts of Arunachal Pradesh Civil Service (Entry
Grade) and D.S.P. – After the Preliminary Examination successful candidates were
required to submit another application for admission to main written examination
indicating their preference for the services/posts – Petitioner had opted for Arunachal
Pradesh Civil Service (Entry Grade) - In the meantime the number of Civil Services
(Entry Grade) posts was increased to 101 from 42 posts and D.S.P. was increased to 7
from 3 and candidates were asked to resubmit their options for the posts – The petitioner
opted for DSP post – Petitioner was among the candidates who opted for DSP post
qualified in the physical test – As against 07 posts of DSP 06 candidates were selected –
The petitioner 7th candidate in the order of merit was not selected on the ground as
against 101 posts advertisement the overall merit position of the petitioner was at
Sl.No.106 therefore, he was not selected against 7th post of DSP – Upholding the
contention of the petitioner that selection should have been made on the basis of relative
merit and not on overall merit as per the options of the candidates allowed the writ
petition and directed the Commission to recommend the name of the petitioner for
appointment as DSP against 7th post.
Held:
Absolute merit is bound to suffer and has to be ignored and relative merit, as per
the option(s) exercised, will be the determinative criteria. It is because of all these
uncertainties in a competitive examination that, time and again, it has been emphasized
by judicial pronouncements that absolute merit is an elusive factor and it is the relative
merit of candidates, as per options exercised, which would determine the final selection.
ORDER
This writ petition has been filed seeking a direction to the Arunachal Pradesh
Public Service Commission (hereinafter referred to as ‘the Commission’) to recommend
the name of the writ petitioner for appointment in the post of Deputy Superintendent of
Police for which a select list has been drawn up by the Commission in which list the
name of the petitioner does not find mention.
2.
The brief facts that will be required to be noticed for the purposes of the present
adjudication may be set out hereunder:
By an advertisement dated 25.7.2006, 42 posts in different Services under the
State including the Arunachal Pradesh Civil Service (Entry Grade) and Arunachal
Pradesh Police Service (D.S.P.) were advertised by the Commission. A preliminary
examination of the eligible candidates was held on 4.2.2007 the result whereof was
declared on 31.3.2007. The successful candidates in the preliminary examination were
required to submit another application for admission to the main written examination. In
the application to be submitted the candidates were required to indicate their
option/preference for the Services/posts which were advertised. The petitioner, at that
stage, had opted for the Arunachal Pradesh Civil Service (Entry Grade). The main
examinations were held between 26.12.2007 and 21.1.2008. The result of the main
examination was declared on 14.10.2008.
The petitioner qualified in the said
examination and became eligible to take part in the viva-voce test which was to follow
thereafter. At this stage, all the candidates were asked to re-submit fresh options for the
particular posts/Service in which they desired to be appointed. Such options were to be
exercised in order of preference. The aforesaid exercise calling upon the candidates to
re-submit their options became necessary in view of the fact that during the period when
the selections remained pending, fresh requisitions were submitted to the Commission by
different departments for additional posts thereby raising the number of total posts to 101
from the initial 42 posts. The petitioner, this time, opted for the Arunachal Pradesh
Police Service indicating the said Service to be his first option. The number of posts of
Deputy Superintendent of Police in the said Service had increased to 7 from the 3
numbers of posts advertised on 25.7.2006.
3.
On being called, the petitioner went through the physical test which was
mandatory for the candidates opting for the Arunachal Pradesh Police Service and also
faced the interview. A total number of 28 candidates who had opted for the Police
Service qualified in the physical test which included the petitioner. The final result of the
selection in respect of all the 101 posts were declared on 17.1.2009. The name of the
petitioner was not included in the list of successful candidates which, however, contained
only 6 names against the 7 available posts of Deputy Superintendent of Police i.e. the
candidates placed at Sl.Nos.16, 58, 61, 69, 70 and 87.
4.
Aggrieved by the exclusion of his name from the list of selected candidates for
appointment in the post of Deputy Superintendent of Police, the petitioner filed an
application under the Right to Information Act, 2005 seeking the marks secured by each
of the 28 candidates who had qualified in the physical test for the post of Deputy
Superintendent of Police. The statement of marks enclosed to the writ petition would
indicate that in terms of the marks secured the position of the petitioner in order of merit
is at Sl.No.7. The first six candidates were declared to be successful. Specifically, the
aforesaid marks indicate that the sixth candidate recommended by the Commission
secured a total of 862.00 marks whereas the petitioner secured 853.50 marks and the next
candidate in order of merit secured 849 marks. It is in the aforesaid circumstances and,
particularly, in view of the availability of one post of Deputy Superintendent of Police
that this writ petition has been filed seeking a direction to the Commission to recommend
the name of the petitioner.
5.
The claims made in the writ petition have been resisted by the Commission by
filing an affidavit.
In the aforesaid affidavit, it has been admitted that with the
additional/fresh requisitions the number of posts of Deputy Superintendent of Police for
which recruitment was made by the Commission had gone upto seven and further than
only six candidates, in order of merit, had been recommended. The Commission, in its
affidavit, has further stated that in order of overall merit vis-à-vis all candidates,
regardless of the options exercised, the position of the petitioner was at Sl.No.106. As
there were candidates better placed in merit than the petitioner, the Commission had
decided not to recommend the name of the petitioner. Instead, one post of Deputy
Superintendent of Police was kept vacant and the names of only six successful candidates
have been recommended for appointment in the post of Deputy Superintendent of Police.
The above exercise, according to the Commission, is in conformity with the requirement
of a Combined Competitive examination based on merit. Therefore, according to the
Commission, the exclusion of the name of the petitioner from the list of recommended
candidates is justified.
6.
Sri. I Choudhury, learned counsel for the petitioner has vehemently argued that
the petitioner having opted for the Police Service as his first choice and his marks having
placed him at the 7th position, the stand taken by the Commission is plainly unacceptable
as in drawing up the merit list for each Service the preference/option indicated by a
candidate has to be taken into account. According to Sri. Choudhury, the exclusion of the
name of the petitioner from the list of recommended candidates discloses an apparent
error resulting in gross injustice to the petitioner which needs to be corrected by the
Court. Accordingly, it is submitted that the direction prayed for in the writ petition is
fully justified in law.
7.
Opposing the contentions advanced on behalf of the writ petitioner, Sri N Tagia,
learned Standing Counsel for the Commission has placed the statements made in the
counter affidavit of the Commission, particularly, those contained in para 8 and 9 to
contend that on an overall consideration of merit the petitioner was placed at Sl.No.106
whereas the recruitment was for 101 posts. According to Sri Tagia, the merit position of
the petitioner in the overall merit list justified his exclusion, notwithstanding the fact that
in so far as the post of Deputy Superintendent of Police is concerned, the petitioner was
placed at Sl.No.7. Sri Tagia further submits that the selection of candidates being on the
basis of overall merit, the petitioner is not entitled to the directions prayed for.
8.
The rival contentions advanced on behalf of the parties have received the most
anxious consideration of the Court. A perusal of the provisions of the Arunachal Pradesh
Public Service Commission Combined Competitive Examination Rules, 2001 including
the procedure contained in the Schedule and the Appendix to the said Rules do not
indicate that in preparing the final select list of successful candidates the option(s) for a
particular Service/Services indicated by a candidate, in order of priority, can be ignored
by the Commission. The absence of such a provision in the Rules, perhaps, is indicative
of the ground realities which can be best illustrated by a specific example:
A candidate may opt, say, for the Arunachal Pradesh Civil Service (Entry Grade)
as his only choice of service. In the final select result he may have been placed, say, at
Sl.No.41. The recruitment could be for 40 posts in the Arunachal Pradesh Civil Service
and coincidentally all the candidates placed above him i.e. 1 to 40 may have opted for the
Arunachal Pradesh Civil Service. In that event, the candidate placed at Sl.No.41 would
stand excluded and cannot be recommended for the Service for which he had opted. As
he had not opted for any other Service his name will not appear in the final select list. As
against the above, there could be a candidate placed at, say, Sl.No.60 who in addition to
opting for the Arunachal Pradesh Civil Service may have opted for some other Service.
Though he cannot be recommended for the Arunachal Pradesh Civil Service, yet, his
merit position i.e. 60 may entitle him to be recommended for some other post unlike the
candidate at Sl.No.41 who did not opt for any other Service. In that event, absolute merit
is bound to suffer and has to be ignored and relative merit, as per the option(s) exercised,
will be the determinative criteria. It is because of all these uncertainties in a competitive
examination that, time and again, it has been emphasized by judicial pronouncements that
absolute merit is an elusive factor and it is the relative merit of candidates, as per options
exercised, which would determine the final selection.
9.
In the present case, the first option exercised by the petitioner was the Police
Service. His merit position amongst the candidates who had opted for the said Service
was at Sl.No.7 which entitled him to be recommended against the 7th vacancy. However,
his overall merit position was at Sl.No.106. In other words,there were five candidates
who were placed above him on overall merit; but none of these five candidates had opted
for the Arunachal Pradesh Police Service. In such a situation, notwithstanding the overall
merit the final recommendation of the Commission should have been on the basis of
relative merit of candidates as per their options. Any other view would not only amount
to ignoring the options exercised by a candidate but is also bound to result in an
anomalous and chaotic situation in preparation of the final select list which must be
avoided.
10.
The petitioner on the basis of his merit amongst the candidates who had opted for
the Police Service being entitled to be recommended against the 7th post, the Court will
have to hold that his exclusion from the list of the recommended candidates suffers from
a fundamental error which has to be corrected.
11.
Accordingly, this writ petition is allowed. The Public Service Commission is
directed to recommend the name of the petitioner for appointment in the Arunachal
Pradesh Police Service against the 7th post which was embraced by the selection process
in question.
12.
The Court is of the view that as the Commission can be expected to make its
recommendations at the earliest, no specific time frame ought to be fixed by the Court to
enable the Constitutional Body to perform its functions as enjoined by law.
13.
The writ petition, consequently, is allowed in terms of the above direction.
***
IN THE GAUHATI HIGH COURT
ITANAGAR BENCH
W.P. (C) NO.408 (AP) OF 2008
D.D. 1.12.2009
Hon’ble Mr. Justice H.N.Sarma
Shri Kirpak Dini
…
Vs.
State of A.P. & Ors. …
Petitioner
Respondents
Recruitment:
When posts are classified streamwise whether selection can be made branchwise on the
basis of merit ignoring streamwise classification? – No.
Whether delay in approaching the Court can be condoned when violation of fundamental
rights under Articles 14 & 16 of Constitution is alleged? – Yes.
Pursuant to requisition of the Appointing Authority for filling up six vacant posts
of Assistant Engineer (Elecl.) i.e. 03 posts for electrical stream, 02 posts for mechanical
stream and 01 post for Electronics/Telecommunication/Computer stream the Commission
issued advertisement dated 15.10.2005 – Petitioner was a candidate for the post of
Electronics/Telecommunication/Computer – After the selection process the Commission
recommended the names of 06 candidates out of which 05 were found from electrical
stream and 01 from mechanical stream – A candidate who was eligible to be selected for
2nd post in mechanical stream filed W.P. (C) No.213 (AP) 2007 and the Court allowed the
writ petition and directed the Commission to select the petitioner in the said case – After
coming to know the result of the above case the petitioner in this case approached the
Court giving reason for the delay – As the 4th respondent selected from electrical stream
scored lowest marks among the selected candidates the Court held that his selection was
required to be quashed and allowed the writ petition of the petitioner with a direction to
the Commission to recommend the name of the petitioner for selection within 03 weeks.
Held:
When the classification of posts indicated in the advertisement is in accordance
with the relevant statutory rules the selection made is not only contrary to the statutory
rules but also offends the provisions of Articles 14 and 16 of the Constitution.
Further held:
When the petitioner has explained the delay in approaching the Court and as the
action complained of is violative of the fundamental rights of the petitioner guaranteed
under Articles 14 and 16 of the Constitution the delay needs to be condoned.
Cases Referred:
AIR 1980 SC 112 - Ashok Kumar Mishra & Anr.Vs Collector Raipur & Ors.
AIR 1987 SC 251 - State of MP & Ors. vs. Nandlal Jaiswal & Ors.
AIR 1992 SC 80 - Ashok alias Somanna Gowda & Anr.Vs. State of Karnataka & Ors.
(2009) 3 SCC 227 - Amlan Jyoti Baruah vs. State of Assam & Ors.
ORDER
Non-recommendation of the petitioner for appointment to the post of Assistant
Engineer in the stream of Electronic/Telecommunication/Computer Engineering in terms
of the notification dated 15.10.2005 issued by the Arunachal Pradesh Public Service
Commission (for short the “Commission”) and filling up of the said post by respondent
No.4 from different stream is the subject matter of challenge in this writ petition.
2.
I have heard Mr K.Ete, learned counsel for the petitioner, Mr N Tagia, learned SC
for the Commission, Mr M.Pertin, learned counsel for the respondent No.3, Mr R Pait,
learned counsel for the respondent No.4 and Ms G.Deka, learned Addl. GA, AP.
3.
On the basis of a requisition received from the Power Department of the
Government of Arunachal Pradesh for selection and recommendation of six number of
candidates for the purpose of appointment as Assistant Engineer in different streams as
per related recruitment rules, the Commission issued the notification dated 15.10.2005
inviting applications from the intending candidates for filing up the aforesaid posts. In
fact, prior to the aforesaid notification an advertisement was issued on 28.5.2005 for
filling up the post of Assistant Engineer and by notification dated 15.10.2005, the ratio of
distribution of the six advertised posts streamwise as per the percentage as indicated by
the Government vide its letter dated 1.10.2005 was fixed as follows:
Electrical Stream
Mechanical Stream
Electronic/Telecommunication/
Computer
-
3 posts
2 posts
1 post
The modality of selection that would be followed by the Commission was also made
know to the candidates in the aforesaid notification. The present petitioner along with
respondent Nos.3 and 4 and some other candidates possessing requisite qualification
applied for selection and appointment in the respective stream. The petitioner is a
Bachelor Degree holder in Computer Engineering and having held all the necessary
qualification to hold the post as mentioned in the related stream has submitted his
application in terms of the said notification. Then respondent No.3 and 4 who are
Bachelor Degree holders in Electrical Engineering have also submitted their applications.
The Commission after scrutiny of the applications eligible candidates were called for
written test and the written test was held on 16th and 17th January 2006 wherein the
petitioner and the private respondent appeared and all of them found to be qualified were
further called for viva voce test held on 18.7.2006. For the lone post of Assistant
Engineer, Electronic/Telecommunication/Computer there were only two candidates,
namely the petitioner and Shri Tao Tana. The result of the selection was notified by the
Commission on 19.7.2006. In the notification the name of the petitioner did not appear.
The petitioner was in the impression that since the selection was made by a Constitutional
authority like State Public Service Commission, the selection must have been made in
due compliance of the relevant recruitment rules and in terms of the advertisement, which
later on found to be not correct. It is also to be noted herein that in the notification
publishing the result of the qualified candidates, the stream of the candidates in which
they were selected have not been mentioned.
In the meanwhile one Sri Rajesh Dawe who was the second highest scorer in the
mechanical stream approached this Court by filing WP(C) No.213(AP)/2007 contending
that although two posts were earmarked for being appointed in the mechanical stream
only one candidate has been recommended by the Commission and he being the second
highest scorer has acquired a right to be recommended and appointed in the remaining
second post. Although the said writ petition was resisted by the respondents, the same
was allowed vide judgment and order dated 12.3.2008. Thereafter, the said candidate
was appointed as Assistant Engineer in the mechanical stream and only at that stage the
present petitioner could find that in fact not a single candidate from the stream of
Electronic/Telecommunication/Computer was selected by the Commission totally
departing from the notification dated 15.10.2005 as well as relevant recruitment rules.
The petitioner made necessary enquiry and by obtaining marks scored by the selected
candidates through Right to Information Act, 2005 noticed that out of two candidates in
the stream of Electronic/Telecommunication/Computer, he scored highest mark. It was
noticed from the selection list so published by the Commission obtained through RTI, it
is found that only one candidate was selected from mechanical stream and other five were
selected from Electrical stream. In WP(C) No.213 (AP)/2007, this Court in spite of
declaring the action of the respondent authorities illegal, unjust and unfair for not having
recommended the name of the said petitioner being second highest mark scorer for
appointment to the post of Assistant Engineer (Mechanical), the illegalities committed in
respect of candidates applied for Electronic/Telecommunication/Computer stream was
not rectified by the respondents, which led the petitioner to approach this Court by filing
this writ petition seeking necessary relief.
4.
A counter has been filed on behalf of the respondent authority. In the said counter
at paragraph-6 has taken the stand as follows:
“That with regard to the contents of the para 3 of the writ petition,
your humble deponent does not admit any of the statements which is
contrary to and inconsistent with the records. The Commission had issued
advertisement vide No.PSC-R/07/05 dated 15.10.2005 for filling up of six
vacant posts of Assistant Engineer (Elecl.) i.e. 3(three) posts for electrical
stream, 2 (two) posts for mechanical stream and 1 (one) post for
Electronic/Telecommunication/Computer stream. But the selection was
made on the basis of merit and as per the Govt. letter No.PWRS/E-93/9596/Vol-III/895, dated 31st May, 2006.”
Further stand of the Commission is that the Government vide letter dated 31.5.2006
indicated that there was not separate streamwise distribution of the posts and the vacancy
required to be filled up is only meant for the Assistant Engineer (Elecl.) which the
Arunachal Pradesh Public Service Commission cannot segregate or resume any post
against any branch/stream on its own. That apart further stand of the Commission is that
the petitioner has not approached this Court with due diligence and having approached
alter considerable delay, he is not entitled to any relief.
5.
Similar stand has also been taken by the respondent Nos.3 and 4 in their counter.
6.
Mr. Ete submits that on the face of the notification dated 15.10.2005, the
Commission is to select and recommend the names of the candidates earmarked for the
different stream on streamwise basis and total non-consideration to select any candidate
from the stream Electronic/Telecommunication/Computer has resulted grave illegality
causing infraction on the advertisement issued by the Commission and the petitioner
having secured the highest marks out of two candidates who qualified themselves to
appear in viva voce, the petitioner is to be recommended and appointed in one of the two
posts earmarked for the steam of Electronic/Telecommunication/Computer. The learned
counsel further submits that the action of the respondent authorities being arbitrary is
violative of Articles 14 and 16 of the Constitution of India, more particularly, when this
Court rectified such defect by issuing suitable direction vide decision rendered in WP (C)
No.213 (A)/2007, the present petitioner approached this court after obtaining necessary
information through Right to Information Act, 2005, and as such he is entitled to the
relief claimed in this writ petition.
7.
Mr. N.Tagia, learned SC, APPSC, in his usual fairness made his submission on
the basis of the statements made in paragraph 6 of the counter affidavit and also submits
that the petitioner not having approached this court with due diligence and required
promptness is not entitled to any relief. Mr M Pertin, learned counsel for the respondent
No.3, raised only question of delay and laches in approaching this Court by the petitioner
and in support of his submission, he referred to the decisions of the Apex Court rendered
in the cases of Amlan Jyoti Baruah vs. State of Assam and others, reported in (2009) 3
SCC 227, Sri. Ashok alias Somanna Gowda and another Vs. State of Karnataka and
others, reported in AIR 1992 SC 80, Ashok Kumar Mishra and another Vs Collector
Raipur and others, reported in AIR1980 SC 112 State of MP and others vs. Nandlal
Jaiswal and others reported in AIR 1987 SC 251. Referring to the ratio of the aforesaid
decisions, Mr. Pertin submits that the present writ petition is not to be entertained by this
Court in view of the fact that the petitioner did not approached this Court with due
diligence. Mr. Pait, learned counsel for the respondent No.4, has adopted the arguments
advanced by Mr. Pertin.
8.
The submissions made by the learned counsel for the parties receives due
attention of this Court. From the facts as eluded hereinabove, the undisputed position is
that there was an advertisement inviting applications from the intending candidates for
recommendation and appointment in six posts of Assistant Engineer in the steam of
electrical,
mechanical
and
Electronic/Telecommunication/Computer
earmarking posts to each steam specifically.
Engineering
As per notification dated 15.10.2005, the
posts earmarked in the Electronic/Telecommunication/Computer is one. Similarly, in the
stream of electrical three posts and in the mechanical two posts. It is also undisputed fact
that no candidate was recommended under the stream of Electronic/Telecommunication/
Computer Engineering although there were two candidates available before the
Commission in the Electronic/Telecommunication/Computer stream including the
petitioner out of which the petitioner secured higher marks than the other candidate in the
selection list.
On the other hand, the Commission recommended the names of six
candidates out of which five were from electrical stream and one from mechanical
stream. The Commission having departed from acting as per notification to recommend
two candidates from the mechanical stream, this Court in WP(C) No.213 (AP)/2007
quashed such action of the Commission and directed to recommend as other candidate
who was the petitioner in the said writ petition.
In the aforesaid situation, the
Commission made flagrant violation of the provision failing to recommend any candidate
under the stream of Electronic/Telecommunication/Computer in terms of the notification
dated 15.10.2005.
The appointment to the posts in question is regulated by a act of statutory rules,
known as, the Arunachal Pradesh Power Engineering Service Rules, 1993. Rule 11 of
the Rules provides for recruitment to the post of Assistant Engineer. Sub-rule (3) of
Rules provides for direct recruitment only such candidates shall be eligible for appearing
at the examination etc. who have atleast an Engineering Degree qualification in
Electrical, Mechanical for the post mentioned in Sub-Rule (i) above, and engineering
Degree qualification in Electronic/Telecommunication/Computer for the post mentioned
in Sub Rule (i) (ii) above. The notification dated 15.10.2005 is perfectly in tune with the
provisions of Rule (1) and (3) and sub-rule (1) and (3) of rule 11 of the recruitment rules.
In view of the aforesaid situation, the Commission committed obvious and
apparent
error
in
not
recommending
any
candidate
from
the
stream
of
Electronic/Telecommunication/Computer in which category, the petitioner applied for.
The aforesaid action of the Commission is not only contrary to the statutory service Rules
but also it offends the provisions of Article 14 and 16 of the Constitution of India.
Accordingly, the petitioner who is the candidate in the said stream and having scored
highest marks amongst two candidates ought to have been recommended and by not
doing so the valuable right of the petitioner has been infringed.
9.
From the marks statements obtained by the petitioner through Right to
Information Act, 2005, it is found that the respondent No.4 is the person who is the
candidate from the electrical stream and scored lowest marks amongst the selected
candidates. In that view of the matter, the appointment of the respondent No.4 has to be
interfered with.
10.
The petitioner has pleaded and explained the cause of delay in filing this writ
petition. The writ petition was filed on 4.10.2008 although the result of the selection was
published vide notification 19.7.2006. The petitioner explained the cause of his not filing
the writ petition in time in paragraphs 12 and 13 of the writ petition. This Court in the
case of Rajesh Dawe set aside such action of the respondents vide judgment and order
dated 12.3.2008 in spite of such judicial pronouncement, the respondent authorities have
not taken any action to provide justice to the petitioner by necessary notification. The
petitioner was of the impression that his case would be considered fairly and he has no
doubt about the fidelity of the Commission it being a highly placed constitutional
authority. The Commission being the constitutional authority is expected to act in a just,
fair and impartial manner.
But after the decision of this Court in WP (C) N.213
(AP)/2007, the petitioner could understand that the action of the commission is contrary
to the provisions of law and accordingly, he approached this court. The principle of law
enunciated in the cases cited by Mr. Pertin is not in dispute but those principles of law are
not applicable in the facts of this case, inasmuch as, the petitioner has explained the delay
in approaching this Court. That apart, the action complained of that the respondent
authorities is violative of the fundamental rights of the petitioner guaranteed under
Articles 14 and 16 of the Constitution of India. In this respect, the decision referred by
Mr K Ete reported in the case of State of Karnataka and others vs. C.Lalitha, reported in
(2006) 2 SCC 747 and Harangthan Mawii Vs Lai autonomous District Council and
others, reported in 2005 (2) GLT 588, are not misplaced.
11.
In view of the aforesaid discussions, I am not inclined to accept the ground of
delay as a stumbling block to provide relief to which the petitioner is entitled in this writ
petition.
12.
In view of the above discussion, this writ petition stands allowed.
The
Commission is directed to recommend the name of the petitioner for appointment as
Assistant Engineer in the Electronic/Telecommunication/Computer stream under the
Power Department in terms of the notification dated 15.10.2005.
Upon such
recommendation the concerned respondent authorities shall provide appointment to the
petitioner without further delay.
In order to accommodate the petitioner, the
recommendation and appointment of the respondent No.4 stands set aside and quashed.
Quashing of the appointment of the respondent No.4, however, would not imply that the
department would be debarred from appointing him in any suitable post subject to the
provisions of relevant Rules, but that shall not be done at the cost of the present
petitioner. The aforesaid Commission shall make such recommendation within a period
of three weeks from today. The State respondents shall issue necessary notification
within three weeks thereafter.
13.
The writ petition stands allowed with the aforesaid observations and directions, as
indicated above.
***
IN THE GAUHATI HIGH COURT
ITANAGAR BENCH
W.P. (C) NO.349 (AP) OF 2008
(W.P. (C) No.2876 of 2008)
D.D. 20.04.2010
Hon’ble Mr. Justice I.A. Ansari &
Hon’ble Mr. Justice A.C.Upadhyay
Shri Tageng Tamuk
Vs.
The State of A.P. & Ors.
…
Petitioner
…
Respondents
Recruitment:
Whether in a recruitment to the post of Principal by direct recruitment as per amended
rules a candidate working as Vice Principal which is the feeder post for promotion to the
post of Principal under unamended rules which governed the recruitment before
amendment can challenge the recruitment under amended rules on the ground that the
vacancy for the post of Principal arose before amendment rules? – No.
The petitioner was appointed as a lecturer on 13.5.1992 – Subsequently as per
guidelines dated 07.07.2006 the petitioner was designated as Vice Principal on 4.12.2006
– Under the Recruitment Rules of 2000 which governed the recruitment to the post of
Principal of Government Colleges the post of Principal was required to be filed up 100%
by promotion on merit cum seniority from amongst selection grade lecturers having 10
years of regular services in the grade – On 18.12.2007 Recruitment Rules 2000 were
amended as per Recruitment (Amendment) Rules 2007 which provided for filling up of
100% vacancies through direct recruitment on the basis of interview – The said Rules
prescribed Ph.D. apart from Master’s Degree – 02 posts of Principal were advertised on
19.03.2008 - The petitioner challenged the advertisement on the ground that as one of
the two vacancies arose before the Amendment Rules he should be considered for
promotion as per the unamended Rules – The Court in view of the fact that as the
petitioner had withdrawn writ petition filed by him seeking similar relief without
reserving liberty to file fresh case held that the present writ petition was barred by
principles of res-judicata and dismissed the writ petition.
Held:
Since the petitioner withdrew from WP (C) 212 AP of 08, without seeking or
being granted permission to file a fresh application, the remedy available to the petitioner
under Art.226 of the Constitution, should be deemed to have been abandoned by him in
respect of the cause of action based on which earlier application was filed.
Cases Referred:
[1987] 1 SCR 200::AIR 1987 (SC) 88 - Sarguja Transport Service v. State Transport
Appellate Tribunal, Gwalior & Ors.
1990 (1) SCC-288 - J.Ranga Swamy –vs- Govt of Andhra Pradesh & Ors
1993 Supp (3) SCC-9 - V.K.Sood –vs- Secretary Civil Aviation and Ors
2003(2) ACC-632 - P.U. Joshi and Ors –vs- Accountant General, Ahmedabad & Ors
2009 (5) SCC -515 - K.A.Nagamani –vs- Indian Airlines & Ors
ORDER
By this application under Article 226 of the Constitution of India, the petitioners
have challenged the Recruitment (Amendment) Rules, 2007 notified on 18.12.2007 and
the corrigendum thereto dated 1.4.2008, for being ultra vires and unconstitutional. The
petitioners have further prayed to set aside the advertisement No.PSC-R-01/2008 dated
19th March, 2008 (vide Annexure-12), issued by the Arunachal Pradesh Public Service
Commission, in terms of the above Rules 2007.
2.
We have heard Mr. A.Apang, learned counsel for the petitioner in WP (C) 349
(AP) 2008, Mr. T.Taki, learned counsel for the petitioners in WP(C) 172 (AP) 2009 and
Mr. R.H.Nabam, learned Senior Govt. Advocate, appearing on behalf of the State
respondents. We have heard Mr. N.Tagia, learned Standing Counsel, appearing on behalf
of the APPSC and Mr. K.Dabi, learned counsel, representing the private respondents.
3.
Both the writ petitions afore noted having raised a common question of law based
on similar factual matrix, we propose to dispose of both the petitions by this common
judgment.
4.
Facts which are common to both the writ petitions may be narrated in brief as
follows:-
The petitioner was appointed as a lecturer on ad-hoc basis in the Department of
‘Education’ of J.N. College, Pasighat on 8.2.1990, initially for 3 (three) months.
Thereafter, on being selected by the Arunachal Pradesh Public Service Commission, in
terms of its merit list dated 2.4.1992, petitioner was appointed as lecturer on 13.5.1992.
5.
Subsequently, in the year 2006, due to increasing complexities in the College
administration, more particularly for increase of enrolment of the students in the colleges,
the State respondents finding it necessary to appoint Vice-Principals in the colleges,
issued guidelines vide Memo No.SEDN-79/93 dated 7.7.2006. The guidelines provided
for designation of the suitable lecturers (Selection Grade) of the colleges as Vice
Principals, in some selected colleges, which among them were (1) J.N. College, Pasighat,
(2) Dera Natung Govt. College, Itanagar and (3) Indira Gandhi Govt. College, Tezu.
However, it was specifically clarified in the ‘guidelines for selection of lecturers
for placement as Vice Principals’, that the designation of Vice Principal of the Colleges
would not be treated as promotion and no benefit as such would be available to such
Vice-Principals, in terms of Rule FR 22 (1) (a) (1). However, the State respondents
reserved the right to select suitable candidates for the post of Vice-Principal on merit cum
seniority basis. Thus, the State Government could again replace the Vice Principal so
designated as a lecturer at any time, without showing any reason. The guidelines also
indicated the method of selection of the lecturer for placement as Vice Principal in the
Government colleges and the duties and responsibilities of a Vice Principal.
7.
The petitioner, fulfilling all the criteria, being eligible for the post of Vice
Principal, was designated as Vice Principal on 4.12.2006 and was posted in the said
capacity in Dera Natung Govt. College, Itanagar.
8.
The Rules of Recruitment to the post of Principal of the Government colleges of
Arunachal Pradesh was governed by the Recruitment Rules 2000, (hereinafter Rules
2000), until it’s amendments on 18.12.2007. The said Rule was made under proviso to
Article 309 of the Constitution of India. In terms of the Rules 2000, the post of Principal
was required to be filled up 100% by promotion on merit-cum-seniority basis from
amongst the Selection Grade Lecturer, having 10 years of regular services in the grade.
9.
In the meanwhile, Mrs. Uma Dutta, the Principal of Dera Natung Govt. College,
Itanagar, retired from service on superannuation. Accordingly, the respondent No.3 by
an Order dated 31.10.2007, directed Mrs. Dutta to handover the charge of the office of
the Principal to the petitioner, upon her release from the service on the eve of retirement.
Consequently, the petitioner was allowed to hold the charge and function as Principal of
the said College until a regular Principal was posted. In view of the above, the petitioner
continued to hold the charge of the Principal till date.
10.
It is submitted on behalf of the petitioner that the petitioner having put in 18 years
of service in the department also held the post of Vice-Principal of the college since
4.12.2006, on the basis of his merit cum seniority position. It has been submitted on
behalf of the petitioner that since there was a vacant post of Principal, which was
available prior to the issuance of amendment Rules, 2007, the respondent authorities
ought to have considered his promotion in the same manner as it was done in other
similarly situated candidates.
11.
On 18.12.2007, the Rule 2000, which governed the service condition for the post
of Principal, was amended. The amendment rule known as Recruitment (Amendment)
Rule 2007 provided for filling up of 100% vacancies through direct recruitment on the
basis of interview to be conducted by the Arunachal Pradesh Public Service Commission,
from amongst the college lecturers under the Government of Arunachal Pradesh
possessing (i) a Master’s Degree with at least 55% of the marks (50% for ST candidates)
or its equivalent grade of B in the 7 point scale with latter grades O, A, B, C, D and F, (ii)
Ph.D. for equivalent qualification and (iii) total experience of 10 years of
teaching/research in University/Colleges and other institutions of higher education.
The amended Recruitment Rules, 2007, reads as follows:
“No.HE/CE-62/99-in exercise of the power conferred by the proviso to
article 309 of the Constitution, the Government of Arunachal Pradesh
hereby makes the following rules further to amend the Recruitment Rules,
2000 to the post of Principal in the Department of Higher Education under
the Government of Arunachal Pradesh namely:1. These rules may be called the Recruitment (Amendment) Rules,
2007 for the post of Principal in the Department of Higher
Education.
2. They shall come into force on the date of their publication in the
Official Gazette.
3. In the Schedule to the Recruitment Rules, 2000 for the entries in
columns 2, 11, 12 and 13, the following shall be substituted,
namely:(i) Column 2: “07 (seven) ‘2007’ subject to variation dependent on
work Load.”
(ii) Column 12: “100% by direct recruitment.”
(iii) Column 12: “By direct recruitment on the basis of interview by
the Arunachal Pradesh Public Srvice Commission from amongst the
college lecturers under the Government of Arunachal Pradesh
possessing:
(1) A Master’s degree with atleast 55% of the marks (50% for ST
candidates) or its equivalent grade of B in the 7 point scale with
latter grades O, A, B, C, D, E and F.
(2) Ph.D. or equivalent qualification.
(3) Total experience of 10 (ten) years of teaching/research in
Universities/Colleges and other institutions of higher education.
4. Column 13 :
“No.HE/CE-62/99/7546
the 1.4.08
“Not applicable”.
Sd/Chief Secretary,
Government of Arunachal Pradesh,
Itanagar.”
Dated Itanagar,
CORRIGENDUM
Sl.No.3, Column No(2) of Govt. Notification issued vide No.HE/CE62/99 dated 18.12.07 for amendment the Recruitment Rules, 2000 to the post of
Principal in the Department of Higher Education under the Govt. of Arunachal
Pradesh may please be read as 11 instead of 12.
Sd/Chief Secretary,
Government of Arunachal Pradesh,
Itanagar.”
12.
In terms of aforesaid amendment, the Arunachal Pradesh Public Service
Commission, Itanagar (APPSC), i.e. respondent No.4, published an advertisement dated
19th March 2008, inviting applications for filling up the posts of Principal in the colleges
of Arunachal Pradesh through direct recruitment, in conformity with the method of
selection, as envisaged in the Recruitment Rules, 2007.
13.
It is submitted, on behalf of the petitioner, that one vacancy, out of 2 (two)
vacancies, for which the impugned advertisement has been issued, arose in the year 2007
due to retirement of Mrs. Uma Dutta, on 31.10.2007, as Principal of Dera Natung
College, Itanagar.
14.
In writ petition WP (C) 172 AP/09, the same petitioner assailed the advertisement,
dated 19.03.2008, issued by the APPSC to fill up 2 (two) posts of Principal of
Government colleges of Arunachal Pradesh, in terms of the amended Rule of 2007, by
which different eligibility criteria were introduced. Learned counsel for the petitioner
emphasized on the fact that the recruitment to the said post of Principal, which had fallen
vacant prior to the introduction of the amendment Rules 2007, would be governed by preamended rules of recruitment and the petitioner, who was eligible for the post of
Principal under the pre-amended rules, deserved to be considered for promotion to the
post of Principal.
15.
The amended Rule of 2007 raised the required qualification for promotion to the
post of Principal in which the petitioner would no longer be eligible, further the amended
Rules, 2007, proposed recruitment to the post of Principal, not by promotion as it stood
prior to the amendment, but only by way of direct recruitment.
16.
The learned counsel for the petitioner further contended that the petitioner, who
had completed more than 10 (ten) years of service in the cadre and was also holding the
post of Vice-Principal in substantive capacity, was eligible to be considered for
promotion/appointment to the post of Principal, since he had all the requisite
qualifications under the said Recruitment Rules 2000, as it stood prior to its amendment
in 2007.
17.
Learned counsel for the petitioner submits that the advertisement, issued by the
APPSC, is violative of Article 14 and 16 of the Constitution of India as it takes away the
fundamental right of the petitioner to be considered for the post of Principal.
18.
The State respondents submitted affidavit contending therein that the petitioner
was placed in Selection Grade from 3.3.2001 vide order, dated 23.3.2005 and not from
13.5.1992 as claimed by the petitioner.
19.
Learned counsel for the respondents/State Government submitted that sometime,
in the month of February, 2007, a conference, on Higher and Technical Education, was
held and it was resolved that since the State Government is following the UGC norms of
pay scales, the recruitment rules for Principal should also be suitably amended by
incorporating the UGC norms. Accordingly, Recruitment Rules of 2000 came to be
amended in 2007 vide notification dated 18.12.2007.
20.
Learned counsel for the State respondents further contended that the last
appointment to the post of Principle was made on 27.10.2006 and, thereafter, no
appointment has so far been made. The State respondents, stated in their affidavit that
two persons, namely, Shri Teyek Talom and Shri Motum Nomuk were appointed as the
Principal prior to the amendment of the rules and there were about 7 (seven) senior most
APST candidates, who were senior to the petitioner, who had not been promoted to the
post of the Principal.
21.
It is admitted by State respondents that the 2 (two) posts of Principal of the
Colleges fell vacant due to retirement of one Smt. Uma Dutta, on 31.10.2007, as
Principal of Dera Natang Govt. College and due to transfer to Shri Tomar Ete, on
deputation, from the post of Principal, Donyi Polo Govt. College, Kamki, to Rajiv
Gandhi University on 18.10.2007, as a Controller of Examination. It is further stated, in
the affidavit submitted on behalf of the State respondents, that the petitioner was afforded
ample opportunity to complete the research works of Ph.D. by granting study leave, but
failed to complete the same.
22.
The guidelines for selection and placement of Vice Principal in the colleges
further clarified that the post of Vice Principal shall not be a substantive post, but
interchangeable post in the same pay scale and not a promotional post. Therefore, the
petitioner was appointed as Vice Principal on the basis of the said guidelines on
4.12.2006 due to increasing complexities in the college administration, but the said posts,
being an interchangeable post of the same grade and pay scale, the petitioner could not
have staked a claim that he was holding the substantive post of Vice Principal and was,
thus, entitled to be considered for promotion to the post of Principal of the College.
23.
Since the post of the Vice Principal was not a feeder post, we are of the
considered view that the guidelines did not confer any right on the petitioner for being
considered for the post of Principal.
The petitioner was selected in terms of the
guidelines for selection of the Vice Principal by fulfilling all the required qualifications.
Out of several candidates in the fray, the petitioner cannot legitimately expect
consideration of his candidature for the next promotion to the post of Principal since the
relevant Recruitment Rules, for the post of the Principal, have different criteria for
promotion.
24.
Learned counsel for the State respondents took the stand that since the
Government of Arunachal Pradesh is following UGC norms in the colleges and
universities, the recruitment rules, for the post of Principal and lecturer, were accordingly
amended from time to time in consonance with UGC norms. The State respondents
refuted the stand of the petitioner regarding non-publication of the amendment rules in
the official gazette by precisely indicating the date of publication of the amendment Rule
2007 in the official Gazette.
25.
Learned counsel for the State respondents pointed out even if, for an argument’s
sake, the old Rule of 2000 is applied for promotion against vacant post of Principal, the
present petitioner could not have been considered on the simple ground that he did not
come under the zone of consideration as there were about 7 (APST) candidates, who
were much senior to the petitioner. However, the recruitment Rule of 2000 having been
amended in 2007 in consonance with the UGC norms, the relevant rules, now, require
Ph.D. Degree for the post of Principal with 10 years of teaching experience, which the
petitioner does not possess. However, the learned counsel for the respondents/State
Government contended that the petitioner with his existing qualifications without the
Ph.D. Degree had ample scope to be considered for promotion to the post of Deputy
Director or Higher Education, which is also a promotional post for all such lecturer, who
did not possess Ph.D. Degree, which may, ultimately, lead the petitioner to further
promotional avenues of the post of Joint Director and Director of Higher Education in the
State Government.
26.
In W.P.(C) No.349 (AP) of 2008 (Shri Tateng Tamuk –vs- the State of Arunachal
Pradesh), the petitioner had challenged the legality and validity of the Recruitment Rules
2007, of Principal of Government Colleges of Arunachal Pradesh, as unconstitutional and
illegal. The petitioner also claimed for filling up of one of the posts of Principal, which
had fallen vacant prior to the amendment of Recruitment Rules, 2007, claiming his
eligibility to the post in terms of the pre-amended Recruitment Rules.
27.
In support of his contention, learned counsel for the petitioner advanced the
following arguments:
The Recruitment Rules of 2000, for selection to the post of Principals, rested on
the selection of posts by 100% promotion from among the lecturers on the basis of meritcum-seniority of such candidates having 10 years regular service in the grade. However,
by introduction of the amendments under the Rules of 2007, the posts of Principal have to
be filled up 100% by direct recruitment from amongst the lecturers of Arunachal Pradesh,
having Ph.D. degree or equivalent qualifications and 10 years experience. Learned
counsel for the petitioner further contended that 100% direct recruitment curtails the
promotional avenues of those lecturers, who had been serving for the last so many years
and were due to retire from service. However, it is submitted on behalf of the petitioner,
that the post of Principal and Deputy Director, being administrative posts, do not require
Ph.D. degree as additional qualification of eligibility.
28.
Learned counsel for the petitioner further submitted that although in their affidavit
the respondents authorities concerned talked about UGC norms being made applicable,
nowhere UGC norms has been incorporated in the Recruitment Rules, 2007, to give the
benefit of 15 years of experience for promotion and retirement age of 65 to the college
lecturers. It is submitted, on behalf of the petitioner, that the authorities cannot mould the
Recruitment Rules so as to suit some selected candidates by forgoing the welfare of all
concerned and public interest.
29.
Resisting the challenge to the vires of the amended Recruitment Rules, 2007,
made by the petitioner, learned counsel for the State respondents relied on the following
decisions of the Apex Court on the question of the power and authority of the state
Governments in making rules in terms of the proviso to Article 309 of the Constitution,
for the welfare of the Government employees, which are as follows:
(i) J.Ranga Swamy –vs- Govt of Andhra Pradesh & Ors, reported in 1990
(1) SCC-288; (ii) V.K.Sood –vs- Secretary Civil Aviation and Ors,
reported in 1993 Supp (3) SCC-9; (iii) P.U. Joshi and Ors –vs- Accountant
General, Ahmedabad & Ors, reported in 2003(2) ACC-632; (iv)
K.A.Nagamani –vs- Indian Airlines & Ors, reported in 2009 (5) SCC 515.
1)
In the case of J.Ranga Swamy –vs- Govt of Andhra Pradesh & Ors,
reported in 1990 (1) SCC-288: The Apex Court held that it is not for the
court to consider the relevance of qualifications prescribed for various
posts. The post in question is that of a Professor and the prescription of a
doctorate as a necessary qualification therefore is nothing unusual.
Petitioner also stated before us that, to the best of his knowledge, there is
no doctorate course anywhere in India in radiological physics. That is
perhaps why a doctorate in nuclear physics has been prescribed. There is
nothing prima facie preposterous about this requirement. It is not for us to
assess the comparative merits of such a doctorate and the BARC diploma
held by the petitioner and decide or direct what should be the
qualifications to be prescribed for the post in question.
2)
In V.K.Sood –vs- Secretary Civil Aviation and Ors, reported in
1993 Supp (3) SCC-9 the Apex Court held that rules framed under Article
309 in the exercise of the rule-making power, the President or authorized
person is entitled to prescribe method of recruitment, qualifications both
educational as well as technical for appointment or conditions of service to
an office or a post under the State. The rules thus having been made in
exercise of the power under proviso to Article 309 of the Constitution,
being statutory, cannot be impeached on the ground that the authorities
have prescribed tailor made qualifications to suit the stated individuals
whose names have been mentioned in the appeal. Suffice to state that it is
settled law that no motives can be attributed to the Legislature in making
the law. The rules prescribed qualifications for eligibility and the
suitability of the appellant would be tested by the Union Public Service
Commission.
3)
In P.U. Joshi and Ors –vs- Accountant General, Ahmedabad &
Ors, reported in 2003(2) ACC-632 Hon’ble Supreme Court held that
determination of conditions of service, alteration thereof by amending
rules pertain to executive policy and within exclusive policy and within
exclusive discretion of the State.
Questions relating to the constitution, pattern, nomenclature of
posts, cadres, categories, their creation/abolition, prescription of
qualifications and other conditions of service including avenues of
promotions and criteria to be fulfilled for such promotions pertain to the
field of policy is within the exclusive discretion and jurisdiction of the
State, subject, of course, to the limitations or restrictions envisaged in the
Constitution of India and it is not for the statutory tribunals, at any rate, to
direct the Government to have a particular method of recruitment or
eligibility criteria or avenues of promotion or impose itself by substituting
its views for that of the State. Similarly, it is well open and within the
competency of the State to change the rules relating to a service and alter
or amend and vary by addition/substraction the qualifications, eligibility
criteria and other conditions of service including avenues of promotion,
from time to time, as the administrative exigencies may need or
necessitate. Likewise, the State by appropriate rules is entitled to
amalgamate departments or bifurcate departments into more and constitute
different categories of posts or cadres by undertaking further
classification, bifurcation or amalgamation as well as reconstitute and
restructure the pattern and cadres/categories of service, as may be required
from time to time by abolishing the existing cadres/posts and creating new
cadres/posts. There is no right in any employee of the State to claim that
rules governing conditions of his service should be forever the same as the
one when he entered service for all purposes and except for ensuring or
safeguarding rights or benefits already earned, acquired or accrued at a
particular point of time, a government servant has no right to challenge the
authority of the State to amend, alter and bring into force new rules
relating to even an existing service.
4)
In the case of K.A. Nagamani –vs- Indian Airlines & Ors, reported
in 2009 (5) SCC -515, the Apex Court referring to the decision of P.U.
Joshi v. Accountant General (supra) held that the competent authority has
power, subject to constitutional limitations, to lay down conditions of
service, amend them and to abolish posts/cadres, amalgamate or bifurcate
departments or to do restructuring. These all matters falls within the ambit
of policy consideration of the Executive. Relevant extract of the
observation made by the Apex Court is reproduced herein below:
“It needs no restatement that the authorities are entitled to determine all
conditions of service, alteration thereof by amending rules, constitution,
classification, abolition of posts, cadres or categories of service,
amalgamation or bifurcation of departments, reconstitution, restructuring
of the pattern etc. as the same pertain to the field of policy within the
exclusive jurisdiction of the employer, subject to limitations or restrictions
envisaged in the Constitution.
37. “10.
…. There is no right in any employee of the State to claim
that rules governing conditions of his service should be forever the same
as the one when he entered service for all purposes and except for
ensuring or safeguarding rights or benefits already earned, acquired or
accrued at a particular point of time, a government servant has no right to
challenge the authority of the State to amend, alter and bring into force
new rules relating to even an existing service. (See P.U. Joshi v.
Accountant General (supra)
30.
On careful consideration of the rival contentions made by the learned counsel for
the parties, it is apparent that Article 309 of the Constitution of India empowers the State
Government to make rules for promotion of its employees. The State has authority to
alter and amend the rules as and when required. Though the petitioner has laid emphasis
on the fact that the Recruitment (Amendment) Rules of 2007 has been tailor-made to
favour some individuals in the department however, the law has been settled by the Apex
Court in the case of V.K.Sood vs. Secretary Civil Aviation and Ors (supra), by holding
that no motives can be attributed to the Legislature in making the law.
It has been contended, on behalf of the petitioner, that the post of Principal of
college is an administrative post and, therefore, degree of Ph.D. is not essential. The
aforesaid contention of the learned counsel for the petitioner was discussed in P.U. Joshi
and Ors –vs- Accountant General, Ahmedabad & Ors (supra), wherein it was held that it
is very well open and within the competency of the State to change the rules relating to
service and accordingly, alter or amend and vary, by addition/substraction, the
qualifications, eligibility criteria and other conditions of service including avenues of
promotion, from time to time, as the administrative exigencies may need or necessitate.
31.
Article 309 of the Constitution of India empowers the State Government to make
rules for its employees. However, in doing so, the State cannot take away such rights,
which the Government servant may have already acquired. The present Recruitment
(Amendment) Rules, 2007, have been made in consonance with the UGC norms in order
to select the best candidates for the post of Principal.
32.
On careful analysis of the arguments advanced by the learned counsel for the
petitioner, the prayer for declaration of the Recruitment (Amendment) Rules, 2007, as
ultra vires and unconstitutional is not sustainable in terms of the decisions of the Supreme
Court, as discussed above, since the petitioner could not attribute any evil design in the
action of the State-respondents in introducing the Recruitment (Amendment) Act, 2007.
All such reasons put forward, on behalf of the petitioner, for the relief sought have been
discussed and disapproved by the Apex Court.
33.
In view of the above discussion, the relief prayed for by the petitioner for setting
aside the Recruitment (Amendment) Rules, 2007, and also for a declaration of the said
rules as ultra vires and unconstitutional, in W.P. (C) no.349 (AP) of 2008, is accordingly
rejected.
34.
In W.P.(C) No.172 (AP) of 2009 (Shri Tateng Tamuk & Anr. –vs- the State of
Arunachal Pradesh & Ors.) the petitioner claimed that the advertisement issued by the
APPSC, dated 19.03.2008, for filling up of 2 (two) posts of Principal is unconstitutional
as the advertised posts fell vacant prior to the amendment of the recruitment rules, and as
such, the petitioner was eligible to be considered for promotion to one of the said 2 (two)
posts of Principal.
35.
Learned counsel for the State respondents raised the plea of res judicata/bar to
subsequent writ petition by specifically indicating, in his submission, that the petitioner,
along with one Shri Sanga Khandu (Vice Principal) had filed a writ petition challenging
the Recruitment (Amendment) Rules of 2007, which gave rise to W.P. (C) No.212 (AP)
2008. However, later on, the petitioner, by filing a Miscellaneous Application No.50
(AP) of 2008, withdrew his name from the petition without taking any liberty to file a
fresh petition to challenge the amended recruitment rules of 2007 and, as such, the
learned counsel submits that the petitioner is barred from raising the same issue, again,
before the Court of law.
36.
Learned counsel further pointed out that the grounds of challenge is the same as
that had been raised in Writ Petition No.212 (AP) 2008, which was dismissed, finally, by
this Court on 10.02.2009. Even the impugned advertisement issued by APPSC, on
19.03.2008, for selection and appointment to 2 (two) vacant posts of Principals, in the
Government colleges, were under challenge in the earlier writ petition.
37.
It is contended, on behalf of the State respondents, that the petitioner after
unsuccessful attempt to obtain interim protection, from the Division Bench in connection
with this case, filed yet another writ petition, which came to be registered as W.P. (C)
No.172 (AP) of 2009. In the said writ petition, the petitioner took the plea that the
APPSC is conducting the said interview by depriving eligible candidates and against the
UGC norms and a stay order has been passed by a single Bench. Therefore, W.P. (C)
No.172 (AP) of 2009 has also been taken up by this Bench for disposal together. On
perusal of the prayer of the writ petitioner in W.P. (C) No.212 (AP) of 2008, it appears
that the petitioner had challenged the said advertisement, dated 19.03.2008, for selection
of 2 (two) posts of Principals of Government college issued by the Arunachal Pradesh
Public Service Commission, Itanagar. The grounds of challenge were also based on the
principle that the posts which had fallen vacant prior to the amendment of the
Recruitment Rules, 2007, shall have to be filled up in terms of the pre-amended Rules.
38.
In W.P. (C) No.212 (AP) of 2008, the learned single Bench of this Court, on
careful consideration of the prayer made by the petitioner in the writ petition and after
threadbare discussions held as follows:
“11. The law regarding applicability of amended or pre-amended Rules
in the matter of promotion is no longer res integra. All the authorities
cited and relief upon by the learned counsel for the petitioner are arising
out of claim for consideration for promotion on the basis of pre-amended
position. However, in the case before me the petitioner is claiming his
right to be considered for direct appointment to be the post of Principal.
Hence, the authorities citied on behalf of the writ petitioner is not
applicable. I am also of the considered opinion that since the source of
filling up the post of Principals of the Govt. Colleges have been switched
over from promotion to direct recruitment, the writ petitioner has no locus
standi to claim for his consideration for direct appointment on the basis of
pre-amended position. Had it been a case of filling up the post of
Principals on promotion after certain changes in the criteria, the writ
petitioner could have certainly raised a question that the promotion should
have been done on the basis of pre-amended Recruitment Rules.
However, the situation is otherwise.
12.
In the result, I find no merit in this writ petition. Consequently, the
same stands dismissed. Interim order passed by this Court earlier stands
vacated.”
Admittedly, the petitioner to this writ petition withdrew himself from the earlier
writ petition without taking leave to file any petition.
39.
We have considered the submissions made by the learned counsel for the
petitioner. On a reading of the order, dated 10.02.09, passed in W.P. (C) No.212 (AP) of
2008, it is crystal clear that the cause of action for these two writ petitions is one and the
same. The claim of the petitioner in both the applications is founded on the same subject
matter and the reliefs sought for both the applications, were in relation to cancellation of
the advertisement issued on 19.03.08.
However, in writ petition W.P. (C) No.349 (AP) of 2009, the petitioner has
challenged the vires of the Recruitment (Amendment) Rules 2007, with consequential
relief of cancellation of the advertisement issued on 19.03.08.
40.
We have already held that there is no merit in the challenge to the vires of the
Rules 2007 made by the petitioner in W.P. (C) No.349 (AP) of 2008. A close reading of
both the writ petitions i.e. W.P. (C) No.212 (AP) of 2008 and W.P. (C) No.172 (AP) of
2009, discloses that the relief sought for by the petitioner is basically for cancellation of
the impugned advertisement issued in respect of recruitment for the post of Principal.
The petitioner apparently abandoned the claim made in W.P. (C) No.212 (AP) of 2008,
when he chose to withdraw himself from the said writ petition by filing formal
application. Based on the petitioner’s request, the learned Single Bench allowed the
petitioner to withdraw from the writ petition. The writ petition W.P. (C) No.212 (AP) of
2008 was finally disposed of on merit rejecting the reliefs claimed by the petitioner
therein. It is thereafter that the present writ petition, W.P. (C) No.172 (AP) of 2009 has
been filed on 14.5.2009, challenging the recruitment process in terms of the
advertisement, dated 19.03.08. The learned single Judge had not, while allowing Mic.
App. No.50 of 2008 to be withdrawn, granted leave to the petitioner to file a fresh
application in respect of the reliefs sought therein. The Apex Court has, in Sarguja
Transport Service v. State Transport Appellate Tribunal, Gwalior and Ors. [1987] 1 SCR
200::AIR 1987 (SC) 88, held that a petitioner cannot, after withdrawing a writ petition
filed by him in the High Court under Article 226 of the Constitution without seeking and
being granted leave to institute a fresh writ petition, file a fresh writ petition in respect of
the same cause of action in the High Court. The Apex Court held as follows:
“9.
The point for consideration is whether a petitioner after
withdrawing a writ petition filed by him in the High Court under Art. 226
of the Constitution of India without the permission to institute a fresh
petition can file a fresh writ petition in the High Court under that Article.
On this point the decision in Daryao’s case (supra) is of no assistance. But
we are of the view that the principle underlying R.1 of O.XXIII of the
Code should be extended in the interests of administration of justice to
cases of withdrawal of writ petition also, not on the ground of res judicata
but on the ground of public policy as explained above. It would also
discourage the litigant from indulging in bench-hunting tactics. In any
event there is no justifiable reason in such a case to permit a petitioner to
invoke the extraordinary jurisdiction of the High Court under Art. 226 of
the Constitution once again. While the withdrawal of a writ petition filed
in High Court without permission to file a fresh writ petition may not bar
other remedies like a suit or a petition under Art. 32 of the Constitution
since such withdrawal does not amount to res judicata, the remedy under
Art. 226 of the Constitution should be deemed to have been abandoned by
the petitioner in respect of the cause of action relied on in the writ petition
when he withdraws it without such permission. In the instant case the
High Court was right in holding that a fresh writ petition was not
maintainable before it in respect of the same subject-matter since the
earlier writ petition had been withdrawn without permission to file a fresh
petition. We, however, make it clear that whatever we have stated in this
order may not be considered as being applicable to a writ petition
involving the personal liberty of an individual in which the petitioner
prays for the issue of a writ in the nature of habeas corpus or seeks to
enforce the fundamental right guaranteed under Art. 21 of the Constitution
since such a case stands on a different footing altogether. We, however,
leave this question open.” (Emphasis supplied).
41.
Applying the principles laid down by the Apex Court in Sarguja Transport
Service v. State Transport Appellate Tribunal, Gwalior and Ors. (supra) to the facts of the
present case, we are of the opinion that since the petitioner withdrew from WP (C)
No.212 AP of 08, without seeking or being granted permission to file a fresh application
in respect of the same subject matter, the remedy, available to the petitioner under
Art.226 of the Constitution, should be deemed to have been abandoned by him in respect
of the cause of action based on which earlier application was filed. More so, when the
subject matter in both writ petition is one and the same. The petitioner did not prefer
appeal against the judgment and order passed in WP (C) 212 AP of 2008. The petitioner
cannot, therefore, be permitted to avail relief by instituting WP (C) No.172 AP of 09 with
regard to the very same subject matter.
In view of the above, we are of the considered view that the relief sought for by
the petitioner in WP (C) No.172 AP of 2009 cannot be granted by this Court in the facts
and circumstances stated above.
42.
With the above observations and directions, both the writ petitions stand
dismissed. However, we do not pass any order as to costs.
***
IN THE GAUHATI HIGH COURT
ITANAGAR PERMANENT BENCH
W.P. (C) NO.413 (AP) OF 2008
D.D. 16.06.2010
Hon’ble Mr. Justice P.K. Musahary
Shri K.Sidhardhan
Vs.
The Govt. of A.P. & Ors.
…
Petitioner
…
Respondents
Recruitment - Limited Departmental Competitive Examination:
Whether procedure prescribed for direct recruitment by competitive examination is
applicable to Limited Departmental Competitive Examination prescribed for the post of
Section Officer in the Government Secretariat? – Yes.
An advertisement was issued for filling up two posts of Section Officer through
Limited Departmental Competitive Examination – Assistants of Arunachal Pradesh
Secretariat who had rendered 06 years of regular service are eligible to apply for the post
as per 2004 Rules – As per O.M. dated 7.1.2008 candidates securing a minimum of 33%
or more marks in each written paper and 45% aggregate shall be eligible for viva-voce –
Result of written test was published on the basis of said O.M. on 15.9.2008 wherein
petitioner’s name appeared at Sl.No.2 – Government as per letter dated 22.9.2008
informed that O.M. dated 7.1.2008 is meant for selection of candidates for viva-voce test
in respect of direct recruitment and it is not applicable to Limited Department
Competitive Examination – On the basis of the said letter revised result was published on
23.9.2008 by adding 03 more names including the name of respondent No.4 who secured
29% as against minimum of 33% in the written test though his name did not figure in the
earlier notification dated 5.9.2008 – Ultimately, after the interview as many as 06
candidates recommended against 01 post under GM the names of Respondent No.4 and
petitioner have been placed at Sl.No.6 and 9 respectively – The said action was
challenged in this writ petition – The High Court interpreting O.M. dated 7.1.2008 that
minimum of 33% marks prescribed as per O.M. dated 7.1.2008 is applicable to the
recruitment through Limited Departmental Competitive Examination has allowed the writ
petition and directed to declare the results as per result notification dated 15.9.2008 in
terms of Government O.M. dated 7.1.2008.
Cases Referred:
AIR 1990 SC 461 - Kumari Anamica Mishra & Anr. –vs- U.P. Public Service
Commission, Allahabad & Ors.
(2007) 9 SCC 497 - P.Mohanan Pillai –vs- State of Kerala
ORDER
Heard Mr. A.Apang, learned counsel for the writ petitioner. Also heard Mr.
R.H.Nabam, learned Senior Govt Advocate, Arunachal Pradesh for State respondents,
Mr. N.Tagia, learned standing counsel for respondent Arunachal Pradesh Public Service
Commission (APPSC) and Mr. A.K.Singh, learned counsel for private Respondent No.4.
2.
An advertisement was issued by the Arunachal Pradesh Public Service
Commission (hereinafter referred to as ‘Commission’ in short) for filling up 2 posts of
Section Officer Gr.-B (Gazetted) in the Arunachal Pradesh Secretariat through Limited
Departmental Competitive Examination. One post is reserved for Arunachal Pradesh
Scheduled Tribe candidate (hereinafter referred to as ‘APST candidate’ in short) while
the other post kept unreserved for general candidates. The Assistants of Arunachal
Pradesh Civil Secretariat (hereinafter referred to as ‘Secretariat’ only) who have rendered
6 years of regular service in the grade are eligible to apply for the said post in prescribed
format. The selections are to be made on the basis of marks secured in the written
examination. The written test was held on 28th and 29th of June, 2008, and the result of
the written test was published on 15.09.2008 wherein the petitioner’s name appeared at
Serial No.2. The candidates who passed the written test were required to appear in the
viva-voce test scheduled to be held on 23.09.2008 which was postponed by a press
release dated 23.09.2008 issued by the Commission. The Commission published another
revised result by its notification dated 23.09.2008 indicating the date of viva-voce test to
be held on 26.09.2008 by adding 3 more names in the list of General Category including
the name of private Respondent No.4, Sri Roman Bora, whose name did not figure in the
earlier notification dated 15.09.2008. The viva-voce test was held on 26.09.2008 and the
result was declared on the same day. The petitioner also appeared in the said test. Thus,
in the result notification dated 23.09.2008, as many as 6 candidates were recommended
against one post in the unreserved/general category. The names of private Respondent
No.4 and petitioner have been placed at Serial Nos.6 and 9 respectively.
3.
The appointment to the post of Section Officer. Group-B (Gazetted), is governed
by “The Arunachal Pradesh Secretariat Recruitment to the post of Section Officer, Rules,
2004 (hereinafter referred to as ‘Rules of 2004’ in short). As per provisions thereunder,
50% percent of the posts is required to be filled up by promotion and the other 50% of the
posts is required to be filled up by Limited Departmental Competitive Examination from
amongst the Assistants of Arunachal Pradesh Secretariat who have rendered 6 years of
regular service in the grade on the basis of merit adjusted from the subjects namely:- (a)
General
Knowledge
(b)
General
English/Noting/Drafting
Essay
Writing
(c)
FRs/SR/GFRs/Pension Rules/CCS (CCA) Rules/General Provident Fund Rules (100
marks in each subjects) and (d) viva voce (50 marks).
4.
The State Government issued an Office Memorandum No.54/2006 dated
07.01.2008 signed by the Secretary (AR), Government of Arunachal Pradesh, providing
that candidates securing a minimum of 33% or more marks in each written paper and
securing 45% of marks out of aggregate total marks in the written examination papers,
shall be eligible for viva-voce. It is specifically provided therein that candidates securing
less than 33% of marks in any of the written examination paper, shall not be eligible for
appearing in the viva-voce test. The respondent No.4 secured only 29 marks out of 100
marks in the written test in the Subject of FRs/SRs/GFRs/Pension Rules etc., whereas the
petitioner secured more than 33% in all the subjects of written examination.
The
Secretary to the Commission, before declaring the results, addressed a letter dated
26.08.2008 to the Secretary (AR) seeking clarification as to whether the Limited
Departmental Examination conducted for the purpose of recruitment of Section Officer
shall attract the provisions of Government O.M. dated 07.01.2008 under which a
candidate is required to secure compulsorily 33% of marks in all the subjects. To the
aforesaid query, the Deputy Secretary (AR), vide letter dated 07.07.2008, clarified that
33% of marks in the minimum clarifying marks and the candidates securing less than
33% of marks do not deserve appointment in Government service. The AR Department
by another letter dated 08.09.2008, signed by the Under Secretary, informed the Secretary
to the Commission that the confusion has already been clarified vide earlier letter dated
07.07.2008. Thereafter, the Under Secretary (AR) by letter dated 22.09.2008 informed
that the O.M. dated 07.01.2008 is actually meant for selection of candidates for viva-voce
test in respect of direct recruitment examination and it does not apply to the Limited
Departmental Competitive Examination and as such, the earlier communication dated
08.09.2008 has been withdrawn to enable the Commission for re-evaluation and redeclaration of result. On the basis of this letter, the Commission re-evaluated and redeclared the result of the candidates as stated earlier vide its notification dated
23.09.2008.
5.
The propriety, authority and legality of the aforesaid actions of the State
respondents are in question in this writ proceeding.
6.
Mr. Apang, learned counsel for the petitioner, submits that although the Rules of
2004 do not prescribe the minimum percentage of marks to be obtained by the candidates
in the written test, it is necessary for a candidate to obtain minimum 33% of marks in
each subjects in the written examination as has been prescribed in the O.M. dated
07.01.2008 irrespective of written examination in the open competition or the Limited
Departmental Competitive Examination, conducted by the Commission.
The AR
Department initially took a right stand which was communicated through its letters dated
07.07.2008 and 08.09.2008, but due to some misrepresentation and mis-advice, it
changed its stand and withdrew the aforesaid letters thereby providing that candidates
securing less than 33% of marks in the Limited Departmental Competitive Examination
would be eligible for appointment to the post of Section Officer.
7.
Mr. Apang, learned counsel, referring to various office notings in the aforesaid
matter which he received through Right to Information Act, 2005, submits that the socalled letter dated 22.09.2008 withdrawing the earlier letters on the stand that the
candidate must secure 33% of marks in all the subjects, is illegal and non-est inasmuch as
the same was issued by a Under Secretary who has no authority to issue such letter
without the approval of the Commissioner or Secretary to the Department concerned.
According to him, the minimum marks of 33% in all the subjects as provided in O.M.
dated 07.01.2008 was circulated as a matter of public policy which was approved by the
State Cabinet and such a decision cannot be changed or amended even by the
Commissioner/Secretary of the Department, not to speak of the Officer like the Under
Secretary, who is at the bottom of the hierarchy. The decision was taken at the levels of
Deputy Secretary and Under Secretary without the approval of the Commissioner or the
Secretary of the Department with some vested interests inasmuch as the aforesaid Deputy
Secretary and the Under Secretary took the initiative after receiving a representation from
one Sri Marnya Angu who was a candidate for the said post but failed to obtain the
minimum 33% of marks in all the subjects. The fact that the aforesaid Marnya Angu
made a representation and the initiative was taken by the officers at the levels of Deputy
Secretary and Under Secretary, according to Mr. Apang, learned counsel, reveals from
the correspondences made between the Secretary and the Commission and the Secretary
(AR), particularly from the Commission’s letter dated 19.09.2008 (Annexure-IX to the
writ petition). The entire exercise was done only to accommodate some candidates,
particularly the private Respondent No.4, who could not secure 33% of marks in all the
subjects in the written examination and to deprive the petitioner of the chance of
appointment although he secured more than 33% of marks. The learned counsel relies on
Kumari Anamica Mishra & Anr. –vs- U.P. Public Service Commission, Allahabad &
Ors., reported in AIR 1990 SC 461 wherein it is held, inter alia, that if no defect is
pointed out in regard to the written examination and if the sole objection is confined to
exclusion of a group of successful candidates in the written examination from the
interview, it would not justify cancellation of the written part in the recruitment
examination.
8.
Countering the submissions advanced by Mr. Apang, learned counsel for the
petitioner, it is argued by Mr. R.H.Nabam, learned Senior Govt. Advocate, that the O.M.
dated 07.01.2008 is a procedure prescribed by the Government to regulate the direct
recruitment examination in which a candidate must secure 33% of marks but in the
instant case, the appointment to the posts of Section Officer is required to be made on the
basis of Limited Departmental Competitive Examination under separate rules namely the
Rules of 2004. For recruitment to the posts in question, the O.M. dated 07.01.2008 has
no application inasmuch as the said Rules of 2004 have prescribed separate method of
recruitment,
are limit and other qualifications and the said Rules do not prescribe
minimum percentage of marks being qualified in the written test. Even in the Arunachal
Pradesh Public Service Commission Competitive Examination Rules, 2001 (hereinafter
referred to as ‘Rules of 2001’ in short) and the procedure prescribed thereunder for
holding the competitive examination, no such minimum percentage of marks in each
subject has been prescribed. The O.M. dated 07.01.2008 provides the minimum 33% of
marks for passing the written examination and for being declared eligible to appear in the
viva-voce test.
9.
As regards the allegation that the stand taken by the Department of AR on
applicability of O.M. dated 07.01.2008 to the Limited Departmental Examination for
recruitment to the post of Section Officer, it has been reiterated by Mr. R.H.Nabam,
learned Senior Govt. Advocate, that the withdrawal letter was issue by the AR
Department after thorough examination and interpretation of the provisions under Rules
of 2004 and O.M dated 07.01.2008 as well as Rules of 2001. He further submits that the
purpose of conducting Limited Departmental Competitive Examination is to appoint
qualified and eligible persons in the Secretariat service to the posts of Section Officer
without involving candidates from the open market and as per the procedure of selection
under O.M. dated 07.01.2008, it cannot be applied giving scope for some relaxation in
the procedure for appointment to the posts in question. As regards the allegation of
impropriety in the matter of giving approval by the Deputy Secretary/Under Secretary of
the AR Department, it has been submitted that as per the manual of office procedure, the
Secretary/Joint Secretary to the State Government are the administrative hierarchy of the
Department and he is the principal advisor to the Chief Secretary/Ministers on all matters
of policy and administration within his department and therefore, approval of a Joint
Secretary to the withdrawal of a letter/communication or decision of the Department
Secretary, would be followed if the Secretary to the Department concerned was on leave
or on tour. In the present case, the Secretary concerned was on tour at the relevant point
of time and as such, the Joint Secretary became the immediate senior officer in the
department and in his capacity as a Joint Secretary, approval was given by him and the
same cannot be faulted with or termed as unauthorized or illegal.
10.
Mr. A.K.Singh, learned counsel appearing for private Respondent No.4, submits
that he would fully adapt the submissions made by Mr. R.H.Nabam, learned Senior Govt.
Advocate, Arunachal Pradesh. The learned counsel, however, would like to add that the
private Respondent No.4 submitted a representation to the Secretary of the Commission
on 18.09.2008 (Annexure A to the counter affidavit filed by the Respondent No.4)
praying for re-scrutiny of answer sheets of the Limited Departmental Competitive
Examination for the post of Section Officer held on 28th and 29th June, 2008, alleging that
the Commission selected only two candidates – one for APST and other for nonAPST/unreserved category for viva-voce test against 2 posts due to wrong application of
the guidelines issued by the AR Department vide O.M. dated 07.01.2008. On receipt of
the aforesaid representation, the Commission sought clarification from the State
Government and a decision was taken and communicated to the effect that minimum
33% of marks in all the subjects would not be required in the Limited Departmental
Competitive Examination and the aforesaid O.M. dated 07.01.2008 has application only
to the open competitive examination.
11.
From the pleadings and submissions of the parties, the following indisputable
position would emerge:
12.
(i)
The posts of Section Officer were sought to be filled up
through a Limited Departmental Competitive Examination.
(ii)
Initially, the Commission selected only 2 candidates namely
Sri Sudharshanan B and K Sidhardhan (writ petitioner) against
one post for unreserved category. The petitioner secured above
33% of marks in all the subjects with aggregate marks 170.16
while Sudarshanan B. could not secure 33% of marks in all the
subjects with 164.5 total marks and he remained out of fray.
(iii)
The private Respondent No.4 Sri. Roman Bora secured only 29
marks out of 100 in the subjects – FRs/SRs/GFRs/Pension
Rules, etc., and 30.50 marks in viva-voce test and thereby, he
could not secure the minim 33% of marks in all the subjects
although he secured in total 178.50 marks. The petitioner
secured less marks in total than private Respondent No.4 but
the petitioner admittedly secured above 33% of marks in all the
subjects.
(iv)
The Government O.M. dated 07.01.2008 provides minimum
33% of marks or more marks in each written examination
paper and 45% of marks in aggregate total marks for being
declared as eligible for viva-voce test with relaxation in the
cut-off marks of 45% in case of non-availability of APST
candidates securing the cut-off marks, and
(v)
There is no provision for securing minimum 33% of marks or
more marks under the Rules of 2004 although the Limited
Departmental Competitive Examination should comprise
written test in – (a) General Knowledge, (b) General English,
notings,
drafting,
essay
writing,
and
(c)
FRs/SRs/GFRs/Pension Rules, etc., 100 marks each in total,
and (d) 50 marks in viva-voce test.
The only question for determination of this court is whether the procedure laid
down in O.M. dated 07.01.2008 should apply to the recruitment of Section Officer
through Limited Departmental Competitive Examination and the respondent authorities
committed illegality in recommending private Respondent No.4 for appointment to the
post of Section Officer as he could not secure 33% of marks in all the subjects.
13.
The Government, by its O.M. dated 07.01.2008, as it appears, wanted to
streamline the procedure for conducting written test and viva-voce test for selection of
candidates and emphasis has been given on securing minimum 33% of marks in each
written examination papers and 45% marks out of aggregate total marks for being
declared eligible for viva-voce test. The purport and intention of the Government could
be better appreciated if the entire O.M. dated 07.01.2008 is extracted, which reads as
follows:
“(TO BE PUBLISHED IN ARUNACHAL PRADESH GAZETTE)
GOVERNMENT OF ARUNACHAL PRADESH
DEPARTMENT OF PERSONNEL, ADMINSITRATIVE REFORMS &
ADMINSITRATIVE TRAINING ADMINISTRATIVE REFORMS
No.OM-54/2006
Dated Itanagar the 7th January, 2008
OFFICE MEMORANDUM
Subject: Selection of candidates for appearing in viva-voce test on
the basis of Recruitment Examination-procedure thereof.
It has been brought to the notice of the Government that various
appointing authorities are selecting candidates for viva-voce test on the
basis of one or two subject of written examination ignoring other equally
important papers and without following a uniform pattern. As a result, the
ratio of candidates selected per vacancy varies from one examination to
other without maintaining common practice on prescription of ratio or cut
off marks even the candidates are selected in the ratio of 1:2:3. The issue
was under examination of the Administrative Reforms Department and
has found that no such procedure had been laid down earlier nor such
procedures have been prescribed in the relevant Recruitment Rules.
After careful examination of the issue and in modification of point
NO.2 and 3 of the OM dated 28.08.2006, the Government of Arunachal
Pradesh has decided to prescribe the following procedures for all direct
recruitment examinations for appointment to Group-A, B
& C
posts/services under the Government of Arunachal Pradesh:1)
For appearing in the viva-voce test, candidates shall be selected in
the ‘ratio’ of 1:3 (meaning 3 candidates shall be selected for each vacancy
or 3 (three) times of the number of vacancies) on the basis of written
examination papers. However, ratio of 1:3 shall not apply in case the
candidates appearing the written examination is less than 3 times of the
number of vacancies. In case of the candidates appearing in the written
examination is less than 3 times of the number of vacancies, all the
candidates securing 33% of marks in each written examination papers
shall be eligible for appearing viva-voce test.
2)
The candidates securing a minimum of 33% or more marks in each
written examination papers and has secured 45% of marks out of
aggregate total marks in the written examination papers shall be eligible
for viva-voce test. On the other, it will further mean that selection for
viva-voce test shall be based on the aggregate total marks secured in the
written examination papers and subject to ratio of 1:3. The candidates
securing less than 33% of marks in any of written examination papers
shall not be eligible for appearing in the viva voce test.
3)
The Selection Committee or Commission may lower the cut off
marks of 45% to certain extent, in case of non-availability of Arunachal
Pradesh Scheduled Tribe candidates securing the ‘cut off marks’.
Therefore, all the appointing authorities are requested to comply
with the above guidelines while conducting recruitment examinations for
appointment to Group ‘A’ ‘B’ & ‘C’ level of posts/services.
Sd/(Y.D.Thongchi)
Secretary (AR)
Government of Arunachal Pradesh.”
14.
The Government after exchange of various communications between the
Commission and AR Department came to a conclusion that the aforesaid O.M. dated
07.01.2008 is not applicable to the selection of candidates through Limited Departmental
Competitive Examination.
The said decision was communicated vide F.No.AR-
77/2008/693 dated 22.09.2008. The said letter is also reproduced for close examination
and coming to a finding:
“F.No.AR-77/2008/693
GOVERNMENT OF ARUNACHAL PRADESH
MINISTRY OF PERSONNEL, ADMINSITRATIVE REFORMS & TRAINING
DEPARTMENT OF ADMINSITRATIVE REFORMS
CIVIL SECRETARIAT, BLOCK NO.18
ITANAGAR
Dated the September 22, 2008
To
The Secretary,
Arunachal Pradesh Public Service Commission
Arunachal Pradesh,
Itanagar.
Subject: WITHDRAWAL OF CLARIFICATION DATED 08.09.2008.
Sir,
I am directed to refer to your letter No.PSC-D/4/2008 dated 19th
September, 2008, on the subject mentioned above.
2.
I am further directed to say that the procedure notified vide
No.OM-54/2006 dated 7.1.2008 is actually meant for selection of
candidates for viva voce test in respect of Direct Recruitment
Examination. The Limited Departmental Competitive Examination is for
selection of candidates for out of turn promotion which is not based on
their seniority but based on the performance in the Limited Departmental
Competitive Examination.
3.
Therefore, I am further directed to withdraw the letter
communication of even number dated 8th September, 2008, forthwith to
enable the Commission for re-evaluation and re-declaration of result etc.
With best regards Sir,
Yours faithfully,
Sd/(Mary Angu)
Under Secretary to the
Government of Arunachal Pradesh”
15.
The post in question is not a promotional post in strict sense of service law. The
post in question belongs to Group-B (Gazetted) as per Rules of 2004. The said post can
be filled up by promotion as well as Limited Departmental Competitive Examination and
from amongst the Assistants of Arunachal Pradesh Secretariat who have rendered 6 years
of regular service in the grade on the basis of merit. The aforesaid Rules of 2004
provides for recruitment of 50% of post by promotion and 50% by Limited Departmental
Competitive Examination. The Government admittedly initiated the recruitment process
for filling up 2 posts of Section Officer; one for APST candidate and other for non-APST
candidate.
The Government entrusted the Commission for holding the written
examination and viva voce test. The comparative marks obtained by the instant writ
petitioner and Respondent No.4, would be seen from their respective mark sheets, which
are reproduced, as under:
“PSC-D/10/2008
ARUNACHAL PRADESH PUBLIC SERVICE COMMISSION
ITANAGAR
Dated Itanagar the 3rd Oct., 2008
To
Shri/Smti K.Sidhardhan
Roll No.28.
Sub:
MARKS OBTAINED IN WRITTEN EXAMINATION/VIVAVOCE TEST FOR THE POST OF SECTION OFFICER UNDER
A.P. CIVIL SECRETARIAT, ITANAGAR.
Sir,
I am directed to refer to your application dated:
on the
subject mentioned above and give below marks obtained by you in the
written examination/viva-voce test for the post of Section Officer under
Arunachal Pradesh Civil Secretariat, Itanagar.
Sl.
No.
1.
2.
3.
4.
Subject
Full
marks
General Knowledge
100
General
English/Noting/Drafting/Essay 100
Writing
FRs/SRs/GFRs/Pension Rules/CCS (CCA) 100
Rules/General Provident Fund Rules
Viva-voce
50
Total
350
Marks
obtained
54
49
33.5
33.66
170.16
Yours faithfully,
Sd/- (B.Koyu)
Under Secretary (R)”
“PSC-D/10/2008
ARUNACHAL PRADESH PUBLIC SERVICE COMMISSION
ITANAGAR
Dated Itanagar the 10.10.2008
To
Shri/Smti Roman Bora
Roll No.23.
Sub:
MARKS OBTAINED IN WRITTEN EXAMINATION/VIVAVOCE TEST FOR THE POST OF SECTION OFFICER UNDER
A.P. CIVIL SECRETARIAT, ITANAGAR.
Sir,
I am directed to refer to your application dated: 10.10.2008 on the
subject mentioned above and give below marks obtained by you in the
written examination/viva-voce test for the post of Section Officer under
Arunachal Pradesh Civil Secretariat, Itanagar.
Sl.
No.
1.
2.
3.
4.
Subject
Full
marks
General Knowledge
100
General
English/Noting/Drafting/Essay 100
Writing
FRs/SRs/GFRs/Pension Rules/CCS (CCA) 100
Rules/General Provident Fund Rules
Viva-voce
50
Total
350
Marks
obtained
59
60
29
30.50
178.50
Yours faithfully,
Sd/- (B.Koyu)
Under Secretary (R)”
16.
The aforesaid position is not disputed. The Rules of 2004 provide for written
examination and viva voce test.
It has been termed as ‘Limited Departmental
Competitive Examination’. It is not akin to procedure adopted for promotion to a higher
post from amongst the eligible officer of a particular feeder post where either ‘merit-cumseniority’ or ‘seniority-cum-merit’ basis is adopted by the Commission or the
Departmental Promotion Committee (DPC) without any written examination. In the
aforesaid context, the written examination conducted by the Commission for recruitment
to the post in question would fall under direct recruitment examination limited amongst
the eligible candidates in the Civil Secretariat of Arunachal Pradesh.
17.
I have gone through the Civil Secretariat File No.AR-77/2008 as maintained by
the AR Department and produced by Mr. R.H.Nabam, learned Senior Govt. Advocate. It
is seen that when the matter was endorsed to the Chief Secretary to the Government of
Arunachal Pradesh, he opined firmly that “33% is the minimum qualifying marks. There
is no question of relaxation. Anybody securing less than 33% of marks is unfit to be in
government service. Please inform APPSC”. Subsequently, when the file was again
endorsed by the Under Secretary to the Commissioner, AR Department, it was further
clarified that:“If the advertisement for Limited Departmental Examination was
published before notification of the procedure mentioned above, it shall
not come under the purview of notification dated 7.1.2008.
However, if the advertisement was issued after 07.01.2008, then it
shall come under it.
No relaxation below 33% can be made by the recruiting agency.
We may inform the Commission as at paras 27, 32, 33 and 34
above.”
From the above notes in the Secretariat File, it is found that the Government
intended to follow the O.M. dated 07.01.2008 as the advertisement for the post in
question, was issued subsequent to publication of aforesaid O.M. dated 07.01.2008.
18.
After the matter was settled at the highest level, some unsuccessful candidates
including private Respondent No.4 filed the representation for re-interpretation of O.M.
dated 07.01.2008 and the aforesaid settled Government policy on cut-off marks i.e.
minimum 33% of marks has been changed at the Deputy Secretary level of the AR
Department, which has been narrated earlier.
In this regard, the Secretary, AR
Department, made a note to the effect that no amendment can be made with retrospective
date already given by the Commission. The Chief Secretary, however, did not agree with
the Secretary (AR) and ultimately, passed an order to the effect that minimum 33% of
marks is applicable to the cases of direct recruitment only and since there is no mention
about 33% minimum qualifying marks, the O.M. dated 07.01.2008 would not be
applicable. The respondent Chief Secretary, while taking the aforesaid decision misdirected himself by accepting that the recruitment to the post of Section Officer is not a
direct recruitment without applying his mind to the fact that the Commission was
entrusted to hold the written examination as well as viva-voce test in the light of O.M.
dated 07.01.2008 requiring the candidates to secure 33% of marks in all the subjects and
45% of marks in aggregate in the written examination. The respondent Chief Secretary
also remained oblivious to the fact that the advertisement for the post in question was
issued on 31.03.2008 i.e. after the issue of O.M. dated 07.01.2008. The view expressed
by the Secretary, AR Department, had bearing with the aforesaid facts, Government
policy and purport of conducting written/viva-voce test by the Commission. In my
considered view, the use of phrase “Limited Departmental Examination” would not bring
the recruitment of Section Officer out of purview of direct recruitment. The sphere of
competition may be confined to the eligible candidates belonging to Civil Secretariat but
they have to compete amongst themselves and the selection would be made on the basis
of merit. The seniority in service amongst the candidates has no relevancy like the direct
recruitment from open market. The appointment of Section Officer although required to
be made through Limited Departmental Competitive Examination, is, therefore, should
be made through direct recruitment amongst the eligible candidates of Civil Secretariat
and as such, the same should be invariably conducted as per the procedure prescribed
under the O.M. dated 07.01.2008.
19.
The post of Section Officer attaches higher responsibility in the Civil Secretariat
and it is, therefore, expected that comparatively more meritorious person from amongst
the Secretariat staff should be brought into service.
Here in this case, the private
Respondent No.4 admittedly failed to show his better merit inasmuch as he could not
even secure minimum 33% of marks out of 100 marks in the subjects –
FRs/SRs/GFRs/Pension Rules, etc., which are important subjects in the day-to-day
administration but he has been preferred over the petitioner who had at least secured the
minimum required cut-off marks in all the subjects. It must be noted that no authority
has been given to the recruitment agency/authority under Rules of 2004 to fix any cut-off
marks in he written examination/viva-voce test. The Rules of 2004 provides only power
to hold written examination/viva-voce test in certain subjects with total marks for each
subject without prescribing any minimum cut-off marks. It is, therefore, implied that the
minimum cut-off marks as prescribed in O.M. dated 07.01.2008 should rule the field.
The aforesaid O.M. dated 07.01.2008 is a supplement to the Rules of 2004. The general
principle of law is that if the competent authority has no power to do certain act, it has no
power to act otherwise. There is a total absence of application of mind by the respondent
Chief Secretary in changing his earlier stand on O.M. dated 07.10.2008 inasmuch as he
has given no valid reason for it and such change of stand has given scope for
accommodating an undeserving candidate like the private Respondent No.4. He also
failed to appreciate that appointment of such undeserving candidate would affect the
efficiency of administration in the Civil Secretariat.
It would naturally drive any
reasonable person to draw an inference of doing an act by the authority concerned in an
unauthorized manner at the behest of vested interests by using his power and authority for
unauthorized purpose.
The settled law is that when a power is exercised for an
unauthorized purpose, the same would amount to malice in law and such unauthorized
action for unauthorized purpose is liable to be interfered with by the court in judicial
review. This has been held so in several decisions of the Apex Court, the latest being
P.Mohanan Pillai –vs- State of Kerala, reported in (2007) 9 SCC 497.
20.
The above decisions and reasoning would lead this court to hold that the O.M.
dated 07.01.2008 is applicable to the recruitment through Limited Departmental
Competitive Examination and the petitioner has been able to make out a case for
interference with the impugned letter of withdrawal of clarification dated 22.09.2008 vide
F.No.AR-77/2008/693, impugned re-notification of revised result dated 23.09.2008 vide
No.PSC-D/4/2008 and impugned recommendation dated 23.09.2008 vide Order No.PSCD/4/2008 and the same are, accordingly, set aside and quashed.
The respondent
authorities are directed to declare results in question as per result notification dated
15.09.2008 vide No.PSC-D/4/2008 strictly in terms of Government O.M. dated
07.01.2008, as expeditiously as possible, preferably, within 1 (one) month from the date
of receipt of a certified copy of this order.
21.
With the above observations and directions, this writ petition stands allowed and
disposed of.
22.
There shall be no order as to costs.
***
IN THE GAUHATI HIGH COURT
ITANAGAR BENCH
W.P. (C) NO.235 (AP) OF 2009
D.D. 23.07.2010
Hon’ble Mr. Justice A.C.Upadhyay
Shri Hage Tabyo
Vs.
The State of A.P. & Ors.
…
Petitioner
…
Respondents
Selection – Reservation under P.H category:
Whether a candidate who was unsuccessful in a recruitment wherein no reservation was
made for PH persons can challenge the selection process and seek direction to select him
against PH vacancy on the ground that one post ought to have been reserved under PH
category under Disabilities Act? - No.
Such PH post should be treated as backlog vacancy in view of Umadevi’s case – (2006) 4
SCC 1.
In the recruitment to Group-A posts in the Government Departments as per
advertisement dated 24.9.2007 reservation of 3% of the posts in terms of Disabilities Act
1995, was not indicated – However, in Clause 7 of the advertisement it is stated that PH
candidate should produce certificate from District Magistrate – Petitioner was a candidate
for the said post enclosing Disability certificate – Petitioner was not selected in the
written test – Subsequent to this advertisement Office Memorandum dated 17.12.2007
instructing the different Departments to reserve posts for PH candidates was circulated –
The Commission took a decision to reserve next vacancies for PH candidates as backlog
– It was admitted that respondent No.24, a G.M. candidate was appointed against a post
meant for disabled candidate – Though the Court held that the petitioner being a disabled
candidate ought to have been selected under PH category since no advertisement was
made to that effect in view of the decision in State of Karnataka vs. Umade Devi (2006) 4
SCC 1, Court disposed of the case with a direction to the State Government to take steps
for selection and appointment of candidates to the post identified for PH persons by
issuing necessary advertisement within three months.
Cases Referred:
(2006) 4 SCC 1 - State of Karnataka vs Uma Devi
(2006) 6 SCC 395 - K.H.Siraj –vs- High Court of Kerala
2007 (2) GLT 754 - Nikhilesh Das –vs- State of Tripura
(2008) 4 SCC 619 - Sadananda Halo & Ors. –v- Momtaz Ali Sheikh & Ors.
(2009) 5 SCC 515 - K.A. Nagmani –vs- Indian Airlines
ORDER
The writ petitioner, who is a physically challenged person, has questioned the
selection and appointment of candidates by the State respondents in terms of the
advertisement No.PSC-R/21/07, dated 24.09.2007 ignoring the rights of the petitioner
guaranteed under the ‘Persons with Disabilities (Equal Opportunities, Protection of Right
and Full Participation) Act, 1995 (hereinafter referred to ‘the Disability Act, 1995’). The
petitioner has prayed for including him among the successful candidates and also to allow
him to participate in the viva-voce test.
2.
The facts, leading to the filing of this writ petition, may be stated in brief, as
follows:
The Arunachal Pradesh Public Service Commission, Itanagar (hereinafter referred
to ‘APPSC’), on 24.09.2007 issued advertisement No.PSC-R/21/07, for recruitment to
the Group-A posts in the Government Departments. It has been also mentioned in the
said advertisement that 80% of the posts shall be reserved for Arunachal Pradesh
Scheduled Tribe candidates. However, reservation of 3% of the posts in terms of the
Disabilities Act, 1995, was not indicated in the advertisement for the physically
disabled/handicapped person, as per exiting law and as per the reservation policy adopted
by the Government of Arunachal Pradesh.
However, in clause 7 of the impugned
advertisement, for a physically handicapped candidate requirement of a certificate from
the District Magistrate, was indicated.
3.
The petitioner in response to the advertisement submitted application in the
prescribed form by enclosing all the required documents together with his disability
certificate, plainly believing that there is reservation for disabled candidates, in terms of
the Disability Act, 1995. Accordingly, the petitioner was called for the written test
examination by the respondent authority. Accordingly, the result of the written test
examination was published on 29.05.2009, by showing only the Roll Numbers of the
successful candidates. However, the petitioner was not selected. Since the petitioner was
not selected in the written test examination, he made an enquiry and came to learn that
3% of posts were not kept reserved for the disabled/handicapped persons, as per the
provision of Section 33 of the Disability Act, 1995, and other existing Office
Memorandum issued by the Government of Arunachal Pradesh from time to time.
4.
The petitioner states that the Government of India, Ministry of Personal, Public
Grievances and Pensions, Department of Personal and Training, New Delhi, issued
Office Memorandum No.36035/16/98-Estt.(Res) dated 13.07.1999, through the Director
(Res), directing to all the Ministry/Departments in Union and States in India to keep
reservation of posts/services for disabled/handicapped person, as per the provision of the
Disability Act, 1995. Further another Notification was issued by the Government of
India vide office D.O.No.16/25/99-NI-I(PWD), dated 14.12.1999 addressed to the
Secretary, Social Welfare, Government of Arunachal Pradesh, Itanagar, suggesting
implementation of the provision of Section 33 of the Disability Act, 1995.
5.
It is stated on behalf of the petitioner that on some occasions, when the APPSC
filed to clearly spell out reservation from handicapped/disabled quota, the State
Commissioner, for Physically Handicapped Persons, Government of Arunachal Padesh,
Itanagar vide Office Memo No.SW-0289/98/999 dated 24.01.2003, had requested the
Secretary, APPSC to comply with the Office Memo No.OM/15/97(Pt-I) dated 20.02.2002
issued by the Commissioner, Department of Personal Administrative Reform and
Training, Government of Arunachal Pradesh, by taking necessary steps for reservation of
posts/services, for physically handicapped persons under the Disability Act, 1995.
6.
It has been further stated on behalf of the petitioner that the Government of
Arunachal Padesh has already adopted the Disability Act, 1995, by its Office Memo
No.OM-15/97(Pt-I)
dated
20.02.2002
and
22.01.2002,
whereby
all
the
Ministries/Departments have been directed to keep 3% of the Group-A, B, C and D posts
reserved for the Arunachal Pradesh Schedule Tribe handicapped persons under the
Government of Arunachal Pradesh. It is alleged by the petitioner that in spite of having
mandatory provision for reservation of posts for handicapped persons in Arunachal
Pradesh, the petitioner has been deprived of his legitimate right to be selected in the
recruitment process. It is submitted on behalf of the petitioner that in view of the specific
provision in the Disability Act, 1995 and the Office Memorandum issued by the
Government of Arunachal Pradesh from time to time, 3% of the vacancies advertised for
the aforesaid posts should be considered to have been reserved for physically
handicapped persons. It is further submitted on behalf of the petitioner that since the
respondent APPSC, knew the reservation policy adopted by the Government of
Arunachal Pradesh, it ought to have reserved 3% of the combined posts under the
aforesaid advertisement for disabled and handicapped persons.
7.
It has been further pointed on behalf of the petitioner that in terms of the
Disability Act, 1995, the reserved vacancy for the physically handicapped has to be filled
up from among the eligible candidates with disabilities and in doing so, candidates lower
in merit than the last candidate in the merit list, but otherwise fit for appointment, should
be considered for appointment, and if necessary, such adjustment should be carried out
by relaxing prescribed standard.
8.
Mr. D.Pangging, learned counsel for the petitioner, specifically pointed out that
the impugned advertisement issued by the APPSC clearly indicated in the column
“method of selection” at Clause-7 that a physically handicapped candidate has to submit
a certificate from the District Magistrate. Such indication in the impugned advertisement
gives implicit approval of the fact that among the posts advertised by the APPSC, some
posts have been reserved for physically handicapped persons.
9.
Learned counsel for the petitioner submitted that the State Government by issuing
Memorandum dated 17.12.2007, directed all the departments to reserve vacancies in
Group-A, B, C and D posts/services, for physically handicapped persons in terms of the
100 point Roster, showing the position of the physically handicapped person clearly as
per the Roster, so circulated by the Government of Arunachal Pradesh, where the first
position among the reserved category has to be kept reserved for the physically
handicapped person. Only in case of non-availability of handicapped persons among the
candidates, the vacancy shall be de-reserved to fill up candidates from other category and
the reservation of handicapped shall be treated as backlog and it is carried forward to the
next year.
10.
Drawing the attention of this Court to paragraph 2 of the Office Memorandum
dated 07.01.2008 issued by the Government of Arunachal Pradesh, Department of
Personnel, Administrative Reforms & Training, Administrative Reforms, Mr.K.Ete,
learned counsel for the respondent No.24 pointed out that a candidate must secure
minimum of 33% or more marks, in each paper, in the written examination and the
aggregate total marks in the written examination has to be 45% or above for being
eligible for viva voce test. Mr. K.Ete, the learned counsel for the respondent No.24
submitted that in Civil Engineering Section-B the petitioner obtained only 24 marks out
of 100 and thus he did not qualify in the written test examination in terms of the above
Office Memorandum, therefore, the petitioner did not qualify for viva voce.
11.
The petitioner by filing an additional affidavit, placed on record the letter issued
by the Under Secretary, APPSC, Itanagar indicating the marks obtained by the petitioner
in the written test examination held by the APPSC in terms of the advertisement
aforesaid, which may be reproduced below:“PSC-R/21/97
ARUNACHAL PRADESH PUBLIC SERVICE COMMISSION, ITANAGAR
Dated, Itanagar, the 15.06.2009
To
Hage Tabyo,
Roll No.00118
Subject: Marks obtained in the Recruitment
Examination of A.E. (Civil) 2008-09.
Sir,
I am directed to refer to your application dated 15.06.09 on the subject mentioned
above and to give below marks obtained by you in the written combined Examination of
A.E. (Civil).
Sl. No.
1.
2.
3.
4.
Subject
Full marks
General English
100
General Knowledge
100
Civil Engineering
100
Section-A
Civil Engineering
100
Section-B
Total
400
Marks obtained
64
51
68
24
207
Yours faithfully,
Sd/(B.Koya)
Under Secretary.”
12.
In reply to the above submission, the learned counsel for the petitioner drew
attention to this court to the advertisement dated 24.09.2007, where, in the column
‘Method of Selection’, among the subjects for written examination, Civil Engineering
carried 200 marks in total and there was no indication in the advertisement regarding
Civil Engineering Section-A and Section-B, which was subsequently shown in the mark
sheet issued by the APPSC in the marks sheet issued by the APPSC also, the marks
obtained by the petitioner in Civil Engineering Section-A and Civil Engineering SectionB has been added up and the total mark has been shown as 92 marks, out of 200 marks
allotted for the subject, which is 46% of total 200 marks allotted for the subject.
Therefore, it clearly transpires that total 200 mark was allotted for Civil
Engineering/Agricultural Engineering and it was treated as one subject with 200 marks,
and, as such, 92% marks secured by the petitioner out of 200 marks is far above the 33%
marks required to qualify in the examination in terms of the guidelines. The petitioner
secured more than 45% in aggregate, therefore, there is no reason of the petitioner being
disqualified.
Therefore, the issue here is not that the petitioner had qualified for the viva-voce
test or not, but it is the eligibility of the petitioner as a handicapped candidate in the posts,
in terms of the impugned advertisement.
13.
It is not in dispute that the Government of Arunachal Pradesh by its Notification
dated 21.05.2007, identified 3% reservation, for physically handicapped persons by
specifically indicating the designation of the post, Department/Office, Group/Service,
physical requirement, for performing the job and categories of disabled persons suitable
for such jobs.
14.
The Government of Arunachal Pradesh by issuing letter of instruction to the
Senior Government Advocate, Gauhati High Court, Itanagar Bench has categorically
indicated that the Office Memorandum, which was circulated on 17.12.2007 instructing
the different departments in respect of the reservation of the posts made for the physically
handicapped persons having been issued after the impugned advertisement, requisition
for reservation could not be made to the APPSC, before the impugned advertisement was
issued. Learned senior Government Advocate has submitted that since the roster position
No.1 is required to be filled up by appointing a physically handicapped person, decision
has been taken by the Government of Arunachal Pradesh to henceforth reserve the next
vacancies for physically handicapped candidates as backlog.
15.
Mr. K.Ete, learned counsel for the respondent No.24 submitted that the petitioner,
who failed to secure pass mark in the written test examination cannot turn around to
challenge the selection process after having been disqualified in the written test
examination. In support of his submission, the learned counsel for the petitioner has
relied on the following decisions: (1) K.H.Siraj –vs- High Court of Kerala, (2006) 6 SCC
395 (2) Sadananda Halo & Others –v- Momtaz Ali Sheikh & Others (2008) 4 SCC 619
(3) K.A. Nagmani –vs- Indian Airlines, (2009) 5 SCC 515 (para 54) and (4) Nikhilesh
Das –vs- State of Tripura, 2007 (2) GLT 754 (para 15).
Hon’ble Supreme Court in Sadananda Halo & Others –vs- Momtaz Ali Sheikh &
Others (2008) 4 SCC 619 held as follows:
It is also a settled position that the unsuccessful candidates cannot turn
back and assail the selection process. There are of course the exceptions
carved out by this Court to this general rule. This position was reiterated
by this Court in its latest judgment in Union of India & Ors. v. S.Vinod
Kumar & Ors [(2007) 8 SCC 100] where one of us (Sinha, J.) was a party.
This was a case where different cut off marks were fixed for the
unreserved candidates and the Scheduled Caste and Scheduled Tribes
candidates. This Court in para 10 of its judgment endorsed the action and
recorded a finding that there was a power in the employer to fix the cut off
marks which power was neither denied nor disputed and further that the
cut off marks were fixed on a rationale basis and, therefore, no exception
could be taken. The Court also referred to the judgment in Om Prakash
Shukha v. Akhilesh Kumar Shukla & Ors. [(1986) Supp. SCC 285] where
it has been held specifically that when a candidate appears in the
examination without protest and subsequently found to be not successful
in the examination, the question of entertaining the petition challenging
such examination would not arise. The Court further made observations in
para 34 of the judgment to the effect:
“There is thus no doubt that while question of any estoppel by conduct
would not arise in the contextual facts but the law seem to be well settled
that in the event a candidate appears at the interview and participates
therein, only because the result of the interview is not ‘palatable’ to him,
he cannot turn round and subsequently contend that the process of
interview was unfair or there was some lacuna in the process.
In para 20 this court further observed that there are certain
exceptions to the aforementioned rule. However, the court did not go into
those exceptions since the same were not material.”
In K.A.Nagmani –vs- Indian Airlines (supra) also Hon’ble Supreme Court held as
follows:
That the appellant admittedly had participated in the similar selection
process for erstwhile grade 15 and 16, Manager (Maintenance/Systems)
and Senior Manager (Maintenance/Systems) respectively.
The
Corporation had given adequate opportunity to the appellant to compete
with all other eligible candidates at the selection for consideration of the
case of all eligible candidates to the post in question. The Corporation did
not violate the right to equality guaranteed under Articles 14 and 16 of the
Constitution. The appellant having participated in the selection process
along with the contesting respondents without any demur or protest cannot
be allowed to turn round and question the very same process having failed
to qualify for the promotion. In Madan Lal & Ors. Vs. State of J & K &
Ors. [ (1995) 3 SCC 486], this court observed: “It is now well settled that
if a candidate takes a calculated chance and appears at the interview, then,
only because the result of the interview is not palatable to him, he cannot
turn round and subsequently contend that the process of interview was
unfair: Therefore, the result of the interview test on merits cannot be
successfully challenged by a candidate who takes a chance to get selected
at the said interview and who ultimately finds himself to be unsuccessful,”
Reference may also be made to the decision of this Court Chandra Prakash
Tiwari vs. Shakuntala Shukla [(2002) 6 SCC 127].
The petitioner, who appeared in the examination, sincerely thought that Clause-7
in the advertisement, which indicated requirement of certificate from the District
Magistrate, for physically disabled person, meant indication of reservations for physically
disabled persons. However, in the facts and circumstances discussed above, since the
petitioner has challenged non-selection of a disabled candidate in terms of the impugned
advertisement, the ratio of the cases cited above may not have any application in the
instant case. Therefore, the petitioner cannot be said to have turned around to challenge
the process after having been unsuccessful in the written test as well as interview
conducted in terms of the advertisement.
17.
However, very interestingly, the respondent No.24, who was selected for
appointment as a general category candidate, was appointed in a post meant for disabled
candidate. If a post identified for disabled candidates was not advertised, question arises,
as to how the respondent No.24, was appointed and adjusted in the said reserved post by
the State respondent. If post meant for disabled candidate was advertised, question arises
as to why the petitioner was not considered for selection. There is no answer to the above
situation.
18.
Mr. N.Tagia, learned counsel for the Arunachal Pradesh Public Service
Commission (APPSC) submitted that the APPSC is not in a position to initiate the
process of recruitment of a particular category of candidate without receiving appropriate
requisition/instruction for such selection and recruitment from the concerned department
of the Government. Learned counsel for APPSC further pointed out that since there was
no requisition for 3% reservation and/or selection for physically handicapped candidates
in terms of the Disability Act, 1995, from the concerned Department, neither it could
indicated in the impugned advertisement issued by the APPSC, nor the petitioner could
be considered as a physically disabled candidates under 35 reservation in terms of the
Disability Act, 1995, by the APPSC for the purpose of selection.
19.
It would be pertinent to mention here that by an interim order dated 31.07.2009
passed in MC No.81 (AP) of 2009, one post of Assistant Engineer (Civil) identified for
physically disabled person, was ordered to be kept vacant by an order of this Court till
disposal of the writ petition. However, the interim order aforesaid was vacated in M.C.
(WP) 128/09 vide order dated 24.11.09 of this Court on condition that if any appointment
is made in favour of respondent No.24 in the post identified for physically handicapped
person, the same shall be subject to result of the writ petition. Accordingly, respondent
No.24 was appointed in the post meant for physically handicapped person by the State
respondent.
20.
Mr. K.Ete, learned counsel appearing on behalf of the respondent No.24
submitted that the petitioner, who is a physically handicapped with one leg, is not entitled
to be accommodated/appointed in the Public Works Department since the post of
Assistant Engineer in the Public Works Department is vacant for physically handicapped
person with one arm, partially deaf and partially blind.
21.
The learned counsel for the respondent No.24 further submitted that the present
petitioner is a physically handicapped with one leg and, as such he is not entitled to be
accommodated in the Public Works Department and at the most he could be appointed in
the Urban Department, where a post for orthopaedically handicapped person may have
been created.
22.
On perusal of the Government Notification No.SW-13/2007 (PIL) dated 21st May,
2007 it transpires that the posts meant for physically disabled persons have been
identified in different departments of the Government, in terms of the requirement of
Disability Act, 1995. The State Government counsel has indicated at the time of hearing
of this case by drawing attention of this court to a written instruction received by the
Counsel, that State Government would take all necessary steps for accommodating
physically disabled persons in all subsequent selections.
23.
It has not been disputed that the State-respondents did not direct the APPSC to
select candidates in any of the posts identified for the purpose of appointment of
physically handicapped person nor it was indicated in the impugned advertisement issued
by the APPSC. It also appears from the affidavit submitted by the State respondents and
the Arunachal Pradesh Public Service Commission (APPSC) that no advertisement was
made, for selection of physically disabled candidates in any of the posts indicated in the
advertisement. The Arunachal Pradesh Public Service Commission further clarified in its
affidavit that when reservation is required to be made for a particular reserved category,
the State Government usually makes requisition.
Therefore, the Public Service
Commission did not take the exercise of selecting the special category candidates, such
as, physically disabled candidate, for having not received any requisition to that effect.
The impugned advertisement issued by the Arunachal Pradesh Public Service
Commission, for recruitment to various posts indicated in the advertisement, neither did
specify reservation of 3% quota for physically disabled persons nor did indicate the posts
in which they would be accommodated. Therefore, it can safely be held that the State
respondents did not advertise any such post, which was identified to be filled up by
physically disabled persons in terms of the provision of the Disability Act, 1995, in the
impugned advertisement.
On the basis of the impugned advertisement made by the Arunachal Pradesh
Public Service Commission, on completion of the selection process a select list of
candidate was provided to the State Government. The State Government being the
appointing authority has to follow the list of candidates selected by the Public Service
Commission for the purpose of issuing appointments.
24.
There is no justification in picking up a post meant for disabled candidate to
accommodate a general category candidate. More so, when the Government made no
requisition to the APPSC for selection of disabled candidate and no advertisement was
issued by the APPSC to that effect.
Therefore, in view of the above discussions,
arguments advanced by Mr. K.Ete, learned counsel to justify appointment of respondent
No.24 in the post identified for disabled candidate cannot be accepted.
25.
Now the question, which arises for consideration, is whether the petitioner can be
directed to be appointed in a post meant for physically handicapped candidate when the
post was not advertised.
26.
Hon’ble Supreme Court in Secretary, State of Karnataka vs Uma Devi, reported in
(2006) 4 SCC 1, observed as follows:
“A class of employment which can only be called ‘litigious employment’
has risen like a phoenix seriously impairing the constitutional scheme.
Such orders are passed apparently in exercise of the wide powers under
Article 226 of the Constitution of India. Whether the wide powers under
Article 226 of the Constitution is intended to be used for a purpose certain
to defeat the concept of social justice and equal opportunity for all, subject
to affirmative action in the matter of public employment as recognized by
our Constitution, has to be seriously pondered over. It is time, that Courts
desist from issuing orders preventing regular selection or recruitment at
the instance of such persons and from issuing directions for continuance of
those who have not secured regular appointments as per procedure
established. The passing of orders for continuance tends to defeat the very
Constitutional scheme of public employment. It has to be emphasized that
this is not the role envisaged for High Courts in the scheme of things and
their wide powers under Article 226 of the Constitution of India are not
intended to be used for the purpose of perpetuating illegalities,
irregularities or improprieties or for scuttling the whole scheme of public
employment. Its role as the sentinel and as the guardian of equal rights
protection should not be forgotten.
27.
This Court has also on occasions issued directions, which could
not be said to be consistent with the Constitutional scheme of public
employment. Such directions are issued presumably on the basis of
equitable considerations or individualization of justice. The question
arises, equity to whom? Equity for the handful of people who have
approached the Court with a claim, or equity for the teeming millions of
this country seeking employment and seeking a fair opportunity for
competing for employment? When one side of the coin is considered, the
other side of the coin, has also to be considered and the way open to any
court of law or justice, is to adhere to the law laid down by the
Constitution and not to make directions, which at times, even if do not run
counter to the Constitutional scheme, certainly tend to water down the
Constitutional requirements.”
27.
In view of the above observation of the Hon’ble Supreme Court in Secretary,
State of Karnataka vs Uma Devi, reported (supra), without a proper advertisement for
selection of physically disabled candidates, any direction by this Court to select and
appoint a disabled candidate would not be legal. Such appointments would defeat the
very Constitutional scheme of public employment. Such direction for appointment would
perpetuate illegalities, irregularities or improprieties by scuttling the complete scheme of
public employment.
Further, in the event of any direction by this Court for
accommodation of the petitioner in any of the posts meant for physically disabled person,
would deprive all such other physically disabled candidate, who would have applied for
the posts and also would have participated in the selection process had there been
indication in the advertisement regarding reservation of 3% quota for the physically
disabled persons. Therefore, proposition for appointment of the petitioner in the terms of
the impugned advertisement is not acceptable.
28.
The petitioner applied for the post in terms of the advertisement issued by the
Arunachal Pradesh Public Service Commission, but such application submitted by the
petitioner cannot be deemed to have been made for selection to any of the posts meant for
physically disabled person, as there was no indication in the advertisement, for
reservation of 3% quota for physically handicapped persons.
29.
There is no explanation from the side of the State Government as to what was the
intrinsic reason for keeping the clause 7 in the advertisement aforesaid. But apparently,
such a clause in the advertisement would only serve the purpose of assessing the extent of
disability of the candidate for his accommodation in the post meant for general category
candidates and not for selection of physically disabled candidate.
30.
Now, in view of the discussions, the petitioner, who is a physically disabled
candidate, was not selected in the normal course, as such, he can be accommodated in the
select list in terms of the reservation policy adopted by the State Government as per
provision of Disability Act, 1995, since no advertisement was made to that effect.
Further such a litigious appointment as has been observed by the Hon’ble Supreme Court
in Uma Devi (supra), would not be in consonance with the Constitutional mandate as
provided under Article 14 and 16 of the Constitution of India.
31.
Fact remains, without having made appropriate advertisement; for the purpose of
selection of physically disabled persons the Government also cannot accommodate any
general category candidate in a reserve post identified for physically disabled persons.
Therefore, any accommodation made by the State Government in a post reserved for
Disabled candidate by a physically fit person, shall have to be treated as a backlog for the
next recruitment. Since appointment has been given to respondent No.24, who is a
selected candidate, accordingly, adjustment shall be made in respect of the post by
treating it to be a backlog for disabled candidates. The State respondent is directed to
take appropriate measure to undo the irregular exercise without wastage of time by
treating one post as a backlog post, meant for physically disabled candidate.
32.
In view of what has been discussed above, the writ petition filed by the petitioner
is disposed of in terms of the direction indicated above. However, the relief sought for on
behalf of the petitioner for consideration of his selection and appointment in terms of the,
in the facts and circumstances of the case is rejected.
Consequently, the State
respondents are directed to take immediate necessary steps for selection and appointment
of the candidates to the posts identified for physically disabled persons, issuing necessary
advertisement, in accordance with law by. This exercise shall be completed by the State
respondents within a span of three months from the date of receipt of certified copy of
this judgment and order.
33.
With the above observations and directions, this writ petition stands disposed of.
However, I pass no order as to costs.
***
PROCEEDINGS OF THE ARUNACHAL PRADESH INFORMATION
COMMISSION, ITANAGAR
Case No.APIC/13/2008
Hon’ble Mr. Habung Payeng, State Information Commissioner
Sri Subu Koyang
…
Vs.
PIO, Arunachal Pradesh PSC …
Appellant
Public Authority
R.T.I. Act:
In this case under R.T.I. Act the appellant sought information viz., marks sheets
of all candidates who have become eligible for interview in Arunachal Pradesh Civil
Service Examination and permission to inspect original documents – As the appellant
was unable to pay the requisite fee in respect of marks sheets he was permitted to inspect
the marks sheet – Regarding 2nd information for inspection of original documents, the
documents being too old were not traced – Aggrieved by the same the applicant filed this
application – However, as the appellant remained absent on four consecutive dates fixed
for hearing the information Commission disposed of the application.
ORDER
Sri. Subu Koyang, Paraline Hapoli, Lower Subansiri District, Ziro filed a RTI
application on 10th December, 2007 seeking the following information in Form ‘A’.
1) Marksheet (statement of marks) obtained in the APPCS Examination, 1993 of
all the candidates who appeared viva voce test, the candidates those who qualified for
appointment to the post of C.O. and those who could not qualified for such appointment
excluding the information that have already obtained. (2) The information regarding the
correspondence made between the Government and the APPCS for the appointment of
Shri Ibom Taw (To be furnished photocopy of the original documents). (3) to give
permission to inspect the original documents of the aforesaid documents.
2.
In response to his application the State Public Information Officer/Under
Secretary, APPSC Sri. N.Tondrang written a letter vide F.No.PSC-07/2007 on 20th
December, 2007 stating that the documents is treated as Top Secret documents and
therefore, your request cannot be considered.
3.
However, Marksheet (Statement of marks of 92 candidates excluding marks
statement of 10 candidates which was supplied earlier) who appeared viva voce test in the
APCS (Jr) held during 1992-93 on remission of Rs.2300/- (Rupees two thousand and
three hundred) only i.e. Rs.25/- per candidate x 92 = Rs.2300 only in the shape of uncrossed IPO in favour of Secretary, APPSC, Itanagar, which is required to be paid as per
APPSC (Limitation of Function Regulation and Office Rules (Manual) 1983. Aggrieved
by the response Sri S.Koyang approached to the AP State Information Commission.
After examining the appealed by Sri. S.Koyang the Commission advised the appellant
vide letter No.APIC/Admin-4/2007 dated 4th January, 2008 that he should file the appeal
to the first Appellate Authority i.e., the authority who is higher than the PIO in his office
or department in case first Appellate Authority is not responded or if not satisfied with
the response of First Appellate authority he should appeal to the Commission.
In
response to the advice from the APIC, the appellant filed appeal to the Secretary, APPSC
who is the first Appellate Authority.
4.
The first Appellate Authority heard on 2nd April 2008 and passed an order stating
that “In pursuance of section 19 of RTI Act 2005, I heard Mr. Subu Koyang appellant and
Sri Nima Tondrang, Deputy Secretary-cum-Public Information Officer of Arunachal
Pradesh Public Service Commission on 2.04.2008. Also I have perused the related
documents.
The appellant applied for the following documents relating to the APPSC
Combined Competitive Examination 1993:
1. Mark statement of all candidates
2. Correspondence documents in connection with appointment of one Shri Ibom Tao
as Circle Officer.
3. Inspection of above documents in original
4. Mark sheet of all candidates including that of Shri Ibom Tao.
Vide his letter No. Nil dated 07.03.2008, the P.I.O. of this office has accepted to
furnish the documents as sought by the applicant subject to deposition of required fee of
Rs.2300/- (Rupees two thousand and three hundred) only @ Rs.25/- (Rupees twenty
five) only per candidate as per the rate fixed by the Govt. vide order No.OM-15/88 dated
08.07.1988 (Par-IV, Rule-11) except few documents which are classified as “top Secret”
as per Commission’s order No.PSC-R/06/2006 dated 30th April, 2007. Aggrieved by the
decision of the P.I.O. the applicant preferred an appeal before me attacking the rate of
fees fixed by the Govt. as excess and arbitrary.
On careful examination it has been found that such rate was fixed much before the
inception of the RTI Act, 2005 and was never questioned by any candidate in the past as
such, the argument put forwarded by the appellant appears to be not genuine. Also a sum
or Rs.25/- per candidate is at all not an excessive rate except as and when an applicant
applied for all candidates, the sum total may appear to be big sum. Therefore, the rule
cannot be changed abruptly for the convenience of only an applicant and accordingly the
plea stands rejected.
In view of above and having regards to the provision of RTI Act, 2005 which has
overriding effecting on all subsidiary acts and rules framed there under, it is hereby
ordered that the documents as sought by the applicant Sri. Subu Koyang shall be
furnished to him immediately on receipt of the fees as stipulated above in the form of
IPO/Treasury Challan.”
5.
Not satisfied with the first Appellate Authority’s decision the appellant once again
filed appeal to the APIC on 9.04.2008 with fee of Rs.50/- through Treasury Challan.
6.
The appealed was admitted and first hearing was heard on 28th May 2008 in
which the appellant was absent and the respondent was represented by Sri. Joram Nagu,
Section Officer, APPSC and informed that the PIO Sri. N.Tondrang was availing on
leave. The hearing was deferred for 6th June 2008.
7.
The second hearing was heard on 6th June 2008 in which both the appellant and
PIO present. The appellant stated that he could not present in the first hearing as he did
not receive the notice serve to him. He submitted before the Commission that the Public
Service Commission conducted APCS (Jr) examination i.e. for recruitment of C.O.
during 1992-93. He stated that 40 posts for C.O.s were selected. He stated that he has
reasonable doubt that he did better than some other candidates in the examination. He
has got better marks in written examination. Therefore, he should be allowed to inspect
the original marksheet statements of all candidates and correspondences with the Govt.
by the APPSC with appointment of Sri. Ibom Tao, CO. He also said that as per the RTI
Rules only Rs.2/- charged per page on the copies provided to the applicant. Thus, he
pleaded that the copy of the marksheet should be charged only Rs.2/- instead of Rs.25/-.
He replied that, the first Appellate Authority is also wrongly interpreted on it. He further
stated that the first Appellate Authority is also silent on inspection of original documents.
He pleaded that the original documents be allowed to inspect and take certified copies.
PIO submitted that the marks sheet of the requested candidates will be provided
as per the norms of APPSC which is laid down in the “Limitation of Function Regulation
and Office Rules (Manual) 1983”. However, PIO has submitted the photo copies of all
the documents sought by the appellants before the Commission in the hearing and
pleaded to give decision by examining the documents.
Reference to the fee of Rs.2/- per page on the copies provided to the applicant, it
was pointed out that as per the Arunachal Pradesh Right to Information Rules, 2005,
section 8(B)(1) “Where the information is available in the form of a price publication”
shall be charged “price so fixed”.
Thus, the Arunachal Pradesh Public Service
Commission (Limitation of Functions) Regulations 1988 Part IV of 11 could be referred
in which it states that “A candidate should be supplied with a copy of the detailed
marksheet of marks obtained by him or any other candidate in the open competitive
examination on submission of an application to the Secretary, Arunachal Pradesh Public
Service Commission with a fee of Rs.25/- (Rupees twenty five) only by Indian Postal
Order within six months of publication of result of such examination. For obtaining the
mark list a fee of Rs.25/- (Rupees twenty five) per candidate will have to be paid. Thus,
this Gazette notification could be treated as the price so fixed for the marks sheet”. The
appellant has agreed on this point to pay Rs.25/- (Rupees twenty five) only on per marks
sheet. In view of the APPSC on rule of destructions of records, that marks sheet will be
kept for 12 years. In case of pending litigation in the court and RTI Act, 2005, the
applicant shall not be denied to furnish this information in pretext of above mentioned
rule which stated within six months of the declaration of the result. However, furnishing
information of marks sheet shall not be denied to any citizen who apply in view of the
mentioned notification which stated that the application be made ‘within six months of
publication of result of such examination.’ This particular sentences shall be override
under section 2(f)(i) 8(j) and Section 8(3) of RTI Act, 2005 by the opinion Information
Commissioner, Section 2(f) prescribed that “information” means any material in any
form, including records, documents, memos, e-mails, opinions, advices, press releases,
circulars, orders, logbooks, contracts, reports, papers, samples, models, data material held
in any electronic form and information relating to any private body which can be
accessed by a public authority under any other law for the time being in force;” and 2(i)
record includes (a) any document, manuscript and file; (b) any microfilm, microfiche and
facsimile copy of a document; (c) any reproduction of image or images embodied in such
microfilm (whether enlarged or not) and (d) any other material produced by a computer
or any other device;” According section 8(3) “Subject to the provisions of clauses (a), (c)
and (i) of sub-section (1), any information relating to any occurrence, event or matter
which has taken place, occurred or happened twenty years from the date on which any
request is made under section 6 shall be provided to any person making a request under
that section:
PROVIDED that where any question arises as to the date from which the said
period of twenty years has to be computed, the decision of the Central Government shall
be final, subject to the usual appeals provided for in this Act.”
8.
It is pertinent to point out that the APPSC has own system of Recording
Clarification, weeding and destruction of Records vide order No.PSC-R/51/90 dated 2nd
May, 1992 in which files containing Results as recommendation of the Commission in
competitive examinations shall be kept for 15 years. The weeding/destruction of records
of competitive examination particulars mark statement, merit list and recommendation
should not be destroyed and be kept permanently. Because such records shall be required
by the employee till superannuation. As such superannuation may take more than 30
years of services. In case of any dispute of seniority, it is responsibility of APPSC to
clear it. Under Section 19(8)(a)(iv) empowers in its decision, the Central Information
Commission or State Information Commission, to take any such steps as may be
necessary to secure compliance with the provision this Act, including by making
necessary changes to its, practices in relation the maintenance, management and
destruction of records. As such it is advised that the APPSC to take corrective measures
in the matter of fee fixation and maintenance of records in conformity with the RTI Act.
9.
The requested information Sl.No.2 to 9 i.e. correspondence documents in
connection with Sri Ibom Tao as Circle Officer is not a secret document. Inspection of
the above mentioned documents including marksheet should not be a top secret document
under RTI Act. Any competitive examination document after the result is declared
should not be a exempted document under RTI Act. Fair and parity can be enlightened
by the transparency. In order to command public confidence in its process of recruitment
through competitive examination conducted by such bodies like APPSC, it should be
transparent and the candidates must have right to information about their performance in
such examination so that they could assess their capability and status. Neither the first
Appellate Authority nor PIO has mentioned under which section of RTI Act, the sought
information could be exempted from furnishing to the appellant.
10.
The appellant requested that the inspection be done in the premises of the
Arunachal Pradesh Information Commission in on appointed day fixed by the
Commission. The PIO agreed that he will arrange for inspection as requested by the
appellant. In view of the above an interim order was passed on 12th August, 2008 as
followed.
(1) The decision of the first Appellate Authority on the matter of the payment of
Rs.25/- (Rupees twenty five) only per candidate is upheld considering that the ‘price so
fixed’ under section 8(b)(i) of the Arunachal Pradesh Right to Information Rules 2005.
(2)
The appellant be allowed to inspect the correspondences documents in
connection with appointment of Sri Ibom Tao as Circle Officer certified copies on
payment of Rs.2/- (Rupees two) only per page.
(3) The appellant also be allowed to inspect the marksheet of all candidates
including Sri Ibom Tao and take certified copies on payment as mentioned above in
Sl.No.1 and charge inspection fee as per rules as interim order passed earlier.
(4) PIO is directed to make necessary arrangement for inspection on 28.7.2008 at
10.30 A.M. in the office of Arunachal Pradesh Information Commission, Secretariat
annexed.
Decision
1. The interim is complied by the PIO
2. It was informed that the appellant inspected the document on 27th August, 2008
and complained that the original documents were not found.
Therefore, the
hearing was fixed on 23.01.09, 11.02.09 and 10.03.09 and 26th March, 2009. In
two occasions the appellant informed that his inability to attend due to his
sickness but in two occasions he did not appeared whereas PIO present in all
hearings. PIO stated that the appellant inspected the documents but the original
papers on which the Board Members signed were not traceable due to very old
case but the photocopies were available in the custody and the appellant has been
inspected. Since, the appellant did not appeared in hearing in four occasions and
seems no more interested in the matter.
3. Therefore, the case is disposed off.
***
ASSAM PUBLIC SERVICE COMMISSION
2002 (1) GLT 363
HIGH COURT OF GAUHATI
W.P. (C) NO.3968 OF 2001
D.D. 07.03.2002
Hon’ble Mr. Justice A.K.Patnaik
Manjura Mohan Kalita
Vs.
Assam P.S.C. & Ors.
…
Petitioner
…
Respondents
Appointment:
Eligibility for the post and suitable for the post are different. Explained.
Service law – Assam Agricultural Service Rules, 1980 – Rule 6, 8, 11, 12 and ScheduleII – Appointment of Director of Agriculture – Administrative experience – Appointment
by direct recruitment – Advertisement issued providing requirement of administrative
experience not below the rank of Deputy Director of Agriculture – Rules providing
experience in administration of agriculture in a senior post only – Rejection of
candidature on the basis of stipulation regarding experience in the advertisement – Held
improper – Writ petitioner not called for interview despite fulfilling the requirement of
administrative experience in terms of the rules – Could appear on the strength of interim
order – Recommended by APSC – Direction given for consideration of recommendation
holding the petitioner to be eligible as regards experience……. Para 7, 8 and 9.
ORDER
A.K.Patnaik, J:
(1)
By an advertisement of the Assam Public Service Commission published in the
newspaper on Sunday, the 14th January, 2001, applications were invited for the post of
Director of Agriculture under the Agriculture Department of Government of Assam. In
the said advertisement, it was, inter alia, stipulated that the candidates must have
experience in administration of agriculture in a senior post atleast for 8 years, not below
the equivalent rank of Deputy Director of Agriculture. The petitioner who initially joined
as Senior Research Assistant in the year 1972 in Assam Agriculture University, Jorhat,
and thereafter, worked there as Associate Director of Students Welfare with effect from
9.9.1988, and who has been working there as the Director of Students Welfare since
15.10.1998 applied for the said post of Director of Agriculture. But the Assam Public
Service Commission, (for short, “the APSC”), did not call the petitioner for selection and
interview for the said post of Director of Agriculture on the ground that the petitioner did
not have the experience in administration of agriculture in a senior post at least for 8
years not below the equivalent rank of Deputy Director of Agriculture. Aggrieved, the
petitioner has filed this application under Article 226 of the Constitution with the prayer
to quash the said advertisement dated 14th January, 2001, published by the APSC, and for
directing the respondents to call the petitioner for selection and interview by the APSC
for the said post of Director of Agriculture.
(2)
On 4.6.2001, this Court while issuing notice passed an interim order directing the
APSC, and respondents-2 and 3 to allow the petitioner to participate in the interview for
selection to the post of Director of Agriculture, but observed in the said order dated
4.6.2001 that the participation of the petitioner in the interview and the result thereof
would abide by such further order that the Court may pass in the writ petition. The APSC
has informed the Court through its counsel Mr T.C.Chutia that pursuant to the said order,
the petitioner has been allowed to participate in the interview and in fact the petitioner
has been placed in the first position in the select list for the post of Director of
Agriculture. The question that remains to be decided in this writ petition is whether the
petitioner had experience in administration of agriculture in a senior post at least for 8
years as stipulated in the rules and the advertisement.
(3)
Mr A.C.Borbora, learned counsel appearing for the petitioner, submitted that the
Assam Agricultural Service Rules, 1980, (for short), “the Rules, 1980”) and in particular
Schedule-II thereof provides that for the post of Director of Agriculture, a candidate must
have experience in administration of agriculture in a senior post at least for 8 years. The
said provision in Schedule-II to the Rules, 1980, does not say that such experience in
administration of agriculture should be in the rank of Deputy Director of Agriculture.
Yet, in the advertisement, it has been stipulated that such experience in administration of
agriculture should be in the rank of Deputy Director of Agriculture. The advertisement
was, therefore, contrary to the Rules, 1980. Alternatively, Mr. Borbora submitted that in
any case the petitioner has worked as associate Director of Students Welfare with effect
from 9.9.1988 and thereafter as the Director of Students Welfare with effect from
15.10.1998, and the scale of pay of the Associate Director of Assam Agriculture
University is Rs.16,400/- Rs.25,000/- whereas the scale of pay of the deputy Director of
Agriculture is Rs.8,001/- Rs.13,025/-. Hence, the experience of the petitioner was above
the rank of Deputy Director of Agriculture. Mr Borbora the certificate of the ViceChancellor of Assam Agriculture University, Jorhat, annexed as Annexure-‘A’ to the writ
petition, certifying that the petitioner in his capacity as Associate Director of Students
Welfare was functioning as a very competent administrator. He also annexure-A(1) to
the writ petition which is a certificate given by the Registrar of Assam Agriculture
University to the effect that the service in the capacity of Associate Director of Students
Welfare was considered as administrative experience in Agriculture. He argued by
referring to the provisions of the Assam Agricultural University Act, 1968, and in
particular section 17 thereof to show that the Director and the Associate Directors are
officers of the University and hold administrative posts. He also notification dated
4.11.1992 annexed to the affidavit-in-reply of the petitioner as annexure-1 which goes to
show that administrative powers have been delegated to Associate Director of Students
Welfare, Assam Agriculture University, Jorhat. Similarly, the notification dated 5.8.1997
annexed to the affidavit-in-reply of the petitioner as Annexure-5(b) to show that
qualifications for the post of Director of Students Welfare which the petitioner is now
holding includes administrative experience in Agriculture as well as other qualities, such
as, initiative, leadership and capacity to plan, organize, co-ordinate and supervise
students’ co-curricular and extra curricular, cultural and different welfare activities etc.
According to Mr Borbora, the materials annexed to the writ petition as well as to the
affidavit-in-reply of the petitioner would go to show that the petitioner had the requisite 8
years experience in administration of agriculture and was therefore eligible to be
considered for appointment to the post Director of Agriculture under the Rules, 1980.
(4)
Mr M.Bhagawati, learned Government Advocate, Assam, on the other hand,
relied on the averments in the affidavit-in-opposition filed on behalf of respondent No.3,
Commissioner and Secretary to the Government of Assam, Agriculture Department. In
para-8 of the said affidavit-in-opposition, it has been stated that the duties and functions
of the officers of the Assam Agriculture University are limited within the jurisdiction of
the University, and there is vast difference between the activities of the officers of the
University and the agricultural activities of the officers working under the Government
departments. In the said para-8 of the affidavit-in-opposition, it has been contended that
experience in administration of agriculture in a senior post for at least 8 years as
stipulated in Schedule-II to the Rules, 1980, covers wide range of activities relating to
agriculture performed by the Government in the entire State of Assam which has multiple
problems in the field of agriculture. Besides field activities, such experience would cover
the knowledge of various Acts, Rules and Regulations as well as the Fertilizer Act, the
Insecticide Act, the Agricultural Market Produce Act and the knowledge of personnel
management, preparation of plan and scheme, F.Rs and SRs, Assam Financial Rules,
Treasury Rules, etc.
The scope of gathering such required experience by holding
Government post is much wider than that of the post under the University. It has further
been stated that the post of associate Director of Students Welfare is not an administrative
post and even though the duties and functions of the post of Director of Students Welfare
are of administrative nature, they have no link in any way with the administration of
agriculture. It was for this reason that a communication dated 25.11.1998 was issued by
the Commissioner and Secretary to the Government of Assam, Agriculture Department to
the Secretary, Assam Public Service Commission to the effect that as per the records
available the petitioner did not possess the required experience in administration of
agriculture as per condition for experience prescribed in Schedule-II to the Rules, 1980.
Along with the said affidavit-in-opposition filed on behalf of respondent No.3, a copy of
the communication dated 24.4.2001 of the Joint Secretary to the Government of Assam,
Agriculture Department to the Deputy Secretary, APSC has also been annexed reiterating
that the petitioner did not possess the required administrative experience in agriculture.
(5)
Mr T.C.Chutia, learned Standing Counsel for Assam Public Service Commission,
relied on the affidavit-in-opposition filed on behalf of respondent no.1 in which it has
been stated that in response to the advertisement, 13 candidates including the petitioner
applied for the post of Director of Agriculture, and by letter dated 31.3.2000 the
petitioner requested the Government to let the APSC know whether the experience
gathered by the petitioner may be treated as experience in administration of agriculture in
the rank of Deputy Director of Agriculture, and in their reply dated 24.4.2001, the
Government of Assam, Agriculture Department informed the APSC that the petitioner
did not have the requisite experience of 8 years in administration of agriculture.
(6)
On a perusal of the Rules, 1980, it appears that Rule 6 provides for direct
recruitment to the cadre of Director in Class-I (Senior grade), and other posts named in
the said rule. As per the said Rule 6 of the Rules, 1980, such direct recruitment is to be
made by the Government on the basis of the recommendations made by the APSC in
accordance with the procedure laid therein. Rule 8 further states that the academic
qualification of the candidates for direct recruitment shall be prescribed by the governor
from time to time and the qualifications and experience prescribed as on the date of
commencement of the rules are those as are given in Schedule-II. For the post of
Director of Agriculture, the qualifications and experience prescribed for direct
recruitment in Schedule-II are as follows:
“1.
For the post of Director of Agriculture. (1) First Class or high
second class degree in agriculture or first class of high second class
honours or master degree in a subject basic to Agricultural Science such as
Botany, Chemistry etc. (2) Experience in research extension work or
teaching for 10 years (3) Experience in Administration of agriculture in a
senior post at least for 8 years. (4) The following qualification will be
treated as a desirable additional qualifications. (a) Post Graduate degree or
Ph.D. or B.Sc., in any of the basic Agricultural Science like Agronomy,
Agriculture, Extension, Horticulture, Botany, Agriculture Botany, Agril.
Chemistry etc. (b) Outstanding research training or extension or evidenced
by published paper (c) Experience in a top position in the field of
Agricultural education or agricultural extension. (d) Intimate knowledge
of problems of Indian Agriculture particularly with reference to any of the
rice growing regions of India.”
A reading of serial No.3 quoted above would show that a candidate must have experience
in administration of agriculture in a senior post at least for 8 years. A reading of serial
No.4 quoted above would show that outstanding research training or extension or
evidenced by published paper, and experience in a top position in the field of Agricultural
education or Agricultural extension would be treated as desirable additional qualifications
for the post of Director of Agriculture. Thus, if a person has outstanding research
training or extension or experience in a top position in the field of Agricultural education
or agricultural extension, the said additional qualifications will be taken into
consideration along with his other essential qualifications and experience, such as
experience in administration of agriculture in a senior post at least for 8 years. Serial
No.3, in any considered opinion, cannot be read in isolation with serial No.4, and so read
it will encompass not only the experience in administration of agriculture under the
Government, but also the experience in administration of agriculture under an Agriculture
University where the outstanding research training or extension and education are carried
on.
Thus the contention of respondent no.3 in the affidavit in opposition that the
experience in administration of agriculture under the Assam Agriculture University is not
the experience in administration of agriculture as stipulated in serial No.4 quoted above is
misconceived.
(7)
Similarly, the contention in the affidavit-in-opposition of respondent No.3 that the
experience in administration of agriculture would mean only the experience under the
Government in administration of agriculture, is not correct for several reasons. Firstly, in
serial No.3 quoted above, it has not been mentioned that experience in administration of
agriculture should be only under the Government, but it has only been mentioned that a
candidate must have experience in administration of agriculture in a senior post at least
for 8 years. The word ‘administration’ in Black’s Law Dictionary, Fifth Edition, has
been defined to mean-management or conduct of an office or employment, the
performance of the executive duties of an institution, business, or the like. In Chambers
Twentieth Century Dictionary, similarly, the words ‘administration’ has been referred to
mean-the act of administering; management etc. Thus, experience in administration
would include not only experience in Government administration but also experience in
other administration, such, as administration under a University. Secondly, Rule 6 of the
Rules, 1980, provides for direct recruitment to the cadre of Director in Class-I (Senior
Grade). The intent of the said statutory rules is that the post of Director of Agriculture
should be filled up not by promotion from amongst the officers under the Agriculture
department holding lower posts than the Director, such as, Additional Director, Joint
Director, Deputy Director, but from amongst the persons having requisite experience and
qualifications including those who do not have administrative experience under the
Department of Agriculture, Government of Assam. In contrast to rule 6, Rules 11 and 12
of the Rules, 1980, provide for promotion to the cadres of Agricultural Extension Officer
and Junior Subject Matter Specialist, and to the cadres of Additional Director, Joint
Director, Deputy Director, Subject Matter Specialist and Sub-Divisional Agricultural
Officer. If the stipulation in serial No.4 quoted above is to be confined to experience in
administration of agriculture under the Department of Agriculture, Government of
Assam, such as experience in the posts of Additional Director, Joint Director, Deputy
Director as contended in the affidavit-in-opposition of respondent no.3, the purport of
Rule 6 of the Rules, 1980, for holding the open direct recruitment to the post of Director
of Agriculture and appointing the best available talent to the said post would be
frustrated. In my considered opinion, therefore, the post of Director of Agriculture is to
be filled up by a competition amongst the candidates having requisite qualifications and
experience provided in Schedule-II to the rules, 1980, and not merely by candidates who
have administrative experience in agriculture under the Government of Assam.
(8)
The petitioner has worked in the administrative post of Associate Director of
Students Welfare since 1988 and thereafter as Director of Students Welfare since 1998.
From the certificate dated 9.4.1998 of the Vice-Chancellor, Assam Agriculture
University in Annexure-A to the writ petition, it appears that the petitioner after serving
as Research and extension Officer and Professor of agronomy has also worked in an
administrative post of Associate Director of Students Welfare. From the certificate dated
16.10.1998 issued by the Registrar, Assam Agriculture University in Annexure-A(1) to
the writ petition, it appears that the administrative experience earned by the petitioner in
the capacity of Associate Director of Students Welfare is considered as administrative
experience in agriculture. It further appears from the certificate dated 2.2.2001 of the
Registrar, Assam Agriculture University that the petitioner being one of the statutory
officers of the University has wide background in administration management as well as
policy procedure. It further appears that by notification dated 5.8.1997, the essential
qualifications for the post of Director of Students Welfare including administrative
experience in Agriculture have been prescribed. The petitioner thus appears to have
administrative experience in Agriculture for the period of 8 years, and the
communications dated 25.11.1998 and 24.4.2001 to the Assam Public Service
Commission that the petitioner has no such administrative experience in Agriculture for
the period of 8 years were not warranted. But as to whether the petitioner is suitable for
the post of Director of Agriculture, Assam, is different from the question as to whether he
is eligible to be considered for the post of Director of Agriculture. The question as to
whether the petitioner is suitable or not for the post of Director of Agriculture is to be
decided by the Assam Public Service Commission and the State Government. Now that
the APSC has found the petitioner to be suitable for the post of Director of Agriculture,
Assam, the Government will now consider the said recommendations of the APSC in
accordance with the Rules, 1980.
(9)
For the foregoing reasons, I dispose of this writ petition by holding that the
petitioner is eligible under Schedule-II to the Rules, 1980, to be considered for selection
to the post of Director of Agriculture, and by directing that the State Government in the
Agriculture Department will consider the recommendations of the APSC selecting the
petitioner for the said post of Director of Agriculture and pass orders thereon within a
period of two months of the date of receipt of a certified copy of this judgment from the
petitioner by the Commissioner and Secretary to the Government of Assam, Agriculture
Department. Considering however the entire facts and circumstances of the case, the
parties, shall bear their respective cost.
***
2009 (4) GLT 648
HIGH COURT OF GAUHATI
W.P. (C) 2755 & 3288 & 3433 & 3590 OF 2009
D.D. 03.09.2009
Hon’ble Mr. Ranjan Gogoi, Chief Justice (Acting)
&
Hon’ble Mr. B.K.Sharma, J.
Ratul Kumar Das & Ors.
Vs.
State of Assam & Ors.
…
Petitioners
…
Respondents
Recruitment:
Allegations of malpractice in conducting examination – Scope of judicial review
explained – The two concepts of horizontal and vertical reservation explained.
(A) Constitution of India – Arts. 14, 16, 226 – Recruitment to Public Service –
Examination conduced by Assam Public Service Commission for filling up vacancies in
Assam Civil Service – Allegations of malpractices in conducting examination – Scope of
judicial review – Courts not to act as appellate bodies – Judges not to assume the role of
super examiners – In the absence of strong compulsion the Courts not to undertake a
review of the actions of the scrutinizers and head examiners in allowing higher or lower
marks – Possibility of abuse inherent in the process without actual proof cannot be a
ground for judicial review of the decision making process.
Service Law – Recruitment – Allegations of malpractices in recruitment process – Scope
of judicial review – Principles restated…….Para 33, 34
(B)
Constitution of India – Arts. 226, 315, 320 – Public Service Commission –
Correctness of its actions in performance of duties of conducting State Civil Service
Examination – Note circulated by one of the members of Commission highlighting
irregularities in the conduct of examination – Concerned member made a party
respondent in the writ petition and he filed affidavits explaining the nature of
irregularities – Plea for striking of the name of concerned member and to ignore his
affidavits in view of Supreme Court Judgment in Bihar Public Service Commission Vs.
Dr. Shiv Jatan Thakur case, rejected – Held, facts stated by the concerned member being
relevant and vital to the credibility of the Commission, should not be ignored. 1994 Supp
(3) SCC 220 referred …. Para 35
(C)
Service Law – Recruitment – Ratio for determining the number of candidates
qualified to sit in the main examination and to call candidates for interview – Fixation of
– Adoption of a higher ratio – Held, cannot vitiate the selection …. Para 36
(D)
Service Law – Reservation – Vertical (social) reservation under Article 16(4) and
horizontal (special) reservation under Articles 16 (1) or 15(3) – Meaning explained –
Reservation in favour of SC, ST and OBC under Article 16(4) are vertical reservations
whereas special reservations for physically handicapped, women etc. under Articles 16(1)
or 15(3) are horizontal reservations – Beneficiaries of vertical reservation may compete
for non-reserved posts and if appointed to the non-reserved posts on their own merit their
numbers are not counted against the quota meant for them – Same principle does not
apply to horizontal reservations.
Constitution of India – Arts. 15(3), 16(1), 16(4) – Reservation – Vertical and horizontal –
Concept explained – Principe restated. (2007) 8 SCC 785, (1995) 5 SCC 173 referred
…… Para 40, 41, 42, 43
(E)
Review – Administration decision – Administrative decision can always be
reviewed notwithstanding absence of an expressed power of review. AIR 1980 SC 1461
followed ….. Para 48
(F)
Service Law – Reservation – Reservation not to exceed fifty percent of the total
posts advertised – Ceiling on fifty percent reservation laid down in Indra Sawhney case
reiterated – For Assam Civil Service the Commission earmarking 51 posts for open
category candidates out of the 116 posts advertised – Action of the Commission in
earmarking 51 posts for the general candidates, held not sustainable …. Para 50
Held:
In the absence of any strong compulsion the Courts will not undertake a review of
the actions of the scrutinizers and head examiners in allowing higher or lower marks to
any particular candidate. The Court will not carry out a review of the entire process to
find out whether similar exercise of increase or decrease of marks should have been done
in case of all candidates.
Further held:
Unless specific instances are brought to the notice of the Court to show that the
actions undertaken are vitiated by malafides or such actions are demonstrated to be
prompted by extraneous or corrupt reasons, interference of the Court will not be justified.
Cases Referred:
1. AIR 1980 SC 1461 – R.R. Verma & Ors. vs. The Union of India & Ors.
2. 1992 Supp (3) SCC 217 – Indra Sawhney & Ors. vs. Union of India & Ors.
3. AIR 1993 SC 1535 – J.S.Jadhav Vs. Mustafa Hazi
4. 1994 Supp (3) SCC 220 – Bihar P.S.C. & Anr. vs. Dr. Shiv Jatan Thakur & Ors.
5. (1995) 5 SCC 173 – Anil Kumar Gupta & Ors. vs. State of U.P. & Ors.
6. (2007) 8 SCC 785 – Rajesh Kumar Daria vs. Rajasthan P.S.C. & Ors.
ORDER
Ranjan Gogoi, CJ (Acting)
(1)
Democratic values enshrined by the Constitution can thrive and prosper only on
public confidence in the institutions that the Constitution has envisaged to act as
necessary support pillars. Public confidence in the Assam Public Service Commission
over the years has eroded and a perception has been generated that all is not well with the
said institution. Though the tenor of the language used may be mild in its suggestion, it
cannot but be observed that lack of public confidence in the Public Service Commission
carries ominous portents for the future. The credibility of any institution primarily
depends on the conduct of the persons manning it.
People of exemplary conduct,
exceptional ability and utmost integrity who share a serious concerned for public good
alone should be inducted into such a body.
There is no room for compromise or
complacency in the matter. Proper induction and necessary cleansing must be done if
any institution has to earn public trust and confidence without which the constitutional
goal will prove to remain an illusion. The above observations have been felt necessary in
view of the perceptions with regard to the functioning of the Commission that had
manifested itself in the course of the deliberations that had taken place at the prolonged
hearing of these cases. The ultimate conclusion that the Court has reached in the present
cases on the basis of application of the settled legal principles is by no means a
vindication of the acts of the Public Service Commission and the above observations
have been felt necessary to awaken a realization in the right quarters the need for an
abundant measure of public confidence in the institution that alone can justify the high
constitutional status bestowed on it. Having stated all that was considered necessary, the
Court will now turn to the facts of the present cases.
(2)
An advertisement dated 10.8.2006 was issued by the Under Secretary of the
Assam Public Service Commission (hereinafter referred to as ‘the Commission’) inviting
applications from eligible candidates to undergo a preliminary examination to screen
candidates for the main written examination to be held for filling up 116 Nos. of posts in
the Assam Civil Service and other allied cadres. Of the 116 posts advertised, 11 posts
were backlog vacancies. By a corrigendum dated 5.9.2006 issued by the concerned
authority of the Commission it was clarified that the provisions of the Assam Women
(Reservation of Vacancies in Services and Posts) Act, 2005 (hereinafter referred to as
‘the Women Reservation Act’) would apply to the instant selection. The preliminary
examination was held on 18.11.2007 in different centers of the State wherein over 23,000
candidates had participated. The result of the preliminary examination was declared on
1.5.2008 and 5569 Nos. of candidates were shown to have qualified for the main
examination. Of the qualified candidates, 4381 appeared in the main examination that
was held on different dates from 7.7.2008 to 31.7.2008.
The result of the main
examination was declared on 1.12.2008 and call letters were issued to 507 candidates to
appear in the interview.
(3)
Two writ petitions i.e. W.P. (C) Nos.5172 and 5368 of 2008 were instituted
before this Court raising a grievance that the petitioners therein, who were female
candidates, had not been called for the interview though male candidates of the category
to which the applicant belonged though had secured lesser marks have been issued call
letters for the interview. Both the writ petitions were disposed of by this Court by an
order dated 6.1.2009 whereby acting on the stand taken by the Commission that there was
an error committed by it in excluding certain candidates and not calling them for the
interview, this Court had permitted the Commission to call additional candidates for the
interview as prayed for. Accordingly, 93 more candidates were called for the interview
by the Commission. Interviews for the 507 candidates initially called were held between
11.12.2008 and 18.12.2008 whereas the additional 93 candidates called by the
Commission were interviewed on 13th and 14th February, 2009.
(4)
In the course of the proceedings of the two writ petitions in question materials
were laid by the Commission before the Court which indicated that in all 51 out of 116
posts were earmarked for the general/open category candidates. The number of such
posts for the general category candidates really ought to have been at least 53 out of 105
posts advertised, the remaining 11 being backlog vacancies. No question with regard to
the aforesaid issue was, however, raised in the writ petitions. That apart, from the
materials placed before the Court it appears that out of the 51 posts earmarked for the
general category candidates, 15 posts were identified to be reserved for women under the
provisions of the Women Reservation Act. The aforesaid 15 posts were put in a separate
compartment and only female candidates, in order of merit, were called for interview
against the said posts. The remaining 36 posts of the open category were to be in another
compartment and candidates of all categories (both males and females) were called for
interview against the said posts on the basis of their relative merit. A similar exercise
was undertaken while calling candidates of each of the reserved categories for interview
against the posts earmarked for such reserved category candidates. In other words, the
posts earmarked for the reserved category candidates were also spilt up into two
compartments-one exclusive for the females and the other for males as well as females.
Such an exercise having given rise to a situation where some female candidate with
higher marks than their male counterparts of a particular category were left out, the writ
petitions i.e. W P (C) Nos.5172 and 5368 of 2008 were filed. In the said writ petitions
the Commission having prayed for leave to call additional candidates, the same were
closed by grant of such leave. No issue with regard to the correctness of the exercise
undertaken in the light of the legal requirement imposed by the Women Reservation Act
was raised in the aforesaid proceedings.
(5)
The interviews of exactly 600 candidates (507 + 93) having been completed by
the Commission, on the eve of publication of the final select list, a writ petition i.e. WP
(C) No.5346/2008 was instituted before this Court calling into question the legitimacy of
the list of candidates called for the interview and the process of final selection on that
basis. It will not be necessary to notice the specific grounds on which the writ petition
was filed and the basis of rejection of the claims of the writ petitioners by the order dated
4.6.2009 save and except that in paragraph 33 of the order passed in the said writ petition
the very same Division Bench of this Court had found fault with the actions of the
Commission in placing female candidates of different reserved categories i.e. OBC, SC,
ST (P), ST (H) etc. in such categories though they had secured more marks than the
female candidates who were included in the list of open category candidates called for the
interview.
Accordingly, it was left open to the Commission to correct what was
perceived by the Court to be an erroneous exercise. The view taken in paragraph 33 of
the judgment and the consequential directions issued are apparently fallacious in view of
the law laid down by the Apex Court in Indra Sawhney and Ors. Vs. Union of India and
Ors., 1992 Suppl. 3 SCC 217; Anil Kumar Gupta and Ors. Vs. State of U.P. and Ors.,
(1995) 5 SCC 173 and in Rajesh Kumar Daria Vs. Rajasthan Public Service Commission
and Ors., (2007) 8 SCC 785.
(6)
The above facts are ample testimony of what has been always said “that judges
are not infallible and are always open to corrections”. Accordingly, the apparent error in
paragraph 33 of the judgment dated 4.6.2009 passed in WP (C) No.5346/2008 is being
suo motu corrected by us by a separate order in the said proceeding. What, however,
needs to be emphasized is the fact that the decisions cited above with regard to the
concepts of ‘vertical and horizontal reservations’ were not pointed out to the Court at the
hearing of WP (C) No.5346/2008. In fact, the Commission had gone ahead to implement
the court’s above directions and in the process has identified 58 more candidates, who
according to the Commission were ‘entitled’ to be called for the interview. However,
none of the said candidates were so called as even if they were to be given full marks in
the interview (200) they would not have made it to the final select list.
(7)
Thereafter, on 15.6.2009 the final select list of the successful candidates was
notified and the same was communicated to the Government. The Government in its turn
issued appointment orders to the selected candidates which obliged the said candidates to
join within the permissible joining time.
At this stage, on 8.7.2009, WP (C)
No.2755/2009 was filed contending that out of the 116 posts advertised excluding 11
backlog vacancies, at least 53 posts should have been earmarked for the general category
candidates in view of the fact that the reservation cannot exceed 50% of the advertised
posts. Against the 53 Nos. of posts that were required to be earmarked for the general
category candidates, only 51 posts were allotted to such candidates. The pleadings in the
writ petition also indicated that the 30% quota for women under the Women Reservation
Act was worked out as a vertical reservation resulting in a situation where 65% of the
posts advertised were treated as reserved posts.
In addition to the above issues
grievances were also raised with regard to allotment of additional marks to two
candidates bearing Roll Nos.32328 (Ms. Mallika Mazumdar) and 33035 (Ms. Sabita
Das). One selected candidate Dipanjali Das, it was alleged, had indirectly canvassed her
candidature by means of a letter written by her community urging for her selection.
According to the petitioners, the aforesaid letter amounted to indirect canvassing by the
candidate which disqualified her. All the aforesaid three candidates were impleaded as
respondent Nos.7, 8 and 9 in the writ petition. In the writ petition filed, one Dr. Sasadhar
Nath, a sitting Member of the Commission was impleaded as the respondent No.10. A
U/o Note dated 12.6.2009 circulated by the aforesaid respondent No.10 was also enclosed
to the writ petition as Annexure-E. In the said U/o Note, the respondent No.10 had
highlighted that call letters for the interview were issued to a large number of ineligible
candidates besides serious anomalies in the evaluation of the answer scripts. Certain
other issues with regard to the conduct of the competitive examination and the inherent
deficiencies in the existing procedure which contained the potential of affecting the
fairness of the selection process were also indicated.
(8)
This Court while entertaining the writ petition i.e. W.P. (C) No.2755/2009 had
issued certain interim directions the result of which is that 116 candidates though selected
and appointed, have not been able to join in the posts in which they have been appointed.
Thereafter, the matter, in view of the issues involved, has been heard out of turn and the
controversy arising in the writ petition is sought to be resolved by the present judgment
and order.
(9)
As already highlighted on the basis of the pleadings contained in the writ petition,
two issues, in the main, arise. The first is with regard to the correctness/fairness in the
evaluation of the answer scripts of the candidates in the main written examination.
Connected with the said question is the issue with regard to award of higher marks to
some of the candidates including the respondent Nos.7 and 8 in the writ petition. The
U/o Note dated 12.6.2009 of the respondent No.10 and the stand taken by the said
respondent in this regard would be covered by the aforesaid first issue identified by the
Court.
The second issue arising in the writ petition is with regard to the preparation of the list of
candidates found eligible for the interview and the final list of successful candidates in
the light of the understanding of the Commission of the law with regard to vertical and
horizontal reservation. The above question would arise from the stand taken by the
petitioners that in giving effect to the 30% reservation provided by the Women
Reservation Act, the actual percentage of reservation provided by the Commission in the
instant selection is 65%. The question with regard to the precise number of vacancies
that should have been earmarked for the open category i.e. 51 or 53 is a connected issue.
(10)
Having identified the issues that arise for consideration in the writ petition, the
Court may now proceed to understand what has been projected in the affidavit of the
respondent No.10, Dr. Sasadhar Nath as well as in the two affidavits dated 10.8.2009 and
13.8.2009 filed by the Commission.
(11)
The respondent No.10, Dr. Sasadhar Nath who is a sitting Member of the
Commission has filed an affidavit dated 31.7.2009 and an additional affidavit dated
5.8.2009. The affidavit dated 31.7.2009 filed by the Respondent No.10 indicates that an
adverse opinion with regard to the conduct of the competitive examination by the Public
Service Commission had been highlighted in the print as well as electronic media which
has “a bad impact in the society”. Dr. Nath had also indicated in the affidavit that there
has been a progressive decline in the number of participants in the civil services
examination and a feeling has been generated in the public mind that selection by the
Commission has been made on extraneous basis and on correct grounds. The respondent
No.10 has pointed out that under the norms in force, against the 116 posts, 12 times the
number of candidates i.e. 1392 should have been declared eligible for the main
examination whereas 5569 candidates have been declared so eligible. Similarly, the
respondent No.10 has pointed out that though the UPSC follows the ratio of 1:2 while
calling candidates for interview, the Assam Public Service Commission had called four
times the number of candidates which should have been 464; yet, 507 candidates have
been so called. In para 9 of the affidavit Dr. Nath had stated that the respondent No.7, Ms
Mallika Mazumdar is a relative of the Controller of Examinations and that the
verification of her answer scripts in the meeting of the Commission held on 11th and 12th
June, 2009 had revealed addition of extra marks. Similar is the position, according to Dr.
Nath, in the case of respondent No.8, Ms Sabita Das. In para 11 of the affidavit, the
respondent No.10 has pointed out that in the meeting of the Commission held on
15.6.2009 the Controller of Examinations had placed before the Commission the answer
scripts of several candidates in whose cases marks were deducted by the Head Examiner.
According to the respondent No.10, such increase and decrease of marks had affected the
fortune of many candidates. In the U.O. Note dated 12.6.2009, which has been enclosed
to the affidavit of the respondent No.10, it has been stated that such change of marks had
been effected at the level of the Head Examiner who at best had examined only 10% of
the answer scripts. That apart, in the affidavit filed, the respondent No.10 has pointed out
other irregularities in the conduct of the selection including constitution of the interview
Boards and the marks to be awarded by the Members of such Boards, particularly the
Advisor. The said facts are contained in another U/o Note dated 4.7.2009 submitted by
the respondent No.10 to the Secretary of the Commission which Note has also been
enclosed to the affidavit filed by the said respondent.
(12)
In the additional affidavit dated 13.8.2009, the respondent No.10 apart from
marking corrections of certain errors appearing in the affidavit dated 31.7.2009, has
further stated that the Controller of Examinations one Sri Balindra Hazarika as well as the
Assistant Controller, one Sri. T.C.Rajkhowa are both undergraduates whereas the similar
posts in other Public Service Commissions like Tamil Nadu, Karnataka are being held by
senior IAS officers.
Along with the additional affidavit the respondent No.10 has
enclosed a note dated 22.6.2009 addressed to the Secretary of the Commission by another
Member informing the said authority of a news item in a local T.V. Channel to the effect
that one particular Head Examiner appointed to examine the answer scripts of Arts
subjects was a Chemistry teacher.
(13)
The Commission in its affidavit dated 10.8.2009 had stated that social reservation
in favour of SC, ST and OBC under Article 16(4) are vertical reservations while special
reservation in favour of women/physically challenged persons are horizontal reservations
under Article 16(1). In case of vertical reservation the candidates belonging to the
reserved categories can compete for unreserved posts and if they are appointed against
unreserved posts on the basis of their merit their numbers will not be counted against the
quota reserved for the respective reserved categories.
In other words, the reservation
quota will remain intact and will be available to other reserved category candidates.
According to the Commission, the said principle applicable to vertical reservation will,
however, not apply to horizontal reservation. In the affidavit dated 10.8.2009 filed by the
Commission, it has been stated that though 464 candidates ought to have been
interviewed on the basis of the ratio of 1:4, in all 600 candidates were interviewed by the
Commission against 116 posts advertised. In para 8 of the affidavit, the Commission had
admitted that it had committed an error in the process of identification of candidates to be
called for the interview. It has been further stated in the said paragraph of the affidavit
that following the law laid down by the Apex Court in Anil Kr. Gupta (supra), the Public
Service Commission has now identified 207 candidates who are qualified for the
interview against 51 general category posts. Similarly, appropriate number of candidates
qualified for interview against different reserved category posts have also been identified.
A compilation of the aforesaid list of candidates qualified to be called for the interview as
per the law laid down in Anil Kumar Gupta (supra) has been enclosed as Annexure-IV to
the affidavit of the Commission.
(14)
Though it has not been expressly stated, it is evident from the contents of para 9,
10 and 11 of the affidavit of the Commission that the correctly identified candidates who
are eligible to be called for the interview had in fact been so called though their names
were included in categories other than such categories against which they should have
been initially included. Specifically, in para 9 it has been stated that four candidates
whose names figure in the select list, namely, (i) Ms. Diptimoni Bora (Sl.No.26 of the list
of ACS (Junior grade); (ii) Ms. Sabita Das (Sl.No.15 of the list of Inspector of Taxes);
(iii) Ms Pinkumoni Gogoi (Sl.No.16 of the list of Inspector of Taxes); and (iv) Ms Elle
Bora (Sl. No.26 of the list of candidates for Assam Police Service) are not eligible to be
included in the final select list and, instead four other candidates bearing Roll Nos.14642
(891 marks), 10521 (OBC-M, 854 marks), 10539 (OBC-M, 853 marks) and 11424
(General-women-881 marks) are eligible for inclusion in the select list of different
categories of posts indicated in the affidavit filed. Furthermore, in para 9 of the affidavit
the Commission has stated that the posts against which 10 candidates have been selected
are liable to change and instead of the posts for which they have been found eligible they
will now be eligible for certain other posts. A copy of such list of candidates has been
annexed to the counter affidavit of the Commission dated 10.8.2009 as Annexure-V.
(15)
In para 10 of the affidavit filed by the Commission, it has been stated that out of
the 116 candidates whose names appear in the final select list dated 15.6.2009 already
published, 113 candidates who have been selected are within the 464 candidates who
were required to be called for the interview. Three candidates bearing Roll Nos.10366,
23330 and 10003 whose names appear at Sl.Nos.525, 697 and 680 of the list prepared
after the written examination are liable to be included in the select list being ST(H)
candidates.
(16)
`In para 11 of the counter affidavit dated 10.8.2009 of the Commission the details
of the number of candidates of each category i.e. General, SC, ST(P), ST (H), OBC etc.
who should have been called for the interview on correct application of the law laid down
by the Apex Court in Anil Kumar Gupta (Supra) and the excess number of candidates so
called have been indicated. Statements have also been made to show that none of the
excess candidates called for the interview had been selected in the final select list dated
15.6.2009. In case of certain reserved category candidates i.e. OBC, MOBC, it has been
indicated that out of the candidates called from that category a certain number were
eligible to be called as general category candidates by virtue of their merit. If the
aforesaid number of candidates are to be included in the general category a few more
candidates were required to be called for the interview from the particular reserved
category. However, any call for interview of such candidates would be an empty
formality as by virtue of the marks secured by them in the written examination even if
they are to be given full credit at the interview (200 marks) they would still not make it to
the lit of finally selected candidates.
(17)
In para 12 of the affidavit of the Commission, it has been stated that three
candidates with Roll Nos.39179 (General, Female-922 marks), 36457 (OBC, Female-853
marks) and 10718 (ST (P), Male-819 marks) though may have been eligible for inclusion
in the final select list, were, however, not eligible for the particular post for which they
have opted. Accordingly, their names are not eligible to be included.
(18)
Pointing out the aforesaid facts in the affidavit filed, it has been stated that the
Assam Public Service Commission may now be allowed by the Court to publish the
corrected select list finalized on the basis of the parameters indicated in the affidavit the
details of which have already been noted.
(19)
In so far as the stand taken by the respondent No.10 is concerned, according to the
Commission, the respondent No.10 was present in the meeting of the Commission dated
15.6.2009 in which it was decided that the select list of candidates should be finally
published. According to the Commission, no dissent was raised by the respondent No.10
who had signed the minutes of the said meeting. That apart, relying on the law laid down
by the Apex Court in Bihar Public Service Commission and Anr. Vs. Dr. Shiv Jatan
Thakur and Ors., reported in 1994 Supp (3) SCC 220 (Para 28). The Commission in the
affidavit filed has contended that the respondent No.10 cannot be allowed to question the
validity of the selection made by the Commission so long he continues to be a Member of
the Commission.
(20)
In the additional affidavit dated 13.8.2009, the Commission acting pursuant to an
order of this Court has clarified that the panel of examiners, scrutinizers and head
examiners appointed in respect of the Combined Competitive (Main) Examination had
been approved by the Commission on 22.12.2006. As the main examination was held in
2008, additional names drawn from the Gauhati University, Dibrugarh University and
other colleges had to be included in the said approved list. In para 5 of the additional
affidavit the details of the procedure followed in evaluating the answer scripts upto the
stage of tabulation of results has been mentioned. In para 6 of the additional affidavit, the
details of the extra marks secured by respondent Nos.7 and 8 have also been indicated. In
para 7 of the additional affidavit, it has been stated that award of marks and re-evaluation
thereof is permitted by the proviso to Rule 42 of the Assam Public Service Commission
(Procedure and Conduct of Business) Rules, 1986 and correction of marks per se without
any other material to indicate any extraneous or oblique purpose will not invalidate the
corrections of marks as made.
(21)
A preliminary consideration of the writ petitions, particularly, the issue with
regard to vertical and horizontal reservation having indicated that some of the selected
candidates, who, in the meantime, have been appointed could be affected by the on going
proceeding in the writ petition, notice in two leading newspapers of the State was
directed to be issued by the Public Service Commission informing all candidates who
may be desirous of participating in the present proceedings to do so. This was done,
particularly, in view of the interim order passed by this Court on 22.7.2009 not to allow
any of the 116 candidates to join in the posts in which they may have been offered
appointment. Naturally at the stage when the interim order was passed on 22.7.2009, the
extent of infirmity, if any, of the selection held, on correct application of the principles of
horizontal and vertical reservation was yet to be conclusively determined.
(22)
Pursuant to the public notice issued a large number of selected candidates i.e. 45
have joined in the present writ petition who have been impleaded as respondent Nos.11 to
55.
Such candidates are represented by Sri N.Dutta, Sri. AB Choudhury and Sri
K.K.Mahanta, learned senior counsels; Sri. B.D.Konwar, Sri. M.Choudhury and Sri.
P.D.Nair, learned counsels.
They have been heard.
We have also heard Sri.
Y.S.Mannan, learned counsel for the petitioners; Sri B.M.Choudhury, learned counsel for
the respondent No.10 and above all, Sri K.N.Choudhury, learned senior counsel
appearing on behalf of the Commission.
(23)
Sri. Mannan, learned counsel for the petitioners has submitted that the two
affidavits filed by the respondent No.10 along with the UO Notes dated 12.6.2009 and
4.7.2009 clearly indicate that gross anomalies had taken place in the conduct of the
written examination by the Commission for which reason the impugned selection as a
whole should be set aside. Pointing out the two affidavits of the Commission, Sri
Mannan has argued that it has been virtually admitted by the Commission that the final
select list dated 15.6.2009 has been prepared without adhering to the principles of
horizontal reservation introduced by the Women Reservation Act. In fact, the said list
has been prepared by treating the reservation provided by the said Act to be a vertical
reservation. Consequently, according to Sri Mannan, more than 50% of the advertised
posts have been made available to the reserved category candidates which is against the
mandate of the law laid down by the Apex Court that reservation, in any case, cannot
exceed 50%. Sri. Mannan has also pointed out that notwithstanding the corrections
sought to be made by the Commission, adherence to the rule of maximum permissible
reservation require 53 posts to be earmarked for the open category whereas in the present
case only 51 of the total posts advertised have been so earmarked. Such an error is
sought to be continued by the Commission, it is argued.
(24)
Sri. N.Dutta, learned senior counsel appearing on behalf of some of the selected
candidates including one Ms. Pinkumoni Gogoi (who according to the Commission is
liable to be excluded from the correct select list), has submitted that inclusion of the
aforesaid candidates in the select list and the offer of appointment given to her on that
basis are facts that ought not to be ignored by the Court while formulating the eventual
relief to be granted in the writ petition. The candidates are in no way responsible for
what had happened. In this regard, it has also been argued by Sri Dutta that the selection
and appointment of Ms. Pinkumoni Gogoi is not under challenge in the writ petition.
There is no allegation against the said candidate. Sri. Dutta has also submitted that the
corrected exercise as indicate in the affidavit has not been performed by the Commission.
In fact, Sri Dutta has drawn support from the stand taken by Sri. Sasadhar Nath,
respondent No.10 that the Commission had not been involved at any stage in undertaking
the aforesaid exercise. Therefore, according to Sri. Dutta, no leave as sought for in the
affidavit of the Commission can be granted by the Court. Sri. Dutta has further pointed
out that in view of the illegalities and irregularities that had now surfaced with regard to
the application of the principles of vertical and horizontal reservation, the next course of
action is to be decided by the Commission in the light of availability of the power of
review to effect corrections in the final select list already published.
(25)
Sri. A.B.Choudhury, learned senior counsel appearing on behalf of some of the
selected candidates including respondent Nos.7 and 8 has raised serious objections with
regard to the role and conduct of the respondent No.10 in taking the particular stand in
the case as already noticed.
Sri. Choudhury has urged that the writ petition is
inordinately delayed, inasmuch as, the result of the written test was declared in the month
of December, 2008 whereas the writ petition has been filed in the month of August 2009.
No explanation for such delay has been offered.
Sri. B.D.Konwar and Sri. N.
Choudhury, learned counsels appearing on behalf of some other selected candidates have
adopted the arguments advanced by Sri N.Dutta and Sri A.B.Choudhury. So as Sri.
P.D.Nair, learned counsel for another set of selected candidates who, however, has
additionally pointed out that the select list does not contain any waitlisted candidate. In
this regard, Sri Nair has pointed out that if the corrected select list is to be published the
said fact should be kept in mind as some candidates may not be willing to join in the
posts that they may be offered to them at this belated stage.
(26)
Sri K.N.Choudhury, learned senior counsel for the Commission has fairly
submitted that the perception of the Commission with regard to the principles of law
governing horizontal and vertical reservation at the time of finalizing the list of
candidates eligible for interview as well as at the time of finalization of the select list was
erroneous. Subsequently, the Commission has understood the correct purport of the law
laid down by the Apex Court in Indra Sawhney, Anil Kumar Gupta and Rajesh Kumar
Daria (supra). Accordingly, an exercise of correction was undertaken which had revealed
that all candidates who were required to be called for the interview were actually so
called though their names were included in categories other than those in which they
ought to have been included. Sri Choudhury has pointed out that taking into account that
by re-arrangement of the names of the candidates called for the interview in proper
categories and by taking into account the marks secured by them in the interview it is still
possible to salvage the situation. As indicated in para 9 of the affidavit only four
candidates will go out and four others will come in their place and in cases of ten
candidates the categories of posts for which they have been selected will have to be
altered. Sri. Choudhury has accordingly submitted that the Public Service Commission
should be allowed to undertake and complete the necessary corrective steps which will be
conducive to larger public interest.
(27)
In so far as the issue with regard to the conduct of the written examination is
concerned, Sri Choudhury has submitted that correction of marks of some candidates
were made by the head examiner in accordance with the prevailing norms and procedure
and unless such corrections are demonstrated to be for oblique, extraneous or corrupt
reasons, the Court should not come to any conclusion adverse to the Commission. Sri
Choudhury has further pointed out that the stand taken by the respondent No.10 in the
matter ought not to be scrutinized by the Court on the law laid down by the Court in
Bihar Public Service Commission and Anr. V. Dr. Shiv Jatan Thakur (supra).
(28)
We have given our most anxious consideration to the facts of the case, the issues
arising therefrom as well as the elaborate submissions advanced by the learned counsels
for the contesting parties.
(29)
In so far as the issue with regard to the conduct of the written examination is
concerned, provisions have been made in the Assam Public Service Commission
(Procedure and Conduct of Business) Rules, 1986 laying down the norms which are
required to be followed for conduct of the written examination and the interviews. Rule
31, 34, 36, 37, 39, 39, 40, 41 and 42 being relevant, are extracted hereinbelow:
“31. The Secretary shall subject to approval of the Chairman prepare a
list of the persons suitable to be appointed as Invigilators and Supervisors
and shall appoint them as such. Provided that Invigilators of the outstation
Centres may be appointed by the Supervisors concerned in consultation
with the representatives of the Commission deputed for the purpose 34.
Ordinarily 3 weeks time shall be allowed for return of assessed answer
books by the Examiners, but the Commission may in special
circumstances and on a request made by the examiner extend the time to a
reasonable period.
……………
36.
The Controller of examination or the officer-in-charge as the case
may be shall in consultation with the Chairman take step for tabulation of
marks as soon as scrutiny of scripts, removal of discrepancies, removal of
variations and correction if any have been done.
37.
The Commission may cause random checking of the tabulation to
ensure correctness and accuracy of tabulation as well as assessment of
answer books.
38.
As soon as the tabulation is completed and thoroughly checked by
the office and the Commission as provided for in the above rules the
Controller/Officer-in-charge shall submit the result before the
Commission in the form and manner to be prescribed by the Commission
to decide how many candidates are considered fit for being admitted to
personality test/interview.
39.
It shall be the responsibility of the Controller of
Examination/Officer-in-charge to ensure correct tabulation of marks and
correct restoration of original Roll Number of candidates.
40.
The Commission shall decide the number of candidates to be
called for interview before a Board or Boards on any day.
41.
When on each day after the interview is over and marks are
awarded to each candidate the mark-sheet prepared shall be placed in
sealed covers and will be kept by the Chairman.
42.
The mark-sheets so obtained shall be opened on the last day of the
interview or immediately thereafter and the marks of interview/personality
test in a competitive examination shall be added to the marks obtained by
the candidate in the written examination. Thereafter on the basis of totals
so obtained the merit list shall be prepared and placed before the
Commission for final declaration of the result:
Provided that the Commission may with a view to eliminate variation in
the marks awarded to candidate on any examination or interview adopt
method, device or formula which they consider proper for the purpose.”
(30)
In addition to the elaborate procedure for evaluation of answer scripts of the
written examination and the manner in which interviews are to be held, the proviso to
Rule 42 of the aforesaid Rules empower the Commission to adopt any method, device or
formula as may be considered proper in order to eliminate variation of marks awarded to
a candidate in any examination or interview.
(31)
In order to explain the precise procedure that was adopted in the conduct of the
written examination, an affidavit has been filed by the Controller of Examination on
13.8.2009 stating that after the answer scripts are received from the respective examiners,
subjectwise scrutiny of the said answer scripts is required to be carried out. Such scrutiny
is to be done initially by the scrutinizers and, thereafter, by the head examiner engaged
for a particular subject. In the affidavit filed, it has been stated that the entire process of
scrutiny is to take place in the office of the Commission and in the course of such
scrutiny apart from ascertaining as to whether all the answers given have been evaluated
and whether the totaling of the marks is correct, the scrutinizers and thereafter the head
examiner(s) also undertake an exercise to find out whether any candidates has been
awarded high/low marks.
It has been further stated, in the affidavit filed, that if
anomalies on any of the aforesaid counts is found by the scrutinizer the matter is brought
to the notice of the head examiner who either removes the anomalies detected by making
the necessary corrections or such task, at times, is performed by the scrutinizer himself
under the instructions of the head examiner. The Controller of Examination in the
affidavit filed has further stated that the above procedure had been followed in the instant
Combined Competitive (Main) Examination conducted by the Commission.
(32)
After due consideration of the statements made in the affidavit dated 13.8.2009
filed by the Controller of Examination, the Court had also insisted on the personal
appearance of the Controller.
Accordingly, the Controller had personally appeared
before the Court and had made a statement that scrutiny of all the answer scripts and
reference to the head examiner, where the same was considered necessary, had been
done. The Controller had also stated before the Court that the aforesaid practice has been
in vogue in the Commission for long.
(33)
In matters pertaining to conduct of examinations either for recruitment to pubic
service as also in the case of examinations conducted by an University, the role of the
Court normally is minimal. The Courts are not to act as appellate bodies in such matters.
Neither the Judges should assume the role of super examiners. In the absence of any
strong compulsion the Courts will not undertake a review of the actions of the
scrutinizers and head examiners in allowing higher or lower marks to any particular
candidate. The Court will also not carry out a review of the entire process to find out
whether similar exercise of increase or decrease of marks should have been done in case
of all candidates. In the present cases the materials on record including the statement
made by the Controller before the Court sufficiently indicates that all the answer scripts
had been subjected to scrutiny and reference to the head examiner, when considered
necessary, were made. Consequently, any review of the process is bound to result in a
roving enquiry which must be avoided. That apart, the Court does not possess the
necessary expertise in matters of evaluation of answer script. Such a task, therefore, must
be left to the experts in the field.
(34)
In the present cases, a process contemplated by the Rules had been undertaken by
the body entrusted with the task. Unless specific instances are brought to the notice of
the Court to show that the actions undertaken are vitiated by malafides or such actions are
demonstrated to be prompted by extraneous or corrupt reasons, interference of the Court
will not be justified. A process of scrutiny of the answer scripts and in appropriate cases
re-examination of the answer scripts by the head examiner takes place in any
examination. Such steps are an integral part of the attempt to objectively decide the merit
of the candidates and, therefore, must be allowed. A possibility of abuse inherent in the
process without actual proof thereof cannot be a ground for judicial review of the
decision making process. In the present cases, the allegation based on the U/o Note of
the respondent No.10 is that the head examiner had re-examined only 10% of the answer
scripts. Doubt, therefore, has been raised with regard to the validity of the marks secured
by the remaining 90% of the candidates. The materials laid before the Court decisively
indicate that all the answer scripts were scrutinized by the scrutinizers and only those
which were felt necessary to be placed before the head examiner (s) were so placed.
Such action can hardly be faulted on the premise that only 10% of the answer scripts
came to be placed before the head examiner.
(35)
This will bring the Court to a consideration of the stand taken by the respondent
No.10, Dr. Sasadhar Nath, a sitting Member of the Commission.
By order dated
13.8.2009 passed in M.C. No.2123/2009 the prayer made by some of the selected
candidates for striking of the name of Dr. Nath from the list of respondents has been
refused by this Court on the grounds and reasons assigned in the said order dated
13.8.2009. In the said order it has also been held by this Court that Dr. Nath has not
challenged any action of the Commission so as to attract the bar imposed by the law laid
down by the Apex Court in Bihar Public Service Commission and Ors. Vs. Dr. Shiv Jatan
Thakur and Ors. (supra). The respondent No.10, Dr. Nath, had circulated the U/o Note
dated 12.6.2009 highlighting certain illegalities/irregularities in the conduct of the main
written examination. The said U/o Note has formed the basis of a major part of the
pleadings in the writ petitions, particularly, in so far as the conduct of the written
examination is concerned. In such circumstances, Dr. Nath was made a party respondent
in the writ petition. In response, he has filed two affidavit explaining what had happened.
The aforesaid action has been stated to be prompted by the need to bring about essential
reforms in the conduct and performance of duties by the Commission. The facts stated
by the respondent No.10 being relevant and vital to the credibility of the Commission, in
our considered view, the same should not be ignored on the basis of the plea advanced
and, instead, the said facts, according to us, should receive due consideration of the Court
keeping in mind that proper conduct of examinations by the Commission is vital to instill
public confidence in the institution. We, therefore, propose to examine the facts stated by
the respondent No.10 in the affidavits filed.
(36)
Adoption of a higher ratio to determine the number of candidates who had
qualified to sit in the main examination and adoption of the ratio of 1:4 to call candidates
for the interview, in a situation where there are no laid down norms in this regard, cannot
be allowed to vitiate the selections held. Calling of more candidates by itself will not be
fatal as long as the candidates eligible to be called had been issued call letters. This
aspect of the matter will be dealt with in a little more details in the discussion that will
follow on the next issue arising in the case.
(37)
The facts pointed by the respondent No.10 with regard to the holding of the
written examination particularly, the role of the scrutinizers and head examiner (s) has
already been dealt with by this Court in a preceding part of this order. In the absence of
any positive proof forthcoming to the effect that the supervisor (s)/head examiner (s) had
conducted themselves on extraneous grounds and reasons, no adverse conclusion can be
drawn merely because the scrutinizers/head examiners had increased or decreased the
marks in some cases.
(38)
The manner of evaluation of the candidates in the interviews held as highlighted
by the respondent No.10 in the affidavits filed; the allotment of marks to the Advisor and
the parameters governing award of such marks as well as the issue with regard to the
qualifications of the Controller and Assistant Controller of Examination are matters to be
dealt with by the appropriate authority in the course of such action as may be considered
necessary to improve the functioning of the Commission. The Court cannot have any
role in such matters.
(39)
Having dealt with the issues highlighted by the respondent No.10, we must
observe that the respondent No.10 was a party to the decision to publish the final select
list which was taken on 15.6.2009. The respondent No.10 had also participated in the
conduct of the interviews. The fact that the respondent No.10 had not raised any voice of
dissent at any earlier stage of the proceedings is another significant fact that cannot be
ignored by the Court. Notwithstanding the above we have deemed it necessary, in public
interest, to deal with the issues contained in the U/o Notes and the affidavits of the said
respondent and record our findings thereon.
(40)
This will bring the Court to a consideration of the issue with regard to the
horizontal and vertical reservation. The aforesaid two concepts have been succinctly
explained by the Apex Court in Rajesh Kumar Daria (supra). Para 9 of the judgment
which clearly lays down the purport of the aforesaid two concepts of reservation may be
usefully extracted herein below:
“9.
The second relates to the difference between the nature of vertical
reservation and horizontal reservation. Social reservation in favour of
SC. ST and OBC under Article 16(4) are “vertical reservations”. Special
reservations in favour of physically handicapped, women etc., under
Articles 16 (1) or 15 (3) are “horizontal reservations”. Where a vertical
reservation is made in favour of a Backward Class under Article 16 (4),
the candidates belonging to such Backward Class, may compete for nonreserved posts and if they are appointed to the non-reserved posts on their
own merit, their number will not be counted against the quota reserved for
respective Backward Class. Therefore, if the number of SC candidates,
who by their own merit, get selected to open competition vacancies,
equals or even exceeds the percentage of posts reserved for SC candidates,
it cannot be said that the reservation quota for SCs has been filled. The
entire reservation quota will be intact and available in addition to those
selected under open competition category. (vide Indra Sawhney,
R.K.Sabharwal Vs. State of Punjab, Union of India v. Virpal Singh
Chauhan and Ritesh R. Sah v. Dr. Y.L. Yamul. But the aforesaid principle
applicable to vertical (social) reservations will not apply to horizontal
(special) reservations. Where a special reservation for women is provided
within the social reservation for Schedule Castes, the proper procedure is
first to fill up the quota for Scheduled Castes in order of merit and then
find out the number of candidates among them who belong to the special
reservation group of Scheduled Caste women.” If the number of women
in such list is equal to our more than the number of special reservation
quota, then there is no need for further selection towards the special
reservation quota. Only if there is any shortfall, the requisite number of
Scheduled Caste women shall have to be taken by deleting the
corresponding number of candidates from the bottom of the list relating to
Scheduled Castes. To this extent, horizontal (special) reservation differs
from vertical (social) reservation. Thus women selected on merit within
the vertical reservation quota will be counted against the horizontal
reservation for women.”
(41)
The Apex Court in Rajesh Kumar Daria (supra) had also taken note of the
following observations in the judgment in Indra Sawhney (supra)”
“all reservations are not of the same nature. There are two types of
reservations, which may, for the sake of convenience, be referred to as
‘vertical reservations’ and ‘horizontal reservations’. The reservations in
favour of Scheduled Castes, Scheduled Tribes and Other Backward
Classes [under Article 16(4)] may be called vertical reservations whereas
reservations in favour of physically handicapped [under Clause (1) of
Article 16] can be referred to as horizontal reservations. Horizontal
reservations cut across the vertical reservations-what is called interlocking
reservations. To be more precise, suppose 3% of the vacancies are
reserved in favour of physically handicapped persons; this would be a
reservation relatable to Clause (1) of Article 16. The persons selected
against this quota will be placed in the appropriate category; if he belongs
to SC category he will be placed in that quota by making necessary
adjustments; similarly, if he belongs to open competition (OC) category,
he will be placed in that category by making necessary adjustments. Even
after providing for these horizontal reservations, the percentage of
reservations in favour of Backward Class of citizens remains and should
remain the same.”
(42)
The observations of the Apex Court in Anil Kumar Gupta (supra) [para 18]
quoted with approval in Rajesh Kumar Daria (supra) may also be extracted hereinbelow:
“18. Now, coming to the correctness of the procedure prescribed by the
revised notification for filling up the seats, it was wrong to direct the
fifteen percent special reservation seats to be filled up first and then take
up the OC (merit) quota (followed by filling of OBC, SC and ST quotas).
The proper and correct course is to first fill up the OC quota (50%) on the
basis of merit; then fill up each of the social reservation quotas, i.e., S.C.,
S.T. and B.C; the third step would be to find out how many candidates
belonging to special reservations have been selected on the above basis. If
the quota fixed for horizontal reservations is already satisfied – in case it is
an overall horizontal reservation – no further question arises. But if it is
not so satisfied, the requisite number of special reservation candidates
shall have to be taken and adjusted/accommodated against their respective
social reservation categories by deleting the corresponding number of
candidates therefrom. (If, however, it is a case of compartmentalized
horizontal reservation, then the process of verification and
adjustment/accommodation as stated above should be applied separately to
each of the vertical reservations. In such a case, the reservation of fifteen
per cent in favour of special categories, overall, may be satisfied or may
not be satisfied.) Because the revised notification provided for a different
method of filling the seats, it has contributed partly to the unfortunate
situation where the entire special reservation quota has been allocated and
adjusted almost exclusively against the OC quota.”
(43)
The law laid down by the Apex Court in the aforesaid cases has clearly and
categorically laid down that reservation for women is a horizontal reservation and is not
over and above the vertical reservation provided to different categories like SC, ST, OBC
etc. Under the Reservation for Women Act of Assam, 30% of the posts are reserved for
women within each category i.e. open, SC, ST, OBC etc. In the event the select list
prepared on the basis of merit for each category already includes the requisite number of
women candidates no further exercise is required to be performed. However, if there is a
shortfall of women candidates in order of merit will replace the last of the male
candidates. This is the essence of the application of the law relating to reservation under
the Reservation for Women Act of Assam.
(44)
What was done by the Commission in the present case is that 30% out of the posts
earmarked in each category i.e. open, SC, ST, OBC etc. were kept open for competition
amongst women candidates only belonging to each of such categories. The remaining
70% of the posts in each category was open to all including female candidates. The list
of candidates eligible for interview and the final select list of successful candidates was
prepared on that basis. What was, therefore, adopted in order to give reservation to
women under the Reservation for Women Act is the principle of vertical reservation
thereby raising the limit of reservation beyond the constitutionally permissible 50%.
Such action being contrary to the law laid down by the Apex Court, as noticed above,
undoubtedly the final select list dated 15.6.2009 is vitiated and has to be set aside. The
question that confronts the Court is whether the Commission should be permitted to re-do
the exercise on the basis of the interviews already held or whether the entire process
should be directed to re-commenced from the stage of interview.
(45)
Certain facts which will be relevant for a just decision on the above issue
highlighted may now be usefully recapitulated:
i) The candidates eligible to be called for interview, upon application of
correct legal principles and parameters, had been so called though such
candidates were placed in categories other than which they were entitled
to be place, as for example, a candidate eligible to be called as a open
category candidate was actually called as a OBC/mobc candidate.
ii) The marks secured by the candidates in the written test were not
before the interview board.
iii) Though excess candidates had been called for the interview, 113 out
of 116 candidates who will get selected upon application of the correct
norms and parameters are amongst the 464 candidates who should have
been called on the basis of the 1:4 ratio.
The remaining three are ST (P) candidates who will make it to the final select list on the
basis of the quota available to them.
iv) None of the excess candidates called for the interview for any of the
categories did figure in the final select list already published.
v) If realignment upon repetition of the exercise by application of correct
parameters is to be done, 4 candidates out of 116 already selected will
have to be excluded and in their place 4 other candidates will come in. In
case of 10 candidates there will be change of the category of posts in
which they have been offered appointment.
(46)
The role of the Court in such matters must be a constructive one. The power to
nullify is a drastic power which must be reserved to be exercised when reconstruction is
not possible. To the extent reconstruction is possible without offending the rights of the
persons involved, the Court must always lean in favour of such a course of action. In the
present case, the materials on record clearly establish that all candidates who were
required to be interviewed upon application of the correct principles and parameters of
the law relating to reservation had actually faced the interview though as candidates of
some other category. The marks of the written test were not available to the members of
the interview board. The exercise, if re-done by application of correct parameters, will
have minimal effect as already indicated and as stated in para 9 of the affidavit of the
Commission dated 10.8.2009. The Court, therefore, would be inclined to lean in favour
of permitting the exercise to be re-done subject to our views on the objections raised in
this regard by the learned counsels for the selected candidates.
(47)
A point has been raised that the result/consequences indicated by the
Commission in para 9 of the affidavit dated 10.8.2009 does not reflect the view of the
Commission as the matter is yet to be placed before the Commission. Naturally, any
further exercise in the matter will have to be performed by the Commission as a body
after a detailed examination of all aspects of the matter to ensure that no further errors
creep in. In other words, the minimal impact of the de novo exercise, as claimed in the
affidavit of the Commission, will have to be verified and approved by the Commission as
a body.
(48)
An argument has been advanced that if the select list dated 15.6.2009 is to be set
aside, the Commission will not have any power to review the names of the selected
candidates in order to effect any alterations therein. Emphasis in this regard has been laid
on the absence of any expressed power of review in the Commission. The argument is
self-defeating. If the select list dated 15.6.2009 is to be set aside by this Court, further
course of action in accordance with law must be left to the discretion of the Commission.
Furthermore, as held by the Apex Court in R.R.Verma & Ors. vs. The Union of India and
Ors., (AIR 1980 SC 1461), the absence of an expressed power of review will not come in
the way of review of a administrative decision.
(49)
The legal right of the four candidates mentioned in para 9 of the Commission’s
affidavit dated 10.8.2009 who, according to the Commission will not be entitled to be
included in the select list upon application of the correct principles and parameters of law
may now be considered. The selection of the aforesaid four candidates being contrary to
the law laid down by the Apex Court with regard to horizontal reservation, cannot vest in
any of such candidates a legal right to insist that their names should continue to remain in
the list of selected candidates. The aforesaid four persons were not initially parties to the
writ petition but three of them have subsequently impleaded themselves as respondent
Nos.8, 28 and 51 in the writ petition pursuant to the notice issued by this Court. The
aforesaid person, therefore are parties to the writ petition who have also participated in
the hearing. Therefore, there cannot be any legal impediment to their exclusion from the
list of selected candidates if such exclusion is justified on application of the correct
principles of law.
(50)
An argument has been advanced on behalf of the petitioners that out of the 116
posts advertised even if the 11 backlog vacancies are excluded, 105 posts will remain in
the fray to which posts the maximum limit of permissible reservation will be applicable.
In that event, at least 53 posts have to be allowed to the open category candidates
whereas, admittedly, the Commission has earmarked 51 posts for the open category
candidates. The point has substance and cannot be ignored on the ground that the
variation is marginal. Even marginal deviations from an established constitutional
principles cannot be tolerated in a system governed by the Rule of law. The action of the
Commission in earmarking 51 posts for the general/open category candidates, therefore,
is not sustainable.
(51)
All relevant facts of the cases having been discussed elaborately, we now deem it
proper to reach the conclusion that the writ petitions should be partly allowed. The select
list dated 15.6.2009 is set aside leaving it open for the Commission to take further steps
in the matter in the light of the observations and findings recorded in the present order
and the law laid down by the Apex Court in the decisions referred to herein.
(52)
Having regard to the facts and circumstances of the case, we are of the view that
the parties should be left to bear their respective costs.
(53)
B.K.SHARMA, J.- While agreeing with the findings an conclusions reached by
his Lordship Hon’ble Mr. Justice Ranjan Gogoi, Chief Justice (Acting) in the judgment, I
would like to give vent to my thoughts generated in my mind on perusal of the materials
on records and upon hearing the learned counsel for the parties.
(54)
My Lord’s opening passage of the judgement highlighting the erosion of public
confidence in the APSC over the years is an apt and timely observation. There is no
gainsaying that an Institution like APSC must be the repository of public confidence.
Once the confidence is shaken, even the right direction of functioning will be doubted.
Such is the importance of public confidence in APSC that the people often question the
credibility of its members on whom the credibility of an Institution like APSC largely
depends. It has rightly been observed that people of exemplary conduct, exceptional
ability and utmost integrity, who share a serious concern for maximum public good
should alone be inducted into such a body. Often it is seen that members of APSC are
inducted on extraneous consideration possessing qualification other than those.
(55)
Much was debated during the course of hearing regarding the role of the
respondent No.10, who is a sitting member of the APSC. It was also suggested that he
being not a necessary party, his name should be stuck off as respondent. It was also
submitted, upon a reference to the decision of the Apex Court in Bihar Public Service
Commission Vs. Dr. Shiv Jatan Thakur reported in 1994 Supp (3) SCC 220 that the
respondent No.10 cannot be allowed to question the very selection of which he was a
party. We, by our order dated 13.8.2009 passed in MC No. 2123/2009, have overruled
the plea on the ground assigned in the order. We have also heard him in person.
(56)
In the counter affidavits filed by the said respondent No.10, certain disturbing
revelations have been made. In his affidavit he has enclosed two U.O. notes dated
12.6.2009 and dated 4.7.2009 addressed to the Secretary, APSC. He has also enclosed
the letter dated 22.6.2009 addressed to the Secretary, APSC by another sitting member.
The contentions raised in the said two affidavits and the U.O. notes, have been generally
discussed in the judgment. It is really disturbing that a sitting member of the APSC has
to state about the allegations that there exists some rackets of brokers or agents inside and
outside the APSC.
(57)
One significant aspect of the matter as highlighted by the said member is that the
candidates were required to furnish their telephone/contract numbers in the prescribed
column of the application forms for the main combined competitive examination, 2006.
There was no such provision for providing phone number by the applicant in the
application form for the preliminary examination but such a provision was made for the
main examination. It is in this context, the said member has alleged that the availability
of the phone numbers in the application forms possibly has linkages to the allegations of
corrupt practices.
(58)
The respondent No.10 has also pointed out the alleged illegality in raising the
marks in the answer script of respondent No.7, stated to be a relation of Controller of
Examination, APSC i.e. the respondent No.6.
(59)
Allegations regarding awarding extra marks to the respondent No.8, another
candidate has also been made. The respondent No.10 has also questioned the very basis
of addition of marks by the Head Examiner. In this connection, he has referred to the
provisions of the Assam Public Service Commission (Procedure and Conduct of
Business) Rules, 1986.
According to him, there is no provision in the rules for
revaluation of the answer scripts by the Scrutinizers or by the Head Examiners.
(60)
Further allegation made by the said member is that in some answer scripts, marks
were reduced by the Scrutinizers or Head Examiners, which according to him had the
potential of two fold objectives, (i) to reduce the marks to remove some candidates from
the run in the competition of inter se merit and (ii) to reduce the burden of adding extra
marks for the entitled candidates for securing slots in the select list as addition of a large
figure of extra marks would require handling of many questions of many scripts that also
with unreasonably high marks. According to him in one script bearing Roll No.8578
(Code No.171) as high as 35 marks were deducted from the marks given by the Examiner
in History paper. Although, he had discussed and pointed out such discrepancies and
anomalies but as per the affidavits filed by him, such discussion was discouraged.
(61)
It is the belief of the respondent No.10, that if deducted marks are reconciled and
original marks given by the examiners restored, some candidates would secure slots in
the select list. Further stand of the respondent No.10 is that if the extra marks added are
eliminated and the original marks given by the examiners restored, the actual successful
candidates would remain in the select list with their actual inter se merit.
(62)
In the additional affidavit filed by the said respondent No.10, he has enclosed the
letter dated 22.6.2009, addressed to the Secretary, APSC by another member. As per the
said letter, there was news item to the effect that while evaluation/revaluation was carried
out by one Science Teacher as Head Examiner, he was asked to examine answer scripts
of Arts subject. Whether such a report was correct or not was requested to be enquired
into to find out the truth. Although, the letter was written on 22.6.2009 but no action
seems to have been taken by the APSC and it was only amidst the hearing of the case,
certain loose sheets terming the same to be the records, had been produced, indicating
furnishing of note etc. in the month of August, 2009, mentioning therein that enquiry was
carried out, which revealed that the news report was incorrect. As to what caused the
delay to immediately show response to the said letter dated 22.6.2009 considering the
seriousness of the matter, is not discernable. It is only after filing of the additional
affidavit on 5.8.2009, enclosing the said letter, the APSC reacted to the same by
preparing office notes thereafter in loose sheets, terming the same to be the records of the
APSC.
(63)
In the U.O. No.1 dated 12.6.2009, the respondent No.10 pointed out the purported
anomalies relating to revaluation of the answer scripts by the Scrutiniser and or the Head
Examiner, so as to reduce and or increase the marks already awarded by the Examiner.
In his further U.O. No.1 dated 4.7.2009, he has indicated that the marks reduced by the
Scrutiniser/Head Examiner from the earlier marks awarded by the examiner in respect of
five candidates. The candidate bearing Roll No.8578 secured 846 marks (including viva
marks). It is the revelation made by the said member that in case of the said candidate, 35
marks were deducted at the stage of scrutiny. The original total marks secured by the
said candidate was 881 but after such deduction, it was reduced to 846, falling short by 3
marks below the Roll No.38546 (885 marks), who secured a slot of un-reserved vacancy
in the select list in the category of Inspector of Labour.
(64)
Another significant aspect of the matter relating to increase and decrease of
marks in the answer scripts as has been revealed by the respondent No.10 is that out of
the two Head Examiners, one had the mind set of increasing by addition while the other
of reducing (by deduction of marks).
(65)
In the U.O. letters dated 12.6.2009 and 4.7.2009, many other anomalies have been
pointed out by the respondent NO.10 and it was urged upon the authority to remove such
anomalies so as to maintain transparency in the matter of selection by the APSC. Some
other aspects of the matter highlighted by the said member have been discussed in the
judgement.
(66)
The Controller of Examination was generally questioned in respect of the
revaluation of the answer scripts by the Scrutiniser/Head Examiner. He in his statement
stated about revaluation of answer scripts totaling to around 500 and odd with the reason
assigned for such re-evaluation. When he was asked to reduce such statement in the form
of an affidavit, he filed the affidavit on 13.8.2009 with the statement that after the answer
scripts are received from the respective examiners, the answer scripts are arranged
subject-wise in the office of the Commission. Thereafter, the procedure of scrutiny
begins for each subject.
One Head Examiner is engaged by the Commission and
depending upon the number of answer scripts, adequate number of Scrutinisers are also
engaged. The scrutinisers scrutinised all the answer scripts subject wise to find out the
following:“a) Whether the candidate has been awarded marks in respect of all the
questions he/she has answered? B) whether the totaling of the marks is
correct? c) whether the candidate has been awarded high/excess marks?
d) whether the candidate has been awarded low marks?”
(67)
The affidavit further states that if the Scrutinisers find any of the aforesaid
anomalies in any of the papers, the same is brought to the notice of the Head Examiners
and then in consultation with the Head Examiner; the said anomalies are corrected either
by the Head Examiner or by the Scrutiniser himself, under the instruction of the Head
Examiner. Thereafter, mark sheet of the candidates are prepared and the tabulation is
done. However, the affidavit is silent on assigning reasons for re-evaluation by the
Scrutinisers/Head Examiners. No records have also been produced.
(68)
According to the respondents, the aforesaid procedure adopted by the
Commission finds support from Rule 37 and proviso to Rule 42, which are quoted
below:“37. The Commission may cause random checking of the tabulation to
ensure correctness and accuracy of tabulation as well as assessment of
answer books.
42.
The mark-sheets so obtained shall be opened on the last day of the
interview or immediately thereafter and the marks of interview/personality
test in a competitive examination shall be added to the marks obtained by
the candidate in the written examination. Thereafter on the basis of totals
so obtained the merit list shall be prepared and placed before the
Commission for final declaration of the result.
Provided that the Commission may with a view to eliminate variation in
the marks awarded to candidate on any examination or interview adopt
method, device or formula which they consider proper for the purpose.”
(69)
In the instant case, it is on record that in at least in some answer scripts final
marks awarded by the examiners were increased or decreased by the Scrutinisers/Head
Examiners. Although, the Commission has referred to the above provisions so as to
justify their action but such course of action adopted does not inspire the confidence of
the Court. Transparency and fair play demand a better methodology. After all the expert
examiners are appointed subject wise.
There is no guarantee that the particular
assessment made by the Scrutiniser and/or the Head Examiner, by way of revaluation is
the better assessment than the assessment made by the expert examiners. The course of
action adopted also makes room for doubt. This requires immediate attention of the
Commission, so as to eradicate any amount of doubt.
Normally, in the University
examinations, the job of the Scrutiniser is to find out any omission in the evaluation by
the Examiner, such as, wrong calculation of total marks, omission to evaluate any answer
etc. Normally, it is not the business of a Scrutiniser to pick up answer scripts to find out
as to whether the Examiner has awarded high or low marks.
Likewise, the Head
Examiner’s business is to supervise the scrutiny and his role is supervisory. But in the
instant case, the Scrutinisers and Head Examiners were given free hands to increase or
decrease the marks, already awarded by the expert Examiners. All these aspects of the
matter need immediate attention of the APSC and for that matter, the authority at the
helm of affairs.
(70)
The Commission in its affidavits filed has not dealt with the anomalies pointed
out in the affidavits filed by the respondent No.10 but has questioned his credibility to
question the validity of the selection made by the Commission, falling back on the
decision in Bihar Public Service Commission (supra). It is true that on the basis of the
available materials, it is difficult to show that the actions undertaken are vitiated by
malafides and/or is the product of extraneous consideration and thus interference with the
selection will not be justified. But at the same time, it needs to be emphasized that the
particular procedure adopted is not a healthy procedure in an Institution like APSC.
Further, although the respondent No.10 has highlighted and has brought on record the
purported anomalies and illegalities committed in the selection but he, as a member of the
Selection Committee, duly participated in the selection process and it was only at a latter
stage, pointed out those illegalities and/or irregularities. This Court cannot be oblivious
of the said position of the respondent No.10.
(71)
This court exercising the power of judicial review under Article 226 of the
Constitution of India, cannot assume the role of the appellate bodies in such matters as
has been observed in the judgement. Neither the judges should assume the role of super
examiner nor undertake any review of the actions undertaken by the APSC, how-so-ever,
suspicious that may be.
But at the same time the apprehensions expressed by the
respondent No.10, though in the realm of suspicion and cannot be said to be concrete
within the parameters of judicial review, cannot be altogether brushed aside as irrelevant.
(72)
`The mistakes which the APSC has admitted in their counter affidavits also have
the potential of eroding the credibility of the APSC in the public eyes.
As a
constitutional body, its primary duty is to conduct important selections pertaining to
public offices. If such a body commits such mistakes as has been admitted by itself, the
general public is bound to raise accusing fingers which is not at all healthy sign in the fair
name of APSC, which otherwise should have been the hall mark.
(73)
My observations made above are in tune with the observations made in the
opening passage of the judgment. The APSC and the authority at the helm of affairs will
do well to remove the general feeling that all is not well with the affairs of the APSC.
(74)
I part with the case records referring to Hary-R-Blythe’s observations (cited in 21
Green Bag 224), which finds mention in AIR 1993 SC 1535 (J.S. Jadhav Vs. Mustaf
Hazi). Although the observation is in the context of cleansing the legal field from poison,
but the same may not be altogether out of place in the present context, as the test for the
members of the APSC and the Institution as a whole, as highlighted in the opening
passage of the judgement is the same.
“great God! The hour has come when we must clear the legal fields from
poisons and from fear; We must remould our standards built them higher,
And clear the air as though by cleansing fire, weed out the damning
traitors to the law, Restore her to her ancient place of awe.”
***
THE GAUHATI HIGH COURT AT GUWAHATI
W.P. (C) NO.2377/2010
D.D. 06.05.2010
Hon’ble Mr. Justice Ranjan Gogoi
Hon’ble Mr. Justice B.P.Katakey
Pritam hazarika
Vs
The State of Assam & Ors.
…
Petitioner
…
Respondent
Examination:
Whether errors, irregularities in the matter of evaluation in respect of one candidate
justify revaluation of scripts of all the candidates? – No.
The petitioner was a candidate for recruitment to Assam Civil Services (Junior
Grade) and Other Allied Services – As the applicant did not clear main examination he
became ineligible for viva-voce – After selection process was over appointments were
issued to selected candidates – At that stage the applicant filed this writ petition seeking
recall of the answer scripts of the main examination of all candidates for evaluation and
re-evaluation and consequential interference with the select list published - High Court
in view of the fact that the total error was in respect of one candidate and other
irregularities pointed out were found to be untenable dismissed the writ petition with
costs of Rs.5,000/- holding that the writ petition was filed without exercising minimum
care and caution and was frivolous.
Held:
Merely because an irregularity has been noticed by the Court in respect of the
marks awarded against one question to one particular candidate, the same will not justify
recall for the answer scripts of all the candidates for the purpose of evaluation.
Further held:
Selection by the Commission is to be made on the basis of merit as determined in
the selection held. If the candidate in question was eligible for selection on the basis of
the merit only because certain groups of citizens had submitted representations for her
selection without her knowledge the candidate cannot be debarred on that ground alone.
ORDER
Ranjan Gogoi, J:
Heard Mr. AM Mazumdar, learned senior counsel for the petitioner and Mr.
C.Baruah, learned Standing Counsel, Assam Public Service Commission.
2.
The petitioner took part in a selection conducted by the Assam Public Service
Commission for selection of candidates for appointment in the Assam Civil Services
(Junior Grade) & Other Allied Services. The selection process was conducted by the
Assam Public Service Commission (hereinafter referred to as “the Commission”)
pursuant to an advertisement dated 10.8.2006. The petitioner did not clear the main
written examination to be entitled to participate in the viva-voce segment of the selection
process. The modified select list in terms of the order dated 1.9.2009 passed in W.P. (C)
No.2755 of 2009 and other connected cases was published on 18th February, 2010. In the
meantime, appointments have been made.
At this stage, the writ petitioner has
approached this Court seeking recall of the answer scripts of the main written
examination of all candidates for evaluation and re-evaluation and consequential
interference with the select list published.
3.
The petitioner has not indicated either in the writ petition filed or in the course of
the oral arguments advanced the precise marks that he had secured in the written
examination and the last of the candidates, in order of merit, who was called for interview
belonging to the category to which the petitioner belongs. The petitioner has also not
exercised his right under the Right to Information Act to have access to the answer scripts
written by him so as to point out the anomalies therein, if any, before the Court. Rather,
the petitioner relies on certain alleged irregularities in respect of some candidates in
support of the prayer made.
4.
The first candidate in respect of whom irregularities with regard to award of mark
mentioned by the petitioner is one Ms. Mallika Mazumdar who has been selected for
appointment in the cadre of Inspector of Taxes. In this regard, the petitioner has pointed
that against one question in addition to 7 marks awarded by the Examiner, the Head
Examiner had added 5 more marks. The resultant total should have been 12, whereas
according to the petitioner, the same was 14. In view of the aforesaid allegation made,
we have called for the answer scripts of the aforesaid Ms Mallik Mazumdar and on
verification thereof we find the statements made by the petitioner to be correct.
However, notwithstanding the above, the result of the selection of Ms. Mallika
Mazumdar will not be affected in any manner inasmuch as the said candidate had secured
a total of 910 marks (including the interview marks), whereas the last of the candidate
selected belonging to the same category has secured 894 marks. Merely because an
irregularity has been noticed by the Court in respect of the marks awarded against one
question to one particular candidate, the same will not justify recall of the answer scripts
of all the candidates for the purpose of evaluation and reevaluation, as prayed for. In this
regard, it must be emphasized that this is the precise reason why the petitioner should
have indicated the marks secured by him and the last of the qualified candidate belonging
to his category so as to enable the Court to exercise its power depending on the difference
of marks as may be existing. The aforesaid exercise not having been done by the
petitioner the Court is not in a position to further appreciate the issue. In any event, as
already indicated, in the decision of this Court in certain other cases, it has been
acknowledged that a candidate who had taken part in a selection conducted by the
Commission has a right under the Right to Information Act to receive photocopies of the
answer scripts. The petitioner should have availed of the said right; examined the answer
scripts and, thereafter, pointed out the anomalies, if any to the Court as may have been
noticed by him. The said course of action was also not attempted by the petitioner.
5.
The second candidate mentioned by the petitioner is one Ms. Dipanjali Das. The
allegation against her is with regard to the fact that the candidature of Dipanjali Das was
canvassed by certain groups which should have resulted in her disqualification. Selection
by the Commission is to be made on the basis of merit as determined in the selection
held. If the candidate in question Dipanjali Das was eligible for selection on the basis of
marks secured by her, as she was, only because certain groups of citizens had submitted
representations for her selection, without her knowledge, the candidate in question cannot
be debarred on that ground alone.
6.
The third candidate mentioned by the petitioner is one Bidyut Das Boro who was
initially selected in the Assam Civil Services as a ST(P) candidate. In a separate writ
proceeding before this court, it has been held that the aforesaid candidate does not belong
to ST(P). It is alleged that in the revised select list dated 18.2.2010 the said candidate has
again been selected, this time, to the Assam Police Service. The petitioner cannot have
any legitimate grievance in this regard inasmuch as though it has been held that the
candidate does not belong to the Schedule Tribe category, the same would not disentitle
him for selection as a general category candidate. Shri Bidyut Das Boro has in fact been
selected to Assam Police Service as open category candidate in the revised select list and
not as a reserved category candidate. This is evident from the select list dated 18.2.2010
enclosed to the writ petition.
7.
The last candidate mentioned by the petitioner is one Alee Bora. According to the
petitioner, in the affidavit filed by the Assam Public Service Commission in W.P.(C)
No.2755 of 2009, it has been stated that on revision of the select list as per the trend of
the exchanges that took place in the course of hearing, 4(four) candidates would be out of
reckoning.
The grievance of the petitioner is notwithstanding the above, the said
candidate Alee Bora has once again been selected for appointment in the cadre of
Inspector of Taxes.
8.
The grievance of the petitioner is wholly untenable. In the affidavit filed by the
Assam Public Service Commission in W.P.(C) No.2755 of 2009 it has been mentioned
that if the select list is to be revised on the lines that had surfaced during the course of
hearing four (4) candidates in all would be out of reckoning. In this regard the name of
Alee Bora has been mentioned. In the original select list the name of the said candidate
appears against the cadre of Inspector of Excise. If Alee Bora was not entitled to be in
the select list of Inspector of Excise consequent to the revision thereof in accordance with
the order of the Court, the same does not mean that the candidate would be ineligible for
any other post even if her marks had permitted her inclusion in the list against any other
cadre. In the revised select list the name of Alee Bora appears in the list prepared for the
cadre of Inspector of Taxes. She had secured 848 marks and was selected as a female
candidate belonging to the OBC. The marks secured by her i.e. 848 did not entitle her to
be eligible for selection in the cadre of Inspector of Excise where the last candidate
belonging to the said category had secured 850 marks. However, the said marks i.e. 848
makes Alee Bora eligible for selection in the cadre of Inspector of Taxes. This is how the
name of Alee Bora appears in the revised select list against the cadre of Inspector of
Taxes.
9.
The above facts would demonstrate that not only the allegations made by the
petitioner are wholly unfounded and untenable, the same have been made without
minimum verification of the basic/essential facts. The marks secured by the petitioner
have been omitted from the writ petition. The writ petition indeed is frivolous and has
been filed without exercising the minimum care and caution. The intended result would
have cast an onerous burden both on the Public Service Commission and the Court. In
such circumstances, we are of the view that not only the writ petition should be
dismissed, a cost of Rs.5,000/- should be imposed on the petitioner.
10.
It is ordered accordingly.
***
THE GAUHATI HIGH COURT AT GUWAHATI
W.P. (C) NO.3054/2009
D.D. 06.11.2009
Hon’ble Mr. Justice Ranjan Gogoi
Hon’ble Mr. Justice B.P.Katakey
Dr. Hamen Bharali & Ors.
Vs.
The State of Assam & Ors.
…
Petitioners
…
Respondents
R.T.I. Act:
Whether answer scripts in the competitive examination held by Public Service
Commission is exempt from disclosure under Section 8(1)(e) of RTI Act on the ground of
fiduciary relationship? – No.
Applications of candidates for Combined Competitive (Main) Examinations held
in July 2008 were rejected by Public Service Commission (P.S.C.) on the ground that the
same are exempt from disclosure under Section 8(1)(e) RTI Act – Aggrieved by the same
the candidates approached Assam Information Commission (AIC) which as per order
dated 03.07.2009 directed the P.S.C. to furnish copies of answer scripts/marks sheets to
the candidates as applied by them – In the writ petition filed by P.S.C. against the said
order Gauhati High Court held that there is no fiduciary relationship involved attracting
exemption under 8(1)(e) of the Act and consequently dismissed the writ petitions.
Aggrieved by the same P.S.C. filed S.L.P. Nos.30858-30861/2009 and Supreme
Court as per order dated 11.12.2009 granted interim stay of the judgment of the High
Court till the next date of hearing – The said order is extended till further orders as per
order dated 12.3.2010.
Cases Referred:
AIR 1966 SC 1942 - B.N.Nagarajan & Ors., Etc. vs. State of Mysore & Ors. etc.,
(1984) 4 SCC 27 - Maharashtra State Board of Secondary and Higher Secondary
Education & Anr. vs. Paritosh Bhupesh kumar Sheth & Ors.
(1997) 4 SCC 306 - Dinesh Trivedi, M.P. & Ors. vs. Union of India & Ors.
(2004) 2 SCC 476 - People’s Union for Civil Liberties & Anr. vs. Union of India & Ors.
(2004) 6 SCC 714 - Pramod Kumar Srivastva vs. Chairman, Bihar P.S.C. Patna & Ors.
(2004) 13 SCC 383 - Board of Secondary Education vs. Pravas Ranjan & Anr.
(2007) 8 SCC 242 - Secy., W.B. Council of Higher Secondary Education vs. Ayan Das &
Ors.
ORDER
Ranjan Gogoi, J.
All the writ petitions having raised common questions of law on similar facts
were heard together and are being disposed of by this common judgment and order.
2.
The sole grievance raised in the writ petitions and the entitlement claimed by the
petitioners is with regard to their right to receive photocopies of the answer scripts in the
concerned subjects for which they had applied under the provisions of the Right to
Information Act, 2005 (hereinafter referred to as the Act). The examination in question
in which the petitioners had appeared is the Combined Competitive Examination, 2006
held by the Assam Public Service Commission (hereafter referred to as the Commission)
for recruitment to vacant posts in the Assam Civil Service, Class-I (Junior Grade) and
allied services. According to the petitioners, on the strength of an order dated 3.7.2009
passed by the State Information Commissioner, Assam, in a proceeding initiated by three
other persons (the petitioner No.2 in WP (C) 3054/2009, Sri Nilav Deva Goswami was a
party to the said proceeding) the State Information Commissioner had issued directions to
the Commission to furnish Xerox copies of the answer scripts to the candidates as and
when applications are made by any candidate. The petitioners have stated and averred
that pursuant to the said order dated 3.7.2009 they had applied for answer scripts of the
papers in which they have been shown to have secured low marks in the statement of
marks supplied to them by the Commission.
However, the Commission had not
furnished photocopies of the answer scripts applied for by the petitioners leaving them
with no option but to move this Court by way of the present writ petitions.
3.
In WP(C) No.3218 of 2009 wherein the Commission is the writ petitioner the
challenge is in respect of another order, also dated 3.7.2009, passed in the case of one
Swaraj Kumar Das who was also a candidate in the aforesaid examination. By means of
the order dated 3.7.2009 the Commission while issuing similar directions, as noted above
had reviewed its earlier order dated 2.4.2009 rejecting the claim of the applicant before it
i.e. Swaraj Kumar Das for being furnished with photocopies of the answer scripts.
According to the writ petitioner-Commission, the initial order dated 2.4.2009 rejecting
the claim of the applicant was passed on the basis of an order dated 23.4.2007 of the
Central Information Commission. However, subsequently the order dated 2.4.2009 was
reviewed by the Assam State Information Commissioner by means of the impugned order
dated 3.7.2009. In this regard, it is the broad contention of the Commission before this
Court that the State Information Commissioner being an authority under the statute had
not been vested with any power of review so as to enable the said authority to recall its
earlier order dated 2.4.2009. It is the further contention of the Assam Public Service
Commission in the writ petition filed before this Court that the exercise by way of review
was without any notice to the Public Service Commission.
4.
The above being the broad features of the case under consideration, recital of
which is considered sufficient for an effective adjudication thereof, the Court has
considered it unnecessary to burden this order with any other specific details, particularly
those pertaining to the marks secured by the candidate in the papers, photocopies of
which have been applied for. The Court may now proceed with the required adjudication
after briefly noticing the rival contentions advanced.
5.
The writ petitioners contend that the answer scripts constitute information within
the meaning of Section 2(f) of the Act and the right to receive such information within
the meaning of Section 2(j) of the Act, extends to a right to be furnished with the
photocopies of the answer scripts.
It is also contended that the Public Service
Commission is a public authority within the meaning of Section 2(h) of the Act obliging
the said authority to make available the required ‘information’ which has been denied in
the present case. The respondent Public Service Commission, who is also the writ
petitioner in WP (C) No.3218/2009, contends to the contrary.
According to the
Commission, the evaluation of the answer scripts of the candidates in the Combined
Competitive Examination, 2006 constitute information received in a fiduciary capacity
and, therefore, the exemption contained in Section 8(1)(e) of the Act dispenses the
obligation to make available photocopies of the answer scripts to the candidates who had
appeared in the said examination. According to the Commission, the right to information
is a facet of the freedom of speech and expression contained in Article 19(1)(a) of the
Constitution and such right is subject to reasonable restrictions that may be imposed
under Article 19(2). Referring to several judgments of the Apex Court, particularly in the
cases of (1) Maharashtra State Board of Secondary and Higher Secondary Education and
another vs. Paritosh Bhupesh kumar Sheth & others, reported in (1984) 4 SCC 27; (2)
Dinesh Trivedi, M.P. & others vs. Union of India and others, reported in (1997) 4 SCC
306; (3) People’s Union for Civil Liberties and another vs. Union of India and others,
reported in (2004) 2 SCC 476; (4) Pramod Kumar Srivastva vs. Chairman, Bihar Public
Service Commission, Patna & others, reported in (2004) 6 SCC 714; (5) Board of
Secondary Education vs. Pravas Ranjan & another, reported in (2004) 13 SCC 383; and
(6) Secy., W.B. Council of Higher Secondary Education vs. Ayan Das & others, reported
in (2007) 8 SCC 242 it is the contention of the Commission that to make available the
answer scripts to a candidate is to permit him to make a self assessment/evaluation of the
answers given by him which is not contemplated by law. According to the Commission,
such a course of action would inevitably delay the finalization of the results making the
system unworkable. According to the Commission, as public policy demands a finality
to the process of any examination it will be in public interest not to acknowledge the
claimed right in favour of the candidates. The preamble of the Act has also been referred
to lay stress on the fact that the right to receive information should be understood in a
balanced manner when such right is likely to come in conflict with different facets of
public interest including efficient operations of the authority. That apart, it is the case of
the Commission that in the writ petition filed no foundation has been laid to substantiate
the right claimed and therefore, the principles of law enunciated by the Supreme Court in
the cases referred to, the core of which have been noticed above, would also apply in the
case of a public examination conducted by a statutory or a constitutional body like the
Public Service Commission while adjudicating claims under the Right to Information
Act, 2005.
6.
The Right to Information Act, undoubtedly, is one of the most potent attempts in
recent years to promote democratic norms and principles. The definition of ‘information’
contained in Section 2(f) is in the widest of terms and must be understood in the light of
the objects sought to be achieved by the legislation in question. Section 6(2) of the Act
even dispenses with the requirement of locus standi; in any case the petitioners
(candidates) in the present case do not suffer from any absence thereof. The right to
receive information vested by the Act enable the decisions taken as well as the decision
making process involved at different levels of the Govt. to be scrutinized by a citizen.
The objection of the Act, therefore, is to enable citizens to scrutinize all Government acts
(except what has been specifically exempted by the Act) so as to ensure greater
accountability and transparency in governmental functioning. The reasonable restrictions
contemplated by Article 19(2) or the balancing act between the right to receive
information and public interest as reflected in the preamble finds manifestation in Section
8(1) of the Act which exempts a public authority from the obligation of disclosure in
cases covered by sub-clauses (a) to (j) of Section 8(1) of the Act.
7.
Having outlined the broad features of the Act and having identified the object that
the Act seeks to achieve the Court may now proceed to analyse and understand the
specific stand taken by the Commission that it is absolved from the obligation to furnish
photocopies of the answer scripts to the candidates as the same is exempted under Section
8(1)(e) of the Act.
8.
Section 8(1)(e) of the Act is in the following terms:
“8.
Exemption from disclosure of information.- (1) Notwithstanding
anything contained in this Act, there shall be no obligation to give any
citizen.….
….
….
(e)
information available to a person in his fiduciary relationship,
unless the competent authority is satisfied that the larger public interest
warrants the disclosure of such information.”
9.
The expression ‘fiduciary relationship’ is defined in the Black’s Law Dictionary
to mean “A relationship in which one person is under a duty to act for the benefit of the
other on matters within the scope of the relationship. Fiduciary relationships – such as
trustee-beneficiary, guardian-ward, agent-principal, and attorney-client – require the
highest duty of care. Fiduciary relationships usu. Arise in one of four situations: (1)
when one person places trust in the faithful integrity of another, who as a result gains
superiority or influence over the first, (2) when one person assumes control and
responsibility over another, (3) when one person has a duty to act for or give advice to
another on matters falling within the scope of the relationship, or (4) when there is a
specific relationship that has traditionally been recognized as involving fiduciary duties,
as with a lawyer and a client or a stockbroker and a customer.”
On the other hand, the word ‘fiduciary’ has been defined in the same Dictionary (Edition)
to mean “1. One who owes to another the duties of good faith, trust, confidence, and
candor <the corporate officer is fiduciary to the shareholders>. 2. One who must exercise
a high standard of care in managing another’s money or property.”
10.
From the above it is clear that a fiduciary relationship comes into existence when
one person places his complete confidence in another in regard to any particular matter.
In the present case under the Regulations pertaining to holding of examinations by the
Commission and the practice followed (noticed in the order dated 1.9.2009 passed in
WP(C) No2755/2009) answer scripts of the candidates are required to be evaluated by
examiners out of a prepared panel or list of such examiners. On completion of the
examination of the answer scripts the evaluated answer scripts are again required to be
scrutinized by moderators, and if so required, finally, by a head examiner. Given the
scheme of the examination of answer scripts as laid down in the Regulations in force and
the procedure followed in practice it is difficult to understand how any fiduciary
relationship between the Public Service Commission and the examiners can be
understood to have come about so as to justify invocation of the exemption from
disclosure contemplated by Section 8(1)(e) of the Act.
11.
Who amongst the persons that are assigned the task of evaluating the answer
scripts of the candidates by the Commission i.e. that the initial examiner, the moderator
or the head examiner is the person in whom the Public Service Commission had placed
its utmost trust and confidence is not known. In a given case the marks awarded by the
initial examiner may be the final award of marks. In another, the marks given by the
same examiner may be superseded by those awarded by the moderator and in the third
case it may the marks as finally awarded by the head examiner which may be the final
determination. Under the Regulations of the Public Service Commission and the practice
followed there is no hard and fast rule to ensure that every answer script undergoes the
aforesaid process i.e. scrutiny by the initial examiner, then by the moderator and finally
by the head examiner. Much would depend on the facts of a given case. The person
whose award of marks is final in any given case, therefore, remains uncertain. If that be
so, the identity of the repository of the trust requiring reciprocal duty of utmost good faith
is absent. That apart, keeping in view the objective that the Act seeks to achieve, the
Court will have no hesitation in holding that the spirit of the Act enjoins disclosure of
information as the general rule and exemption therefrom as an exception. In such a
situation, making available the answer scripts without disclosing the identity of the
person(s) whose award of marks has been construed to be the final determination in any
given case, a situation contemplated by Section 8(1)(g) of the Act, can easily take care of
the situation. If the requirement of disclosure of information can be met by adequately
taking care that the identity of the examiner is not disclosed such an interpretation must
be adopted and the more drastic course of action i.e. to recognise an exemption must be
avoided.
12.
An argument has been advanced on behalf of the Public Service Commission that
leaving aside the specific exemptions enjoined by the provisions of the Act the
requirement of disclosure can still be balanced by the authority on a fine scale wherein
competing public interests can be weighed along with the necessity of the disclosure. We
are afraid we cannot subscribe to the aforesaid proposition. In B.N.Nagarajan and others,
Etc. vs. State of Mysore and others, etc., reported in AIR 1966 SC 1942, the Constitution
Bench of the Apex Court has observed:
“We see nothing in the terms of Article 309 of the Constitution which
abridges the power of the executive to act under Article 162 of the
Constitution without a law. It is hardly necessary to mention that if there
is a statutory rule or an act on the matter, the executive must abide by that
act or rule and it cannot in exercise of the executive power under Article
162 of the Constitution ignore or act contrary to that rule or act.”
If the rights of the citizen to receive information and the situations in which the
obligation to furnish such information has been dispensed with by the provisions of a
comprehensive statute, we do not see how the respective rights and obligations of the
State vis-à-vis its citizens can be measured by any other yardstick except by reference to
the provisions of the statute. On the above conclusion that we have reached it will not be
necessary for us to refer to the details of the cases cited by the learned counsel for the
Public Service Commission in support of the plea that furnishing of answer scripts of the
candidates would not be in public interest.
13.
The above would require the Court to consider the specific plea advanced on
behalf of the Public Service Commission that the order dated 3.7.2009 passed by the
State Information Commissioner in the matter of Swaraj Kr. Das will not be authorized in
the absence of any power of review vested in the said authority by the provisions of the
Act. While the learned counsel for the Commission may be correct in raising the issue,
yet, in our considered view, the question has become largely academic in view of the fact
that irrespective of the order dated 3.7.2009 passed by the State Information
Commissioner in the matter of Swaraj Kr. Das, the views recorded therein finds
manifestation in another order of the same date i.e. 3.7.2009 passed by the State
Information Commissioner in a separate proceeding involving other candidates including
the petitioner No.2, Sri. Nilav Deva Goswami in WP (C) No.3054/2009. It is on the basis
of the views recorded in the said independent proceeding that the direction has been
issued to the Commission to furnish to all candidates in the public examinations held by
it Xerox copies of the answer scripts that may have been or may be applied for. Even if
we are to technically set aside the review order dated 3.7.2009 passed by the State
Information Commissioner in the case of Swaraj Kr. Das, on the conclusions that we
have reached, the views recorded by the State Information Commissioner in the order of
the same date passed in the case of other candidates and the consequential directions
issued by the said authority will not be liable to any interference.
14.
We, therefore, allow the writ petitions i.e. WP(C) Nos.3054/2009, 3382/2009 and
3702/2009 and dismiss WP (C) No.3218/2009 filed by the Assam Public Service
Commission leaving the parties to bear their own costs.
***
PROCEEDINGS OF THE ASSAM INFORMATION COMMISSION
KP(M) 28/2010
D.D. 26.02.2010
Sri. D.N.Dutt, Assam State Chief Information Commissioner
Shri Srikana Kalita
…
Vs.
Secretary, APSC, Guwahati …
Complainant
Public Authority/PIO
R.T.I. Act:
In the recruitment to the post of lecturer in English in Government Colleges the
complainant sought several items of information – Public Information Officer informed
the complainant that the information at Sl.Nos.(i) to (iv) relating to educational
qualification of the selected candidates could not be furnished as the relevant documents
were sent to the Appointing Authority along with the select list – Information at
Sl.No.(vi) viz., the names of the Members and Experts of the Interview Board was
declined on the ground that the same was exempted from disclosure under Section
8(1)(g) RTI Act – PIO furnished information at Sl.Nos.(v) & and (vii) viz., interview
marks of two selected candidates and total marks obtained by them in various papers –
The Information Commission held that the PIO may intimate the complainant the names
of the Members of the Interview Board – But as far as the names of Experts of the said
Board are concerned Information Commission upheld the decision of the Commission
that the said information is exempt from disclosure under Section 8(g) of the Act.
ORDER
Both the petitioner and the respondents are present. The brief of the case is that
the petitioner, Shri Srikanta Kalita had submitted a RTI application dated 18.9.09
followed by another petition dated 19.10.09 seeking information from the SPIO of the
office of the APSC with regard to the selection of candidates for the post of lecturer in
English in the Govt. Colleges/Institutions of Assam under the Education (Higher)
Department, Govt. of Assam. The information sought for point-wise is as follows:
1. The mark sheets (of HSLC, HSSLC, BA, MA) of both the candidates (and
M.Phil., Ph.D. if any).
2. Experience certificates of teaching of both the selected candidates, along
with the level (UG/PG) of teaching.
3. Specialisations, (certificate, diploma course of the two candidates) if any,
and from which university/institution (along with mark sheets and
certificates)
4. Certificates of NET/SLET of the two candidates.
5. Interview marks of the two candidates (along with that of Jovial Kalita).
6. Names of the Members and the Experts of the Interview Board, of both the
candidates selected (along with that of jovial Kalita).
7. Total marks (item wise) obtained by the two selected candidates (along
with that of Jovial Kalita) in (1) HSLC, (b) HSSLC, (c) BA, (d) MA, (e)
NET/SLET/M.Phil./Ph.D., (f) Service/Experience, (g) Additional
Qualification, (h) General Bearing (i) Knowledge of the subject.
The SPIO, APSC, Shri A.K. Das had informed the applicant that the required
information at Sl.(i) to (iv) could not be furnished as the applications of the selected
candidates with all the relevant documents sere sent to the Education (Higher)
Department along with the recommendation list. The applicant was further informed that
the information at Sl.(vi) (viz. the names of the members and experts of the Interview
Board) was exempted from disclosure under Section 8(i)(g) of the RTI Act, 2005 and that
the information at Sl. (v) and (vii) were furnished as sought for.
The Commission is of the considered view that since the information asked for
vide Sl.(i) to (iv) could not be furnished by the APSC and had instead been transferred to
the Education (Higher) Department for furnishing the same, the petitioner is advised to
take up the matter with the Education (Higher) Department, Govt. of Assam. The
Commissioner & Secretary, Education (Higher) Department is requested to consider the
petitioner’s case and furnish the information if not exempt from disclosure as third party
information under Section 11(1) of the RTI Act, 2005.
The Commission is also of the view that the SPIO, APSC may intimate to the
petitioner, the names of the Members of the Interview Board. With regard to the names
of the Experts of the said Board, however, the Commission accepted in principle the
contention of the SPIO, APSC as well as the Secretary, APSC, Shri Sunil Dutta that this
information was covered by the provisions of Section 8(g) of the RTI Act, 2005 and
hence exempt from disclosure under the said Act. The petitioner also wanted certain
clarifications with regard to the information furnished against Sl.(v) of his petition. The
SPIO, APSC is directed to furnish the required clarifications to the petitioner accordingly
with intimation to the Commission.
Copy of his order be sent to both the parties.
***
BIHAR PUBLIC SERVICE COMMISSION
IN THE HIGH COURT OF JUDICATURE AT PATNA
CWJC NO.577 OF 2008
D.D. 22.02.2008
Hon’ble Mr. Justice Chadramauli Kr. Prasad &
Hon’ble Mr. Justice Jayanandan Singh
Digvijay Kumar
…
Vs.
The Chairman, BPSC & Ors. …
Petitioner
Respondents
Examination:
Whether a candidate is entitled to inspect the original answer sheets? – No.
The petitioner an unsuccessful candidate for appointment to the post of Civil
Judge has filed this writ petition for a direction to respondent to permit inspection of the
answer sheets of two papers viz., Law of Contract and torts and Elementary General
Science alleging that he has not been awarded marks as expected – High Court in view of
the fact that there is no provision for permitting a candidate to inspect the answer sheets,
has dismissed the writ petition.
ORDER
This application has been filed for issuance of a writ in the nature of mandamus
commanding the respondent to inspect the original answer sheets of two papers i.e. Law
of Contracts and torts as well as Elementary General Science.
Petitioner appeared in the examination conducted by the Bihar Public Service
Commission for appointment to the post of Civil Judge. On the basis of the marks
obtained by him in the written examination, he was called for viva voce test but
ultimately he has not been selected. According to him, he had performed well in the Law
of Contracts and torts and Elementary General Science but he has not been awarded
marks as expected by him. Accordingly his prayer is for issuance of a direction to the
respondent to permit inspection of the answer sheets of the aforesaid two papers.
When question as to which provision entitles the petitioner to inspect the answer
sheets, Mr. Ajit Kumar Singh, appearing on behalf of the petitioner is unable to point out
any.
Mr. Sanjay Pandey, appearing on behalf of the respondent submits that there is no
provision for permitting a candidate to inspect the answer sheets.
In view of aforesaid, we are of the opinion that the prayer made by the petitioner
is without any merit and deserves to be rejected.
Application stands dismissed.
***
CHHATTISGARH PUBLIC SERVICE COMMISSION
HIGH COURT OF CHHATTISGARH, BILASPUR
WRIT APPEAL NO.241 OF 2009
D.D. 21.08.2009
Hon’ble Shri Dhirendra Mishra &
Hon’ble Shri R.N.Chandrakar, JJ
Dr. Prakash Chandra Prasad …
Vs.
The Secretary (Examn.),
Chhattisgarh PSC & Ors.
…
Appellant/Petitioner
Respondents
Qualification:
Rejection of application for not possessing the qualification prescribed before the cutoff
date is held justified.
For the post of Ayurvedic Medical Officer Clause 10(b) of the advertisement
mentioned that certificates with respect to educational qualification, experience of the
candidates and requisite experience certificate should be compulsorily produced on the
last date for filing the application i.e. 28.10.2006 – The requisite certificate produced by
the petitioner was issued only on 30.10.2006 hence his application was rejected – Writ
petition filed by the petitioner was dismissed by the learned single Judge - This writ
appeal filed by the petitioner was also dismissed.
Held:
Rejection made by the PSC in respect of the petitioner is just and proper and does
not warrant any interference.
ORDER
This writ appeal is directed against the order dated 26.6.2009 passed in Writ
Petition (S) No.6029 of 2007, whereby the petition preferred by the petitioner has been
dismissed and it has been held that that it cannot be said that the petitioner has obtained
the internship certificate as on 28.10.2006.
The rejection order passed by the
respondent/Public Service Commission (for short, “PSC”) in respect of the petitioner is
just and proper and the same does not warrant any interference.
From perusal of the pleadings in the writ petition, it appears that the petitioner
applied for the post of Ayurvedic Medical Officer. The application of the petitioner was
rejected by the respondent/PSC on the ground that he does not possess the requisite
qualification certificate.
On 28.9.2007 interim order was passed in favour of the
petitioner and the respondent/Public Service Commission was directed to allow the
petitioner to appear in the interview as a special case on 29.9.2007. However, the
petition was finally dismissed by the impugned order on the ground that the petitioner did
not fulfill the criteria mentioned in Clause 10(c) (which should have been Clause 10(b) )
of the advertisement as he did not possess the requisite certificate of the qualification on
the last date i.e., 28.10.2006 and the certificate filed by him was issued to the petitioner
only on 30.10.2006.
Learned counsel for the appellant submits that the appellant has already appeared
in the examination on the strength of the interim order passed in his favour by this court,
however, his result has not been declared and the respondent/PSC ought not to have
rejected the application of the petitioner on technical ground that the petitioner did not
possess the requisite certificate on the last date for applying to the post.
We have heard learned counsel for the petitioner, perused the impugned order and
also perused the Clause 10(b) of the advertisement which clearly mentions that
certificates with respect to the educational qualification, experience etc., of all the
candidates, necessary for the corresponding post and the requisite experience certificate
should be compulsorily produced on the last date for filing of the application.
In view of the above condition in the advertisement, we are of the opinion that the
learned single Judge has rightly held that the rejection order passed by the
respondent/PSC in respect of the petitioner is just and proper and the same does not
warrant any interference.
We find no substance in this writ appeal and the same deserves to be and is
accordingly dismissed.
***
HIGH COURT OF CHHATTISGARH, BILASPUR
WRIT PETITION (S) NO.4636 OF 2009
D.D. 09.09.2009
Hon’ble Mr. Justice S.K.Agnihotri
Amita Singh Thakur
Vs.
C.G. P.S.C. & Anr.
…
Petitioner
…
Respondents
Age Limit:
Cutoff date for determination of age is the date specified in the notification.
PSC issued an advertisement in September 2008 inviting applications for various
posts from eligible candidates – The Cutoff date for determination of age of the candidate
was fixed as 1.1.2009 in the advertisement – The petitioner purchased application for
appearing for the preliminary examination from the Postal Department – In the receipt of
the application form issued by Postal Department the cutoff date was mentioned as
1.1.2010 – The petitioner appeared for preliminary examination and qualified for main
examination and at that stage the application of the petitioner was rejected on the ground
of overage – PSC took necessary steps for making corrections with regard to the cutoff
date through press release, website etc. – In view of this fact the writ petition was
dismissed.
Held:
If there is any typographical error/clerical mistake, PSC has every right to correct
the same within the stipulated period and the candidates cannot take advantage of the
mistake.
ORDER
Heard learned counsel for the parties.
2.
The petitioner, by this petition, seeks quashing of the order dated 17.8.2009
(Annexure-P/1) passed, in respect of the petitioner, by the Under Secretary
(Examination), Chhattisgarh Public Service Commission, through Rojgar Aur Niyojan
and also prays that the petitioner may be allowed to appear in the main examination,
which is going to be held on 12th & 13th September, 2009.
3.
The facts, in nutshell, as projected by the petitioner, are that the respondent –
Chhattisgarh Public Service Commission (for short “the PSC”) issued an advertisement
in the month of September, 2008 inviting applications for various posts from the eligible
candidates. In the advertisement, the cut off date for determination of the age of the
candidates has been fixed as 1.1.2009, which is evident from Annexure – P/2. The
petitioner purchased the application form to appear in the preliminary examination from
the Postal Department. In the receipt of the application form issued by the Postal
Department the cut off date has been mentioned as 1.1.2010, which is evident from
Annexure – P/3.
4.
Thereafter, the petitioner applied and appeared in the preliminary examination.
She qualified in the said examination.
Before calling the candidates for main
examination, the PSC scrutinized the application forms and in that process, the
application form of the petitioner has been rejected and declared the petitioner ineligible
to appear in the main examination on the ground that the petitioner has not attained the
age of 21 years as on 1.1.2009. Thus, this petition.
5.
Shri. Otwani, learned counsel appearing for the petitioner, submits that after
succeeding in the preliminary examination, the PSC declared the petitioner as ineligible
to appear in the main examination on the ground that the petitioner has not attained the
age of 21 years as on 1.1.2009 whereas in the receipt provided by the Postal Department
the cut off date has been fixed as 1.1.2010. The action of the PSC is illegal, arbitrary and
unreasonable. Shri Otwani further submits that due to the inaction on the part of the PSC,
the petitioner will suffer a great hindrance in the path of advancement of her career.
6.
Per contra, Shri Sharma, learned counsel appearing for the PSC submits that
inadvertently in the receipt issued by the Postal Department at the time of issuance of
application form, the cut off date has been mentioned as 1.1.2010. However, when the
aforesaid mistake came into the knowledge of the authorities of the PSC by a press
release dated 26.9.2008 (Annexure-R/1) the same has been rectified as 1.1.2009. The
PSC requested the Postal Department to delete the cut off date i.e. 1.1.2010 from the
receipt of the application form. Even the same has also been published in the various
newspapers as also uploaded in the website of the PSC, which is evident from Annexures
– R/2 & R/3. The cut off date i.e. 1.1.2009 has been fixed by the PSC after following due
process of law and as per the Rules and Regulations and the PSC never intends to change
the cut off date i.e from 1.1.2009 and 1.1.2010.
7.
I have heard learned counsel appearing for the parties, perused the pleadings and
the documents appended thereto. Admittedly in the advertisement issued by the PSC, the
cut off date for determination of the age of the candidates has been fixed as 1.1.2009 and
on the basis of that the petitioner purchased the application form. Though in the receipt
issued by the Postal Department, the cut off date has wrongly been mentioned as
1.1.2010, the PSC immediately took necessary steps for making necessary corrections
with regard to cut off date through various means viz., press release, website etc. The
petitioner has applied before the PSC as per the terms and conditions of the advertisement
only and not on the basis of the receipt issued by the Postal Department, therefore, she
cannot take the advantage of one mistake committed by the PSC as her right. If there is
any typographical error/clerical mistake, the PSC has every right to correct the same well
within the stipulated period and the same has been done.
8.
For the reasons mentioned hereinabove, there is no illegality or infirmity in
declaring the candidature of the petitioner as ineligible on the ground of under age i.e.
below 21 year as on 1.1.2009. The same is just, proper and warranting no interference by
this Court.
9.
In the result, the petition is liable to be and is hereby dismissed at the motion
stage itself.
***
HIGH COURT OF CHHATTISGARH, BILASPUR
WRIT PETITION (S) NO.4151 OF 2009
D.D. 26.10.2009
Hon’ble Shri Satish K. Agnihotri, J
Sangharsh Kumar Mishra
Vs.
Chhattisgarh PSC & Anr.
…
Petitioner
…
Respondents
Recruitment:
Whether prescribing qualifying test (screening test) after commencement of selection
process amounts to change of selection process? – No.
PSC notified for recruitment to the posts of Drug Inspector as per notification
dated 26.11.2008 specifying the mode of selection on the basis of educational
qualification, experience and interview – Subsequently as per notification dated 6.7.2009
having regard to the number of applications received viz., 2313 for total 55 posts PSC by
notification dated 6.7.2009 prescribed qualifying written test and thereafter calling for
interview three times the number of seats available – The petitioner challenged
subsequent notification alleging that the subsequent notification was a clear change in the
selection process – High Court in view of Clause 12.1 of the earlier notification dated
26.11.2008 which provided for conducting qualifying written examination for shortlisting of candidates held that there was no irregularity in holding written test and
consequently dismissed the application.
Held:
Even in the absence of statutory provision for holding screening test the same can
be held on the basis of administrative instructions for the purpose of elimination and short
listing of huge number of candidates provided the action is otherwise bona fide and
reasonable. [The decision in Tridip Kumar Dingal & Ors. vs. State of West Bengal &
Ors. - (2009) 1 SCC 768 followed.]
Cases Referred:
(2009) 1 SCC 768. - Tridip Kumar Dingal & Ors. vs. State of West Bengal & Ors.
JUDGMENT
Challenge in his petition is to the notification dated 6.7.2009 (Annexure - P/1)
whereby the written test for qualifying and thereafter calling for three time of the seats
available to interview on the basis of result of written test was provided in a selection
process for appointment on the posts of Drug Inspectors & Food Inspectors.
2.
By notification dated 26.11.2008 (Annexure – P/2) published in Rojgar Aur
Niyojan, applications were invited for appointment on the posts of Drug Inspectors, Food
Inspectors and Drug Analyst (Second) in the Department of Health and Family Welfare
by the Chhattisgarh Public Service Commission (for short “the PSC”).
3.
Clause 12 of the notification dated 26.11.2008 provides for selection process. In
clause 12.1 it was clearly provided that the candidates having required educational
qualification and experience would not be entitled to interview directly in the event
applications received are more in number, the PSC may undertake selection process on
the basis of higher qualification or experience or both or on the basis of written
examination for short listing of the candidates.
4.
Accordingly, after receiving the applications, on 6.7.2009 the notification was
issued for holding written examination on objective basis. The syllabus was also notified
in the notification dated 6.7.2009. Being aggrieved the petitioner has filed this petition,
impugning the prescription for written test by subsequent notification dated 6.7.2009.
5.
Shri Tripathi, learned counsel appearing for the petitioner, would submit that the
subsequent notification is a clear change in the selection process, as in earlier notification
the test was to be made on the basis requisite qualification, training and experience. The
written test was provided, without processing the applications, on the basis of receipt of
number of applications, provision for written test cannot be done and the same is invalid
and unconstitutional.
6.
On the other hand, Shri Kachhawaha, learned counsel appearing for the PSC,
would submit that the petitioner applied for the post of Food Inspector. On receipt of
more than 2313 applications for total 55 posts, as notified in the earlier notification dated
26.11.2008, having regard to the number of applications, it was decided in the light of
clause 12.1 of the notification dated 26.11.2008 to have a written test for short listing of
the candidates and thereafter three times of the posts available would be called for
interview on the basis of strict merit list prepared on the basis of written test. Thus, there
is no irregularity or illegality in the selection process.
7.
In support of his contention, learned counsel appearing for the PSC placed his
reliance upon the decision of the Supreme Court rendered in Tridip Kumar Dingal and
Others vs. State of West Bengal and Others – (2009) 1 SCC 768.
8.
I have heard learned counsel appearing for the parties, perused the pleadings and
the documents appended thereto.
Reading both the notifications together, as the
subsequent notification was in continuation of earlier notification, which cannot be held
that the rules of the game have been changed after the game was played. Short listing of
the candidates was proposed to be done on the basis of number of applications received
as was clearly provided in clause 12.1 of the earlier notification dated 26.11.2008.
9.
In Tridip Kumar Dingal (supra), the Supreme Court observed as under:
“38. Having heard learned counsel for the parties, in our opinion, the
appeals deserve to be partly allowed. The contention on behalf of the
State Government that written examination was for short listing the
candidates and was in the nature of “elimination test” has no doubt
substance in it in view of the fact that the records disclose that there were
about 80 posts of Medical Technologists and a huge number of candidates,
approximately 4000 applied for appointment. The State authorities had,
therefore, no other option but to “screen” candidates by holding written
examination. It was observed that no recruitment rules were framed in
exercise of the power under the proviso to Article 309 of the Constitution
and hence no such action could be taken. In our opinion, however, even in
absence of statutory provision, such an action can always be taken on the
basis of administrative instructions-for the purpose of “elimination” and
“short listing” of huge number of candidates provided the action is
otherwise bona fide and reasonable.”
The action of the State authorities was upheld by the Supreme Court in Tridip Kumar
Dinga (supra).
10.
In the present case there was a clear provision in the first notification dated
26.11.2008 for holding written test in case number of applications received was more. It
is evident that for 55 posts, 2313 applications were received, therefore, there is no
irregularity in holding the written test and, as such, calling for interview for three times
of the posts available on the basis of written test. The contention of the learned counsel
for the petitioner, as stated above, has no merit and the same is noted to be rejected.
11.
Looking from all angles and for the reasons mentioned hereinabove, there is no
merit in this petition. The writ petition is accordingly dismissed. No order as to costs.
***
HIGH COURT OF CHHATTISGARH, BILASPUR
WRIT APPEAL NO.57 OF 2009
D.D. 27.10.2009
Hon’ble Shri Sunil Kumar Sinha and
Hon’ble Shri R.L.Jhanwar, JJ
Chhattisgarh P.S.C.& Anr.
…
Vs.
Dr.Smt. Mamta Singh Sengar & Ors. …
Appellants
Respondents
Recruitment:
Whether application dispatched by speed post before the last date for receipt of
applications but received by PSC after the last date is liable for rejection? – Yes.
The petitioner’s application for the post of Ayurvedic Medical Officer was to be
submitted before the last date i.e., 5.7.2008 – But the same sent by speed post on
1.7.2008 was delivered in the office of PSC on 7.7.2008 – Hence it was rejected –
Aggrieved by the same the petitioner filed writ petition which was allowed – Against the
said order this writ appeal was filed and as per interim order the direction issued in writ
petition to permit the petitioner to appear for the examination was stayed – As the
petitioner could not appear for the examination which had already taken place on
26.4.2009 writ appeal was allowed and the order of learned single Judge was set aside.
Held:
Normally such reliefs which are in the nature of final relief are not granted by the
Courts at interim stages.
ORDER
The appellants/respondents No.2 & 3 in the Writ Petition have filed this appeal
against the order dated 07.01.2009 passed in W.P. (S) No.6943/2008 by the learned
Single Judge of this Court. By the said order, the learned Single Judge, after hearing the
parties, directed that the petitioner’s (respondent No.1) application shall be duly
processed and if found in order, she shall be permitted to appear in the selection process.
2.
A perusal of the writ petition would show that the vacancies were advertised for
selection to the post of Ayurvedic Medical Officer by the Chhattisgarh Public Service
Commission on 28.05.2008. As per the advertisement, the application for the said post
was to be submitted upto 05.07.2008.
According to the petitioner, she sent her
application on 01.07.2008 by speed post from Baikunthpur which was delivered in Public
Service Commission’s Office at Raipur on 07.07.2008.
Since the application was
delivered in the Office of the Public Service Commission after cut off date of 05.07.2008,
therefore, the application of respondent No.1 was rejected and a communication dated
16.10.2008 (Annexure P/1 in the writ petition) was served upon respondent
No.1/petitioner.
After receiving the said communication, respondent No.1/petitioner filed a Writ
Petition for the following reliefs:
“10.1.
This Hon’ble Court may graciously be pleased to issue a writ of
mandamus/certiorari to quash the rejection letter dated 16.10.08
(Annexure P-1) and further to direct the Respondent No.1 to 3 to accept
the application of the petitioner allowing her to appear in the examination
of Ayurvedic Medical Officer.
10.2.
That a command/direction may kindly be issued to take the appropriate
action against the delinquent employee of the Postal Department calling
for the entire records pertaining to the case of the petitioner granting
compensation in a tune of Rs.1 Lac to the petitioner from the Postal
Department.
10.3.
Any other relief which this Hon’ble Court may deems fit and proper also
be awarded to the petitioner including the cost and expenses of the
petition.”
It is in the said Writ Petition the learned Single Judge passed the impugned order
and made directions as above.
3.
The said order was challenged before the Division Bench in the present Writ
Appeal and the Division bench passed an interim order on 04.03.2009 by which the effect
and operation of the impugned order dated 07.01.2009 was stayed until further orders.
Thereafter, the respondent No.1 filed another application i.e. I.A.No.3/2009 for grant of
permission to appear in the examination scheduled to be held on 26.04.2009. The said
application (I.A.No.3/2009) was also dismissed by the Division Bench on 23.04.2009.
As an effect of the aforesaid orders passed by the Division Bench in this Writ Appeal, the
application form of respondent No.1/petitioner in the Writ Petition was not processed and
ultimately she was not allowed to appear in the examination which was conducted on the
scheduled date i.e. 26.04.2009.
A perusal of the Writ Petition and the relief granted in the impugned order would
show that the relief granted vide impugned order dated 07.01.2009 is the main relief
sought by the petitioner/respondent No.1 in para 10.1 of the Writ petition and normally
such reliefs which are in the nature of final relief are not granted by the Courts at interim
stages.
4.
In view of the above and in the prevailing facts and circumstances of the case,
when the examination has already taken place on 26.04.2009 and respondent
No.1/petitioner could not appear in the examination, we deem it appropriate to set aside
the impugned order passed by the learned Single Judge.
5.
Accordingly, the Writ Appeal is allowed and the impugned order dated
07.01.2009 passed by the learned Single Judge is set aside. There shall be no order as to
costs.
***
BEFORE THE HIGH COURT OF CHHATTISGARH AT BILASPUR
WRIT PETITION NO.2549/2009 & CONNECTED CASES
D.D. 15.03.2010
Hon’ble Shri Satish K.Agnihotri &
Hon’ble Shri Manindra Mohan Shrivastava, JJ
Dr. Ranjit Kumar Guru
…
Vs.
State of Chhattisgarh & Ors. …
Petitioner
Respondents
Recruitment:
Age Relaxation:
Whether persons working on contract basis can claim in a direct recruitment age
relaxation or separate examination for them reserving quota? – No.
Petitioners who were originally appointed as Ayurveda Chikitsa Adhikari on
contract basis for a period of 3 years and continued subsequently challenged the
recruitment to the posts of Ayurveda Chikitsa Adhikari initiated as per advertisement
dated 22.5.2008 – It was among others sought by the petitioners that reservation for 50%
of posts for those working on contract basis, increase in age limit, grant of weightage etc.
– Court after considering several decisions relied upon by the parties rejected the
applications of the petitioners in view of the decision in Secretary, State of Karnataka &
Ors. vs. Umadevi & Ors. (2006) 4 SCC 1 and dismissed the writ petition.
Held:
Unless the petitioners are eligible under the rules, they cannot claim as of right,
participation in the process of recruitment or consideration of their candidature.
Further held:
Prescription of lower and upper age limit is essentially legislative function of the
Governor in exercise of Constitutional Authority under Article 309 of the Constitution.
In the absence of violation of any Constitutional provisions, the Court would not issue
any command to the Rule Making Authority to prescribe any particular lower or upper
age limit for recruitment to any post under public employment. It is the exclusive domain
of the Rule Making Authority.
Cases Referred:
(1990) 2 SCC 707 - Mallikarjuna Rao & Ors. Vs. State of Andhra Pradesh & Ors.
1992 Supp (2) SCC 318 - R.S.Bansal & Ors. Vs. Union of India & Ors.
(1998) 3 SCC 495 - A.B.Krishna & Ors. Vs. State of Karnataka & Ors.
(1998) 6 SCC 165 – State of M.P. & Anr. V. Dharan Bir
AIR 2001 S.C. 308 - Kuldeep Kumar Gupta & Ors vs. Himachal Pradesh State Electricity
Board & Ors.
(2003) 2 SCC 632 - P.U. Joshi & Ors. V. Accountant General, Ahmedbad & Ors.
AIR 2005 SC 3180 - Jacob Mathew vs. State of Punjab & Anr.
(2009) 6 SC 611 - Mohd. Abdul Kadir & Anr. vs. Director General of Police, Assam &
Ors.
ORDER
Per Manindra Mohan Shrivastava, J.
In this batch of petitions filed under Article 226 of the Constitution of India,
common question of law and facts arise for consideration at the instance of the
petitioners, who are all working as contract Ayurveda Chikitsa Adhikari in various
Ayurvedaic Hospitals in the State.
2.
Petitioners in all the petitions are contract Ayuveda Chikitsa Adhikari. In order to
cope-up with requirement of post of Ayurveda Chikitsa Adhikari for various Ayurvedic
Hospitals in the State, appointments were made by the Chief Executive Officer of various
Janpad Panchayat as also Collector pursuant to Government decision circulated vide
Circular No.1459/152/01 dated 5.4.2002 issued by the Department of Public Health and
Family Welfare and Medical Education, Govt. of Chhattisgarh. Petitioners in these three
petitions are those were appointed in various Ayurvedic Hospitals/Dispensaries as
Ayurveda Chikitsa Adhikari on contract basis and on fixed pay of Rs.8,000/- per month
for a period of 3 years. The appointment orders of all the petitioners which have been
placed on record show that all these petitioners were appointed on contract basis and their
services could be terminated by giving one month notice or in lieu thereof one month
salary upon completion of period of contract service.
3.
Petitioners in the aforesaid petitions were appointed on various posts in the year
2003, initially for a period of 3 years. Later on, as the need continued and regular
selection could not be made as per the recruitment rules, services of these petitioners
were continued from time to time.
4.
For the purposes of making regular appointment of 106 posts of Ayurveda
Chikitsa Adhikari, process of recruitment was set in motion in accordance with the
M.P./C.G. Public Health (Indian System of Medicine and Homoeopathy (Gazetted)
Service Recruitment Rules, 1987 (hereinafter referred to as ‘the Recruitment Rules of
1987’) by issuing advertisement dated 22.5.2008.
Recruitment Rules of 1987 were
amended vide notification dated 5.7.2008. Thereafter, a Corrigendum was issued on
17.7.2008 published in the Newspaper on 23.7.2008 (Annexure R-2/1) filed in W.P.(S)
No.1990/09. By amendment made in the Recruitment Rules of 1987, one time relaxation
of age up to 5 years was granted to medical officers working or possessed of experience
having worked in the Government, Semi-Government and other recognized Medical
Institutions. The number of post for which selection were to be made were increased to
297 vide Public Service Commission advertisement dated 31.12.2008 in continuation of
earlier advertisement dated 22.5.2008, 17.7.2008 and 21.8.2008.
5.
In all the three petitions, petitioners are those Ayurveda Chikitsa Adhikaris who
were appointed on contract basis and have been continuing in service for the last 6 years
on contract basis. They all are commonly aggrieved by the eligibility criteria laid down
in the advertisement dated 22.5.2008. In all the three petitions, petitioners have prayed
that the advertisement dated 22.5.2008 (Annexure P-1 in W.P.(S) 1746/09) be declared
illegal.
In W.P. (S) 1746/09 & 2549/09, petitioners have also challenged the
Constitutional validity of provision contained in Rule 7(1) of the M.P./C.G. Civil
Services (General Condition of Service), Rules 1961 (henceforth ‘the Rules of 1961’) and
Rule 6(1)(a) of the Recruitment Rules of 1987. Petitioners have also prayed for issuance
of suitable instructions/directions to conduct separate examination for the petitioners
reserving the quota of post in view of judgment of Supreme Court in the case of Jacob
Mathew vs. State of Punjab & another, AIR 2005 SC 3180. Further prayer has been
made for issuance of direction to the State to extend the upper age limit up to 48 years.
Petitioners have also prayed for issuance of direction for reserving 50 per cent post for
those Ayurveda Chikitsa Adhikaris who are presently working on contract basis and
allowing the petitioners to take part in the examination irrespective of upper age limit of
45 years. In W.P. (S) No.1990/09 petitioners have prayed for declaration that action of
respondents/State is discriminatory, arbitrary and illegal being ultra vires of Article 14 &
16 of the Constitution of India. In addition to prayer for quashing advertisement dated
25.5.2008, they have also prayed for issuance of direction to State to increase age limit
and grant weightage to the petitioners in respect of services extended by them.
6.
In W.P. (S) No.1746/09 & W.P. (S) 2549/09, learned counsel for the petitioners
argued that while initiating process of recruitment for regular appointment on the post of
Ayurveda Chikitsa Adhikari, interest of Ayurveda Chikitsa Adhikari working since last
many years has not been taken care of. It is further submitted that they are entitled to
preferential right in view of their long experience of working as contract Ayurveda
Chikitsa Adhikari and in the absence of their being any such provision in the rules and in
the advertisement, the method of competitive examination is irrational and
unconstitutional and contrary to the law laid down by the Supreme Court in the case of
Kuldeep Kumar Gupta & Ors vs. Himachal Pradesh State Electricity Board & Ors., AIR
2001 S.C. 308. It is contended that the process of recruitment is liable to be set aside on
the ground that it does not categorize the direct recruitment and limited recruitment and
therefore the same is irrational. It is further contended that it is not in consonance with
the recruitment process adopted in the case of Chhattisarh Higher Judicial Service
(Recruitment and Conditions of Service) Rules, 2006 (henceforth ‘the Rules of 2006’).
Petitioners contended that the same scheme of recruitment as laid down in the Rules of
2006 shall be applied in the case of the recruitment to the post of Ayurveda Chikitsa
Adhikari.
According to the learned counsel for the petitioners, petitioners, having
continued as contract Ayurveda Chikitsa Adhikari for last several years, have legitimate
expectations which has been completely ignored rendering the entire process of
recruitment and selection violative of Article 14 & 16 of the Constitution of India.
Placing reliance on the decision of the Supreme Court in the case of Mohd. Abdul Kadir
& Anr. vs. Director General of Police, Assam & Ors. (2009) 6 SC 611, it has been argued
that though in the case of policy matters having financial and other implications, judicial
review is not permissible, in case of arbitrariness and unreasonableness, interference
would be warranted by issuing suitable directions/guidelines drawing attention of the
authorities concerned to the issue involved.
7.
Learned counsel for the petitioners appearing in W.P. (S) No.1990/09 has also
made similar submission
It has been argued that petitioners, having continued as
contract Ayurveda Chikitsa Adhikari, are entitled to appropriate weightage.
In the
absence of such weightage either in the relevant statutory recruitment rules or in the
advertisement, they have been illegally and arbitrarily denied equality of opportunity. It
is contended that the maximum age limit has been fixed illegally and arbitrarily.
8.
Opposing the reliefs claimed by the petitioners, learned counsel for the State
submitted that the matter, relating to creation and abolition of post, formation and
restricting of cadres, prescribe the source/mode of recruitment and qualifications, criteria
of selection, valuation of service record of the employees, fall within the exclusive
domain of the employer. What steps should be taken for improving efficiency of the
administration is also domain of the employer. Learned counsel for the State submits that
the power of judicial review can be exercised in such matters only if it is demonstrated
that the action of the employer is contrary to any constitutional or statutory provisions or
is patently, arbitrary or vitiated due to malafide. He further submits that the Court has no
role in determining the methodology of recruitment or laying down criteria of selection.
Matter relating to prescription of eligibility criteria and age limit is subject matter of
policy and as long as the Rule Making Authority exercise its power within the
constitutional limits, petitioners are not entitled for issuance of any direction to
alter/change the eligibility criteria with regard to age in such a manner so as to make the
petitioners, who are otherwise ineligible, eligible for the purpose of selection and
appointment to the post of Ayurveda Chikitsa Adhikari. It has also been argued that the
provision contained in the rules and reflected under the age eligibility criteria in the
advertisement dated 22.5.2008 make appropriate provision for relaxation which is
statutory scheme.
Petitioners have no right to claim prescription of age limit or
weightage for experience or relaxation in any manner desired by the petitioners.
9.
Respondent-Chhattisgarh Pubic Service Commission has filed its separate return
submitting that the recruitment is being made by issuing advertisement dated 22.5.2008
in accordance with the provisions contained in the Recruitment Rules of 1987. It has also
been highlighted that in view of amendments made in the Rule vide notification dated
5.7.2008, a corrigendum was issued on 17.7.2008 that those who are beneficiary of the
amended provision regarding relaxation of age, may submit their application by 31st July,
2008.
It is the submission of learned counsel for the P.S.C. that now after the
amendment, vide notification dated 5.7.2008, one time age relaxation up to 5 years has
been provided in respect of medical officers who are working or have acquired
experience of working in Government, Semi-Government and other recognized Medical
Institutions. It has also been submitted that the upper age limit is 35 years and relaxation
has
been
provided
to
OBC/Permanent/Temporary
various
Worked
category
of
Charged
persons
like
Contingency
Woman/ST-SC,
Employee
of
Corporation/Project/Ex-Military Persons, relaxation of age up to 5 years as one time
measure has also been provided to medical officers.
Petitioners are not entitled to
issuance of any direction/mandamus for fixing any age limit or relaxation to their choice
and suitability, as it is, being purely policy matter, for the Government to fix the
norms/criteria, rules of appointment to public services on various posts in different
department of the Government. Reliance has been placed on the decision of Supreme
Court in the case of P.U. Joshi & Others V. Accountant General, Ahmedbad and Others,
(2003) 2 SCC 632. In W.P. (S) No.1990/09, respondent No.8 has filed is separate return
supporting the action of the respondents/State and the Pubic Service Commission. It is
submitted that petitioners are only contract employees and mere continuance in service
for few years does not confer upon the petitioners any indefeasible right to compete for
regular appointment during the process of recruitment under the relevant statutory rules if
the petitioners are otherwise not fulfilling eligibility criteria with regard to age at the time
when the process of recruitment has been initiated.
Petitioners at the time of their
appointment were fully aware of the nature of their appointment that they are contract
employees and not regular employees. It was fully known to the petitioners that their
appointment will continue only till regular appointments are made in accordance with
rules. If the petitioners are not fulfilling eligibility criteria prescribed under the rules,
their mere continuance as contract medical officers, would not entitled them to seek any
direction to alter/amend the such criteria or to claim relaxation or weightage so as to
make them eligible.
10.
We have given our anxious consideration to the submission made by the learned
counsel for the respective parties and perused the record.
11.
In order to appreciate the submission made by learned counsel for the respective
parties with reference to statutory scheme of recruitment to the post of Ayurveda Chikitsa
Adhikari, we consider it appropriate to consider the statutory scheme of recruitment,
eligibility criteria qualification etc.
12.
Recruitment to various posts including post of Ayurveda Chikitsa Adhikari is
governed by the Recruitment Rules of 1987. Rule 5 thereof provides that qualifications,
the number of post included in the service and the scale of pay attached thereto shall be in
accordance with the provision contained in Schedule-I. A perusal of Schedule-I shows
that the post of Ayurveda Chikitsa Adhikari is also included in the service. Recruitment
to the service, after the commencement of the Recruitment Rules of 1987 can be made by
any of the method specified in Clauses (a), (b) or (c) of Rule 6 of the Recruitment Rules
of 1987.
One of such method is by direct recruitment by selection/competitive
examination. Rules make elaborate provisions with regard to conditions of eligibility of
direct recruits. Rule 8(1) of the Recruitment Rules of 1987 deals with the criteria relating
to age. The same being relevant for the purpose of present case is being reproduced as
under:
“8. Conditions of legibility of direct recruits.- In order to be eligible to
be selected/compete the examination, a candidate must satisfy the
following conditions, namely:(1) Age (a) He must have attained the age in column (3) and not attained
the age in column (4) of Schedule III on the first day of January next
following the date of commencement of the selection.
(b) The upper age limit shall be relaxable up to a maximum of 5 year if a
candidate belongs to a Scheduled Castes or a Scheduled Tribe.
(c)
The upper age limit will also be relaxable in respect of candidates
who are or have been employed of the Madhya Pradesh Government to the
extent and subject to the conditions specified below:(i) A candidate, who is holding a post temporarily and applying for
another post should not be more than 38 years of age. This
concession shall also be admissible to work-charges employees,
contingency paid employees and persons employed in the Project
implementing Committees.
(ii) A candidate, who is retrenched Government servant will be
allowed to deduct from his age the period of all temporary service
previously rendered by him up to a maximum limit of seven years
even if it represents more than on spell provided that the resultant
age does not exceed the upper age limit by more than 3 years.”
(d) A candidate, who is an ex-serviceman will be allowed to deduct from
his age the period of all defence services previously rendered by him
provided that the resultant age does no exceed the upper age limit by more
than three years;
(e) The upper age limit will be 35 years in respect of widow candidates.
(f) The upper age limit shall also be relaxable up to 2 years in respect of
Green Card holder candidates under Family Welfare Programme.
(g) The upper age limits shall be relaxable up to 5 years in respect of
awarded couple of Inter-caste Marriage Incentive Scheme under the Tribal
Harijan and Backward Classes Welfare Department.
(h) The general upper age limit shall also be relaxable up to 15 years in
respect of Vikram Award holder candidate.
(i) The upper age limit shall also be relaxable in respect of employees of
Madhya Pradesh State Corporations/Boards up to 38 years.
(j) The upper age limit will be relaxed in the case of voluntary Home
Guards and Non-Commissioned Officer of the Home Guards for the
period of service rendered so by them subject to the limit of 8 years, but in
no case their age should not exceed 38 years.”
Perusal of the aforesaid provision would show that in order to be eligible for
appointment, a candidate must have attained the age in column (3) and not attained the
age of column (4) of Schedule III on the first day of January next following the date of
commencement of the selection.
Sr.No.6 of Schedule-III shows that the minimum age limit is 22 years, whereas
the maximum age limit is 32 years. Clauses (b), (c), (d), (e), (f), (g), (h), (i) and (j) of
Rule 8 provide for relaxation in upper age limit in respect of different classes/categories
of candidates. Clause (c) makes provision for relaxation of upper age limit in respect of
candidates who or have been employed in the M.P./C.G. Government to the extent and
subject to conditions specified in sub clause-(i) and (ii). It is also admitted position on
record that vide notification dated 5.7.2008, the aforesaid recruitment rules have been
amended the following has been inserted in column (7) corresponding Sr.No.4, 6 & 8 of
Schedule-III.
“Omitted as the matter is in Hindi”
From the perusal of aforesaid statutory scheme engrafted in the Recruitment
Rules of 1987, it is clear that relaxation of upper age limit has been granted to different
categories. While Clause (c) generally provides for relaxation of upper age limit in
respect of candidates who are or have been employed by the M.P./C.G. Government, the
newly added provision provides for relaxation of age to the entire class of medical
officers working or having worked in Government, Semi-Government and recognized
Medical Institutions.
13.
While different recruitment rules have been made for various departments of the
Government by the Governor in exercise of power under Article 309 of the Constitution
of India, general conditions of service applicable to all the Government servants
irrespective of their individual departments are regulated by the Rules of 1961. Rule 7
thereof specifies various methods of recruitment. It embodies three well-known methods
of recruitment namely, direct recruitment, promotion, transfer of person or persons
already employed in another service or post.
14.
Recruitment Rules of 1987 also specifies three well recognized mode and
methods of recruitments i.e. by direct recruitment, by promotion, by transfer of persons.
Rules 6(4) of the Recruitment Rules of 1987 further provides that notwithstanding
anything contained in sub rule (1) if in the opinion of the Government, the exigencies of
the service so require, the Government may with prior concurrence of the Department of
Personnel and Administration Reforms and Training adopt such method of the
recruitment of service other than those specified in sub-rule (1), as it may be order issued
in this behalf, prescribe.
15.
In the Advertisement No.07/2008 dated 22.5.2008 published on 28.5.2008
inviting applications for appointment to the post of Ayurveda Chikitsa Adhikari,
minimum and maximum age limit has been prescribed as 22 & 35 years respectively.
The note appended to Clause (4) clarifies that as per G.A.D. Circular dated 28.6.2007, the
upper age limit for direct recruitment has been prescribed as 35 years. It further provides
that the benefit of relaxation of age to various categories will continue as before but with
such relaxation the maximum age limit should not exceed 45 years. Clause (5) of the said
Advertisement makes provisions with regard to relaxation in age consistent with the
statutory scheme engrafted in Rule 8 of the Recruitment Rules of 1987. Further, in view
of special provision, 10 years relaxation in upper age limit has been provided for female
candidates. In view of GAD Notification dated 2.6.2004, relaxation in upper age limit
has also been provided to Shiksha Karmis in respect of the number of years they have
remained in Government service subject to maximum upper age limit of 45 years.
Provision for relaxation has also been provided in respect of voluntary Home Guards,
Widows, Destitute, Divorced Female, Green Card Holders, Recipients of reward under
Inter Caste Marriage Incentive Scheme and Recipient of other rewards and the cutoff date
has been provided as 1st January 2009.
In view of the amendment made in the Recruitment Rules of 1987, the Public
Service Commission issued a Corrigendum on 17.7.2008 (AnnexureR-2/1 in W.P.(S)
No.1990/09) allowing even those medical officers who are entitled to compete in view of
newly added provision providing for relaxation of age to the extent of 5 years and were
granted opportunity to submit their application forms by 31st July, 2008. An analysis of
the provisions contained in the Recruitment Rules of 1987 shows that an elaborate and
comprehensive statutory scheme for relaxation of age to different classes and categories
of candidates has been provided including medical officers who were working or have
worked in Government, Semi-Semi Government or recognized medical institutions.
Rule 7(1) of the Rules of 1961 and Rule 6 of Recruitment Rules of 1987 prescribe
well recognized three methods of recruitment to public employment namely direct
recruitment; promotion; transfer from other services. In the Recruitment Rules of 1987,
it has also been provided that notwithstanding the aforesaid normal method of
recruitments to various posts classified under the rules, in the exigencies of service,
Government may, with the prior concurrence of the Department of Personnel and
Administration Reforms and Training may adopt any other method of recruitment of
service.
16.
The petitioners in these three petitions who were appointed on contract basis for a
limited period, have challenged the legality and validity of the provisions contained in
Rule 7(i) of the Rules of 1961 and Rules 6(1)(a) of the Recruitment Rules of 1987 on the
ground that the rules are unconstitutional as they do not contain any provision for limited
departmental competitive examination. The submission advanced by the learned counsel
for the petitioners in these three petitions is common i.e. the Recruitment Rules are
unconstitutional, irrational and arbitrary as no provision has been made for conducting
separate competitive examination for the class of contract Ayurveda Chikitsa Adhikari
alike the petitioners. In order to lay strength to their submission, petitioners have referred
to the statutory scheme of recruitment under the Chhattisgarh Higher Judicial Service
(Recruitment and Conditions of Service) Rules, 2006.
17.
The backbone of petitioners’ arguments is that petitioners were working as
contract medical officer and therefore there was a legitimate expectation that their
services will be regularized. They made representations on 22.1.2009. It is submitted
that there is discrimination between the fresh candidates and the petitioners/contract
Ayurveda Chikitsa Adhikaris.
18.
It is also urged by the learned counsel for the petitioners that the upper age limit is
required to be extended up to age of 48 years so that the petitioners/contract Ayurveda
Chikitsa Adhikari become eligible so as to compete in the examination.
Further
submission is that looking to the fact that they have been working as contract employees
for the last six years they are entitled to relaxation in upper age limit as they have attained
the age just beyond the upper age limit prescribed under the Advertisement. In order to
ensure that they are not ousted from the process of selection, learned counsel for the
petitioners argued that under any circumstances, they are entitled to grant of appropriate
weightage in respect of number of year they have worked as contract employees.
19.
Prescription of lower and upper age limit, relaxation in age, mode and method of
recruitment in the case of recruitment to the post of Ayurveda Chikitsa Adhikari is
prescribed under the recruitment rules framed by the Governor under Article 309 of the
Constitution of India, which is legislative function under the constitutional authority. The
law with regard to judicial review of any legislative act is now well settled. The grounds
of judicial review of legislative act are much more restricted then those of the judicial
review of administrative action.
Unless it lacks competence or is violative of the
constitutional provisions; it is not open to challenge. The Supreme Court in the case of
R.S.Bansal & Ors. Vs. Union of India & Ors., 1992 Supp (2) SCC 318, while examining
the scope of judicial review of a legislative action, particularly with reference to
recruitment rules framed by the Governor under Article 309 of the Constitution of India,
has authoritatively pronounced as under:
“21.
Now it is true that rules made under the proviso to Article 309 of
the Constitution being legislative in character cannot be struck down
merely because the Court thinks that they are unreasonable, - and that they
can be struck down only on the grounds upon which a legislative measure
can be struck down.”
Further in the case of A.B.Krishna & Ors. Vs. State of Karnataka & Ors. (1998) 3
SCC 495 it was observed as under:
“7.
The rule-making function under the proviso to Article 309 is a
legislative function. Since Article 309 has to operate subject to other
provisions of the Constitution, it is obvious that whether it is an Act made
by Parliament or the State Legislature which lays down the conditions of
service or it is a rule made by the President or the Governor under the
proviso to that article, it has to be in conformity with the other provisions
of the Constitution specially Articles 14, 16, 310 and 311.”
20.
We shall now examine the submission made on behalf of the petitioners that
unconstitutional validity of relevant provisions contained in Rule 6(1) of the Recruitment
Rules of 1987 on the touchstone of legal principles as annunciated by the Supreme Court
as above. Undeniably, the rules have been framed by the Governor in exercise of his
constitutional authority under Article 309 of the Constitution of India and neither from
the pleadings nor from the submission made into this dispute that the Governor lacked
competence to make such provisions prescribing different methods of recruitment to the
post of Ayurveda Chikitsa Adhikaris.
It has also not been demonstrated that the
provision is in any manner inconsistent with any Parliamentary or State Legislation in
force providing for some other mode/method of recruitment thereby rendering provision
contained in Rule (6)(I)(a) in the Recruitment Rules of 1987 and 7(i) of the Rules of 1961
inconsistent and inoperative. The entire submission rests on an assumption, though
erroneous in law, that by virtue of continuance for about 6 years as contract employees,
they acquire an indefeasible right to be considered for regular appointment, irrespective
of the eligibility criteria relating to age prescribed under the existing recruitment rules.
We are unable to accept that merely because the rules do not provide for limited
competitive examination only for contract employees, the rules are violative of any
constitutional provision. Even, at the time when the petitioners were appointed on the
contract basis, the existing recruitment rules providing for regular recruitment to the post
of Ayurveda Chikitsa Adhikaris prescribed various mode/methods as also provision
relating to relaxation in upper age limit. While the petitioners were continued on contract
basis, they crossed the upper age limit by the time the process of regular recruitment
started. What should be the mode and methodology and procedure for recruitment is
entirely for the Rule Making Authority and it is not the function of the Courts to suggest
or evolve any new/alternative mode/method or procedure for recruitment in public
employment. It is relevant to note that Rule 6(4) of the Recruitment Rules of 1987
reserve the power to the Government Authority to adopt any other mode for recruitment
than those enumerated, namely direct recruitment; promotion; transfer from other
services. It would thus be seen that the mode of recruitment provided in Rule 6 of the
Recruitment Rules of 1987 are not only well recognized, accepted and known mode and
method of recruitment to public service but also such other mode and method may be
adopted as the exigencies of the administration require. Such a comprehensive and
foolproof statutory scheme engrafted in the rules cannot be said to be in any manner
violative of Article 14 and 16 of the Constitution of India merely because it does not
specifically provides for limited competitive examination for the class/category of
contract employees. In the case of P.U. Joshi (supra) the Supreme Court held as under:
“10.
We have carefully considered the submissions made on behalf of
both parties. Questions relating to the constitution, pattern, nomenclature
of posts, cadres, categories, their creation/abolition, prescription of
qualifications and other conditions of service including avenues of
promotions and criteria to be fulfilled for such promotions pertain to the
field of policy is within the exclusive discretion and jurisdiction of the
State, subject, of course, to the limitations or restrictions envisaged in the
Constitution of India and it is not for the statutory tribunals, at any rate, to
direct the Government to have a particular method of recruitment or
eligibility criteria or avenues of promotion or impose itself by substituting
its views for that of the State……”
21.
As we have mentioned hereinabove, the entire basis for such submission proceeds
on alleged equitable consideration that as the petitioners had continued for six years as
contract employees, the State is under constitutional obligation to ensure their
continuance by making appropriate provisions in the recruitment rules by holding limited
competitive examination. We are of the firm opinion that petitioners are not entitled to
any such equitable consideration merely on the basis of their continuance as contract
appointees for few years. It would be apposite to refer to what was authoritatively
pronounce by the Supreme Court while negativing claims of regularization of
temporary/ad-hoc employees on long continuance in service in the case of Secretary,
State of Karnataka & Ors. Vs. Umadevi (3) & Ors. (2006) 4 SCC 1:
“43. Thus, it is clear that adherence to the rule of equality in public
employment is a basic feature of our Constitution and since the rule of law
is the core of our Constitution, a court would certainly be disabled from
passing an order upholding a violation of Article 14 or in ordering the
overlooking of the need to comply with the requirements of Article 14
read with Article 16 of the Constitution. Therefore, consistent with the
scheme for public employment, this Court while laying down the law, has
necessarily to hold that unless the appointment is in terms of the relevant
rules and after a proper competition among qualified persons, the same
would not confer any right on the appointee. If it is a contractual
appointment, the appointment comes to an end at the end of the contract, if
it were an engagement or appointment on daily wages or casual basis, the
same would come to an end when it is discontinued. Similarly, a
temporary employee could not claim to be made permanent on the expiry
of his term of appointment. It has also to be clarified that merely because
a temporary employee or a casual wage worker is continued for a time
beyond the term of his appointment, he would not be entitled to be
absorbed in regular service or made permanent, merely on the strength of
such continuance, if the original appointment was not made by following a
due process of selection as envisaged by the relevant rules. It is not open
to the court to prevent regular recruitment at the instance of temporary
employees whose period of employment has come to an end or of ad hoc
employees who by the very nature of their appointment, do not acquire
any right…..”
“45.
While directing that appointments, temporary or casual, be
regularised or made permanent, the courts are swayed by the fact that the
person concerned has worked for some time and in some cases for a
considerable length of time. It is not as if the person who accepts an
engagement either temporary or casual in nature, is not aware of the nature
of his employment. He accepts the employment with open eyes. It may
be true that he is not in a position to bargain-not at arm’s length-since he
might have been searching for some employment so as to eke out his
livelihood and accepts whatever he gets. But on that ground alone, it
would not be appropriate to jettison the constitutional scheme of
appointment and to take the view that a person who has temporarily or
casually got employed should be directed to be continued permanently.
By doing so, it will be creating another mode of public appointment which
is not permissible. If the court were to void a contractual employment of
this nature on the ground that the parties were not having equal bargaining
power, that too would not enable the court to grant any relief to that
employee. A total embargo on such casual or temporary employment is
not possible, given the exigencies of administration and if imposed, would
only mean that some people who at least get employment temporarily,
contractually or casually, would not be getting even that employment
when securing of such employment brings at least some succour to them.
After all, innumerable citizens of our vast country are in search of
employment and one is not compelled to accept a casual or temporary
employment if one is not inclined to go in for such an employment. It is
in that context that one has to proceed on the basis that the employment
was accepted fully knowing the nature of it and the consequences flowing
from it. In other words, even while accepting the employment, the person
concerned knows the nature of his employment. It is not an appointment
to a post in the real sense of the term. The claim acquired by him in the
post in which he is temporarily employed or the interest in that post cannot
be considered to be of such a magnitude as to enable the giving up of the
procedure established, for making regular appointments to available posts
in the service of the State. The argument that since one has been working
for some time in the post, it will not be just to discontinue him, even
though he was aware of the nature of the employment when he first took it
up, is not one that would enable the jettisoning of the procedure
established by law for public employment and would have to fail when
tested on the touchstone of constitutionality and equality of opportunity
enshrined in Article 14 of the Constitution.”
“47. When a person enters a temporary employment or gets engagement
as a contractual or casual worker and the engagement is not based on a
proper selection as recognized by the relevant rules of procedure, he is
aware of the consequences of the appointment being temporary, casual or
contractual in nature. Such a person cannot invoke the theory of
legitimate expectation for being confirmed in the post when an
appointment to the post could be made only by following a proper
procedure for selection and in cases concerned, in consultation with the
Public Service Commission.
Therefore, the theory of legitimate
expectation cannot be successfully advanced by temporary, contractual or
casual employees. It cannot also be held that the State has held out any
promise while engaging these persons either to continue them where they
are or to make them permanent. The State cannot constitutionally make
such a promise. It is also obvious that the theory cannot be invoked to
seek a positive relief of being made permanent in the post.”
22.
The aforesaid discussion therefore leads to inevitable conclusion that petitioners
did not even acquire any right either on equity or under equality clause which the State is
under Constitutional obligation to enforce by framing appropriate scheme for recruitment
through limited examination.
Unless the petitioners are eligible under the rules
governing the regular recruitment, they cannot claim as of right, participation in the
process of recruitment or consideration of their candidature. Having found that they have
crossed the upper age limit at the time when the process of regular recruitment was
initiated, such far-fetched claim without any constitutional or legal sanction has been
raised.
23.
To buttress their contention that the recruitment rules ought to have contained
provisions with regard to limited competitive examination, petitioners placed reliance on
the decision of the Supreme Court in the case of Kuldeep Kumar Gupta (supra). The
dispute in the aforesaid case centered around the question as to whether it is permissible
for the employer to frame regulations providing a separate quota of promotional avenues
for the less qualified junior engineers in preference to the claim of qualified diploma
holders junior engineers. The service regulations provided that 46 per cent of the posts in
the cadre of Assistant Engineers was available for promotion quota and out of the same,
28 percent were to be filled up by junior engineers qualified, 8 percent by junior
engineers (unqualified), 6 percent from amongst those who have passed Section A & B
examination of the Institute of Engineers (Service), 4 percent from draftsman. The
aforesaid provision providing different quotas for promotion, was challenged by direct
recruits qualified junior engineers praying for quashing of the quota rule vis-à-vis them
and the unqualified junior engineers. It was in the said context, while examining the
validity of such provision, it was held that there can be separate consideration for the
promotee unqualified matriculate junior engineers in the matter of promotion to the post
of Asst. Engineer and the regulations providing a quota for them cannot be held to be
violative of Article 14 of the Constitution of India. The other contention raised that
providing a quota tantamounts to reservation was also negatived. In that process of
consideration, it was observed that providing a quota is not a new in service
jurisprudence and whenever the feeder category itself consist of different category of
persons and when they are considered for any promotion, the employer fixes a quota for
different category so that the promotion would be equi-balance and at the same time each
category of persons in feeder category would get the opportunity of being considered for
promotion. The aforesaid authority does not help the petitioners to propound that the
Rule Making Authority is under a constitutional obligation to necessarily provide for a
special recruitment process by way of limited competitive examination in respect of
contract employees in public employment. It is neither the jurisdiction nor the function
of the constitutional Court to issue a command to the State to provide by legislation any
particular mode/method of recruitment for class of contract employees in addition to the
mode and method of recruitment provided under the statutory rules framed under Article
309 of the Constitution of India. We, therefore, reject petitioners’ contention that on
account of absence of provision for recruitment by limited competitive examination in
respect of contract appointees, Rule 6(1)(a) of the Recruitment Rules of 1987 or Rule
7(1) of the Rules of 1961 are in any manner unconstitutional or ultra vires constitutional
provision.
24.
Petitioners have also placed reliance upon the decision of Supreme Court in the
case of Mohd. Abdul Kadir (supra). That was a case where ad-hoc appointments were
made under prevention of Infiltration of Foreigner’s Scheme. Later on, a process of
termination and reappointment was introduced which was subject to challenge by ad-hoc
employees.
The petitioners therein challenged system of appointment on one year
contract basis and also claimed relaxation in service.
Claim of regularization was
negatived by holding as below:“14. The fact that the Scheme had been in operation for some decades or
that the employee concerned has continued on ad hoc basis for one or two
decades would not entitle the employee to seek permanency or
regularisation. Even if any posts are sanctioned with reference to the
Scheme, such sanction is of ad hoc or temporary posts coterminous with
the Scheme and not of permanent posts.”
“15. On completion of the project or discontinuance of the scheme, those
who were engaged with reference to or in connection with such project or
scheme cannot claim any right to continue in service, nor seek
regularization in some other project or service. [see Bhagwan Dass v.
State of Haryana, Delhi Development Horticulture Employees Union v.
Delhi Admn., Hindustan Steel Works Construction Ltd. V. Employees’
Union, U.P. Land Development Corpn. V. Amar Singh, Madhyamik
Shiksha Parishad, U.P.v. Anil Kumar Mishra, state of Karnataka v.
Umadevi (3), Indian Council of Medical Research v. K.Rajyalakshmi and
Lal Mohammad v. Indian Railway Construction Co. Ltd.] In view of this
settled position, the appellants will not be entitled to regularization”
What was deprecated was system of termination and reappointment every year
and it was observed that it should be avoided and the ad-hoc employee should be
continued as long as scheme continued but purely on ad-hoc temporary basis coterminous
with the scheme. Taking into consideration the fact that PIF Scheme had been in force
for merely 5 decades and observing that the task under the scheme is perennial, it was
expressed that there is no point in executing the same as temporary scheme.
It was in
those circumstances that the Supreme Court observed that if those working as ad-hoc or
temporary staff and are converted to regular staff, that would boost their morale and
efficiency. We are unable to hold that such observation made by the Supreme Court in
the peculiar circumstances of that case help the petitioners to advance submission that the
recruitment rules are bad because they do not contain provision relating to recruitment by
way of competitive examination of contract employees/petitioner who have continued for
6 years.
25.
The petitioners have also sought issuance of writ in the nature of mandamus for
issuing command to respondents to enhance the upper age limit up to 48 years or to grant
relaxation in age corresponding to the number of year of contract service rendered by the
petitioners.
26.
As we have already discussed above, prescription of lower and upper age limit is
essentially legislative function of the Governor in exercise of constitutional authority
under Article 309 of the Constitution of India. In the absence of violation of any
constitutional provisions, this Court would not issue any command to the Rule Making
Authority to prescribe any particular lower or upper age limit for recruitment to any post
under public employment. It is the exclusive domain of the Rule Making Authority. It is
pertinent to observe that an exhaustive legislative scheme has been provided under the
Recruitment Rules of 1987 prescribing not only minimum upper age limit but also
relaxation in upper age limit to various categories of candidates including medical
officers. If the petitioners are not eligible even by invoking those relaxation clauses
provided for medical officers, no direction can be issued to the Rule Making Authority to
invent a relaxation clause to make petitioners eligible to compete in the examination. In
the case of Mallikarjuna Rao & Ors. Vs. State of Andhra Pradesh & Ors. (1990) 2 SCC
707 it was held as under:
“13. The Special Rules have been framed under Article 309 of the
Constitution of India. The power under Article 309 of the Constitution of
India to frame rules is the legislative power. This power under the
Constitution has to be exercised by the President or the Governor of a
State as the case may be. The High Courts or the Administrative
Tribunals cannot issue a mandate to the State Government to legislate
under Article 309 of the Constitution of India. The courts cannot usurp
the functions assigned to the executive under the Constitution and cannot
even indirectly require the executive to exercise is rule making power in
any manner. The courts cannot assume to itself a supervisory role over the
rule making power of the executive under Article 309 of the Constitution
of India.”
27.
It is useful to refer to the observation made by the Supreme Court in the case of
State of M.P. and another Vs. Dharam Bir, (1998) 6 SCC 165
“34. The respondent having working in an ad hoc capacity on the post of
Principal might have gained some administrative experience but the same
cannot be treated as equivalent to his knowledge in the field of
Engineering. A compounder, sitting for a considerably long time with a
doctor practisting in modern medicine, my have gained some experience
by observing the medicine prescribed by the doctor for various diseases or
ailments but that does not mean that he, by that process, acquires
knowledge of the human anatomy or physiology or the principles of
pharmacology or the field of action of any particular medicine or its side
effects The compounder cannot, merely on the basis of experience, claim
a post meant exclusively for persons having MBBS or other higher
degrees in medicine or surgery. The plea of experience, therefore, must
fail. Moreover this would amount to a relaxation of the Rule relating to
educational qualification. Power to relax the Rule vests exclusively in the
Governor as provided by Rule 21. This power cannot be usurped by the
court or the tribunal.”
28.
In the result, we do not find any merit in these petitions. They are liable to be
dismissed and are accordingly dismissed.
29.
There shall be no order as to costs.
***
HIGH COURT OF CHHATTISGARH, BILASPUR
WRIT PETITION (S) NO.4856 OF 2009
D.D. 25.03.2010
Hon’ble Shri Dhirendra Mishra and
Hon’ble Shri R.N.Chandrakar, JJ
Hemanand Mani Tripathi & Ors.
Vs.
State of Chhattisgarh & Ors.
…
Petitioners
…
Respondents
Eligibility:
Whether eligibility criteria with regard to age can be changed after commencement of
selection process? – Yes.
Whether higher upper age limit for local candidates amounts to discrimination? – No.
The State Government as per Circular dated 2.6.2008 gave relaxation of two years
(30+2 years) in upper age limit to all the candidates participating in PSC examination –
Relaxation was to be valid till 31.5.2009 – PSC issued advertisement on 3.9.2008 and
invited applications for various civil posts – The State Government as per Circular dated
16.9.2008 modified its earlier circulars and prescribed upper age limit of 30 years and
allowed relaxation of five years to local residents of the State – PSC issued fresh
advertisement for recruitment superseding its earlier advertisement prescribing age
criteria as per Circular dated 16.9.2008 – Petitioners who successfully participated in the
preliminary examination submitted their applications with requisite fee for the main
examination – PSC declared the petitioners ineligible for main examination – Aggrieved
by the same petitioners filed this writ petition – It was among others contended by the
petitioners that the Circular according age relaxation to the local residents has not been
issued with the prior concurrence of the Parliament and as such ultra vires Article 16(2)
of the Constitution – High Court after considering contentions of the parties held that
State to make suitable provisions in the relevant rules keeping in view the interest of the
candidates belonging to the State and the same cannot be termed arbitrary etc., dismissed
the petition.
Held:
Relaxation of five years in the upper age limit was extended to the local residents
of the State in the peculiar circumstances that PSC examination could not be convened
annually in the last nine years. Relaxation of age only enabled certain candidates
belonging to the State of Chhattisgarh to fall within the zone of consideration. The
concession in age does not, in any manner, favour the candidates of Chhattisgarh in
preparation of final merit list as they are also required to participate in the preliminary
examination and then in the main examination together with the successful candidates of
other States.
Further held:
Discrimination in prescribing maximum age limit for recruitment to the State
Civil Services is not based solely on the ground of place of residence. Hence not bad.
Cases referred:
1. AIR (38) 1951 Madras 120 (C.N. 14) - Srimathi Champakam Dorairajan & Anr. Vs.
State of Madras
2. AIR 1993 SC 1365 - Union of India and Others Vs. Sanjay Pant and Others \
3. AIR 2001 SC 467 - K.Thimmappa & Ors. vs. Chairman, Central Board of Directors,
SBI & Anr.
4. 2002 (6) SCC 562 - Kailash Chand Sharma Vs. State of Rajasthan & Ors.
5. AIR 2003 SC 1266 - Welfare Association, A.R.P., Maharashtra & Anr.Vs. Ranjit P.
Gohil & Ors.
6. (2006) 8 SCC 671 - Kendriya Vidyalaya Sangathan & Ors. Vs. Sajal Kumar Roy &
Ors.
7. JT 2010 (1) SC 177 - Jitendra Kumar Singh & Anr. Vs. State of U.P. & Ors.
ORDER
Dhirendra Mishra, J
The petitioners have prayed for quashing of Circular dated 16th September 2008
(Annexure P/1) issued by the General Administration Department of the State of
Chhattisgarh prescribing maximum age limit for direct recruitment on services/posts of
the State Govt. as also for quashing of advertisement dated 20th September, 2008
(Annexure P/2) whereby applications have been invited from eligible candidates for State
Services Examination, 2008 by the Chhattisgarh Public Service Commission (in short
“PSC”).
2.
Briefly stated, facts of the case are that the State Government vide its Circular
dated 2nd June, 2008 gave relaxation of two years (30+2 years) in upper age limit to all
the candidates participating in PSC examination. The relaxation was to be valid till 31st
May, 2009.
PSC issued an advertisement on 3rd September, 2008 and invited
applications for various civil posts in the State. The State Govt. vide its Circular dated
16th September, 2008 (Annexure P/1) modified its earlier circulars and prescribed upper
age limit of 30 years and allowed relaxation of five years to local residents of the State.
PSC issued fresh advertisement for recruitment superseding its earlier advertisement
dated 3rd September, 2008 and the age criteria in the subsequent advertisement was fixed
as per Circular dated 16th September, 2008 i.e. maximum age limit of 30 years with
relaxation of five years for the local residents of the State of Chhattisgarh. The petitioner
submitted their applications in response to the advertisement and successfully
participated in the preliminary examination held on 2nd February, 2009. They were
informed about the result by the Examination Controller vide its Memo dated 18th May,
2009. The petitioners were called upon to deposit requisite fee along with application for
appearing in PSC Main Examination. The petitioners submitted their applications with
requisite fee. However, PSC declared the petitioner ineligible for Main Examination,
2008 on the ground of overage, vide Annexure P/9.
3.
Shri Awadh Tripathi, learned counsel for the petitioners argued that the
petitioners submitted their applications in response to the advertisement dated 3rd
September, 2008 and successfully participated in the preliminary examination. They
were not aware about the subsequent advertisement issued on 20th September, 2008.
They submitted their applications for Main Examination as directed by PSC vide their
communication dated 18th May, 2009. However, after commencing selection process
vide advertisement dated 3rd September, 2008, the eligibility criteria with respect to age
was changed and the maximum age was reduced from 37 years to 30 years vide Circular
dated 16th September, 2008 (Annexure P/1), which discriminates between the candidates,
who are permanent residents of State of Chhattisgarh and the candidates out of the State
of Chhattisgarh, and the same is illegal, unconstitutional and in violation of sub-clause
(2) of Article 16 as also Articles 14 & 15 of the Constitution of India. The local residents
of Chhattisgarh are entitled for relaxation of seven years in the upper age limit whereas
the petitioners, who are otherwise eligible as per advertisement dated 3rd September, 2008
were declared ineligible on the basis of Circular dated 16th September, 2008.
The
Circular of Annexure P/1, according relaxation to the local residents has not been issued
with the prior concurrence of the Parliament and as such, ultra vires Article 16(2) of the
Constitution.
4.
Reliance is placed on the judgments in the matter of Kendriya Vidyalaya
Sangathan and Others Vs. Sajal Kumar Roy and Others (2006) 8 SCC 671 and Union of
India and Others Vs. Sanjay Pant and Others AIR 1993 SC 1365.
5.
Shri U.N.S. Deo, learned Govt. Advocate appearing for the State/Respondent
No.1 & 3, would argue that the Circular of Annexure P/1 or the advertisement (Annexure
P/2) issued on the basis of Circular of Annexure P/1 does not bar any citizen of India
from participating in the recruitment process, provided he fulfils the eligibility criteria as
prescribed in the advertisement, which has been issued in accordance with the relevant
rules and circulars of the State Govt.
Article 16(2) of the Constitution prohibits
discrimination only on the ground of religion, race, caste, sex, descent, place of birth and
residence in respect of any employment or office under the State. In the instant case, the
petitioners being the residents of other States and not conforming to the eligibility criteria
with respect of maximum age under the Rules, have been held to be ineligible not only on
the ground that they belong to other States, but also on the ground that they are overage,
and the same cannot be termed to be in violation of Article 16(2) of the Constitution.
It was further argued that the action of the State in extending relaxation in upper
age limit to the bonafide residents of the State of Chhattisgarh in the matters of
employment cannot be termed to be arbitrary or discriminatory as the candidates
belonging to State of Chhattisgarh are class apart from the candidates belonging to other
States. The State of Chhattisgarh, keeping in view the special circumstances that PSC
examination for civil services could not be held annually in the past in the State of
Chhattisgarh as after 2000, examinations were held only in 2003, 2005 and 2008, with a
purpose to address the difficulties faced by the unemployed youths of the State, provided
age relaxation to the candidate who are bonafide residents of Chhattisgarh, which is an
affirmative action of the State.
6.
Reliance is placed on the judgments in the matter of Srimathi Champakam
Dorairajan and another Vs. State of Madras – AIR (38) 1951 Madras 120 (C.N. 14),
Welfare Association, A.R.P., Maharashtra and another Vs. Ranjit P. Gohil and others –
AIR 2003 SC 1266 and K.Thimmappa and others vs. Chairman, Central Board of
Directors, SBI and another – AIR 2001 SC 467.
7.
Advancing similar arguments, Shri Sanjay K. Agrawal with Shri Abhishek Sinha,
learned counsel appearing for PSC & Shri Mateen Siddiqui, learned counsel for
respondent No.5 would argue that recruitment for the State Civil Service posts is made by
State Services Examination, which is held annually by PSC. The State of Chhattisgarh
published the State Services Examination Rules (in short ‘Examination Rules’) dated 22nd
September, 2008. Rule 5 provides for eligibility conditions. Rule 5(c) prescribes that a
candidate must have attained the age of 21 years and must not have attained the age of 30
years on 1st January next following the date of advertisement. The relaxation in age is
provided under Rule 5(c)(b), according to which relaxation of five year in upper age limit
is allowed to a candidate domiciled in the State of Chhattisgarh. Rule 5(c)(b) clearly
provides that the age limit shall be applicable as per Circular dated 16th September, 2008
issued by the Govt. of Chhattisgarh.
The State Govt. vide its Circular dated 16th
September, 2008 fixed the upper age limit at 30 years with relaxation of five years to the
local residents of Chattisgarh.
PSC superseding its earlier advertisements dated 3rd
September, 2008 issued fresh advertisement on 20th September, 2008 fixing age criteria
in accordance with Circular of the State Govt. dated 16th September, 2008. Thus, the
petitioners in response to the above advertisement issued by PSC, having participated in
the recruitment process, cannot be permitted to challenge the same, including age criteria
on the ground of discrimination.
8.
Repelling the argument that the candidates belonging to other States have been
discriminated as against the candidates domiciled in the State of Chhattisgarh, it was
argued that through advertisement dated 20th September, 2008, PSC invited applications
from all eligible candidates, who fulfilled the eligibility criteria as prescribed in the
advertisement, no candidate has been declared ineligible on the ground of place of
residence. Relaxation in age to the permanent residents of the State of Chhattisgarh is in
the public interest and no violative of Article 16(2) of the Constitution and there is no
discrimination by granting relaxation of age to the local residents of Chhattisgarh. The
petitioners having participated in the recruitment process on the basis of advertisement
dated 20th September, 2008, which clearly prescribes maximum age limit for the
candidates of State of Chhattisgarh and other candidates, without any demur or protest,
cannot now be permitted to challenge the aforesaid condition in the advertisement on any
ground whatsoever. Referring to additional affidavit filed on behalf of the PSC dated 20th
January, 2001, it was argued that out of 7609 candidates, who qualified for Main
Examination, 1494 candidates are resident of the States other than Chhattisgarh.
9.
Reliance is placed on the judgments in the matters of Kailash Chand Sharma Vs.
State of Rajasthan & Ors. – 2002 (6) SCC 562 and Jitendra Kumar Singh and another Vs.
State of U.P. and others – JT 2010 (1) SC 177.
10.
We have heard learned counsel for the parties and perused the material available
on record.
11.
On the basis of averments of the respective parties and arguments advanced, the
following questions emerge for decision of this writ petition:
“1. Whether the respondent/PSC after commencing recruitment process
vide advertisement dated 3rd September, 2008, was justified in altering the
eligibility criteria and lowering the upper age limit from 35 years to 30
years for the candidates belonging to the States other than the State of
Chhattisgarh by issuing fresh advertisement dated 20th September, 2008
(Annexure P/2) on the basis of Circular dated 16th September, 2008
(Annexure P/1), issued by the State?”
2. Whether prescribing different upper age limits i.e. 35 years for the
candidates belonging to the State of Chhattisgarh and 30 years for the rest,
is an act of discrimination in the matters of employment as contemplated
under Article 16(2) of the Constitution?
3. Whether providing different upper age limits for the candidates
belonging to the State of Chhattisgarh and other States in the matters of
employment is arbitrary and discriminatory and in violative of Articles 14
& 15 of the Constitution?
1.
Whether the respondent /PSC after commencing recruitment
…………………………..issued by the State?”
12.
So far as the first issue is concerned, PSC vide its advertisement dated 3rd
September, 2008, issued on the basis of earlier circulars and Examination Rules framed
by the State, invited applications for PSC Examination to be held under the Examination
Rules for civil services in the State of Chhattisgarh, in which any candidate whose age
was between 21 to 37 years could apply for the post, other than the post of Dy.
Superintendent of Police. Category of candidates, who were entitled for relaxation in age
was to be provided, subject to maximum age limit of 45 years. The State Govt. vide its
Circular dated 16th September, 2008 (Annexure P/1) superseding all its earlier circulars
regarding maximum age limit for direct recruitment, prescribed maximum age limit for
the candidates of Chhattisgarh as 35 years and for others 30 years. In pursuance of the
aforesaid circular, PSC canceling the earlier advertisement dated 3rd September, 2008
(Annexure P/3), issued fresh advertisement dated 20th September, 2008 and prescribed
maximum age limit of 35 years for the candidates, who are bonafide residents of
Chhattisgarh, and 30 years for the candidates of other States. The last date for receiving
the applications was fixed as 3rd November, 2008.
Thus, recruitment process was
commenced fresh vide advertisement dated 20th September, 2008 and therefore,
contention of the petitioners that the eligibility criteria was changed in the midst of
selection process to the detriment of the candidates is without any substance.
11. Whether prescribing different age ………………………under Article
16(2) of the Constitution?
13.
As to the second question indisputably, direct recruitment to various civil posts of
the State is made by combined competitive examination – State Services Examination
which is conducted under the Examination Rules. Rule 5 of the Examination Rules deals
with eligibility conditions. Rule 5(c) prescribes minimum and maximum age limit of the
candidates, who may be eligible to participate in the recruitment process. It confers
power upon the State Government to vary the lower and upper age limit for any of the
services included in the Examination Rules, looking to the exigencies of services.
The State Government vide its Circular dated 16th September, 2008 superseding
its earlier circulars regarding maximum age limit, prescribed maximum age limit of 30
years for all the candidates other than the State of Chhattisgarh and 35 years for the
candidates, who are local residents of State of Chhattisgarh. The Examination Rules
have been accordingly amended and notified on 22nd September, 2008 and it has been
specifically clarified that the age limit shall be applicable as per Circular dated 16th
September, 2008 of the Govt. of Chhattisgarh, General Administration Department.
14.
Article 16 of the Constitution guarantees equality of opportunity for all citizens in
the matters relating to employment or appointment to any office under the State. Subclause (2) of Article 16 reads as under:
“(2) No citizen shall, on grounds only of religion, race, caste, sex,
descent, place of birth, residence or any of them, be ineligible for,
or discriminated against in respect of, any employment or office
under the State.”
15.
In Kendriya Vidyalaya, pursuant to the advertisement issued by the appellant-
Sangathan for recruitment to the post of Lower Division Clerk (LDC), the respondents
applied for appointment to the post of LDC. They were permitted to appear at the
examination and typing test even though they were overage, in contravention of the
relevant recruitment rules. The age limit prescribed therefore was 18 to 25 years as on
the appointed date, which was, however, relaxable. The higher authorities of the school
were moved for cancellation of the recruitment of LDCs. The Central Administrative
Tribunal directed the appellant – Sangathan to relax the age of the candidates. On appeal
by the Sangathan against the order of the Tribunal, the High Court held that the Tribunal
could not have directed for relaxation of age for appointment of the private respondents
until and unless the appointing authority exercises the power of relaxation of age limit
and directed the appointing authority to consider the case of the respondents for
relaxation of age limit, include their names in the select list and thereafter, issue
appointment orders to them in accordance with law on the basis of merits of the
candidates.
On further appeal by the Sangathan, the Supreme Court held that recruitment
rules as well as advertisement provides for age limit of 25 years for appointment to the
post of LDC. Relaxation could also be granted in favour of those, who fall within the
descriptions given in second part of Article 45 of the Education Code for Kendriya
Vidyalayas in deserving cases.
Since the appellants were bound by the rules, the
discretionary jurisdiction could be exercised for relaxation of age provided for in the
rules and within the four corners thereof. Since the respondents do not come within the
purview of the exception contained in Article 45 of the Education Code, the Tribunal or
the High Court could not issue any direction regarding relaxation of age.
16.
In the matter of Sanjay Pant, the respondent was granted scholarship for
prosecuting his studies by the Andaman Nicobar Administration. He had to execute a
personal bond to serve Andaman and Nicobar Administration for a minimum period of
three years. The respondent appeared before the interview board for selection to the post
of Statistical Assistant. However, he was not selected on the ground that he did not have
10 years continuous education in Andaman and Nicobar Islands and since he was not a
local candidate, he was not offered a regular appointment. The Tribunal allowed appeal
of the respondent on the ground that requirement of residence in a particular territory is
opposed to Article 16(2) of the Constitution; such restriction could be imposed only a law
made by Parliament under Article 16(3) of the Constitution. Dismissing the appeal of the
Sangathan, the Supreme Court confirmed the order of the Tribunal.
17.
In the matter of Kailash Chand Sharma, the Hon’ble Supreme Court interpreting
the use of word “only” in Article 16(2) held thus:
“An analysis of Article 16 indicated two things: firstly,
discrimination only on the ground of residence (or place of birth) insofar
as public employment is concerned, is prohibited: secondly, Parliament is
empowered to make the law prescribing residential requirement with a
State or Union Territory, as the case may be, in relation to a class or
classes of employment. That means, in the absence of a parliamentary
law, even the prescription of requirement as to residence within the State
is a taboo. However, the prohibitory mandate under Article 16(2) is not
attracted if the alleged discrimination is on grounds not merely related to
residence, but the factum of residence is only taken into account in
addition to other relevant factors. This, in effect, is the import of the
expression “only”.
18.
In the matter of Srimathi Cahmpakam Dorairajan, Government’s order in the
matter of admission in the Madras University was questioned on the ground that it was
inconsistent with Articles 15 & 29(2) of the Constitution as it constitutes discrimination
on the consideration of religion, race, caste, language etc. The Full Bench of the Madras
High Court, interpreting Articles 15(1) and 29(2) of the Constitution, held that the
aforesaid Articles would apply only if the persons of a particular religion, race or caste is
totally excluded on the ground of their religion, race or caste, but would not apply when
no person of any religion, race or caste is denied admission as such.
Hon’ble Shri Somasundaram J, in his concurring judgment, interpreting the word
“only” occurring in Article 15(1) and 29(2) held that any action of the State would be
prohibited under the aforesaid provisions only when discrimination or denial is solely on
the ground of religion, race, caste or language etc. It follows therefore that one of the
grounds of discrimination or denial may be on the basis of religion, race, caste, language,
but it should not be the sole ground.
19.
If we examine the facts of the present case in the light of above principles of law
laid down, we find that in the instant case, discrimination in prescribing maximum age
limit for recruitment to the State Civil Services is not based solely on the ground of place
of residence. The candidates of other States are also eligible to apply for the posts
advertised, provided they conform to the eligibility criteria prescribed under the
Examination Rules. The candidates of Chhattisgarh have been given relaxation in upper
age limit under the special circumstances. It has been contended by the respondents and
not disputed by the petitioners that 1494 candidates of other States have been found to be
eligible to participate in the Main Examination after result of the Preliminary
Examination and therefore, we are unable to accept the challenge to the Circular of
Annexure P/1 and fresh advertisement dated 20th September, 2008 issued on the basis of
Circular of Annexure P/1 on the ground that it is violative of Article 16(2) of the
Constitution.
III.
20.
Whether providing different upper age …………………… violation
of Articles 14 & 15 of the Constitution?
In the matter of Jitendra Kumar Singh, a dispute between the petitioners and the
respondents revolved around the issue of reservation of posts for Backward Classes,
Scheduled Casts, Scheduled Tribes, Woman candidates and Sports Persons. Under the
relevant rules, provisions for relaxation in fee and upper age limit of five yeas to OBC
etc. candidates were made. The Supreme Court considering that all the candidates i.e.
candidates belonging to Woman and OBC etc. categories as also the General category
were required to appear for Preliminary Written and Physical Test and Main Written
Examination and Interview, held that these were merely eligibility conditions for being
permitted to participate in the selection process. Thereafter, the candidates had to appear
in the Preliminary Written Test and after being successful to undergo Physical Test. A
candidate was also required to secure 50% or more marks. It was only those candidates
who qualified in the preliminary written test and the physical test became eligible to
appear in the main written test and only such candidates, who secured 40% or above
would be declared successful and only after being successful in interview, final merit list
was to be prepared on the basis of marks secured in the main written test and the
interview and thus, it is quite apparent that the concession in fee and age relaxation only
enabled certain candidates belonging to the reserved category to fall within the zone of
consideration. The concession in age did not in any manner tilt the balance in favour of
the reserved category candidates, in the preparation of final merit/select list.
It is
permissible for the State in view of Articles, 14, 15, 16 and 38 of the Constitution of
India to make suitable provisions in law to eradicate the disadvantages of candidates
belonging to socially and educationally backward classes.
Article 14 does not bar rationale classification.
It permits reasonable
classification for the purpose of legislation and prohibits class legislation. A legislation
intended to apply or benefit a “well defined class” is not open to challenge by reference
to Article 14 of the Constitution on the ground that the same does not extent a similar
benefit or protection to other persons, as has been held in Welfare Association, A.R.P.,
Maharashtra.
21.
In K.Thimmappa, while considering the prohibition under Article 14 of the
Constitution, the Hon’ble Supreme Court held that Article 14 prohibits class legislation
and not reasonable classification for the purpose of legislation. If the rule Making
Authority takes care to reasonably classify persons for a particular purpose and if it deals
equally with all persons belonging to a well defined class, then it would not be open to
the charge of discrimination. But to pass the test of permissible classification two
conditions must be fulfilled:(a) that the classification must be founded on an intelligible differentia which
distinguishes persons or things which are grouped together from other left
out of the group; and
(b) that the differentia must have a rational relation to the object sought to be
achieved by the statute in question.
22.
If we examine the facts of the present case in the light of law laid down by the
Supreme Court in the aforesaid judgments, we find that relaxation of five years in the
upper age limit was extended to the local residents of the State of Chhattisgarh, vide
Circular of Annexure P/1, in the peculiar circumstances that PSC examination could not
be convened annually in the last nine years. Relaxation of age only enabled certain
candidates belonging to the State of Chhattisgarh to fall within the zone of consideration.
The concession in age does not, in any manner, favour the candidates of Chhattisgarh in
preparation of final merit list as they are also required to participate in the preliminary
examination and only those candidates, who are successful in the preliminary
examination, participate in the main examination together with the successful candidates
of other States. They are also to face interview after being successful in the main
examination. In these circumstances, we are of the opinion that it is permissible for the
State to make suitable provisions in the relevant rules keeping in view the interest of the
candidates belonging to the State and the same cannot be termed arbitrary, discriminatory
or in violation of Articles 14 & 15 of the Constitution of India.
23.
In the result, the instant petition being without any substance deserves to be
dismissed and is hereby dismissed.
No order as to costs.
***
HIGH COURT OF CHHATTISGARH, BILASPUR
WRIT PETITION (S) NO.1673 OF 2010
D.D. 26.04.2010
Hon’ble Shri Satish K. Agnihotri, J
Dr. Kamlesh Kumar Jain
Vs.
The State of Chhattisgarh & Anr.
…
Petitioner
…
Respondents
Recruitment:
Defects in the application for the post – Whether candidates are entitled to intimation
before rejection? – No.
The petitioner was a candidate for the post of Assistant Professor (Non-clinical
Department) in Department of Health and Family Welfare (Medical Education) - As
there was no post in the Department of Health and Family Welfare (Medical Education)
in the advertisement the application of the petitioner was rejected – The petitioner filed
writ petition contending that those who have applied for the post of Demonstrator were
intimated that they should submit separate application for separate subject – Court in
view of the fact that as the Departments were clearly given in the advertisement form for
the post of Assistant Professor (Non-Clinical Department) High Court dismissed the
petition holding that the petitioner cannot take advantage on the ground that
Demonstrators were intimated for submission of applications to their specific branches.
ORDER
By this petition, the petitioner seeks a direction to the respondent No.2 to issue a
call letter in favour of the petitioner to participate in the examination, which was to be
held on 19.04.2010.
2.
The petitioner appearing in person submits that pursuant to the advertisement
dated 27.09.2008 (Annexure P/4), the petitioner applied for the post of Assistant
Professor, PSM. After lapse of two years, call letters were issued to the candidates to
participate in the selection process to be held on 19.04.2010. According to the petitioner,
by letter dated 08.04.2010 (Annexure P/7) the respondent/Public service Commission (for
short “PSC”) informed one Poornima Raj about rejection of her candidature. When the
petitioner has not received any communication/call letter from PSC, he approached the
authorities concerned and enquired about the non-receipt of call letter and he also made a
representation and requested for issuance of call letter, but till date the same has not been
issued.
The petitioner further submits that the action of the respondents is illegal,
arbitrary and bad in law. The petitioner has been deprived from participating in the
selection process without any rhyme and reason.
3.
The petitioner, pursuant to the advertisement dated 27.9.2009 (Annexure P/4),
made an application for consideration and selection on the post of Assistant Professor
(Non-clinical Department) in Department of Health and Family Welfare (Medical
Education) on 25.10.2008 (Annexure R-2/2). In the advertisement, there was no post in
the Department of Health and Family Welfare (Medical Education). Thus, according to
the learned counsel appearing for the respondent No.2, the application of the petitioner
was rejected and thereafter, no information was sent to the petitioner, as there is no
provision for intimation, thereon.
4.
On the perusal of the advertisement, it is found that there is no Department like
Health and Family Welfare (Medical Education).
In clause 20 and 21 (A) of the
advertisement, it is clearly provided that the application forms should be filled up
carefully and if information as required in the application is not given, the application
would stand rejected. Clause 21 and 21 (A) reads as under:
“Omitted as the matter is in Hindi”
5.
Dr. Jain, petitioner in person submits that the subject mentioned in the application
form i.e. Department of Health and Family Welfare (Medical Education) comes within
the department of PSM, thus it cannot be held that the petitioner has not given the
concerned department wherefor, he has applied for appointment on the post of Assistant
Professor.
6.
Contention of the petitioner is noticed to be rejected on the simple ground that a
candidate was required to fill only department, mentioned in the advertisement. There is
no such department like Department of Health and Family Welfare (Medical Education),
in clause B of the posts in non-clinical department in the advertisement.
7.
The next contention of the petitioner that in case of Demonstrators, an opportunity
to the candidates were given to specify branch in the application within a period of 15
days, the same was not done in case of the petitioner.
8.
Mr. Sharma, learned counsel for PSC, in response, submits that there is no
provision for entertaining any rectification in the application. In case of Demonstrators in
the original advertisement dated 27.9.2008, there were eight branches specified in the
advertisement.
A candidate who has obtained MBBS degree was eligible for
consideration of appointment and selection on the post of Demonstrator in all the
branches. Since maximum number of the candidates did not mention their branches, by
advertisement and corrigendum dated 15.5.2009 (Annexure P/5) it was specially
mentioned in para 7 & 8 that the candidates making an application for the post of
Demonstrators have to submit separate application for separate subject. Accordingly,
notices were issued to the Demonstrators, this was not done in the case of Assistant
Professor (non-clinical department), as, there is no ambiguity in the advertisement or
application form.
9.
Heard the petitioner in person and learned counsel appearing for the parties, it
appears that the Demonstrators, having regard to their essential qualifications, being the
same for all the eight branches applied for all branches. Thus, it became necessary to
intimate them that they should make an application for one branch only.
10.
In case of appointment of the Assistant Professor (non-clinical department) which
is the subject matter of the petition, the qualification for different departments were
different, as is evident from clause 4(B) of the advertisement. Even otherwise, the
petitioner cannot claim intimation enabling him to correct the department in the
application form, as the departments were clearly given in the advertisement form and
even the education qualification, which were different for all the departments, were
specifically provided in the advertisement. The petitioner cannot take any advantage on
the ground that demonstrators were intimated for submission of their specific branches.
The petitioner has failed to fill up correct department in the application for which he
made an application for selection and recruitment.
Thus, the action of the
respondent/PSC cannot be faulted with.
11.
For the reasons mentioned hereinabove, the petition fails and is hereby dismissed.
***
GOA PUBLIC SERVICE COMMISSION
IN THE HIGH COURT OF BOMBAY AT GOA
WRIT PETITION NO. 93/2002
D.D.04.09.2002
Hon’ble Mr. Justice S.Radhakrishnan &
Hon’ble Mr. Justice P.V. Hardas
Adv. Aires Rodrigues
Vs.
State of Goa & Ors.
…
Petitioner
…
Respondents
Public Service Commission – Composition and eligibility to be appointed as
Chairman/Member.
Petitioner a practising Advocate filed this petition as a public interest litigation
challenging the appointment of 2nd respondent as Chairman and 3rd respondent as
Member – The ground alleged is that they are not competent, honest and independent
persons whose integrity is extremely doubtful relying on the decision in Ashok Kumar
Yadav & Ors. v. State of Haryana & Ors. (1985) 4 SCC 417 – Regarding respondent
No.2 it is alleged there was a departmental inquiry against him in which he was punished
with penalty of compulsory retirement and on appeal he was exonerated merely on a
technical ground – Regarding respondent No.3 it is alleged that he being a retired Bank
Officer is not an eminent person – Regarding respondent No.2 it was found by the Court
that he was exonerated from the charge in the departmental inquiry not merely on
technical ground as alleged – Regarding respondent No.3 Court held that there is no
material to even remotely suggest that he is of doubtful integrity – Consequently writ
petition was dismissed by awarding costs of ` 5,000/- each to respondents No.2 and 3.
Held:
A public interest litigation should always be a bona fide vindication of grievance
which should not be coloured by either mala fide or personal vendetta or for seeking
personal gains.
Further held:
Filing of a public interest litigation cannot be used as a vehicle for gaining
publicity.
Further held:
Reckless allegations which cannot be substantiated on the basis of documents,
should never be made in a Court of law, particularly by the petitioner who is not a novice
to the law of pleadings and proof.
Further held:
The words "as nearly as may be" occurring in proviso to Article 316(1) suggests
that the proportion of 50% of the service members is not exact but approximate and is
meant not to be mandatory but directory. The said proviso does not, in terms, say that in
no case and at no point of time, the said proportion should either go above, or fall below
50%.
Cases Referred:
(1985) 4 SCC 417 - Ashok Kumar Yadav & Ors. v. State of Haryana & Ors.
(1993) 2 SCC. 597 - Jai Shankar Prasad v. State of Bihar & Ors.
ORDER
(Per HARDAS, J.)
Rule. Rule made returnable forthwith by consent of parties. The respondents
waive service. By consent taken up for final hearing.
2.
This petition filed in public interest by a practising Lawyer of the State of Goa,
takes exception to the Notification dated 23.5.2001, appointing the second respondent as
Chairman of the Goa Public Service Commission (hereinafter, for the sake of brevity,
referred to as ‘GPSC’) with effect from the date of his taking charge and the Notification
dated 2.7.2001, appointing respondent No.3 as a Member of the GPSC from the date of
his taking charge.
3.
Public interest litigation by now has come to be accepted as an effective remedy
to challenge public wrong or public injury caused by various acts or omissions of the
Government or Public Authorities. The general norms regarding pleadings and locus
standi are relaxed for public interest litigation because of the very nature of the challenge
and the person who files public interest litigation. A public interest litigation should
always be a bona fide vindication of grievance which should not be coloured by either
mala fide or personal vendetta or for seeking personal gains.
4.
The present petitioner who, as stated earlier, is a practising Advocate, has
challenged the appointment of respondents No.2 and 3 on the ground that respondents
No.2 and 3 are not persons of integrity, honesty and are not eminent persons in their field.
It is faintly urged that both the respondents are staunch supporters of the ruling
Government in Goa. In respect of respondent No.3, apart from making bald assertions
regarding not possessing integrity, honesty, etc., it is urged that his appointment is in
violation of Article 316 of the Constitution of India. A scathing attack has been made by
the petitioner in respect of the resume of the third respondent which is annexed to the
affidavit of the third respondent.
5.
In respect of the second respondent, it is stated that the second respondent, in a
Departmental Inquiry, had been punished with penalty of compulsory retirement by
Order dated 16.8.1999. On an appeal being preferred against this order by respondent
No.2, the Appellate Authority, by its order dated 4.2.2000 had exonerated him. The
petitioner has submitted that the exoneration was merely on technical grounds and while
so exonerating him, the Appellate Authority in the last paragraph of its Order, held as
follows:
" The delinquent official has deposited by way of cheque the amounts
which he has obtained towards the House Building Advance or portion
thereof as has remained unpaid. In case the said cheque has not been
encashed by the Corporation, and the same is valid, I order the
Corporation to encash the same and adjust towards the loan account of the
delinquent official. In case validity period of the cheque has expired, I
direct the delinquent official to deposit the necessary amounts within 15
days from receipt of this Order."
6.
Thus, relying on the observations of the Apex Court in the case of Ashok Kumar
Yadav and others v. State of Haryana and others, (1985) 4 SCC 417 and particularly on
the observations of the Apex Court at para 30, it is submitted before us that the
respondents No.2 and 3 ought not to have been appointed as Chairman and Member of
the GPSC, as they are not competent, honest and independent persons, whose integrity is
extremely doubtful. A relief of issuance of a writ of quo-warranto in terms of prayer
clause is prayed for along with a writ of mandamus for quashing appointments of
respondents No. 2 and 3.
7.
On notice being issued by this Court, the respondents have filed their affidavits.
The affidavit of respondent No.3 discloses that respondent No.3 has served in various
capacities in Bank of India. He had retired from the Bank as Deputy Chief Officer,
Departmental Head of Public Relations Department. It is also stated in the affidavit that
the respondent No.3 was posted in 1989-90 at the Bank’s London Branch. The second
respondent, in his affidavit has brought to fore and has highlighted the reasons for
initiation of the Departmental Proceedings. He has also given reasons as to why the
amount of House Building Advance was not paid to the builder. These facts are not
germane for deciding the controversy in the Petition, as these facts were placed before the
Appellate Authority who has accepted them and exonerated respondent No.2. The
petitioner had chosen not to file the full text of order of the Appellate Authority, but only
a brief portion of the order was referred to in para 3 of the petition. The full text of the
order of the Appellate Authority has been annexed by the respondents. A perusal of the
Appellate Order would show that the Appellate Authority had not exonerated the
respondent No.2 on mere technical grounds. In paragraph 20 of the order of the Appellate
Authority, the Appellate Authority, has observed as under:
" (20) From the file it is evident that after remarks of the Chief
Development Officer dated 4.10.1999, there has been no serious follow up
by the Corporation. The apparently closed issue is raked up after 8 years,
only when the delinquent official makes certain statements against the ExManaging Director. The complaint from Nizari appears to have been
procured by the ex-Managing Director, as the complaint itself states that
the same is pursuant to the query raised by the ex-Managing Director. The
ex-Managing Director does not depose as to what was the occasion for
raising such a query. Nizari also is not examined during the course of
inquiry. All these coupled with the defence evidence that complaint was
procured by the ex-Managing Director for victimizing the delinquent
official for having made representation dated 19.11.1997, does create
serious doubt of motive of initiating the disciplinary proceedings."
8.
In the factual matrix, therefore, the petitioner has urged before us that admittedly,
the second respondent was proceeded against departmentally and the Inquiry Officer had
found the second respondent guilty and had imposed a penalty of compulsory retirement.
It is further urged before us by the petitioner that the Appellate Authority has exonerated
the Second Respondent, but while exonerating the second respondent, the Appellate
Authority has directed that the cheques deposited by the second respondent towards the
repayment of the house building advance should be encashed by the Economic
Development Corporation. The Appellate Authority has further observed that in case the
validity of the cheques had expired, the Delinquent Official, meaning the respondent
No.2, would deposit the necessary amount within 15 days from the receipt of the order.
Relying upon the observations of the Appellate Authority, it is contended by the
petitioner before us that the integrity of respondent No.2 is extremely doubtful and
respondent No.2 ought not to have been appointed as a Chairman of the GPSC.
9.
We will take up the case as is made out against respondent No.2 at a later stage
and we will examine whether the petitioner has been able to make out any case against
respondent No.3 first to justify the exercise of the power of judicial review of the action
of respondent No.1 in appointing respondents No.2 and 3. The petitioner did concede
during the hearing that there was no material against respondent No.3. Apart from saying
that the appointment of respondent No.3 is in direct violation of Article 316 of the
Constitution of India, the petitioner has not been able to substantiate any ground
challenging the appointment of the third respondent. Though in the petition at para 6 it is
stated that the appointment of respondents No.2 and 3 as Chairman and the Member of
the GPSC was a subject matter of a public outcry and also representation made by the
Leader of Opposition, the petitioner was quick enough in submitting that the
representation was in respect of the appointment of respondent No.2 and not as against
respondent No.3. The only ground which is urged before us in respect of the appointment
of respondent No.3 is that he cannot be classified as ‘an eminent person’ deserving to be
appointed as a Member of the GPSC. According to the petitioner, respondent No.3 was
concerned mostly in discharging his duties in the department of public relations. The
submission of the petitioner is wholly misplaced. A perusal of the resume respondent
No.3 had a total experience of working in a Bank for 23 years and 11 months.
Respondent No.3 has held various positions, including that of the Branch Manager and
Officer Incharge, Foreign Exchange Cell. Suffice it to say that in the absence of any
valid challenge, in a writ of quo warranto, the Court cannot go on a fishing expedition in
trying to ascertain the respondents’ requisite qualifications for being appointed. The
appointment of respondent No.3 is, to a certain extent, challenged in a very casual
manner. There is a total absence of material to even remotely suggest that respondent
No.3 is of doubtful integrity. We cannot substitute our opinion on the mere ipse dixit of
the petitioner. There is no yardstick to judge whether respondent No.3 is or is not suitable
for holding the post to which he is appointed. The positions earlier held by respondent
No.3 do not in any manner suggest that respondent No.3 is not qualified to hold the post.
There is no material against respondent No.3 which would disentitle him to hold this
post. The challenge to the appointment of respondent No.3, therefore, must fail. 10.
Turning to the allegations which have been made against respondent No.2, it is to be
noticed that though respondent No.2 was held guilty in departmental proceedings, the
respondent No.2 was ultimately exonerated by the Appellate Authority by a detailed and
a reasoned order. The Appellate Authority has come to a conclusion that the respondent
No.2 was victimized and the entire inquiry had been initiated because of a representation
which had been filed by respondent No.2 complaining therein various nefarious activities
which were being carried on in the department with the blessings of the then Managing
Director. The petitioner is, therefore, not correct in urging before us that respondent No.2
has been exonerated on technical grounds. The Appellate Authority has given a clean
chit to respondent No.2 and his exoneration wipes out any blemish or stigma in relation
to his integrity which was existing because of the order of the Inquiry Officer.
Respondent No.2, therefore, emerges unblemished after the order of the Appellate
Authority. The Appellate Authority, in respect of the direction regarding deposit of
amount, reproduced in para 3 of the petition, has clarified it in its order, which reads as
under :
" It is clarified that the order for reinstatement with all
consequential benefits is not linked to the direction to deposit of amount
and the Delinquent Official shall be reinstated with all consequential
benefits forthwith."
Thus, the direction of the Appellate Authority to deposit the amount in no way casts a
cloud on the integrity of the second respondent. To do so would be drawing an inference
from an order of an Authority which did not intend any inference to be drawn.
11.
It is urged before us by the petitioner that though respondent No.2 was exonerated
by the Appellate Authority, the fact remains that respondent No.2 was proceeded against
departmentally and an officer who was proceeded against departmentally should not be
considered to the highest office of the Chairman of GPSC. It is true that respondent No.2
was proceeded against departmentally. The fact that the departmental inquiry had been
held against him, cannot be used as lever to disentitle respondent No.2 from being
appointed to the highest office of the GPSC merely on that score. Respondent No.2 has
been exonerated by the Appellate Authority and the reasons of the Appellate Authority
cannot be lost sight of. The Appellate Authority has clearly held that respondent No.2
was victimised because of his complaints. The order of the Appellate Authority,
therefore, has the effect of wiping the slate clean and no stigma can be attached to
respondent No.2 because of the departmental action or inquiry that had been initiated
against him.
12.
Learned Advocate General, appearing on behalf of the State has drawn our
attention to the Judgment of the Apex Court in Jai Shankar Prasad v. State of Bihar and
others, (1993) 2 SCC. 597. This Judgment is a complete answer to the challenge of the
petitioner in respect of Article 316 of the Constitution of India.
13.
Article 316 of the Constitution of India, reads as under :
" 316. Appointment and term of office of members. - (1) The
Chairman and other members of a Public Service Commission shall be
appointed, in the case of the Union Commission or a Joint Commission,
by the President, and in the case of a State Commission, by the Governor
of the State;
Provided that as nearly as may be one-half of the members of
every Public Service Commission shall be persons who at the dates of
their respective appointments have held office for at least ten years either
under the Government of India or under the Government of a State, and in
computing the said period of ten years any period before the
commencement of this Constitution during which a person has held office
under the Crown in India or under the Government of an Indian State shall
be included.
[(1A). If the office of the Chairman of the Commission becomes
vacant or if any such Chairman is by reason of absence or for any other
reason unable to perform the duties of his office, those duties shall, until
some persons appointed under clause (1) to the vacant office has entered
on the duties thereof or, as the case may be, until the Chairman has
resumed his duties, be performed by such one of the other members of the
Commission as the President, in the case of the Union Commission or a
Joint Commission, and the Governor of the State in the case of a State in
the case of a State Commission, may appoint for the purpose.]
(2) A member of a Public Service Commission shall hold office for
a term of six years from the date on which he enters upon his office or
until he attains, in the case of the Union Commission, the age of sixty-five
years, and in the case of a State Commission or a Joint Commission, the
age of [sixty-two years, whichever is earlier:
Provided that (a) a member of a Public Service Commission may, by writing
under his hand addressed, in the case of the Union Commission or a Joint
Commission, to the President, and in the case of a State Commission, to
the Governor of the State, resign office;
(b) a member of a Public Service Commission may be removed
from his office in the manner provided in clause (1) or clause (3) of article
317.
(3) A person who holds office as a Member of a Public Service
Commission shall, on the expiration of his term of office, be ineligible for
a re-appointment to that office. "
The petitioner states that when respondents No.2 and 3 were appointed, the mandate of
Article 316 of the Constitution had been fulfilled as Mr. P.A. Debras was the other
Member of the GPSC. According to the petitioner, on retirement of Mr. Debras, the
mandate of Article 316 is not fulfilled as neither the second respondent nor the third
respondent can be said to have held the office for at least 10 years either under the
Government of India or under the Government of State. Countering this submission of
the petitioner, the learned Advocate General has referred to Jai Shankar Prasad’s case.
The Apex Court, in paragraph 9 of the Judgment has held as under :
" 9.
It is apparent from these provisions that the Chairman and other
members of the State Public Service Commission are appointed by the
Governor of the State. The appointments are obviously made on the advice
of the Council of Ministers of the State. The proviso to clause (1) of the
Article requires that "as nearly as may be", one half of the members of the
Commission shall be persons who on the dates of their respective
appointments have held office for at least ten years either under the
Government of India or under the Government of a State. For brevity’s
sake we may refer to this category of members as service members. The
expression "as nearly as may be" itself suggests that the proportion of 50
% of the service members is not exact but approximate and is meant not to
be mandatory but directory. The said proviso does not, in terms, say that in
no case and at no point of time, the said proportion should either go above,
or fall below 50 %. In the very nature of things, a strict adherence to the
said direction is not practicable at any particular point of time. In the first
instance, the superannuation age of the member of the Commission is 62
years and his total tenure as a member cannot exceed six years. He has to
vacate his office either when his tenure comes to an end or when he attains
the age of 62 years whichever is earlier. When the members are appointed,
they are bound to differ in age, whether they belong to the service
category or the non-service category. In the normal course, they would
retire at different points of time. If it is insisted, as is done on behalf of the
appellant, that the said requirement must be followed strictly at all times, it
would be well nigh possible to do so. Every time a member, whether
belonging to the service or the non-service category, retires, there should
be available a suitable person from the same category to be appointed in
his place. It is not always possible to make an advance list of persons of
either category who are suitable for such appointments. Hence, the total
strength of the Commission as well as the number from each of the
categories, are bound to vary from time to time. At any given point of
time, therefore, it may not be possible to maintain the proportion between
the two categories strictly in accordance with the direction given in the
Constitution. It appears that it is for this reason that the words "at least
half" used in the proviso to Section 265(1) of the Government of India
Act, 1935, corresponding to the present proviso to Article 316(1), have
been substituted by the words "as nearly as may be one-half"."
14.
The vacancy created by the retirement of Mr. Debras, in no way affects the
validity of the composition of the present GPSC and it could not be said that the
composition is in direct breach of Article 316 of the Constitution. During the course of
arguments, learned Advocate General has produced before us the confidential file relating
to the appointment of the incumbent in the vacancy created on retirement of Mr. Debras
and on a perusal, we are satisfied that on such appointment being made, the mandate of
Article 316 of the Constitution would be satisfied. In fact, the learned Advocate General
did stress before us that the appointment of the incumbent in place of Mr.Debras would
not be in breach of Article 316 of the Constitution.
15.
We have given our careful and anxious consideration to the submissions advanced
by the petitioner and the learned Counsel appearing on behalf of the respondents.
Reckless allegations which cannot be substantiated on the basis of documents, should
never be made in a Court of law, particularly by the petitioner who is not a novice to the
law of pleadings and proof. In the present case, we are constrained to observe that
though the appointments of respondent No.2 and 3 were made in May, 2001 and July
2001 respectively, a petition ostensibly in public interest seeking to challenge their
appointments, came to be filed in this Court on 8.3.2002. Undisputedly, in a public
interest litigation, the petitioner does derive certain publicity. In the present case, the
contents of the petition appeared in the news-papers on the very next day of its lodging
and even before the matter was placed before the Bench for passing appropriate orders.
The contents of the petition appeared in the news papers even before the petition was
formally registered in the Registry. A petitioner in a public interest litigation should not
normally be a person who is seeking publicity of the fact of having filed a petition in the
Court. Certain degree of publicity is attendant to the decision of such petition which
normally challenges sensitive issues before the Court. Filing of a public interest litigation
cannot be used as a vehicle for gaining publicity. Be that as it may, the petitioner has
assured us that he would, in future, not be privy to passing information to the press,
regarding the contents of a petition yet to be registered. We accept the explanation of the
petitioner and do not propose to dilate on this issue, any further.
16.
Though the petitioner has taken exceptions against respondents, challenging the
appointments of respondents No.2 and 3, we have reached to a conclusion that there is no
substance at all in the challenge which is made against respondents No.2 and 3. The
petition is founded on reckless allegations, which cannot be substantiated on the basis of
record. Only an extract of the order of the Appellate Authority was reproduced in the
petition, without annexing the full text of the order. The Courts cannot countenance any
attempts to mislead the Court, much less in a public interest litigation. We have,
therefore, no other alternative but to dismiss the petition and though we do not agree with
the submission of learned Advocate General for imposing exemplary costs, we dismiss
the petition awarding costs of Rs.5,000/- each to respondents No.2 and 3.
17.
Writ Petition No. 93/2002 is dismissed, with costs. Rule is discharged.
***
IN THE HIGH COURT OF BOMBAY AT GOA
WRIT PETITION NO.116 OF 1998
D.D. 30.12.2003
Hon’ble Mr. Justice A.M. Khanwilkar &
Hon’ble Mr. Justice P.V. Hardas
Shri R.R. Sohani & Ors.
Vs.
Shri V.O. Madhavanunni,
Surveyor of Works,
Government of Goa, Goa & Ors.
…
Petitioner
…
Respondents
Promotion- Seniority:
Whether initial appointment of the petitioner in 1978 as Assistant Engineer in the Soil
Conservation Division, Directorate of Agriculture, can be counted for determining
seniority in P.W.D.? – No.
The petitioner was initially appointed on temporary basis as Assistant Engineer in
the Soil Conservation Division, Directorate of Agriculture, on 17.4.1978 and regularly
appointed in the same Department on 3.11.1980 by making clear that he will not be
entitled for any benefits to past service – This writ petition is filed seeking quashing of
Government Order dated 25.9.1997 in so far as it places respondents No.3, 5, 6, 8, 9, 10
and 12 above the petitioner and accords them seniority in the grade of Executive
Engineer in the Irrigation Department – As per the Recruitment Rules of 1969, the post of
Executive Engineer was to be filled by promotion, failing which by transfer on deputation
and failing both, by direct recruitment – For promotion minimum of 8 years service as
Assistant Engineer - By order dated 17.2.1983 the technical posts of Soil Conservation
Division, Directorate of Agriculture were transferred to PWD and as a result the
petitioner was assigned to PWD – The Government cancelled its earlier order dated
17.2.1983 transferring technical posts of Soil Conservation Division to PWD - In view of
this, the services of the petitioner stood restored to the Soil Conservation Division – Only
by order dated 18.5.1989 the petitioner was transferred to Irrigation Department in public
interest - The petitioner’s entry in Irrigation Department will have to be reckoned from
18.5.1989 – The petitioner was ineligible for being considered in the Irrigation
Department at least for a period of 8 years therefrom which means he was to be eligible
only in 1997 – After examining the whole matter the Court held that several grounds
alleged by the petitioner for quashing the seniority list are not tenable and consequently
dismissed the writ petition.
Held:
The effect of decision of dereservation of vacancy will have prospective effect
and not relate back to the year of that vacancy.
Cases Referred:
1972 SLR 44 - State of Assam & Anr. vs. Raghava Rajgopalchari
AIR 1977 SC 781 - State of Haryana & Anr. vs. The Karnal Distillery Co. Ltd.,
(1980) 3 SCC 29 - Dr.N.C. Singhal vs. Union of India
1981 (2) SLR 253 - Varghese & Ors. vs. State of Kerala & Ors.
1987 SC 2291 - K. Madhavan v/s. Union of India
1988 (3) SCC page 570; Assistant Commissioner of Commercial Taxes (Asst.) Dharwar
& Ors. vs. Dharmendra Trading Company & Ors.
AIR 1990 SC 442 - Union of India & Ors. vs. Vadera & Ors.
1994 Supp.(2) SCC 491 - Harish Chandra Ram vs. Mukh Ram Dubey & Ors.
AIR 1994 SC 1808 - J&K Public Service Commission etc, vs. Dr.Narinder Mohan & Ors.
(1997) 9 SCC 287 - Union of India & Ors. v/s. N.R. Banerjee & Ors.
(1999) 8 SCC 16 - Maharaja Chintamani Saran Nath Shahdeo vs. State of Bihar & Ors.
AIR 2000 SC 594 - S.I. Rooplal & Anr. vs. Lt. Governor
(2001) 4 SCC 675 - Vinod Kumar Sharma vs. State of U.P. & Anr.
(2001) 8 SCC 676 - Bharathidasan University & Anr.vs. All India Council for Technical
Education & Anr.
2002(2) GLT 213 - M/s. Consolidated Distributors Pvt. Ltd. vs. Economic Development
Corporation & Anr.
(2002) 2 SCC 712 in G.N. Nayak vs. Goa University & Ors.
(2002) 3 SCC 586 - K. Shekar vs. V. Indiramma & Ors.
ORDER
(Per KHANWILKAR, J.)
By the present writ petition, the petitioner prays for quashing the order of the
Government of Goa No.3/25-20/85/IRRG/729 dated 25.9.1997 published in the Official
Gazette Ser. II No.37 dated 11.12.1997 in so far as it places the respondents No.3, 5, 6, 8,
9, 10 and 12 above the petitioner and accords them seniority in the grade of Executive
Engineer in the Irrigation Department, higher than the petitioner. The petitioner further
prays for the direction against the Government of Goa to issue a fresh order promoting
the petitioner and respondents No.1 to 15 to the post of Executive Engineer in the
Irrigation Department granting seniority to the petitioner and placing the petitioner higher
than the abovenumbered respondents.
2.
The petition, as filed, asserts that the petitioner and the respondents No.1 to 15
were appointed as Assistant Engineers with the dates of respective appointments as stated
in para 3(d) of the petition. It is stated that the petitioner and respondents No.1 to 8 and
14 to 16 were promoted as the Executive Engineers in the Irrigation Department on ad
hoc basis on the dates as mentioned in paragraph 3(e) of the petition. In substance, it is
the case of the petitioner that he was appointed as Assistant Engineer on 3.11.1977 and
was promoted as Executive Engineer in the Irrigation Department, on ad hoc basis, on
8.2.1988. Whereas, the concerned contesting respondents were appointed on later dates.
It is further stated that all the incumbents who were appointed as ad hoc promotees were
continued as such till the date of the impugned order. It is then asserted that the
Departmental Promotion Committee (DPC)/Goa Public Service Commission (GPSC) for
recommending regular promotions of Assistant Engineer/Assistant Surveyor of Works to
the post of Executive Engineer/Surveyor of Works in the Irrigation Department did not
meet in the years 1985 to 1986, so as to recommend the promotions to the post of
Executive Engineer available in those years. It is stated that in the year 1987 to 1989, the
posts of Executive Engineer were available for being filled in by regular promotions. It is
further stated that under Office Memorandum No.3/12/85/Irg/87 dated 23.4.1986, the
Government finalised the seniority list of the Assistant Engineers in the Irrigation
Department. On these averments, the petitioner has challenged the impugned order
granting seniority to the abovenumbered contesting respondents, thereby granting them
seniority over the petitioner in the grade of Executive Engineer. The grounds on which
the impugned order has been challenged can be culled out from para 4 of the petition.
According to the petitioner, the DPC/GPSC/Government have acted unconstitutionally,
arbitrarily and illegally as:- (1) they have acted in breach of the guide-lines of the DPC
contained in Office Memorandum 12/14/89-PER dated 26.9.1990, in particular,
paragraphs 6.4.1, 6.4.2 and 6.4.3 thereof; (2) that nine posts of Executive Engineer were
available for recommending regular promotions for the year 1987-88. The petitioner was
eligible for promotion in the year 1987-88; whereas, the respondents 3, 5 and 6 were
ineligible for promotion as they had not completed the required 8 years’ regular service in
the feeder grade as per the recruitment rules called "Government of Goa, Daman and Diu,
Irrigation Department, Group "A" and "B" Gazetted Post, Recruitment Rules, 1986.".
Notwithstanding this, the respondents No.3, 5 and 6 were considered along with the
petitioner as against the Vacancies in the year 1987-88 and on comparative evaluation on
their merits, Respondents No.3 and 5 have been promoted to the posts of Executive
Engineer against the year 1987-88; (3) if the said respondents No.3 and 5 were not to be
considered for promotion to the post of Executive Engineer for the year 1987-88 along
with the petitioner, the petitioner would have found place in the impugned order higher
than the respondents No.3, 5, 6, 8, 9, 10, 11 and 20. These are the only grounds on which
the impugned order has been assailed by the petitioner.
3.
In response, the respondent No.3 filed affidavit dated June 29, 1998. According to
the respondent No.3, the petition is liable to be rejected as the petitioner has suppressed
the material particulars and also because the petition is barred by laches. It is then averred
that the respondent No.3 has reason to believe that the petitioner does not possess the
basic qualification of a Degree in Civil Engineering which is essential for promotion to
the post of Executive Engineer. It is then asserted that the petitioner has wrongly stated
that he was appointed to the post of Assistant Engineer on 3.11.1977; whereas, the
petitioner was appointed vide Order dated 17.4.1978 on temporary basis to the post of
Assistant Engineer in the Soil Conservation Division, Directorate of Agriculture, Panaji
with effect from 2.3.1978. It is the case of the respondent No.3 that the initial
appointment of the petitioner as Assistant Engineer being in the Soil Conservation
Division of the Directorate of Agriculture, the same has no nexus with the Public Works
Department (PWD), which came to be subsequently bifurcated as PWD and Irrigation
Department in 1984. Respondent No.3 has then averred that by Order dated 17.2.1983,
with a view to streamline the working of the Soil Conservation Division, the Government
was pleased to transfer the technical posts in the Soil Conservation Division along with
the incumbents to the PWD with immediate effect. Thereafter, by Notification dated
2.4.1984, Public Works Department was bifurcated into PWD and Irrigation Department
and by order dated 1.12.1984, the technical posts of the Soil Conservation Division of the
Directorate of Agriculture were included in the cadre of Irrigation Department.
Subsequently, by order dated 22.12.1987, the Government was pleased to cancel the
order dated 17.12.1983 in terms whereof the technical posts in the Soil Conservation
Division of the Directorate of Agriculture along with the incumbents were transferred to
PWD. It is stated further that in terms of order dated 22.12.1987, the technical posts and
the incumbents holding the same stood transferred to the Soil Conservation Division of
Directorate of Agriculture, on account of which the petitioner ceased to be the Assistant
Engineer in the Irrigation Department to that effect from 22.12.1987 and instead became
the Assistant Engineer in the Soil Conservation Division of Directorate of Agriculture. It
is further averred that in fact, by order dated 8.2.1988, the petitioner was promoted, on ad
hoc basis, to the post of Executive Engineer in the Soil Conservation Division of the
Directorate of Agriculture. Respondent No.3 thus, states that the case made out by the
petitioner that he ought to have been considered as against the vacancy that arose in the
year 1987-88 to the post of Executive Engineer in the Irrigation Department is totally
misplaced because during the year 1987-88, the petitioner was not at all in the cadre of
Assistant Engineer in the Irrigation Department. It is urged by respondent No.3 that all
these vital facts have been suppressed by the petitioner, on which count alone the
petitioner is disentitled to any reliefs. It is then stated in the reply-affidavit filed by
respondent No.3 that Government of Goa by Order dated 18.5.1989 was pleased to
transfer in ‘public interest’ certain Executive Engineers, including the petitioner from the
Soil Conservation Division of the Directorate of Agriculture to the Chief Engineer’s
Office, Irrigation Department. According to respondent No.3, therefore, the entry of the
petitioner in the cadre of the Assistant Engineers in the Irrigation Department, at best, can
be stated to have taken effect from 18.5.1989 and not from 3.11.1977 as is contended by
the petitioner. Respondent No.3 has then adverted to order dated 28.6.1989, which
indicates that there was no question of exercise of any option by the employees of the
Soil Conservation Division, Directorate of Agriculture, as there was no bifurcation as
such. It is asserted that no option was afforded to the employees of the Social
Conservation Division, at the time of issuance of the order dated 17.2.1983 or for that
matter at the time of issuance of order dated 22.12.1987. It is further stated that the
petitioner accepted his posting in the Directorate of Agriculture pursuant to the order
dated 22.12.1987 and further was promoted as Executive Engineer vide order dated
8.2.1988. It is next averred that in terms of the Recruitment Rules, promotions to the post
of Executive Engineer (Civil) in the Irrigation Department is from amongst the Assistant
Engineers (Civil)/Asst. Surveyor of Works, with eight years regular service in the
respective grades. Besides, the note appended to the Recruitment Rules specifies that the
eligibility for promotion regular service as Assistant Engineer in the PWD prior to
transfer to the Irrigation Department and regular service in the Irrigation Department
shall be counted. It is the case of the respondent No.3 that therefore, it is clear that the
service rendered in the Directorate of Agriculture can, by no means, be counted for the
purposes of eligibility for promotion to the post of Executive Engineer in the Irrigation
Department. On this premise, it is asserted that the petitioner can, at the highest, claim
regular service in the Irrigation Department with effect from 17.2.1983 upto 22.12.1987
and thereafter, with effect from 18.5.1989 onwards. It is the case of the respondent No.3
that for the purposes of determining the eligibility of the petitioner, only the period from
18.5.1989 can be reckoned and on that basis, the petitioner would become eligible for
promotion to the post of Executive Engineer in the Irrigation Department only in the year
1997. It is then contended that even if the earlier period of almost 4 years between
17.2.1983 to 22.12.1987 was to be counted, even then the petitioner would become
eligible to be considered for promotion to the post of Executive Engineer (Civil) only in
the year 1993; and therefore, there was no question of considering the petitioner for
promotion in respect of the vacancies which arose prior to the year 1993. In para 13 of
the reply-affidavit the details regarding yearwise vacancy position from the year 1984 has
been furnished. Similar information has been provided in the reply-affidavit filed on
behalf of the respondent No.16, GPSC to which we shall make a reference a little later. In
paragraph 15 of the reply-affidavit, respondent No.3 has stated that he was appointed as
Assistant Engineer in the PWD on ad hoc basis with effect from 9.11.1972, which
appointment was with the concurrence of the Union Public Service Commission. It is
then stated that he came to be regularly appointed as Assistant Engineer in the PWD with
effect from 3.11.1980 and his services came to be allotted to the Irrigation Department
with effect from 2.4.1984. It is averred that in terms of Recruitment Rules, the services
rendered by him in the PWD prior to formation of Irrigation Department are liable to be
considered for the purposes of determining the eligibility. According to respondent No.3,
his initial appointment in the year 1972 was in terms of the Recruitment Rules then
prevalent and moreover, the same was in concurrence with the UPSC. It is also asserted
that when he came to be appointed at that time there was a regular vacancy. On this
premise, it is stated that it was obligatory on the part of the Government to treat his
appointment with effect from 9.11.1972 as being regular appointment and at any rate, in
terms of the Office Memorandum dated 25.6.1980 itself, the DPC is required to make
appointment in respect of the anticipated vacancies. On the above basis, the respondent
No. 3 has opposed this writ petition.
4.
Respondent No.16, GPSC filed affidavit of Shri D.M. Borkar, Under Secretary
dated 6.7.1998 for opposing this writ petition. It is stated in this affidavit that the
petitioner has not properly projected the relevant facts of the present case and instead
countered that the DPC has followed the correct procedure. It is then averred that the
selections made by the GPSC are in accordance with the established and legitimate
procedure and there is nothing wrong whatsoever in the selection of the candidates. It is
stated that in the month of June, 1997, the Government had sent a proposal to consider
the Assistant Engineers for promotion to the post of Executive Engineer/Surveyor of
Works in the Irrigation Department against 20 existing vacancies pertaining to 9
recruitment years from the year 1984 to 1997. The details of the said vacancies are
furnished in paragraph 3, which read as under :
Sr.
No.
Year
of
Vacancies
Recruitment General S.C.
S.T.
Total
1
2
3
4
5
6
1.
1984
-
1
-
1
2.
1985
1
-
-
1
3.
1986
1
-
1
2
4
1987
2
-
-
2
5.
1988
2
1
-
3
6.
1989
2
-
-
2
7.
1993
1
-
-
1
8.
1996
1
1
-
2
9.
1997
5
-
1
6
Total
15
3
2
20
It is then stated by the respondent No.16 that the vacancies for the year 1984 and 1985
were to be filled in as per the recruitment rules framed in 1969 and the remaining
vacancies were to be filled in as per the Recruitment Rules framed in the year 1986. As
per the Recruitment Rules of 1969, the post of Executive Engineer was to be filed by
promotion, failing which by transfer on deputation and failing both, by direct recruitment.
Assistant Engineers (Soil Conservation) with 5 years service in the grade and Assistant
Engineer/Assistant Surveyor of Works with 8 years service in the grade and possessing
degree in Civil Engineering were to be considered for promotion. It is stated that
consequent to the formation of separate Department of Irrigation, Recruitment Rules of
1969 were revised and published in the Official Gazette dated 3.10.1986. According to
the revised Rules of 1986, the Post is to be filled in by promotion, failing which transfer
on deputation and failing both, by direct recruitment. In so far as eligibility criteria is
concerned, service rendered by the Assistant Engineers (Civil)/Assistant Surveyor of
Works (Civil) to PWD prior to their transfer to the Irrigation Department is also to be
counted. It is stated that the post of Executive Engineer is a Selection Post. In paragraph 8
onwards the procedure followed has been referred to. It is stated that both the sets of
Recruitment Rules were taken into consideration by the DPC for recommending officers
for promotion according to the Recruitment Rules as applicable relating to the year of the
vacancy. It is stated that the post of Executive Engineer being a Selection Post, as per the
DPC guide-lines, the vacancies occurred in the year 1984 to 1989, and 1996 and 1997 are
to be filled in on the basis of merit and the vacancy of 1993 is to be filled on the basis of
bench mark grade, prescribed for the purpose, which in this case is "Good". It is stated
that the DPC which met on 8.8.1998 considered the eligible officers coming in the zone
of consideration for the promotion yearwise. It is further stated that the Government had
reported that there were no officers belonging to the reserved categories in the feeder post
and action was afoot to dereserve the reserved vacancies. In para 11 of this affidavit, it is
stated that in normal course as per the DPC procedure, the DPC has to consider 24
officers who come in the zone of consideration for promotion for 20 vacancies of
different years. However, the Department had reported that there were only 23 officers
eligible who fulfill the requirement of Recruitment Rules for promotion to the post of
Executive Engineer. In para 12, it is stated that the DPC considered the eligible officers
yearwise as per the details given therein. It is further stated that 8 years Confidential
Reports preceding the year of the vacancies/vacancy were assessed by the DPC for
recommending the officers for promotion. In so far as one vacancy of 1985 (General) as
well as one vacancy of 1986 (General), it is stated that the petitioner did not come in the
zone of consideration. In so far as two vacancies of 1987 (General), after deleting the
name of the officer recommended for promotion against the vacancy of 1986, 5 eligible
officers including the petitioner in order of their seniority, were considered for
promotion. The petitioner was assessed as "Good" by the DPC, but could not get selected
as the officers junior to him (respondents No.1 and 2 herein) who secured higher grading
were recommended for promotion by the DPC. It is specifically averred that respondents
No.3, 5 and 6 were not considered against these vacancies as they did not have 8 years’
service in Grade of Assistant Engineer. It is then stated that in so far as two vacancies of
1988 (General), after deleting the names of the officers recommended for promotion
against the vacancies of the year 1987, eight eligible officers including the petitioner, in
order of their seniority, were considered for promotion. The petitioner was assessed by
the DPC as "Good", but could not get placing in the select list because the DPC
recommended one officer (respondent No.3 herein) junior to the petitioner who had
secured higher grading and another officer (the respondent No.4) with grading equal to
that of the petitioner, but senior to the petitioner. It is stated that respondent No.3 was
considered for promotion as he had completed 8 years’ service in the year 1988, as his
date of appointment being 3.11.1980. In so far as two vacancies of 1989 (General) are
concerned, after deleting the names of the two officers recommended for promotion
against the vacancies of 1988, eight eligible officers, including of the petitioner in order
of merit, who formed the zone of consideration, were considered for promotion against
two vacancies of 1989. The petitioner was also considered for promotion along with
others and he was assessed by the DPC as "Good". It is stated that the petitioner could not
get placing in the select list as two officers (respondents No.5 and 6 herein) who were
junior to him had obtained higher grading and were recommended for promotion. In so
far as one vacancy of 1993 (General) is concerned, after deleting the names of two
officers recommended against 1980 vacancies, five eligible officers, including of the
petitioner in order of seniority, who came within the zone of consideration, were
considered for promotion by the DPC. The selection was on the basis of bench mark
grade prescribed for promotion which is "Good". The petitioner was assessed as "Good",
but the officer senior to the petitioner in the feeder grade who was also assessed as
"Good" was recommended by the DPC for promotion to the post of Executive Engineer,
on account of which the petitioner did not get placing in the select list against this
vacancy. It is further stated that the recommendation of the said officer is in a sealed
cover as he is under cloud. In so far as one vacancy of 1996 (General), after deleting the
name of the officer recommended against the vacancy of the year 1993, five eligible
officers, including the petitioner in order of seniority coming in the zone of consideration,
were considered for promotion by the DPC. The petitioner was assessed by the DPC as
"Good" and was not recommended for promotion as respondent No.7 herein senior to
him in the feeder grade, who was given higher grading by the DPC, was recommended
for promotion. In so far as five vacancies of 1997 (General) are concerned, after deleting
the names of the officers recommended for promotions against the vacancy of the
previous year (1996), remaining 13 eligible officers, inclusive of the petitioner in order of
their seniority, were considered for promotion. It is stated that the zone of consideration
for five vacancies is 14 officers. It is further stated that the petitioner was assessed by the
DPC as "Good". and the officers (respondents No.8, 9, 10, 11 and 12 herein) who were
junior to the petitioner in the seniority, who were assessed with the higher grading, were
recommended for promotion against these five vacancies and the petitioner was
superseded by them. It is stated that the DPC has followed the proper procedure to
consider the officers for promotion, as the officers were considered yearwise, as required
under the Rules and eight years Confidential Reports preceding the year of the
vacancy/vacancies were considered by the DPC. It is stated that since the selection being
on merit (except for the year 1993), the petitioner could not get the placing in the select
list against the general vacancies and for the vacancies of the year 1993, the Officer
senior to him and possessing bench mark "Good" was recommended for promotion and
so the petitioner could not get selected even to this vacancy. It is then stated that the
Government had already initiated action to dereserve the five reserved vacancies to be
filled in by general candidates. The Government therefore, desired that five officers
(General Category) to be recommended against the five reserved vacancies which would
be available for filling up by the general candidates soon after the de-reservation. It is
stated that the Government had made this proposal as there was acute shortage of officers
in the cadre of Executive Engineers and further because the recommendation of some
Officers had to be kept in sealed covers as they were under cloud and their integrity could
not be certified by the Government. The reserved vacancies would be available in 1997 to
be filled up by the general candidates after they were actually dereserved by the
Government. It is stated that the DPC accordingly recommended five officers for being
kept in waiting list who would be considered for promotion after de-reservation of
vacancies reserved for Scheduled Caste and two for Scheduled Tribes. These were the
officers who could not get placing against the vacancies of 1997, but could be selected
for the five vacancies as per the seniority, since the remaining 8 officers considered
against 1997 vacancies were assessed as "Good" by the DPC. The name of the petitioner
is shown as sr. No.1 in the said waiting list; whereas the respondents No.14, 18, 24 and
27 are shown at sr. Nos. 2 to 5 respectively. It is stated that the petitioner along with four
other officers referred to above were kept on the waiting list to be promoted after
reserved vacancies were de-reserved by the Government and that the petitioner came to
be promoted by the Government vide order No.3/25-20/85/ Irr-98/729 dated 25.9.1997
along with other officers. It is stated that one more order was issued by the Government
on 16.4.1998 promoting Shri M.K. Pant, Assistant Engineer, and respondents No.13, 14
and 15 as recommended by the DPC, are shown at sr. Nos. 2 to 5 respectively. It is
further stated that the recommendations in respect of three officers are in the sealed
covers as they are under the cloud. On the above averments, it is stated that the DPC had
followed the correct procedure as laid down by the Government and no irregularity has
been committed. In para 16, it is once again asserted that respondents No.3, 5 and 6 were
not considered for promotion in the year 1987. Whereas, respondent No.3 came in the
zone of consideration and was considered for the two vacancies of 1988 along with others
as he had completed 8 years’ service in 1988. It is also stated that exact dates of the
vacancies were not furnished by the Department and the date of appointment of
respondents No.3, 5 and 6 was furnished by the Department was 3.11.1980. On the above
basis, respondent No.16 contends that there is no substance in the present writ petition.
5.
After the aforesaid replies were filed, the petitioner filed rejoinder sworn in on
13.7.1998. For the first time, the rejoinder/affidavit made out a new case that three posts
of Executive Engineer fell vacant in the year 1988. It is averred that on 7.11.1988, Shri
G.R. Karandikar and Shri S.M. Nadkarni promoted on regular basis to the posts of
Superintending Engineer and in October, 1988 Shri B.N. Nawalawal, Executive Engineer
left the service under Goa Government to join Planning Commission, Government of
India. Besides in para 6 of the rejoinder in affidavit, it is once again stated that respondent
No.3 was not eligible for being considered for promotion to the vacancy which occurred
in the year 1988. In para 7 it is stated that the Government misdirected itself in law in
purporting to dereserve three vacancies in the post of Executive Engineer in the year
1997. Whereas the vacancies reserved for Scheduled Castes/Scheduled Tribes in the
years 1984 to 1988 should have been treated as unreserved in those years. On this
premise, it is stated that since the petitioner being one of the only three Assistant
Engineers eligible, was found fit ought to have been promoted in the year 1988. The
petitioner filed further affidavit on 3.3.2001 stating that certain material which has come
to his notice needs to be placed on record. A new case is made out in this petition,
asserting that two posts of Executive Engineer were available for being filled up in 1990
and one post in 1991. It is stated that the post of Cost Analyst is in the grade of Executive
Engineer (3000-4500) and that respondent No.14 was promoted to the said post in 1990
and one K.S. Sheshadri, Surveyor of Works was deputed as Executive Engineer (Civil) to
Sports Authority of Goa in 1991 and repatriated as Cost Analyst in August 1993. It is
then stated that carrying forward of reservations relating to SC/ST is not permitted in
promotion from Group B posts to Group A Posts.
6.
Respondent No.17 contested the petition by filing affidavit of Shri S.D. Sayanak,
Chief Engineer of the Water Resources Department, previously known as the Irrigation
Department, as also the Addl. Secretary to the Government of Goa. Suffice it to mention
that affidavit reiterates the position stated in the affidavit filed on behalf of the GPSC
(respondent No.16 herein). It also reinforces the stand taken in the affidavit of respondent
No.3. It is clearly asserted that the petitioner could not have claimed any right to the
technical post in the Irrigation Department prior to his transfer along with the post vide
order dated 1.12.1984 or in any event, prior to his transfer in the composite PWD vide
order dated 17.2.1983. With regard to the correct date of appointment of the petitioner as
Assistant Engineer, explanation has been offered in para 9 of this affidavit placing
reliance on the appointment order which indicates that the petitioner was appointed as
Assistant Engineer in the Soil Conservation Division of the Directorate of Agriculture
with effect from 2.3.1978. In so far as the new case made out by the petitioner regarding
dereservation of the reserved vacancies and the vacancy would relate back to the
concerned year, it is stated in para 11 of the affidavit that correct procedure has been
followed on the recommendation of the DPC/GPSC and appointments were done only
after the vacancies were dereserved. It is clarified in para 12 of the affidavit that the DPC
could not meet as composite seniority list of the Asst. Engineers was under challenge
before the Apex Court and only after disposal of the said appeal, the final seniority list
was drawn, as thereafter the names of the eligible candidates could be sent to the GPSC
for selection to the post of Executive Engineer. With regard to the grounds as articulated
in the memo of writ petition, the reply asserts that the same are misconceived and without
any substance as the said grounds are founded on erroneous assumptions.
7.
In response to the affidavit filed by the respondent No.3, the petitioner filed
rejoinder on 12.3.2001. By this affidavit, the petitioner disputes the stand taken by the
respondent No.3 that the petitioner did not possess necessary qualification. The petitioner
filed rejoinder/affidavit in response to the affidavit of respondent No.17 sworn in on
17.3.2001 thereby reiterating his earlier stand and placed reliance on the documents to
support his case.
8.
Respondent No.17 filed further affidavit of S.D. Sayanak, Chief Engineer sworn
in on 13.10.2003 relating to the qualifications possessed by the petitioner.
9.
Mr. Kakodkar for the petitioner mainly argued that respondent No.3 was not
eligible for being considered against the general vacancy in the year 1988. According to
him, respondent No.3 was appointed on 3.11.1980 and 8 years period would be
completed only in November, 1988. Whereas, respondent No.3 has been considered for
the vacancy which had arisen earlier thereto. It is next contended that in 1988 there were
three general vacancies to the post of Executive Engineer and not two as has been stated
by the respondents. Reliance is placed on the averments made in the affidavit filed by the
petitioner asserting that position. It is then contended that the stand taken on behalf of the
Government that the vacancy of reserved candidates which was then dereserved in 1997,
it would relate back to the actual year when that vacancy had become available and not
when it was dereserved in 1997, as is contended on behalf of the respondents. It is argued
that there can be no carrying forward of reservation in respect of the said post and even
for that reason the vacancy of reserved post would relate back to the actual year. It is next
contended that in any case the Government was obliged to dereserve the reserved
vacancies within reasonable time and there was no reason to wait till 1997 when it was
amply clear that no candidate belonging to the reserved category was available from the
feeder grade. It is contended that if this contention was to be accepted, then the action of
the Government of dereserving the reserved vacancy as late as in 1997 cannot stand the
test of judicial scrutiny, for which reason also the dereservation would relate back to the
actual year. It is next contended that two vacancies became available in 1990 and one in
1991 which have not been referred to either by respondent No.16 or respondent No.17
and the details whereof are furnished in the affidavit of the petitioner. It is contended that
if that is so, it was obligatory on the part of the respondents to consider the petitioner
along with other eligible candidates for appointment against the said vacancies in 1990 or
at least in 1991 and not in 1997 against the dereserved post. It is further contended that
the stand of the Government that the services rendered by the petitioner in the Soil
Conservation Division of the Directorate of Agriculture cannot be reckoned, is wholly
untenable as there is no express provision in the Rules to do so. Learned Counsel further
contends that the replies filed by the respondents No.16 and 17 do not reveal that the
procedure prescribed under the DPC Guidelines has been complied with as it is
obligatory to convene the DPC at regular annual intervals. Besides, in terms of the Clause
3.1 of the Guidelines, the vacancy is required to be filled up in accordance with the
recruitment rules in force on the date of vacancy. It was contended that in terms of this
provision, the vacancy cannot be ascribed to the "year", but the "date of vacancy" has to
be kept in mind while preparing the panel for making promotions against the vacancy
occurred during the course of a year; and the vacancies prior to 1st April of that year
should be the basis. Learned Counsel has placed reliance on the Full Bench Judgment of
the Kerala High Court in 1981 (2) SLR 253, in the case of Varghese and ors. vs. State of
Kerala and others to contend that the eligibility should be on the date of occurrence of
vacancy and not the time of the order of promotion. Reliance is also placed on AIR 1987
SC 2291 in K. Madhavan v/s. Union of India to contend that transfer of an employee
from one Government Department to another Department, as in this case, cannot wipe
out his length of service from the post from which he has been transferred. To support the
said proposition, reliance is also placed on (2001) 4 SCC 675 in the case of Vinod Kumar
Sharma vs. State of U.P. and anr. as well as AIR 2000 SC 594 in S.I. Rooplal and anr. vs.
Lt. Governor through Chief Secretary, Delhi and ors. It is contended that in view of Rule
6.4.1, when the DPC meets after number of years, it has to ascertain the eligible
candidates as of 1st of April or May of the respective year of vacancy, though the
consideration is at a subsequent point of time. In so far as the preliminary objection taken
by the respondent No.3 in the reply/affidavit for dismissing the petition, it is contended
that there is no suppression of material facts, as all the relevant dates can be culled out
from the seniority list which has been appended to the petition and non-mentioning of
those dates in the petition by itself would make no difference. Learned Counsel further
submits that it is not open to the Government to argue contrary to its own order and
seniority list regarding eligibility of the petitioner. Reliance is placed on 1972 SLR 44,
State of Asssam and anr. vs. Raghava Rajgopalchari (para 13) and 1988 (3) SCC page
570; Assistant Commissioner of Commercial Taxes (Asst.) Dharwar and ors. vs.
Dharmendra Trading Company and ors. (para 5) in support of this proposition. Learned
Counsel further submits that if the argument of respondent No.3 regarding eligibility of
the petitioner was to be entertained, that would result in allowing the respondent No.3 to
challenge the seniority list inspite of the fact that he has accepted the said list without any
demur. Accordingly, it is submitted that even respondent No.3 cannot be permitted to
question the eligibility of the petitioner which is found otherwise and considered his case
along with other eligible candidates for the relevant period. Learned Counsel has placed
reliance on (1997) 9 SCC 287 reported in Union of India and others v/s. N.R. Banerjee
and ors., to contend that when there is delay in convening DPC as to the procedure to be
observed and the zone of consideration. According to the petitioner, in the present
petition, required procedure has not been observed.
10.
Per contra, Mr. Nadkarni, learned Advocate General for the respondents No.16
and 17 has confined his arguments to three issues; first regarding eligibility of respondent
No.3, second issue regarding the effect of dereservation of reserved vacancies and
whether it relates back to the actual year or to be considered only as vacancy when the
decision of dereservation is finally reached by the appropriate authority. The third issue is
regarding the plea taken by the petitioner regarding vacancies in 1990-1991 respectively.
In so far as eligibility of respondent No.3 is concerned, he contends that from the scheme
of various provisions of the Rules and the Guide-lines as applicable, it would appear that
the panel has to be drawn on the basis of yearwise vacancies separately and in the present
case, panel was prepared on the basis of said provision. According to him, no fault can be
found with the inclusion of respondent No.3 in the panel for considering vacancy of 1988
because, admittedly, respondent No.3 was regularly appointed on 3.11.1980 and would
complete 8 years in November, 1988. In so far as the issue relating to dereservation is
concerned, learned Counsel has placed reliance on 1994 Supp.(2) SCC 491 Harish
Chandra Ram vs. Mukh Ram Dubey and ors. (paras 5, 6 and 7) to contend that it was not
incumbent upon the Government to dereserve the vacancy as soon as the vacancy arises,
so as to fill up the same by appointing candidates belonging to general category. He
contends that dereservation needs to be carried forward at least for three successive
recruitment years. Even in the above noted case, the vacancy had occurred in the year
1980, and the recruitment was made in the year 1987. Reliance is also placed on an
unreported decision in Writ Petition No.127/1999 in Sada X. Raut Dessai vs. State of
Goa and ors, decided on 12.8.2003 (para 3) in support of the proposition that steps to
denotify or dereserve the reserved vacancy can be initiated only after expiry of three
years when candidates of reserved category were not available. Reliance is also placed on
AIR 1994 SC 1808 in the case of J. & K. Public Service Commission, etc., vs.
Dr.Narinder Mohan and ors. etc., etc. (para 12) to contend that it is settled law that the
Government need not immediately notify the vacancies as soon as they arose. It is open,
as early as possible, to inform the vacancies existing or anticipated to the PSC for
recruitment and that every eligible person is entitled to apply for and to be considered of
his claim for recruitment provided he satisfies the prescribed requisite qualifications.
Reliance is also placed on AIR 1990 SC 442, in the case of Union of India and ors. vs.
Vadera and others, (para 4) wherein it has been held that there is no statutory provision
that the promotion to the post of Scientist "B" should take effect from 1st July of the year
in which the promotion is granted. The Apex Court has held in para 4 that there is no law
or any rule under which a promotion is to be made effective from the date of creation of
the additional posts. After a post falls vacant for any reason whatsoever, a promotion to
that post should be with effect from the date of creation of the promotional posts. In the
same way when additional posts are created, permissions to those posts can be granted
only after the Assessment Board has met and made its recommendations for promotions
is granted. If, on the contrary, promotions are directed to become effective from the date
of the creation of posts, then it would have the effect of giving promotions even before
the Assessment Board has met and assessed the suitability of the candidates for
promotion. Reliance is also placed on Clause 6.4.4 to contend that the promotions will
have only prospective effect even in cases relating to earlier years. In so far as the case
made out by the petitioner about availability of vacancies in 1990 and 1991 is concerned,
learned Advocate General contends that this case is made out for the first time in the
rejoinder/affidavit and for which reason, the same need not be examined. Besides placing
reliance on the relevant records, the learned Counsel pointed out that there was no
vacancy in 1990 or 1991 as is claimed. On the above arguments, learned Counsel submits
that the petition is devoid of merits.
11.
Mr. Sonak, the learned Counsel appearing for the respondents No.3, 8 and 9
contends that the petition deserves to be dismissed on the preliminary objections taken on
behalf of respondent No.3. He submits that none of the relevant dates have been
furnished in the writ petition as filed. Besides, some intentional misleading statements
have been made in the Memo of Petition as can be discerned from grounds 3(a), 3(d),
3(e) and 3(f) of the Petition. The averments in the petition suggest as if the petitioner was
appointed as Assistant Engineer in the Irrigation Department. It is contended that the
petitioner has given only selective facts for which reason the petitioner is guilty of
suppression of material and relevant facts and is disentitled for any relief from this Court.
Moreover, all new grounds have been pressed into service for the first time across the
Bar, without their being foundation laid in the pleadings in that behalf. The learned
Counsel has placed reliance on 2002(2) GLT 213 in M/s. Consolidated Distributors Pvt.
Ltd. vs. Economic Development Corporation and anr. (paras 18 and 19) to buttress the
argument that in view of the suppression of material facts, the petition deserves to be
dismissed. Reliance is also placed on AIR 1977 SC 781 in case of State of Haryana &
Anr. vs. The Karnal Distillery Co. Ltd., to contend that the petitioner is disentitled for any
relief because of the misleading statements made in his pleadings. Besides, learned
Counsel contends that the petitioner was ineligible for consideration in the year 1988.
This argument proceeds on the basis that Rules of 1986 postulate that the incumbent
should possess 8 years experience in the feeder grade. The said rules provides as follows :
" For the purpose of eligibility for promotion, regular service as Assistant
Engineer (Mech./Elect.)/Asst. Surveyor of Works (Civil) in the PWD prior
to transfer to the Irrigation Department of the Goa Administration and
regular service in the Irrigation Department shall be counted."
It is contended that these Rules are specifically framed to regulate the recruitments in the
Irrigation Department alone and only the Department which had some nexus with the
Irrigation Department such as PWD, service rendered by the incumbent were of some
relevance and, therefore, required to be reckoned for considering the eligibility for
promotion in the Irrigation Department. Whereas, the petitioner was appointed on
temporary basis in the Soil Conservation Division of the Directorate of Agriculture on
17.4.1978 and regularized vide order dated 3.11.1980, which clearly provides that the
petitioner shall not get any benefit of his past service. In any case, the transfer of
technical post from Soil Conservation Division of Directorate of Agriculture to PWD was
only on 17.2.1983, which order was, however, cancelled on 22.12.1987 restoring the
position, whereby the petitioner was assigned to the Soil Conservation Division. It is only
by order dated 18.5.1989 the petitioner was in regular service of the Irrigation
Department and his service on and from that period alone can be the basis for considering
his eligibility and 8 years service, in which case, the petitioner would become eligible for
consideration as per the Rules only in 1996 and not earlier. It is pointed out that assuming
that the petitioner was entitled to get benefit of the period spent in PWD after transfer of
technical posts from 17.12.1983 till 22.12.1987, even then the petitioner would become
eligible for consideration only in 1993. It is, therefore, submitted that even accepting the
stand of the petitioner that he was assigned to PWD after transfer of technical post of the
Soil Conservation Division of the Directorate of Agriculture in February 1983 even then
the petitioner at the best would qualified for promotion after 1993-1994. It was also
contended that the petitioner was transferred to the Irrigation Department on his own
volition as can be seen from the letter dated 22.6.1988 and in that event, the petitioner
cannot get any benefit of his transfer to the Irrigation Department. It is also a matter of
record that after the petitioner was assigned to the Soil Conservation Division,
Directorate of Agriculture on 22.12.1987, he was promoted on ad hoc basis to the post of
Executive Engineer in the same Department on 8.2.1988 and which promotion was
accepted by the petitioner without any demur. It is submitted that accordingly, the
services rendered by the petitioner prior to 1989 would be of no avail. On this premiss, it
is argued that since the petitioner was ineligible to be considered for the vacancy in 1988,
it is not open to the petitioner to set up challenge to the promotion of respondent No.3
made for the vacancy arisen in the said year. In support of this proposition, reliance is
placed on (1980) 3 SCC 29, in the case of Dr.N.C. Singhal vs. Union of India, (para 21).
It is further contended that it is open to the respondent No.3 to question the eligibility of
the petitioner so as to resist this petition and persuade this Court to refuse to grant any
relief to the petitioner. It is argued that as the petitioner being ineligible by virtue of the
Rules as mentioned earlier, he is not entitled to maintain this petition. To buttress this
proposition, reliance is placed on (2001)8 SCC 676, in the case of Bharathidasan
University and anr. vs. All India Council for Technical Education and anr. Reliance is
also placed on (1999) 8 SCC 16, in the case of Maharaja Chintamani Saran Nath Shahdeo
vs. State of Bihar and ors. (paras 12 to 14 and 38) to contend that no writ of mandamus
be issued to effectuate eligibility in favour of the petitioner who was ineligible for being
considered for promotion in the year 1988 or for that matter in the year 1990 or 1991 as
the case may be. Reliance is also placed on the decision reported in (2002) 3 SCC 586 in
the case of K. Shekar vs. V. Indiramma and ors., (para 22). Learned Counsel further
contends that the reliance placed on the decision in the case of K. Madhavan and anr. vs.
Union of India and ors (supra) is misplaced. It is submitted that in that case there was no
express Rule unlike in the present Rules referred to above, which restrict the criteria for
considering eligibility to the service rendered in PWD and the Irrigation Department
only; and having regard to the fact that Rules of 1986 have been framed only to meet the
requirement of the Irrigation Department. Mr. Sonak has adopted the arguments of
learned Advocate General that the vacancy will have to be reckoned on the basis of
"year" in which it arose and not from the "actual date" as is contended by the petitioner.
Reliance is placed on the decision in (2002) 2 SCC 712 in G.N. Nayak vs. Goa
University and ors, (Paras 27 & 28) to buttress the argument that the experience in Soil
Conservation Division of the Directorate of Agriculture or for that matter service
rendered in that division can be of no consequence for considering the requirement of the
Irrigation Department as the Soil Conservation Division has no nexus with the Irrigation
Department. On the above basis, Mr. Sonak contends that the petition be dismissed being
devoid of merits.
12.
Having considered the rival submissions, we propose to first deal with the
grounds as pressed into service in the Memo of Writ Petition. The first ground is that
there has been breach of the guidelines on DPC contained in the Office Memorandum
dated 26.9.1990, in particular paragraph 6.4.1, 6.4.2 and 6.4.3. In our opinion, there is no
foundation laid in the Memo of Writ Petition as to the nature of breach committed by the
authorities. On the other hand, reference is only made to the provisions and nothing more.
Going by the ground as taken, we are at a loss to discern the basis on which the petitioner
pleads that there has been breach of the concerned guidelines. Even on a fair reading of
the petition as a whole, all that can be culled out in this context is the grievance made in
para 3(f) of the petition which states that the DPC did not meet in the years 1985 to 1986
to recommend the regular promotions to the post of Executive Engineer available in those
years for making recommendations. As has been stated in the reply filed by the
respondent No.17 that the DPC could not meet because of the challenge to the combined
seniority list pending before the Apex Court; And as soon as the appeal before the Apex
Court was decided, final seniority list was prepared and the names of eligible candidates
were sent to the DPC. Moreover, going by the affidavits of the respondent No.16 as well
as respondent No.17, we have no hesitation in taking a view that the same graphically
describe the steps taken from time to time which, in our view, are in accordance with the
requirement of the relevant provisions. From the arguments as were canvassed on behalf
of the petitioner, all that could be suggested in the context of challenge under
consideration is that the panel which has been drawn does not consist of the candidates
who were eligible prior to 1st April of the relevant year. There is no substance in this
submission either. In our opinion, on examining the relevant provisions of the guide-lines
and on conjoint reading of paragraphs 3.1, 4.1, 6.4.1, 6.4.2 and 6.4.3, it postulates that a
panel has to be drawn of the candidates who became eligible for being considered for the
vacancy in the course of the year. There is nothing to show that it restricts preparation of
such panel upto 1st April of that year, as is contended. To appreciate this aspect, it will be
apposite to reproduce the relevant paragraphs referred to above:
"3.1 The DPCs should be convened at a regular annual intervals to draw
panels which could be utilised on making promotions against the
vacancies occurring during the course of a year. For this purpose it is
essential for the concerned appointing authorities to initiate action to fill
up the existing as well as anticipated vacancies well in advance of the
expiry of the previous panel by collecting relevant documents like CRs,
integrity certificates, seniority list etc. for placing before the DPC. DPCs
could be convened every year if necessary on a fixed date e.g., 1st April or
May. The Departments should lay down a time schedule for holding DPCs
under their control and after laying down such a schedule the same should
be monitored by making one of their offices responsible for keeping a
watch over the various cadre authorities to ensure that they are held
regularly. Holding of DPC meetings need not be delayed or postponed on
the ground that recruitment rules for a post are being reviewed/amended.
A vacancy shall be filled in accordance with the recruitment rules in force
on the date of vacancy, unless rules made subsequently have been
expressly given retrospective effect. Since Amendments to recruitment
rules normally have only prospective application, the existing vacancies
should be filled as per the recruitment rules in force.
4.1
It is essential that the number of vacancies in respect of which a
panel is to be prepared by a DPC should be estimated as accurately as
possible. For this purpose the vacancies to be taken into account should be
the clear vacancies arising in a post/grade/service due to death, retirement,
resignation, regular long term promotion and deputation or from creation
of additional posts on a long term. As regards vacancies arising out of
deputation, only those cases of deputation for periods exceeding one year
should be taken into account due note, however, being kept also of the
number of the deputationists likely to return to the cadre and who have to
be provided for. Purely short term vacancies created as a result of officers
proceeding on leave, or on deputation for a shorter period, training etc.,
should not be taken into account for the purpose of preparation of a panel.
In cases where there has been delay in holding DPCs for a year or more,
vacancies should be indicated year-wise separately.
6.4.1 Where for reasons beyond control, the DPC could not be held in an
year (s), even
though the vacancies arose during that year (or years), the first DPC that
meets thereafter should follow the following procedure:
(i) Determine the actual number of regular vacancies that arose in
each of the previous year(s) immediately preceding and the actual number
of regular vacancies proposed to be filled in the current year separately.
(ii) Consider in respect of each of the years those officers only who
would be within the field of choice with reference to the vacancies of each
year starting with the earliest year onwards.
(iii) Prepare a ‘select list’ by placing the select list of the earlier
year above the one for the next year and so on;
6.4.2 Where a DPC has already been held in a year further vacancies
arise during the same year due to death, resignation, voluntary retirement,
etc. or because the vacancies were not intimated to the DPC due to error or
omission on the part of the Department concerned, the following
procedure should be followed :(i) Vacancies due to death, voluntary retirement, new creations,
etc., clearly belonging to the category which could not be foreseen at the
time of placing facts and material before the DPC. In such cases, another
meeting of the DPC should be held for drawing up a panel for these
vacancies as these vacancies could not be anticipated at the time of
holding the earlier DPC. If, for any reason, the DPC cannot meet for the
second time, the procedure of drawing up of year-wise panels may be
followed when it meets next for preparing panels in respect of vacancies
that arise in subsequent year(s).
(ii) In the second type of cases of non-reporting of vacancies due to
error or omission (i.e. though the vacancies were there at the time of
holding of DPC meeting they were not reported to it) results in injustice to
the officers concerned by artificially restricting the zone of consideration.
The wrong done cannot be rectified by holding a second DPC of preparing
an year-wise panel. In all such cases, a review DPC should be held
keeping in mind the total vacancies of the year.
6.4.3. For the purpose of evaluating the merit of the officers while
preparing year-wise panels, the scrutiny of record of service of the officers
should be limited to the records that would have been available had the
DPC met at the appropriate time. For instance for preparing a panel
relating to the vacancies of 1978 the latest available records of service of
the officers either upto December 1977 or the period ending March, 1978
as the case may be, should be taken into account and not the subsequent
ones. However, if on the date of the meeting of the DPC, departmental
proceedings are in progress and under the existing instructions sealed
cover procedure should be observed even if departmental proceedings
were not in existence in the year to which the vacancy related. The
officer’s name should be kept in the sealed cover till the proceedings are
finalised."
13.
Paragraph 3.1 refers to drawing panel which could be utilised for making
promotions against the vacancies occurring "during the course of a year". Indeed the said
paragraph refers to the DPC to be convened every year, if necessary on a fixed date, e.g.
1st April, or May, but, in our opinion, that provision does not mean that the panel which
has to be drawn of eligible candidates ought to be only upto 1st April of that year. Even
paragraph 6.4.1 refers to a situation in cases where there has been delay in holding the
DPC for a year or more, vacancies should be indicated year-wise separately. If vacancies
are to be indicated yearwise separately, there can be no justification for restricting the
preparation of panel only upto 1st April, or May of that year. Whereas, the date of 1st
April or May has been indicated in paragraph 3.1 only as illustrative. The substance of
that provision is that DPC should be convened at regular annual intervals. There is no
express provision so as to curtail the panel only upto 1st April of a given year. Much
emphasis was placed on paragraph 6.4.1.Indeed this paragraph deals with the aspect of
preparation of year-wise panel by the DPC, where they have not met for a number of
years. But again as mentioned earlier, there is no express provision that the panel should
be drawn of eligible candidates only upto 1st April or May of the relevant year. Reliance
was placed on paragraph 6.4.3; however, in our opinion, that is a provision relating to
evaluation of merit, as it stipulates that for the purpose of evaluating merit of the officers
while preparing the year-wise panel, the scrutiny to the record of service of the officers
should be limited to the records that would have become available, had the DPC met at
the appropriate time. It provides for illustration by observing that for preparing a panel
relating to the vacancies of 1978, the latest available records of the service of the officers
either upto December 1977 or the period ending March, 1978 as the case may be, should
be taken into account and not the subsequent one. This does not mean that the candidates
who have become eligible for consideration on or after 1st April of the relevant year,
cannot be included in the panel, but it is a provision that only the record upto December
of the previous year or upto March of the relevant year, as the case may be, ought to be
taken into account. We are of the view, therefore, that there is no substance even in the
argument canvassed before us relating to the breach of any of the guidelines as contended
by the petitioner.
14.
The next ground of challenge in the memo of writ petition is that nine posts of
Executive Engineer were available in the year 1987-88 and the cases of respondents
No.3, 5 and 6 have been wrongly considered although they were ineligible for being
considered against the vacancies of that year. This ground is entirely misconceived. The
affidavits filed by respondents No.16 and 17 clearly assert that cases of the respondents
No.3, 5 and 6 were not considered for the vacancies of 1987 as they had not completed 8
years’ of service in the feeder grade. In view of this categorical stand taken by the
respondents, the said ground canvassed is untenable. The grievance founded on such
erroneous basis is also, therefore, unavailable and untenable. The affidavits filed on
behalf of respondents No.16 and 17 clearly assert that the claim of respondents No.3 was
considered for the first time against the vacancy of 1988, having completed 8 years’ of
service in that year. Whereas, the respondents No.5 and 6 were considered along with the
petitioner for the vacancies of 1989. There is no specific challenge that none of the
respondents No.3, 5 and 6 were eligible to be considered for the vacancies of 1988. But
the pleadings filed by the petitioner proceeds on the premise that the said respondents
were wrongly considered against the vacancy of "1987 only". Accordingly, there is no
substance even in second ground as taken in the writ petition.
15.
The third ground is the consequence of the second ground articulated in the memo
of writ petition. As the same is devoid of merits, even this ground should necessarily fail.
Going by the petition as filed, the same has to fail on the above reasonings. Even if we
were to examine the other aspects raised in the subsequent affidavits filed by the
petitioner or for that matter during the course of oral arguments across the Bar, even then
the petition deserves to be dismissed for the reasons recorded hereinafter.
16.
We have reproduced the pleadings in extensor which would clearly demonstrate
the nature of averments made by the petitioner in the Memo of Writ Petition. Going by
the said averments and particularly in the context of the preliminary objections raised on
behalf of the respondent No.3, we have no manner of doubt that the petition as filed is an
attempt of suppressing material and relevant facts, necessary for full, complete and
effectual adjudication of the issues between the parties. Indeed, Counsel for the petitioner
made an unsuccessful attempt in persuading us that all the relevant facts were stated in
the Memo of Writ Petition and the relevant events could be discerned from the
accompanying documents. According to him, it was possible to take the view that the
detailed facts disclosed in the reply filed on behalf of the respondent were
inconsequential. In our opinion, chronology of events which can be culled out from the
reply filed by the respondents if were to be indicated in the Memo of Writ Petition, the
petition could have been thrown out at the threshold. There is also substance in the
argument canvassed on behalf of the respondent No.3 that the petitioner has deliberately
made misleading statements in the Memo of Writ Petition. In para 3(d), no disclosure has
been made that the appointment of the petitioner was as Assistant Engineer in the "Soil
Conservation Division of the Directorate of Agriculture", which fact was a relevant fact
because the Soil Conservation Division had no nexus either with the PWD or the
Irrigation Department. Besides, the petitioner has reproduced incorrect date of
appointment as 3.11.1976 taking advantage of the incorrect date mentioned in the
tentative seniority list instead of the correct date as 2.3.1978. Even if nothing turns out on
this date, the fact remains that the petitioner taking advantage of incorrect date of
appointment mentioned in the tentative seniority list has stated the said date. Once again
in para 3(a), the petitioner has made a false and misleading statement by not asserting that
he was promoted as Executive Engineer in the Irrigation Department instead of Soil
Conservation Division. Again in para 3(f) of the petition, relevant fact is suppressed that
between 1985-86 the DPC could not meet on account of the seniority list as the issue was
subjudice before the Apex Court. Whereas, the argument canvassed before this Court on
behalf of the Petitioner was on the basis of such misleading statements made in the writ
petition. In this view of the matter, we would have no hesitation to non-suit the petitioner
for having made false, incorrect and misleading statements as also for suppression of
material and relevant facts, as such a litigant does not deserve any indulgence in exercise
of extra-ordinary jurisdiction.
17.
Assuming that we were to consider the other grounds now pressed into service on
behalf of the petitioner, we are of the view that none of those grounds are available to the
petitioner. From the reply filed on behalf of the respondents, it is amply clear that the
petitioner was initially appointed on temporary basis as Assistant Engineer in the Soil
Conservation Division, Directorate of Agriculture on 17.4.1978. He came to be regularly
appointed in the same Department on 3.11.1980. While doing so, it was made clear that
the petitioner will not be entitled for any benefits of his past service. Be that as it may, it
is only pursuant to the order dated 17.2.1983, the technical posts of the Soil Conservation
Division, Directorate of Agriculture were transferred to PWD, on account of which the
petitioner was assigned to PWD. The composite PWD was bifurcated into PWD and
Irrigation Department on 2.4.1984. On 3.10.1986, separate recruitment rules for the
Irrigation Department were brought into force. On 22.12.1987, the Government cancelled
its earlier order dated 17.2.1983 transferring technical posts of the Soil Conservation
Division to PWD. In view of this development, services of the petitioner stood restored to
the Soil Conservation Division. By this order, it was made clear that the Soil
Conservation Division would continue to be an absolute constituent of the Agriculture
Department which, obviously, had no nexus with the PWD or for that matter the
Irrigation Department. It is only by order dated 18.5.1989, the petitioner was transferred
to the Irrigation Department along with other officers "in public interest". Accordingly,
the petitioner’s entry in the Irrigation Department will have to be reckoned from this date
i.e. 18.5.1989. If that is so, then obviously the petitioner was ineligible for being
considered for promotion to the post of Executive Engineer in the Irrigation Department
at least for a period of 8 years therefrom, which means that he was to be eligible only in
1997. It necessarily follows that the petitioner has no locus or right to question the
promotion of the private respondents and in particular of respondent No.3 which were for
earlier period. Assuming that the services rendered by the petitioner from 17.2.1983 to
22.12.1987 in the Irrigation Department could be counted for the purpose of eligibility
for promotion, even then 8 years’ of service in the feeder post would complete only after
July, 1992, which means that he would become entitled for being considered for the first
time against the vacancy of 1993. Even so, the petitioner would not be entitled to
question the promotion of the private respondents, in particular, respondent No.3 which
was in respect of much earlier period. To overcome this position, Counsel for the
petitioner contends that it was not open to the Government to argue contrary to its own
order regarding eligibility of the petitioner. In fact no such argument was canvassed on
behalf of the respondents No.16 and 17. The argument regarding eligibility of the
petitioner has been pressed into service by the respondent No.3. Reliance has rightly been
placed by the Counsel for the respondent No.3 on the decision reported in the cases of
Bharathidasan University and anr. vs. All-India Council for Technical Education and ors.
(supra); Maharaja Chintamani Saran Nath Shahdeo (supra) and K. Shekar vs. V.
Indiramma and ors. (supra). In our opinion, it was open to the respondent No.3 to resist
this petition or to argue that the petitioner was not entitled to any relief in law on the basis
of the established facts and the relevant Rules. We find substance in the argument of the
respondent No.3 that the petitioner was in fact ineligible even for being considered,
though the authorities have considered his claim for the vacancies of 1987 onwards
instead for vacancy of 1997 or at any rate for the first time against the vacancy of 1993.
The learned Counsel for the petitioner, however, contends that the respondent No.3 has
not challenged the seniority list and having accepted that seniority list, it was not open
even to the respondent No.3 to argue that the petitioner was ineligible for being
considered till 1997 or till 1993. For, if that argument was to be accepted, it would
tantamount in permitting respondent No.3 to challenge his seniority list. We find no
substance in this submission. On the other hand, we are of the view that the respondent
No.3 was not challenging the seniority list as such, but questioning the eligibility of the
petitioner for being considered for promotion to the post of Executive Engineer in the
Irrigation Department which was a selection post.
18.
To counter the claim of respondent No.3, the Counsel for the petitioner, at the
foremost contended that the respondent No.3 was ineligible for being considered against
the general vacancy of 1988 as he had not completed 8 years service in the feeder grade.
We have already briefly dealt with this aspect in the foregoing paras. In our opinion, this
arguments is clearly one of desperation. It is seen from the record that the respondent
No.3 was appointed as Assistant Engineer in PWD on 9.11.1972. He was regularly
appointed on 3.11.1980 as Assistant Engineer in PWD. After bifurcation of composite
PWD, the respondent No.3 was allocated to the Irrigation Department on 2.4.1984.
Having regard to the note appended to the Rules, the regular services rendered by the
respondent No.3 in the PWD as well as the Irrigation Department will have to be
counted. As mentioned earlier, respondent No.3 was regularly appointed on 3.11.1980.
Eight years period from that date was to expire in 1988. If that is so, for the reasons
already recorded above, the respondent No.3 was plainly eligible for being considered for
promotion to the post of Executive Engineer in the Irrigation Department against the
vacancy of 1988. Accordingly, there is no substance in the challenge to the eligibility of
respondent No.3 against the vacancy of 1988, which contention obviously has been
argued only across the Bar, without there being any foundation in the pleadings in that
behalf.
19.
We shall now advert to the other argument canvassed on behalf of the petitioner.
According to the petitioner, the action of the Government in de-reserving the reserved
vacancy of 1988 in the year 1997 was inappropriate. The argument proceeds on the basis
that the Government had knowledge about the factual position that no candidate
belonging to the reserved category was available in the feeder post, therefore, could not
have waited till 1997 for de-reserving the reserved vacancy of 1988. As is noted earlier,
from 1988 till 1996, the entire matter was in limbo on account of the pendency of appeal
before the Apex Court, challenging the composite seniority list. It is only after the final
decision in that appeal, the Government proceeded to prepare the final seniority list. Only
on preparation of the final seniority list in 1997, the process for promotion to the post of
Executive Engineer/Surveyor of Works in the Irrigation Department against 20 existing
vacancies belonging to 9 recruitment years from 1984 to 1997 was and could be initiated.
Besides referring proposal to the GPSC/DPC, the Government simultaneously initiated
procedure for dereserving the reserved vacancies of the respective years. As is mentioned
earlier there is no foundation for the ground agitated by the petitioner before us. On the
other hand, in our view, there was valid justification for not taking steps for dereserving
the reserved vacancies for the respective years. In any case, the petitioner has been
considered for the first time against the vacancy of 1987. Accordingly, we find substance
in the argument of the learned Advocate General that the action of the Government of
dereserving the reserved vacancies of different years in 1997 cannot be questioned.
Reliance has rightly been placed on the decisions in the case of Harish Chandra Ram vs.
Mukh Ram Dubey and ors. (supra) as well as the dictum in the unreported decision of
this Court in the case of Shri Sada X. Raut Dessai vs. State of Goa and ors. (supra).
Learned Counsel for the petitioner, however, submits that there could be no carry forward
of the reserved vacancy when it was amply clear that the candidate of reserved category
was unavailable in the feeder post. In the first place, no foundation has been laid in the
writ petition to set up this challenge. Secondly, as mentioned earlier, the petitioner
became eligible for consideration only in 1997 and at any rate, against the vacancy of
1993, the petitioner cannot be allowed to question the promotion of the private
respondents on the argument that the reserved vacancy of 1988 or of earlier years ought
to have been dereserved.
Because, even if that argument was to be accepted, the
petitioner was ineligible for being considered for promotion against those vacancies. On
the other hand, we find substance in the argument of the respondents that the reserved
vacancy will be thrown open to the general candidates on and from the decision of
dereservation is finally taken and not for any earlier period. In other words, the effect of
decision of dereservation of vacancy will have prospective effect and not relate back to
the year of that vacancy. Therefore, in our view, the petitioner would become entitled to
be considered for promotion qua the reserved vacancies only after the same were
dereserved in 1997, as he was at serial No.1 in the waiting list prepared by the DPC.
20.
That takes us to the ground taken in the rejoinder-affidavit dated 13.7.1998 filed
by the petitioner. It is stated that three posts of Executive Engineer fell vacant in the year
1988. In so far as this assertion is concerned, no case is made out by the petitioner. In any
case, there is no substance in this contention. Affidavits have been filed both by
respondent No.16 as well as by respondent No.17, giving vacancy position on yearwise
basis. There is no room to doubt the figures stated on affidavit which are supported by the
official record. In our opinion, this contention, therefore, is devoid of merit. Besides for
the reasons already recorded earlier, the petitioner was ineligible for being considered for
promotion in 1988, therefore, there is no substance even in this ground.
21.
In the subsequent affidavits filed by the petitioner dated 3.3.2001, once again a
new case is made out asserting that two posts were vacant and were available for being
filled up in 1990 and one post in 1991. Again there is no substance in this grievance. The
learned Advocate General has rightly contended that this is a new case made out for the
first time on affidavit filed in 2001. Besides, he has offered an explanation which is
supported by the record that Mr. S. Sekhar, Assistant Engineer was reckoned as against
the vacancy of 1989 and that there was no other vacancy in 1990 or 1991. In other words,
the ground sought to be made out for the first time on affidavit is without any basis and
unsubstantiated from the record. In any case, assuming that this vacancy was available,
even then the petitioner was ineligible for being considered against the said vacancy at
the relevant time. Accordingly, there is no substance even in this contention pressed into
service on behalf of the petitioner. For the view that we have taken, it is not necessary to
advert to the several decisions cited across the Bar or to each of the incidental points
canvassed before us.
22.
For the aforesaid reasons, this petition being devoid of merits, the same is
dismissed with costs.
***
GOA STATE INFORMATION COMMISSION AT PANAJI
APPEAL NO.129/SCIC/2010
D.D. 24.09.2010
Shri. M.S.Keny, Chief Information Commissioner
Edwin Rodrigues
…
Vs.
PIO, Goa PSC & Anr. …
Appellant
Respondents
R.T.I. Act:
In this case under RTI Act the appellant sought two items of information (1)
Names and addresses of selected candidates and (2) Application forms and documents
attached to the application form by the selected candidates – 1st item of information is
furnished and 2nd item of information refused on the ground that it was third party
information and the same has been already forwarded to the Government – Information
Commission allowed the appeal upholding that the application of the appellant ought to
have been transferred to the concerned Department as provided under Section 6(3)(ii) of
RTI Act.
Held:
The solemn obligation cast upon the public Authority to which the application for
information is made has to transfer the application to the public authority which holds the
information as provided under Section 6(3) of the Act - The objective behind enacting
this provision is perhaps to lessen the travails of an information seeker lest he is lost in
the labyrinth of procedural technicalities.
ORDER
1.
The Appellant, Edwin Rodrigues, has preferred the present appeal praying that the
Public Information Officer be directed to furnish the information sought by the Appellant
vide his letter dated 16/11/2009 at Sr. No. 2 at the earliest free of cost and/or that the
transfer the application dated 16/11/2009 under section 6(3) (i) & (ii) of the Right to
Information Act to impose cost/fine on the Public Information Officer.
2.
The brief facts leading to the present appeal are as under:That the Appellant filed an application dated 16/11/2009 before Respondent No. 1
seeking certain information under Right to Information Act 2005 (‘RTI’ Act for short).
That by reply dated 11/02/2009 the Public Information Officer refused to furnish to the
Appellant the information sought at Sr. No. 2 on the ground that it is third party
information. That the denial of information has to be treated as non-furnishing/denying
the information sought by the Appellant. Being not satisfied the Appellant preferred the
Appeal before the First Appellate Authority (‘FAA’ for short). That the stand of the
Respondent No. 1 was that information sought is third party and secondly that the
relevant document were sent to the Government. That the F.A.A upheld the decision of
the Public Information Officer and directed the Appellant to approach the Government
Authority. That Public Information Officer did not state as to which Government
Authority the said documents were sent and that Public Information Officer was duty
bound to forward the application to the concerned Government Authority. Being
aggrieved the Appellant has preferred the present appeal on the grounds as set out in the
memo of Appeal.
3.
The Respondent resist the Appeal and the reply of the Respondent No. 1 is on
record. It is the case of the Respondent that the Appeal filed by the Appellant is
misconceived and not maintainable in law. That the said application was duly considered
by the Respondent No. 1 and Appellant was furnished the information sought at Sr. No. 1
and Sr. No. 2 could not be furnished being third party information. The Respondent No.
1 admits about First Appeal and also about filing the Reply in the said Appeal. That there
is no challenge to the order of the First Appellate Authority and as such directions sought
cannot be issued. It is the case of the Respondent No.1 that the information which has
been sought by the Appellant is already been forwarded to the Government and the
Respondent No. 1 is, therefore, not in a position to furnish the said information and that
the Appellant will have to file an appropriate application before the concerned
department of the Government if at all the Appellant is entitled to the said information.
That the Respondent No. 1 rightly rejected the said information.
4.
Heard Shri Rui Ferreira, the representative of Appellant and Adv. Shri H. D.Naik
on behalf of the Respondent No. 1. Both sides submitted in similar vein as per their
pleadings.
5.
I have carefully gone through the records of the case and also considered the
arguments advanced by the parties. The point that arises for my consideration is whether
the relief prayed is to be granted or not?
It is seen that by an application dated 16/11/2009 the Appellant sought certain
information from the Public Information Officer/Respondent No.1. The information
consisted of about names and addresses of selected candidates recommended to the
Government and the same was granted. The other part of information was about
application forms and documents attached to the form by candidates who have been
recommended for appointment for the said post. This piece of information was not
furnished on the ground that it is third party information. This was conveyed to the
Appellant by letter dated 11/12/2009 which is within stipulated period.
It is the contention of the Appellant that the reply that the information cannot be
furnished under Right to Information Act that it is third party information is misleading
and irrelevant as the Public Information Officer has denied the information available with
G.P.S.C. in utter violation of the Right to Information Act.
It is to be noted here that in respect of information relating to a third party the
concerned Public Information officer must give notice to the third party and if such third
party makes submissions then to consider the same.
Section 11 of the Right to Information Act relates to third party information Third
party has been defined under section 2(n) to mean a person other than the citizen making
a request for information required to be disclosed as confidential that authority is required
to give a written notice to such third party of the request. In case such information is not
held as confidential no written notice is required to be given. To be noted further that
whenever any applicant is asking for information involving third party such information
is to be given under section 7 of the Act only after following procedure prescribed under
section 11(1) of the Act and also keeping in mind section 7(7) of the Right to Information
Act.
6.
From the submissions made it appears that information is not with respondent
No.1.
As per section 6(3) where a public Authority to whom an application for
information is made, finds that the information demanded is held by another public
authority, it is duty bound to transfer the application of information to the concerned
Public Authority within 5 days of receipt of the application and the information seeker is
to be informed of this immediately. Breach of the same would be violation of section 6
(3) of the Act.
This is solemn obligation cast upon the public Authority to which the application
for information is made. The objective behind enacting this provision is perhaps to lessen
the travails of an information seeker lest he is lost in the labyrinth of procedural
technicalities.
7.
Looking at the factual matrix of this case the matter is to be referred to the Public
Information Officer. So that Public Information Officer complies section 6(3) (ii) of
Right to Information Act.
8.
Appellant prays for penalty. The same requires to be considered adequately. The
said issue is kept open to be adjudicated. The Appellant can agitate the same
subsequently.
9.
In view of the above I pass the following order:ORDER
The Appeal is partly allowed. The Respondent No. 1 is hereby directed to transfer
the application of the Appellant dated 16/11/2009 in respect of Sr. No. 2 to the concerned
department/Authority under section 6(3)(ii) of the Right to Information Act 2005 within 5
days from the receipt of the order under intimation to the Appellant and the Appellant to
deal with the same.
***
GOA STATE INFORMATION COMMISSION AT PANAJI
Complaint No. 534/SCIC/2010
D.D. 15.12.2010
Shri M. S. Keny, State Chief Information Commissioner
Dr. Kalpana V. Kamat
V/s
Mrs. Seema Malkarnekar,
P.I.O. Goa P.S.C.
…
Complainant
…
Opponent
R.T.I. Act:
The Complainant a candidate for the post of Teacher Grade-I (Biology) under
Directorate of Education sought information viz., copies of screening written examination
papers of three candidates, marks sheet copies and copies of teaching experience
certificates - PIO furnished written examination papers of the complainant and in respect
of other two candidates the request for marks sheets and teaching experience certificate
was rejected under Section 8(1)(e) of the Act – Information Commission after
considering several decisions touching the matter in issue allowed the complaint and
directed the opponent-Public Information Officer to furnish Sl.No.2 and 3 of the
complainant’s application for information within 20 days – Regarding Sl.No.1
information viz., written examination papers of two candidates PIO was directed to show
only evaluated answer scripts without showing the names and other things.
ORDER
1.
The Complainant, Dr. Kalpana V. Kamat, has filed the present complaint praying
that inquiry be made with respect to reply of respondent dated 15.09.2010 and for
directing Respondent to furnish complete information to the petitioner as per letter dated
10.08.2010.
2.
The brief facts leading to the present Complaint are as under :That the Complainant applied for the post of Teacher Grade-I (Biology) ADEI
under Directorate of Education. The Selection to the post of Teacher Grade-I (Biology) is
conducted through Goa Public Service Commission exams. That on completion of oral
interview of shortlisted candidates held on 8th June and 9th June 2009, four candidates
were selected. It is the case of the Complainant that on 10.08.2010 the Complainant
sought certain information under Right to Information Act, 2005 (‘R.T.I.’ Act for short)
from the Opponent. That the Complainant received incomplete information from the
Opponent on 15.09.2010 which provides only written examination papers based on
Subjective and General knowledge in respect of Complainant alone. That other
information was not furnished. Being aggrieved, the Complainant has filed the present
Complaint.
3.
The Opponent resists the Complaint and their say is on record. It is the case of the
Opponent that the Complaint filed by the Complainant is not maintainable in law. That
the Complainant ought to have filed the First Appeal before the First Appellate Authority
as provided under section 19 of the R.T.I. Act and, therefore, the present Complaint may
not be entertained. That the present Complaint has been filed only to harass the
Opponent. It is the case of the Opponent that Complainant was furnished part of the
information and was also informed that the other could not be provided as the same was
exempted under section 8(1) (e) of R.T.I. Act. That certificates of other candidates as
sought are in fact personal information and disclosure of which has no relationship to any
public activity or interest and no public interest would be served in disclosing this
information. It is further the case of the Opponent that furnishing copies of evaluated
answer papers would compromise the fairness and impartiality of the selection process.
In short it is the case of the Opponent that Opponent was justified in not issuing the
information sought by the Complainant, and, therefore, the Complaint is liable to be
dismissed.
4.
Heard the arguments. Shri Jowett D’Souza, representative of the Complainant
argued on behalf of the Complainant and Adv. H. D. Naik argued on behalf of the
Opponent.
Representative of the Complainant referred to the facts of the case in detail and
submitted that the information asked can very well be given. According to him part of the
information is furnished and the other part could be given. He relied on a judgment of
Delhi High Court. He also referred to the rejoinder which is on record.
Advocate for the Opponent submitted that Complaint is not maintainable.
According to him reply was furnished on 15.09.2010 and that no Appeal is filed. He
referred to section 19. He next submitted that answer papers cannot be given. According
to him Complaint may not be entertained.
5.
I have carefully gone through the records of the case and also considered the
arguments advanced by the parties. The point that arises for my consideration is whether
the relief prayed is to be granted or not?
It is seen that the Complainant, vide her application dated 10.08.2010 sought
certain information from the Public Information Officer/Opponent. The information
consists of copy of entire screening written examination papers of three candidates, mark
sheet copies and copies of teaching experience certificates. By reply dated 15.09.2010 the
Public Information Officer furnished written examination papers of the Complainant and
in respect of other two candidates the request was rejected under section 8(1)(e) of the
R.T.I. Act. Regarding mark sheet copies and teaching experience the request was rejected
on the ground that it was personal information and disclosure of which has no
relationship to any public activity or interest and no public interest would serve in
disclosing this information.
The grievance of the Complainant is that such an information ought not to have
been rejected as the same can very well be given under the R.T.I. Act.
6.
It is to be noted here that right to know is a basic right of citizens of a free
country. Long back Aristotle observed that people desire to know. Without adequate
information a person cannot form an informed opinion. The Right to Information Act
2005 has been enacted to provide for a legal right to information for citizens to secure
access to information under the control of public authorities, in order to promote
transparency and accountability in the working of every public authority. The citizens
and information seekers have, subject to few exemptions, an overriding right to be given
information on matters in possession of State and Public Agencies that are covered by the
Act.
7.
It is pertinent to note that integrity of examination system should not be
compromised. Conduct of examination for identifying and shortlisitng of candidates in
terms of competence, attitude, skills, etc. is a highly confidential activity and, therefore,
answer sheets are normally not disclosed. Multiple type of examinations are conducted at
different levels like those in schools, professional colleges, Departments, Public Service
Commissions, etc where purpose varies from admission to selection or promotion in
services. Many applications under R.T.I. Act have been filed and exemption has been
claimed either under section 8(1) (e) – Fiduciary relationship with the examiner and
section 8(1) (j) - Personal information of examiner. These matters have come before
Central Information Commission and various State Information Commissions.
(i) In Ms. Threesa Irish v/s. Kerala Postal Circle in ICIB/A-2/COC/2006 it has
been observed that when the answer papers are evaluated, the authority
conducting the examination and the examiners evaluating the answer sheets
stand in a fiduciary relationship between each other. Such a relationship
warrants maintenance of confidentiality by both of the manner and method of
evaluation. This decision was cited with approval in other cases. The fiduciary
relationship between the examiners and the authority conducting the
examination is personal and it can extend only in so far as the disclosure of
the identity of the examiner is concerned.
(ii) In G. Gurunadham v/s. BSNL, Hyderabad (CIC/AD/A/09/00162 dated
26.03.2009) it was observed as under:
“.... In regard to public examinations conducted by institutions established by the
Constitution like UPSC or institutions established by any enactment by the
Parliament or Rules made thereunder like CBSE, Staff Selection commission,
Universities., etc, the function of which is mainly to conduct examinations and
which have an established system as fool-proof as that can be, and which, by their
own rules or regulations prohibit disclosure of evaluated answer sheets or where
the disclosure of evaluated answer sheets would result in rendering the system
unworkable in practice and on the basis of rationale followed by the Supreme
Court in the above two cases, we would like to put at rest the matter of disclosure
of answer sheets. We therefore decide that in such cases, a citizen cannot seek
disclosure of the evaluated answer sheets under the RTI Act, 2005.
Insofar as examinations conducted by other public authorities, the main
function of which is not of conducting examinations, but only for filling up of
posts either by promotion or by recruitment, be it limited or public, the rationale
of the judgments of the Supreme Court may not be applicable in their totality, as
in arriving at their conclusions, the above judgments took into consideration
various facts like the large number of candidates, the method and criteria of
selection of examiners, existence of a foolproof system with proper checks and
balances etc. Therefore, in respect of these examinations, the disclosure of the
answer sheets shall be the general rule but each case may have to be examined
individually to see as to whether disclosure of evaluated answer sheets would
render the system unworkable in practice. If that be so, the disclosure of the
evaluated answer sheets could be denied but not otherwise. However, while doing
so the concerned authority should ensure that the name and identity of the
examiner, supervisor or any other person associated with the process of
examination is in no way disclosed so as to endanger the life or physical safety of
such person.”
I have also perused the rulings relied by the Advocate for the Opponent, i.e. (i)
Shri J. Shahbudeen v/s. Director of Postal Services Kerala Circle; (ii) Shri George Paul
v/s. Bharat Sanchar Nigam Ltd and Ms. Treesa Irish v/s. Kerala Postal Circle,
Trivandrum. The sum and substance of these rulings is that evaluated answer sheets need
not be disclosed and that furnishing copies of the evaluated answer papers would be
against public interest and would compromise the fairness and impartiality of the
selection process.
8.
I have also perused some other rulings. (i) Maharashtra State Board of Secondary
and Higher Secondary Education and anr. v/s. Paritosh Bhupesh Kumarseth, etc. AIR
1984 SC 1543. It was observed that finality ought to be attached to the results of public
examinations, that an examinee has no right to inspect his answer script and that
principles of natural justice have no applications in such cases. (ii) Secretary W.B.
Council of Higher Secondary Education v/s. Ayan Das (2007) 8 SCC 242 and (iii)
University of Calcutta & Others v/s. Pritom Rooj (Calcutta High Court). Central
Information Commission in a catena of judgments/orders have held that a citizen cannot
seek disclosure of the evaluated answer sheets under R.T.I. Act, 2005.
Be that as it may, however, in my view the Commission has to adopt a beneficial
approach in tune with the interests of the information seekers whereby allowing R.T.I.
Act to have its full play thereby allowing the Complainant to see the answer sheets so that
any doubt or suspicion is cleared or wiped out and faith in the system is restored.
However, this Commission feels that a fine balance has to be struck between the
imperatives of confidentiality of information with the right of the citizen to get
information. In the instant case no balance will be affected if papers of other candidates
are shown to the Complainant.
I have also perused the rulings of the Hon’ble High Court of Delhi relied by the
Complainant as well as Arunachal Pradesh Public Service Commission v/s. Arunachal
Pradesh Information Commission and Another 2010 (2) id 582 (Gauhati H.C.).
9.
Point No. 2 is regarding copies of mark sheet of other two candidates and point
No. 3 is about their teaching experience certificates. This was rejected on the ground of
personal information and disclosure of which has no relationship to any public activity or
interest and no public interest would serve in disclosing this information. Apparently the
same was rejected under section 8(1) (j).
Section 8 is as under:8. Exemption from disclosure of information: Notwithstanding anything
contained in this Act there shall be no obligation to give any citizen –
(j) information which relates to personal information the disclosure of
which has no relationship to any public activity or interest, or which
would cause unwarranted invasion of the privacy of the individual unless
Central Public Information officer or the Appellate Authority as the case
may be, is satisfied that the larger public interest justifies the disclosure of
such information.
Provided that the information, which cannot be denied to the Parliament or
a State Legislature, shall not be denied to any person.
From the above provision it is evident that the exemption is attracted under two
circumstances. Firstly if the information is personal in nature and has no relationship to
any public activity or interest. Secondly furnishing the same would cause unwarranted
invasion of privacy of an individual. Again the same is subject to the opinion of State
Public Information Officer or Appellate Authority that the larger public interest justifies
the disclosure.
In the instant case the information sought is in respect of selection conducted by
Public Authority to fill a public post. Once certain documents are produced before the
Public Authority in connection with this post the same cannot be considered as personal.
Again furnishing of such information would certainly instill confidence in the process.
Again, candidates are selected/appointed on the basis of mark sheets, experience and
other criteria/qualifications prescribed and hence these qualifications have direct
relationship with their duty and, therefore, exemption from disclosure of information
under section 8(1) (j) is not available or attracted.
I am fortified in the above by the observations of the Hon’ble Allahabad High
Court in Surender Singh v/s. State of U.P. and Others 2009 (1) ID 208. The relevant
observations are in para 6, 7 and 8.
“6. Section 11 of the Act relates to third party information. Third party has
been defined under section 2 (n) to mean a person other than the citizen
making a request for information and includes a public authority. It is only
when the third party treats the information required to be disclosed as
confidential that the authority is required to give a written notice to such
third party of the request. In case such information is not held as
confidential no written notice is required to be given. Such provisions in
section 11 appear to be for the purpose of preventing the Act from
becoming a tool in the hands of a busy body only for the purpose of
setting personal scores or other oblique motives.
7.
The information sought by the appellant in the present case relates
to six Assistant Teachers of the institution in question and the educational
certificates submitted by them for being appointed as Assistant teachers.
Since the institution in question and the committee of Management
managing the institution is a public authority as defined in the Act the
Assistant Teachers working therein are also performing the duties of
imparting education to the society. Consequently when the Assistant
Teachers are performing public activity the information sought by the
applicant is with relation to such activity and cannot be said that the
teaching work done by the six Assistant Teachers has no relationship to
any public activity or interest. The information sought by the appellant
cannot also be said to cause unwarranted invasion of the privacy of such
Assistant Teachers in the institution inasmuch as their educational
certificates are matter of record of the institution on the strength of which
they have obtained appointments as Assistant Teachers and are performing
public activities by imparting education in the institution. By no stretch of
imagination can it be held that the information regarding their appointment
and educational certificates would be unwarranted invasion of their
privacy. Their educational qualifications are not privy to them but are
records available with the institution which is a public authority within the
meaning of the Act.
8.
The information sought in the present case cannot also be brought
within the meaning of being confidential to the third party. The records of
educational certificates of the six Assistant Teachers are available with the
public authority and have relationship to their performing their duties as
such. They were appointed by virtue of their qualifications and hence such
qualifications have direct relationship to their duties. As such the
exemption from disclosure of information under section 8(j) is not
available in the present case.”
10.
If one travels down the memory lane and observe the authoritative judgments of
Hon’ble Supreme Court right from Raj Narain case to S.P. Gupta and Other, one would
invariably come across two milestones - one that in a Government of responsibility like
ours there can be but few secrets and secondly for openness in the Government. The spirit
of the Constitutional right to expression and information may not be lost.
It is pertinent to note that Commission’s discipline/judicial discipline demand
deference to precedents. Society has progressed and progressive legislation like RTI Act
has been enacted, aspirations of public have risen and veils of secrecy as well as curtains
that were put in its infancy or adulthood have been removed/raised and secrets of
yesteryears have come into the open and it would not be a cry in wilderness if this
Commission aspires that a day will come when secrecy would be a relic of the past and
answer papers would accompany mark sheets thereby fulfilling the mandate of R.T.I.
Act.
11.
It was contended by Advocate for the Opponent about maintainability of the
complaint. According to him Complainant has not preferred First Appeal and instead
filed the Complaint which is not maintainable.
In the case before me the position is, the information is sought and the Opponent
partly furnished the same and rest was not furnished on account of section 8. Thereafter
the present Complaint is filed. According to the Complainant, the Complaint is
maintainable. To my mind the remedy lies of First Appeal. Even assuming for a while the
ground of rejection is not valid yet the fact remains that good or bad, Public Information
Officer acted within law. I have perused some of the rulings of Central Information
Commission wherein matters were not entertained without approaching First Appellate
Authority. In some complaints were entertained under certain circumstances.
Under section 18(1) Complaint may be filed if sub-section (a) to (f) are attracted.
Complaint can be filed in case the Public Information officer does not respond within the
time limits specified under the law. The reply of Public Information Officer has the
backing of law.
I do agree with the Advocate for the Respondent No. 1 on this count. But in the
instant case I am inclined to entertain the Complaint firstly because of the factual matrix
of this case and secondly because R.T.I. Act is a people friendly, user friendly Act and to
deny information on such ground is not in true spirit of the Act.
However, it is made
clear that this should not be cited as precedent. I find some support regarding this in Life
Insurance Corporation of India & Others V/s. Central Information Commission & Others
(W.P. No. 8708/2008 Delhi High Court). No doubt facts are different but the principle is
attracted in the factual backdrop of this case.
12.
In view of all the above, I am of the opinion that request of the Complainant can
be granted partly. Hence, I pass the following Order:
ORDER
The Complaint is allowed and the Opponent is hereby directed to furnish the
information in respect of point/Sr. No. 2 and 3 of the Complainant’s application dated
10.08.2010 within twenty days from the date of receipt of the Order. In respect of point
No. 1 written examination papers of Kum. Ansari Azra Z. A. and Shri Kotker Devidas
Vithoba be shown to the Complainant to ascertain that the same have been evaluated.
Names and other things need not be disclosed. No copies, photocopies are allowed to be
taken. The Complaint is accordingly disposed off.
***
GUJARATH PUBLIC SERVICE COMMISSION
IN THE HIGH COURT OF GUJARAT AT AHMEDABAD
LETTERS PATENT APPEAL NO.4 OF 2009
IN
SPECIAL CIVIL APPLICATION NO.15960 OF 2008
WITH
CIVIL APPLICATION NO.23 OF 2009
IN
LETTERS PATENT APPEAL NO.4 OF 2009
WITH
LETTERS PATENT APPEAL NO.82 OF 2009
IN
SPECIAL CIVIL APPLICATION NO.15567 OF 2008
D.D. 06.03.2009
Hon’ble Mr. Justice J.R.Vora
Hon’ble Mr. Justice M.R.Shah
Mistry Rajeshkumar Ramanlal
Vs.
State of Gujarat & Anr.
…
Appellant
…
Petitioners
Reservation:
Whether candidates claiming reservation under SEBC category can claim to be selected
under G.M. category on their failure to produce non creamy layer certificate in support of
claim of reservation under SEBC category? – No.
In the direct recruitment for the post of Assistant Public Prosecutor in the
advertisement candidates seeking reservation under SEBC category were required to
produce non creamy layer certificates along with the application – The appellant claimed
reservation under SEBC category but failed to produce non creamy layer certificate - His
application was rejected as incomplete – Special Civil Application filed challenging the
said order was dismissed by the learned Single Judge as per order dated 30.12.2008 –
Letters Patent Appeal was filed against the said order - The appellant contended that he
did not intend to get age relaxation and as such he did not produce non creamy layer
certificate and his case was required to be considered under General merit – High Court
distinguishing two decisions R.K.Sabharwal & Ors Vs. State of Punjab & Ors. AIR 1995
SC 1371 and Barot Jignesh Lakshmansinh & Ors. Vs. State of Gujarat & Ors. – 2003 (2)
GLR 1727 held that the said decisions were not applicable and consequently dismissed
the appeal.
Held:
Therefore, the submission on behalf of the appellants that on non-production of
the non-creamy layer certificate along with the applications, GPSC was required to
consider their cases in the general category automatically, cannot be accepted. It is not so
provided in the advertisement. GPSC has no suo-motu power and/or it is not the practice
of the GPSC to automatically consider case of the candidate belonging to reserved
category in the general category on non-production of requisite caste certificate etc. And
therefore on non-production of non-creamy layer certificate by the appellants their
applications are found to be incomplete. It can be done provided the same is mentioned in
the advertisement and/or the same is the practice of the GPSC. Therefore, the contention
on behalf of the respective appellants / petitioners that on non-production of the non
creamy layer certificate by them their cases were required to be considered in the general
category, cannot be accepted.
Cases Referred:
1)
2)
AIR 1995 S.C. 1371 - R.K. Sabharwal & Ors. Vs. State of Punjab & Ors.
2003(2) GLR 1727 – (relevant para 36 to 39) Barot Jignesh Lakshmansinh & Ors.
Vs. State of Gujarat & Ors.
ORDER
1.
As common question of facts and law arise in both these Letters Patent Appeals,
they are being disposed of by this common judgement and order.
2.
Letters Patent Appeal No. 4 of 2009 with Civil Application No. 23 of 2009 is
filed by the appellant herein - original petitioner challenging the impugned order passed
by the learned Single Judge dtd.30/12/2008 in Special Civil Application No. 15960 of
2008 in dismissing the same and in not directing the respondents to consider the
application of the appellant - original petitioner for the post of Assistant Public
Prosecutors in the State of Gujarat as per law and not allowing the appellant to appear in
preliminary examination. Letters Patent Appeal No. 82 of 2009 is preferred by the
appellant - original petitioner challenging the impugned order dtd.30/12/2008 passed by
the learned Single Judge in Special Civil Application No. 15567 of 2008 in dismissing
the said Special Civil Application and in not directing the respondent No.2 - Gujarat
Public Service Commission (“GPSC” for short) to issue Call Letter of test to the
appellant.
3.
Before considering the Letters Patent Appeals on merits, few facts and the history
which lead to filling up the posts of Assistant Public Prosecutor in the State of Gujarat
deserves consideration, and the same are as under:3.1
Criminal Misc. Application No.13937 in Criminal Appeal No.487 of 2006 was
placed before the Division Bench of this Court to condone the delay of 1695 days caused
in filing the Criminal Appeal against the judgement and order of acquittal passed by the
learned trial court, wherein one of the ground in the application for condonation of delay
was shortage of Assistant Public Prosecutor and one Additional Public Prosecutor was
required to attend more than one courts and in most of the cases in different Talukas and
the resultant effect was delay in disposal of the criminal trials, as Assistant Public
Prosecutor was attending the particular Criminal Court in the Taluka only once a week
and it was found that on rest of the days, the Additional Public Prosecutors were not
available in the Court due to which the work of the Court was suffering and therefore, the
Division bench asked the State to file detailed reply stating that how many posts of
Assistant Public Prosecutors in the State are vacant. By order dtd.19/2/2008, Division
Bench directed the Legal Department of the State Government to file detailed reply
giving particulars of the statement with respect to the Additional Public Prosecutors in
the entire State with respect to each of the Sessions Court, Additional Sessions Court and
Magistrate's Court by observing that in so many cases, grievances is made that there is
shortage of Additional Public Prosecutors and due to shortage of Additional Public
Prosecutor, Judicial Magistrate First Class's Courts, Additional Sessions Court, Sessions
Courts, Magistrate's Court have to adjourn the matters which is one of the reason for
arrears.
3.2
Initially Dy. Secretary, Legal Department filed reply giving particulars with
respect to the Criminal Courts in each District and how many sanctioned posts of
Additional Public Prosecutors are there and how many posts of Additional Public
Prosecutors are vacant. It appeared that there are no sufficient Additional Public
Prosecutor / Assistant Public Prosecutors. It was found that in Vadodara District, there
were total 25 Courts taking up criminal matters against which sanctioned post of
Assistant Public Prosecutors was only 8 and out of which one was vacant, therefore, it
was found that at least 17 to 18 criminal courts in Vadodara District were without
Additional Public Prosecutors/Assistant Public Prosecutors. Similar was the position with
respect to other Sessions Court in each District. Therefore, Division Bench directed the
Legal Department, State of Gujarat to come out with a specific instructions when each
court would be having Additional Public Prosecutors/Assistant Public Prosecutors, so
that Criminal Courts can proceed further with the matter and to that the Secretary, Legal
Department filed the reply that the State Government intends to create 270 Posts of
Assistant Public Prosecutors and other Assistant Public Prosecutors may be appointed in
three different phases. It was also pointed out that even against the sanctioned strength of
236 Assistant Public Prosecutors, 60 posts of Assistant Public Prosecutors were vacant, it
was proposed that the said vacant posts would be filled in by GPSC by undertaking
appropriate selection procedure. Even to meet with the emergent situation, the State
Government also took a decision to appoint Additional Public Prosecutors on adhoc
basis, so that the said adhoc Additional Public Prosecutors may appear before the
Magistrate Court.
3.3
The Division Bench was not satisfied with the reply filed by the Legal
Department and the Division Bench in its order dtd.11/3/2008 in the aforesaid Criminal
Misc. Application made the following observations in para 5:“5. ..... Even against the sanctioned strength of 236 Additional Public
Prosecutors, 60 posts are vacant. Even the sanctioned posts are less than
the requirement considering each Court. It cannot be disputed that without
the Public Prosecutors may be Additional and/or Assistant Public
Prosecutors, the Criminal Courts cannot proceed further with the matter,
and as stated above, when one Additional Public Prosecutor is in charge of
three Criminal Courts at various places/Talukas, it will not be possible for
an Additional Public Prosecutor to remain present in each Court every
day, and if an Assistant Public Prosecutor is in charge of three Taluka
JMFC Courts, he can remain present only before one Court and other two
Courts will have to wait and/or adjourn the matters. This can be the reason
for arrears and delay in disposal of criminal cases. In the reply filed on
behalf of the State, it is not stated that when and how vacant posts are
likely to be filled in and even when the sanctioned posts are likely to be
increased.”
3.4
The Division Bench directed the Secretary, Legal Department to file detailed
reply dealing with the aforesaid aspects.
3.5
That thereafter the State of Gujarat responded positively and Notifications
dtd.5/8/2008 and 6/98/2008 came to be issued creating / sanctioning 180 additional posts
of Assistant Public Prosecutor and the State Government also responded positively to fill
up the vacant posts against the sanctioned posts. It was further stated that looking to the
work and the strength of the Court and the judges, there was a further requisition for 198
posts from various courts and the same will be looked into by the State Government.
3.6
When the aforesaid application came up for hearing before the Division Bench on
27/8/2008, the learned Advocate General submitted that 180 posts of Additional Public
Prosecutors are already sanctioned by the State Government inclusive of the Finance
Department and accordingly considering 180 additional posts of Additional Public
Prosecutors and 60 posts of Assistant Public Prosecutors vacant against sanctioned posts,
State Government sent requisition to the GPSC for 240 posts of Assistant Public
Prosecutors. The learned Advocate General also stated on behalf of the State that as one
time programme and with a view to see that the posts of Assistant Public Prosecutors are
filled in as early as possible, it has been decided to have the examinations of paper Nos.2,
3 and 4 by Objective Type of 200 marks. The State Government also came out with a
time bound schedule so that the posts of Assistant Public Prosecutors are filled in at the
earliest and there are sufficient Assistant Public Prosecutors in most of the criminal
courts.
3.7
The learned Advocate General, in consultation with the Secretary, Legal
Department; Dy.Secretary, GAD and Secretary GPSC, submitted that Draft Rules for
filling of posts of Assistant Public Prosecutors as one time measure will be published on
or before 20/10/2008. It was submitted that as one time measure for filling 242 posts of
Assistant Public Prosecutors which is proposed to have been one question paper of 200
marks for various subjects inclusive of Law Paper-I, Law Paper-II, which will be for
framing of issues, framing of charges, principles of pleadings, Code of Civil Procedure,
Code of Criminal Procedure, Evidence Act, Indian Penal Code. It was also further
submitted that the question paper with respect to English, Gujarati will be there and
separate marks will be allotted for different subject. It was further submitted in
consultation with the Secretary, GPSC that the advertisement inviting applications for
242 posts of Assistant Public Prosecutors in the local news paper as per the rules shall be
published on or before 20/10/2008 and the last date for submitting application would be
10/11/2008 and GPSC would further require one month for scrutiny of those applications.
It was also submitted by the learned Advocate General approximately three times of
number of posts to be filed in starting from top to bottom, the applicants will be called for
oral interview. It was also further submitted that minimum qualifying marks will be
prescribed and the aforesaid will also be reflected and/or notified in the advertisement. It
was submitted that scrutiny of the applications would be done on or before 10/12/2008
and simultaneously setting of the question paper would also be done on ior before
10/12/2008. It was further submitted that the examination will be conducted on or before
16/1/2009 and a further six weeks time is required by GPSC for assessing answer papers
and that would be done on or before end of February, 2009. It was further submitted that
thereafter oral interview of the eligible candidates would commence from 15/4/2009 and
approximately six weeks' time would require to complete the oral interview. It was
further submitted that within 15 days on completion of oral interviews immediately list
would be sent to the State Government, more particularly Legal Department. The learned
Advocate General, in consultation with the Secretary, Legal Department submitted that
approximately minimum 30 days time would require for police inquiry, medical test etc.
and immediately thereafter all the vacancies of Assistant Public Prosecutors will be filled
in.
3.8
Considering the above, the Division Bench directed all concerned to act as per the
above schedule. Considering the above, it was hoped that the entire process of filling of
aforesaid 242 posts of Assistant Public Prosecutors in the State of Gujarat would be
completed on or before 31/7/2009, if the aforesaid time schedule is strictly followed and
acted upon.
3.9
In the above background, process to fill up the 242 posts of Assistant Public
Prosecutors started.
4.
The GPSC invited applications for 242 posts of Assistant Public Prosecutors from
the eligible candidates by giving advertisement in local news paper on 17/10/2008. In the
said advertisement, it was specifically mentioned that out of 242 posts of Assistant Public
Prosecutors, Class-II, 122 posts are for General category, 15 posts are reserved for the
candidates belonging to Scheduled Caste and 36 posts are reserved for candidates
belonging to Scheduled Tribe candidate and 69 posts are reserved for the candidates
belonging to SEBC candidates. Even reservation for the Women candidates was also
provided in the said advertisement. In the said advertisement, age relaxation with respect
to reserved category candidate was also provided and the candidates were required to fill
in the form in the prescribed format and were required to produce necessary documents
as mentioned in the said advertisement inclusive of necessary marksheet and educational
experience certificate issued by the concerned criminal courts, caste certificate and
creamy layer and non-creamy layer (in case of reserved SEBC category candidate). It was
specifically provided in the said advertisement that candidate belonging to General
category and Women candidates were required to deposit Rs.100 as fees and candidates
belonging to reserved category were not required to pay/deposit the aforesaid Rs.100 as
fees. It was also provided in the advertisement that if a reserved category candidate even
wants to apply in General category and does not want to get benefit of reservation, in that
case also he is not required to pay/deposit Rs.100 as fees. It was also provided in the said
advertisement that candidates belonging to SEBC category who have produced noncreamy layer certificate will also be given age relaxation. It was specifically provided in
the said advertisement that incomplete applications will be rejected and such candidate
will not be called for written examination.
4.1
It is also to be noted that in the form, there was a specific column with respect to
which category candidate belongs to i.e. General category, Scheduled Caste, Scheduled
Tribes, SEBC etc. and candidate belonging to a particular category was required to put
tick mark against the said category, so that their applications can be scrutinized and
considered in that particular category.
5.
In response to the aforesaid advertisement, both the appellants - petitioners who
are practicing advocates submitted applications for the posts of Assistant Public
Prosecutors, Class-II, on or before the date mentioned in the said advertisement. Both the
appellants belong to SEBC Community and in the forms they have also put a specific tick
mark against SEBC Category. Candidates belonging to SEBC category were required to
submit creamy layer / non-creamy layer certificate issued by the competent authority for
considering their case in the said reserved category. Both the appellants did not submit
non-creamy layer certificate though the same was must and therefore, vide
communication dtd.8/12/2008 both the appellants were informed by GPSC that both of
them are considered ineligible for appearing in preliminary examination as having not
produced non-creamy layer certificate and their application being incomplete.
The preliminary examination was to be conducted on 11/1/2009. As both the appellants
were considered ineligible for appearing in the preliminary test/examination for the posts
of Assistant Public Prosecutors, Class-II, both the appellants preferred the aforesaid
Special Civil Applications.
6.
The appellant of Letters Patent Appeal No. 4 of 2009 preferred Special Civil
Application No. 16960 of 2008 praying only the following final relief in para 10(a):“8 (a). To issue a writ of mandamus and/or certiorari and/or any other
writ, order or directions to the respondents to consider the application as
per law and allow the petitioner to give the preliminary examination.”
6.1
The appellant of Letters Patent Appeal No. 82 of 2009 preferred Special Civil
Application No. 15567 of 2008 and asked for the following final relief in para 8 (D):“8(D). Be pleased to direct resp. No.2, to issue call letter of test to the
present petitioner, and allow him to appear in the test.”
6.2
It was contended on behalf of the original petitioners that in the advertisement in
para 7.4 it was mentioned that whomsoever falling in the category of SEBC has to submit
non-creamy layer certificate, if he wants to take advantage of age relaxation and as the
appellants did not want to take advantage of age, they had not produced non-creamy layer
certificate and therefore, on that ground, the application of the petitioners ought not to
have been rejected. It was submitted that in the advertisement production of non-creamy
layer certificate was not compulsory and it was only for the candidate who wants to take
additional benefit of age relaxation and not for all the SEBC candidates. It was further
submitted that in any case, Even if non-creamy layer certificate was required, then,
respondent No.2 ought to have asked the petitioners to produce such certificate within the
prescribed time, as so done in many cases. No other submissions were made.
6.3
That both the aforesaid Special Civil Applications came to be heard by the
learned Single Judge on 30/12/2008 and the learned Single Judge dismissed both the
aforesaid Special Civil Applications by observing that the respondent No.2 GPSC was
justified in rejecting the applications of the petitioners as the respective petitioners have
not produced non-creamy layer certificates and their applications were incomplete. Being
aggrieved by the impugned order dtd.30/12/2008 passed in Special Civil Application
Nos.15960 and 15567 of 2008, respective appellants have preferred present Letters Patent
Appeals before this Court.
7.
Mr.Asim Pandya, learned advocate appearing on behalf of both the appellants has
submitted that SEBC Candidate who were not seeking age relaxation were not required to
produce non-creamy layer certificate and in the advertisement non-creamy layer
certificate was required from SEBC candidate for age relaxation only. It is submitted that
SEBC candidate not seeking any age relaxation, was not required to submit non-creamy
layer certificate for appearing in the examination.
7.1
It is further submitted that it is well settled proposition of law that the person
belonging to SEBC Category is entitled to compete in General Category. If on merit a
reserved category candidate gets higher marks then general category candidate and he can
claim his selection/admission or appointment in open/general category. He cannot be
compelled to be considered against the reserved quota only. It is submitted that on the
same analogy if the present petitioners did not fulfill OBC criteria, their cases are
required to be considered in open category subject to the eligibility criteria prescribed for
general category.
7.2
Mr.Asim, learned advocate appearing on behalf of the appellants has relied upon
the following decisions in support of his above submissions :(i)
(ii)
7.3
AIR 1995 S.C. 1371 (R.K. Sabharwal and others Vs. State of Punjab and
others).
2003(2) GLR 1727 (relevant para 36 to 39) (Barot Jignesh Lakshmansinh &
Ors. Vs. State of Gujarat & Ors.).
It is further submitted by Mr.Pandya learned advocate appearing on behalf of the
appellants that even otherwise advocates and other such professional falling within the
category of “Creamy Layer” of OBC, as per the prevalent policy, if their income during
preceding three years is more than the income provided in the policy of a particular State
for obtaining a non-creamy layer certificate, a procedure has been prescribed by the State
of Gujarat and the procedure includes submission of an application in the prescribe form;
recording of a statement of the applicant, preparation of Panchnama and submission of
affidavit. Form No.4/37 is an affidavit where the deponent has to state on
oath/affirmation that in his/her family nobody is a professional namely, a doctor, a
surgeon, an advocate, a CA, an Income Tax Consultant etc. It is submitted that said
prescribed form 4/37 is defective. Affidavit in the prescribe form could not have been
given by the father/mother of a lawyer applying for judicial posts or posts connected with
the judiciary, since the appellants are advocates. Hence, it is unjust to insist submission of
a false affidavit for obtaining a non-creamy lawyer certificate in the form prescribed by
the State Government from the advocates. It is submitted that all those advocates who
could procure a non-creamy layer certificate have submitted defective affidavits, which,
in law, cannot be accepted as affidavits.
7.4
It is further submitted by the learned advocate appearing on behalf of the
appellants that even advertisement published by the respondent was vague and does not
clearly specify the requirement to be fulfilled by the SEBC candidate. No specific option
given to the SEBC to fill up separate form if intending to compete in open category. On
the contrary an indication is given that even if a candidate belonging to SEBC wants to
compete in general category, such candidate need not pay Rs.100 fees. Therefore, the
decision of rejection shows total non-application of mind.
7.5
Lastly it is submitted that the candidates belonging to SEBC not seeking age
relaxation, were not required to furnish non-creamy layer certificate and therefore, their
applications ought not to have been rejected on that ground. Therefore, it is submitted
that if the appellants are not allowed to appear in the examination, it will cause
irretrievable loss to them as by the next time, when such advertisement will be issued, the
appellants would be age barred, whereas no such loss would be caused to anyone if the
appellants are permitted to appear in the examination.
7.6
Additional Affidavit is filed in Letters Patent Appeal No. 82 of 2009 pointing out
instances of two candidates namely Suresh Santuji Thakore and one another person, who
had filled up forms for the post of Assistant Public Prosecutor in pursuance of the
advertisement in question in SEBC category and whose form came to be rejected by
GPSC, and subsequently were allowed to appear in the examination conducted by the
GPSC on 10/11/2009. Thus it was submitted that those persons who were possessing
political influence, were allowed to appear in the examination in the same category,
whereas other candidates in the same category were not allowed to appear in the
examination. Thus, it was sought to be contended that the aforesaid act of GPSC is
discriminatory and violative of Article 14 of the Constitution of India. In the said
Additional Affidavit, it was also submitted that it is normal practice of the GPSC to shift
person applying in the SEBC category to the General category, if the candidate applying
in SEBC does not fulfill requirement for applying in SEBC. In support of his above
submission, appellants has also produced entry receipts and forms submitted by one
Chauhan Mayurika for the post of Clerk, Class-III Examination.
Submitting accordingly, it is requested to allow both these appeals.
8.
Both these Letters Patent Appeals are opposed by the learned advocate appearing
on behalf of GPSC as well as State Government. Mr.D.G. Shukla, learned advocate has
appeared on behalf of the GPSC and Mr.Umesh Trivedi, learned Additional Government
Pleader has appeared on behalf of the State. Learned advocate appearing on behalf of the
GPSC has submitted that both the appellants belong to SEBC category and both of them
put tick mark against SEBC category in the prescribed form, and they did not produce
non-creamy layer certificate along with the application. Thus, the applications submitted
by the respective appellants were incomplete as they have not submitted non-creamy
layer certificate, therefore, GPSC is justified in rejecting the applications of the appellants
and considering them as ineligible for appearing in the preliminary text/examination for
the post of Assistant Public Prosecutor, Class-II. It is further submitted that therefore, the
learned Single Judge has not committed any error in dismissing the petitions.
8.1
It is further submitted by the learned advocate appearing on behalf of the
respondents - GPSC and the State Government that for getting benefit of reservation
under SEBC, a candidate has to produce non creamy layer certificate as, as per the
reservation policy for those candidates who are falling within the definition of noncreamy layer, are entitled to get benefit of reservation in the reserved category of SEBC.
It is submitted that candidate belonging to reserved category of SEBC under the
reservation is not only required to be given age relaxation only but his case is required to
be considered along with the candidates belonging to SEBC Category and cut off marks
etc. would be different. Therefore, it is submitted that non-creamy layer certificate was
required only for age relaxation, cannot be accepted.
8.2
It is further submitted that even the contention on behalf of the appellants that the
advertisement was vague is also not true. It is submitted that the advertisement is as
clearly as it can be and every necessary particulars are mentioned in the advertisement. It
is submitted that in all 4985 candidates submitted applications, out of which 1699
candidates were belonging to SEBC Category and in fact 966 candidates belonging to
SEBC category, submitted non-creamy layer certificate. It is submitted that if those
candidates could understand the advertisement and submit non creamy layer certificates
which is required for the candidate belonging to SEBC category, in that case, the
appellants also ought to have produced non creamy layer certificate.
8.3
It is further submitted that so far as the submission with respect to defect in the
Form No.4/37 for getting non creamy layer certificate and the affidavit required, it is
submitted that first of all, such a contention has not been raised before the learned Single
Judge. It is submitted that even there are no pleadings or averments in the memo of
petitions/appeals. It is further submitted that even otherwise on merits also, the same has
no substance. It is submitted that only in a case where even a professional who is having
a particular income can get non creamy layer certificate and affidavit was required to that
effect with respect to income. It is submitted that it is not that no professional can get
non-creamy layer certificate. It is submitted that as such, as stated above, 966 applicants
who are professionals (advocates) have got non-creamy layer certificate. It is further
submitted that as such it was not the case on behalf of the appellants that because they are
advocates, they could not get non-creamy layer certificate.
8.4
Now, so far as the contention on behalf of the respective appellants that as they
did not want to get benefit of age relaxation which is available to candidates belonging to
SEBC category and as they did not intend to get benefit of reservation in the reserved
category of SEBC candidate and therefore, they did not produce non-creamy layer
certificate and therefore, their cases were required to be considered along with other
general category candidates is concerned, learned advocate appearing on behalf of the
respondents more particularly GPSC has stated that as such there is no power vested with
the GPSC to suo motu consider the applications of the candidate belonging to reserved
category automatically along with the general category candidate as non-filling the
requirement by reserved category candidate. It is further submitted that the decision
relied upon by the learned advocate appearing on behalf of the appellants in the cases
of R.K. Sabharwal and others Vs. State of Punjab and others (supra) and Barot Jignesh
Lakshmansinh & Ors. Vs. State of Gujarat & Ors. (supra) will not be applicable as in the
aforesaid cases, applications from the candidate belonging to reserved category were
found to be complete and in order and they were found to be more meritorious than
general category candidate and therefore, it was held that their cases should be considered
along with the general category candidate first as they are found to be more meritorious
than general category candidate and their cases is not to be considered in the reserved
category quota. It is submitted that in none of the cases it is held that irrespective where
their applications were found to be complete or not, and moment applications submitted
by the respective reserved category candidate found to be incomplete, automatically their
cases are required to be considered along with the general category candidates in general
category.
8.5
It is further submitted that if intention of the appellants was to consider their cases
along with general category candidate in general category and they did not intend to get
benefit under the reservation, in that case, they should have specifically mentioned in the
application (as has been done by some SEBC Candidates) that though they belong to
SEBC category, they do not intend to get benefit of reservation and their cases be
considered along with the general category candidate in general category and therefore,
they have not produced non creamy layer certificate. It is submitted that in the case of
Suresh Shantuji Thakore and another, in the form itself the said candidates had mentioned
“SEBC-General” “SEBC-Unreserved” respectively and therefore, it was decided to
consider their cases and consider them in general - unreserved category.
8.6
By way of additional affidavit in reply it is specifically denied that it is not the
practice of the GPSC to automatically treat applications of the reserved category
candidate in general category having been found that their applications are incomplete
and necessary certificates and caste certificates are not produced. It is submitted that
instances mentioned in the additional affidavit in reply in Letters Patent Appeal No. 82 of
2009 are not by the GPSC.
9.
It is submitted by the learned advocates appearing on behalf of the respective
respondents - GPSC as well as State Government that examination has already been
conducted on 11/1/2009 and now GPSC is required to declare results and they are
required to act as per the schedule fixed by the Division Bench of this Court to fill up 242
posts of Assistant Public Prosecutors. Therefore, it is submitted that it is now not possible
for the GPSC to take fresh examination of the appellants, otherwise it would further delay
the process of filling up the posts of Assistant Public Prosecutor in the State which is the
need of the day to clear arrears, as on account of non-availability of the Assistant Public
Prosecutors, in most of the cases, the courts have to adjourn criminal cases. Therefore, it
is requested to vacate ad-interim relief granted earlier and permit GPSC to declare the
results so that further process can be initiated.
Submitting accordingly, it is requested to dismiss both the present appeals.
10.
Heard the learned advocates appearing on behalf of the respective parties at
length.
11.
At the outset it is required to be noted that against the 236 sanctioned posts of
Assistant Public Prosecutors in the State 64 posts of Assistant Public Prosecutors were
found to be vacant. Looking to the strength of the criminal courts in the Courts, in the
Sessions Court, Additional Sessions Courts, Magistrate's Court etc. it was found that
sufficient number of Assistant Public Prosecutors are not available and it was also found
that one Additional Public Prosecutor is in charge of 3 to 4 Magistrate Courts and that too
in different Talukas and even one Assistant Public Prosecutor was attending that court
once in a week and therefore, due to non-availability of the Assistant Public Prosecutor in
the particular Taluka Courts, Courts are required to either wait for the Assistant Public
Prosecutors or adjourn the criminal cases and it was found to be the major reason / cause
for the delay in disposal of the criminal cases. The State Government responded
immediately to meet with the aforesaid situation and sanctioned 180 additional posts of
Assistant Public Prosecutors. Necessary Draft Rules came to be published immediately;
necessary sanction from the Finance Department was also obtained immediately, and
process was started to fill up 242 posts of Assistant Public Prosecutors in the State of
Gujarat in the time bound schedule and it was hoped that if everything is done within
stipulated schedule, 242 posts of Assistant Public Prosecutors will be filled in by
September 2009 and that is why process of filling up posts of Assistant Public Prosecutor
started. Applications came to be invited by giving advertisement in local news paper from
the eligible candidates (advocates) with all required particulars as mentioned in the
advertisement. There is specific column mentioned in the prescribed form with respect to
each category i.e. General category, Scheduled Caste, Scheduled Tribe, SEBC etc.
Candidates belonging to each category were required to put tick mark against the
respective category. It was also specifically pointed out that the candidate belonging to
SEBC has to produce non creamy layer certificate. It was specially pointed out that
candidates belonging to general category and women candidate, are required to pay
Rs.100 fees and candidate belonging to reserve candidate is not required to pay such a
fee. It was also mentioned that even if a candidate belonging to reserved category does
not want to get benefit under the reserved category, in that case also such candidate is not
required to pay the aforesaid fee. It is specifically provided in the said advertisement that
if candidate belong to SEBC category has not produced the non-creamy layer certificate,
he shall not be entitled to the age relaxation of five years. Considering the above, it is
sought to be contended on behalf of the respective appellants that as they did not want to
get benefit of age relaxation, they did not submit non-creamy layer certificate. However,
it is to be noted that reserved category candidate belonging to SEBC under the
reservation policy, will not be getting age relaxation only but there will be different
merits for such category candidate and cut off marks would also be different. It is also to
be noted that in the advertisement it was specifically mentioned that those applications
without requisite certificates and particulars will be considered as incomplete and those
candidates would be considered ineligible for the preliminary tests.
12.
Admittedly both the appellants had not submitted non-creamy layer certificate
which is must for getting benefit under the reserved category of SEBC. It cannot be
disputed that only those candidates who belong to non-creamy layer will be entitled to the
benefit under the reserved category of SEBC. As both the appellants did not submit noncreamy layer certificate along with the applications, their applications are considered
incomplete and accordingly they are held ineligible for appearing in Preliminary Test
/Examination and are not permitted to appear in the Preliminary text/ examination.
13.
It is the contention on behalf of the petitioners that even if they had not produced
the non-creamy layer certificate in that case, GPSC was required to consider their cases
in general category. In support of above submission, learned advocate has relied upon the
decision in the cases of K. Sabharwal and others (supra) and Barot Jignesh Lakshmansinh
& Ors. (supra). However, on considering the above decisions, there is no absolute
proposition of law laid down by the Hon'ble Supreme Court in the said decisions that
even in a case where application of the candidate belonging to reserved category is found
to be incomplete on non-production of the required caste certificate etc. their cases are
required to be considered automatically in general category. In the case before the
Hon'ble Supreme Court a candidate belonging to reserved category whose application
was found to be complete and who had submitted necessary caste certificate etc. was
found to be more meritorious than general category candidate and to that the Apex Court
held that his case was required to be considered in the quota of general category
candidate and his case is not required to be considered in the reserve quota. Therefore,
the submission on behalf of the appellants that on non-production of the non-creamy
layer certificate along with the applications, GPSC was required to consider their cases in
the general category automatically cannot be accepted. It is not so provided in the
advertisement. GPSC has no suo-motu power and/or it is not the practice of the GPSC to
automatically consider case of the candidate belong to reserved category in the general
category on non-production of requisite caste certificate etc. And therefore on nonproduction of non-creamy layer certificate by the appellants their applications are found
to be incomplete. It can be done provided the same is mentioned in the advertisement
and/or the same is the practice of the GPSC. Therefore, the contention on behalf of the
respective appellants/petitioners that on non-production of the non creamy layer
certificate by them their cases were required to be considered in the general category,
cannot be accepted.
14.
Now, the contention on behalf of the appellants that as they did not intend to get
age relaxation and that they did not intend to consider their cases in the reserved category
and therefore, they did not produce non creamy layer certificate and therefore, their cases
were required to be considered in the general category, cannot be accepted. If the
intention of the respective appellants was that their cases may not be considered in the
reserved category and their cases were required to be considered in general category in
that case, first of all, they should have specifically mentioned in the application that
though they belong to SEBC category, they do not intend to get benefit of reservation and
their cases be considered along with general category candidate. It is also required to be
noted that in the form the respective appellants have specifically put tick mark in the
SEBC Category, therefore, the intention of the respective appellants was very much clear
to consider their cases in the reserved category of SEBC. Now, on non production of the
non-creamy layer certificate which is must for getting benefit under the reserved category
of SEBC and having found their applications incomplete and they are held ineligible for
preliminary test, the appellants have now come out with a story that as they did not intend
to get age relaxation and the benefits in the reserved category, they have not produced
non-creamy layer certificate, which cannot be accepted. The said contention is nothing
but an afterthought.
15.
Now, so far as the contention on behalf of the respective appellants with respect
to the defective form No.4/37 for getting non-creamy layer certificate is concerned, it is
required to be noted that such a contention is not raised before the learned Single Judge
and even there are no averments and/or pleadings in the petitions. It is also to be noted
that in all 966 candidates belonging to SEBC category, who were also
professionals/advocates, have produced non-creamy layer certificate. It is also to be noted
that it is not that no professional will be eligible for non-creamy layer certificate. If a
professional is having more than a particular income, is not entitled to non-creamy layer
certificate. Therefore, necessary affidavits are required with respect to income of the said
professionals and considering the same, authority is required to issue non-creamy layer
certificate. Even such a submission on behalf of the appellants is nothing but an
afterthought and even self-contradictory. On one hand, as stated above, it is contended on
behalf of the appellants / petitioners that as they did not intend to get age relaxation and
benefit under reservation of SEBC, they did not produce non-creamy layer certificate and
on the other hand, now they have come out with a case that it was a defective
form/affidavit and their parents do not intend to file false affidavit. It is also to be noted
that it is not the case of the appellants that even they applied for non-creamy layer
certificate, and they could not get it.
16.
Now, so far as discriminatory treatment alleged to have been given to the
appellants on the ground that two candidates belong to SEBC category namely Suresh
Santuji Thakore and another, though did not submit non-creamy layer certificate, they are
found to be eligible and therefore, their cases are considered, is concerned, as per the
reply filed by the GPSC it was found that those candidates in the prescribed form have
specifically stated that SEBC-General” – “SEBC-Unreserved” and therefore, their cases
have been considered in the general category. It is submitted that had appellants stated in
the applications that their cases be considered in the general category, their cases would
also have been considered along with other two candidates. It is submitted that is not that
only with respect to aforesaid two candidates but with respect to other 42 similarly
situated candidates though they belong to SEBC, as they have specifically mentioned in
the form that their cases be considered in the General category, on non-production of
non-creamy layer certificates, their cases are considered in General Category.
17.
The contention on behalf of the appellants - petitioners that the application form
was vague, cannot be accepted. In all 4985 candidates have submitted applications and
none of the candidate except the petitioners, have made any grievance that the application
is vague. Even considering the advertisement, it cannot be said that the advertisement is
vague as sought to be contended by the appellants - petitioners. All the necessary
particulars have been mentioned in the advertisement inclusive of the requirement of noncreamy layer certificate etc.
18.
Now so far as reliance placed upon the recent decision of the Hon'ble Supreme
Court in the case of Bihari Lal Rada Vs. Anil Jain (Tinu) & Ors. rendered in Civil Appeal
No.976 of 2009 is concerned, the same will also not be applicable to the facts of the
present case. The question posed before the Hon'ble Supreme Court was whether
provisions of Haryana Municipal Act 1973 and the rules framed thereunder curtail and
put any embargo on the right of the Municipal Councilors elected of President
Municipality if by virtue of roaster such office is notified to be filled in by the members
belonging to the general category? That the Hon'ble Supreme Court considered that the
General candidate simply means the same is not reserved for any particular class of
persons or community and it is open to all classes who have been elected as members of
the ward irrespective of the fact whether they have been elected from the wards were
reserved for Scheduled Tribe and the backward classes or from the wards which
remained unreserved. Such a situation is not in the present case. The aforesaid decision
would have been applicable if the petitioners belonging to reserved category (SEBC),
have requested to consider their case in the general category and in a case where a
candidate who has filed complete form and who is found to be eligible for appearing in
the examination and has appeared and is found to be more meritorious than general
category candidate.
19.
Now, so far as the contention on behalf of the appellants that no prejudice would
be caused if examinations are taken so far as appellants are concerned and their cases are
considered as general category. It is to be noted that only 10 to 12 candidates have made
grievance and have approached this Court against 4985 candidates, out of which
applications of 1302 candidates have been rejected on various grounds and/or they are
found to be not eligible and therefore, for few of them who have come before this Court,
separate examination cannot be conducted. Even otherwise, the same is likely to upset the
schedule fixed by the Division Bench and further process of filling up the post of
Assistant Public Prosecutors would be delayed and it will result in further delay in
disposal of the criminal cases. As and when in future applications are invited, the
appellants can submit fresh applications along with necessary certificate etc. and their
case can be considered and there shall not be any question of age bar, as they belong to
SEBC category and they are entitled to age relaxation of five years and both the
appellants are aged 30 years. Under the circumstances, the prayer of the appellants to
conduct their examination, cannot be accepted.
20.
For the reasons stated above, as the applications submitted by the respective
appellants were incomplete and they have not produced the requisite non-creamy layer
certificate along with the application form though they belong to SEBC category and so
stated in the application form and thereby they are held to be ineligible for appearing in
the Preliminary Text/Examination for the posts of Assistant Public Prosecutor, Class-II
and even the examinations are already conducted on 11/1/2009 and the resultant is to be
declared and further process to fill up the posts of Assistant Public Prosecutors is to be
taken, both the Letters Patent Appeals deserve to be dismissed and are accordingly
dismissed. Any interference at this stage is likely to further delay the process to fill up the
post of Assistant Public Prosecutors in the State, which is the need of the day to clear the
backlog and the arrears of criminal cases.
21.
In view of dismissal of the main Letters Patent Appeals, no orders in the Civil
Application. Ad-Interim relief granted earlier stands vacated forthwith.
In the facts and circumstances of the case, there shall be no order as to costs.
After pronouncement of the judgement, Mr.Hiren Modi, learned advocate
appearing on behalf of the appellant has requested to continue the ad-interim relief
granted earlier.
For the reasons stated in the judgement and considering the fact that the Written
Examination is already conducted on 11/1/2009 and Answer Sheets are already checked
and now only the result is to be declared and if any further stay is granted, the same is
likely to further delay the appointments of Assistant Public Prosecutors, which is the
need of the day, as due to shortage of Assistant Public Prosecutors in the Criminal Courts
in the State, there is a delay in disposal of trial, the prayer of the appellants to continue
the ad-interim relief, is hereby rejected.
***
HARYANA PUBLIC SERVICE COMMISSION
IN THE HIGH COURT OF PUNJAB AND HARYANA AT CHANDIGARH
Civil Writ Petition No.9721 of 2010 C/W 10304 of 2010
D.D. 10.01.2011
Hon’ble Mr. Justice Mehinder Singh Sullar
Haryana P.S.C. & Anr.
Vs.
State Information Commissioner & Anr.
…
Petitioners
…
Respondents
R.T.I. Act:
In the recruitment to 184 posts of Haryana Civil Services (Executive Branch) and
Allied Services, 2009, respondent No.2 sought information namely, photocopy of
question booklet of Zoology subject, photocopy of question booklet of General Studies
and model key code – The same was declined by the Commission on the ground of
confidentiality – The 1st appeal was also rejected – In the 2nd appeal the Information
Commission directed to furnish the information sought – In this writ Petition filed by the
Commission against the order of the Information Commission the High Court after
considering the relevant provisions with reference to case law upheld the decision of the
Information Commission and dismissed the writ petition.
Held:
A co-joint reading of the provisions would reveal, only that information is
exempted, the disclosure of which, has no relationship to any public activity or interest,
or which would cause unwarranted invasion of the privacy of the individual, unless the
authorities are satisfied that the larger public interest justifies the disclosure of such
information.
Further held:
Section 4 of RTI Act mandates that every public authority shall maintain all its
records duly catalogued and published within one hundred and twenty days from
enactment of the Act, all the particulars and in the manner stipulated under the Section.
Cases Referred:
AIR 1984 SC 1543 - Maharashtra State Board of Secondary & Higher Secondary
Education & Anr. Vs. Paritosh Bhupesh Kumarsheth etc.,
(2007) 1 SCC 603 - President Board of Secondary Education, Orissa & Anr. vs.
D.Suvankar & Anr.
ORDER
Mehinder Singh Sullar, J.
As identical questions of law and facts are involved, therefore, I propose to
dispose of above indicated writ petitions vide this common order, in order to avoid the
repetition. However, the relevant facts, which need a necessary mention for deciding the
core controversy involved in these petitions, have been extracted from CWP No.9721 of
2010 titled as Haryana Public Service Commission and another Versus State Information
Commissioners, Haryana and another for ready reference.
2.
The epitome of the facts culminating in the commencement, relevant for disposal
of the present writ petitions and emanating from the record is that petitioner-Haryana
Public Service Commission (for brevity “the petitioner-HPSC”) invited the applications
from the eligible candidates for the recruitment of 184 post of Haryana Civil Services
(Executive Branch and Allied Services, 2009, vide Advertisement dated 04.01.2009. One
Wazir Singh Dalal (respondent No.2) applied for the indicated post and ultimately he
appeared, but could not clear the preliminary examination held on 26.04.2009 in this
regard.
3.
In the wake of application (Annexure P-1) dated 20.7.2009, respondent No.2
sought the (information) Photocopy of question booklet of Zoology subject (Optional),
photocopy of question booklet of general Studies (Preliminary Exams HCS-2009)
(Executive Branch and Allied Services) and photocopy of response of paper
setter/examiner, Zoology (optional), invoking the provisions of the Right of Information
Act, 2005 (hereinafter to be referred as “the Act”). The informations were denied to him
by the State Public Information Officer-cum-Secretary (for brevity “the SPIO”), vide
single line order dated 19.10.2009 (Annexure P-2) on the ground of its confidentiality.
The first appeal filed by him before the First Appellate Authority (for short “the FAA”)
met with the same fate.
4.
Aggrieved by the action of the SPIO and the FAA, respondent No.2 filed the
second appeal, which ultimately came to be disposed of and the following directions were
issued by the State Information Commissioner (for brevity “the SIC”) by virtue of
impugned order dated 23.02.2010 (Annexure P-10):“(a) The question paper and the Model Key Code as requested by Sh.
Wazir Sing Dalal in respect of Zoology (optional), and General Studies
Preliminary Exam HCS-2009 (Executive Branch and Allied Services) will
be provided to him by the respondent.
(b) The State Information Commissioner rejected the request of the
appellant with regard to photo copies of the comments of the question
paper setter on the ground of safety an security of the individual.”
Sequelly, aggrieved by the action of the SPIO and the FAA (in a similar matter),
private respondents, namely, Sudhir Kataria, Suman Balhara and Saroj Bala in CWP
No.10304 of 2010 filed another appeal, which was allowed by the SIC, vide impugned
order dated 14.01.2010 (Annexure P-8). The operative part of which is, as under:“In view of the above the following orders are passed:(a) Haryana Public Service Commission may allow all candidates to retain
question booklet (question paper) after completion of the time of the
paper.
(b) The Haryana Public Service Commission will make the model key
(answer to question paper) public by displaying the same on the
HPSC/Govt. Website after the results of the examinations have been
declared.
(c) The Haryana Public Service Commission will allow inspection/scrutiny of
OMR sheets (answer sheet) or answer booklets to any candidate who
desires to do so under proper supervision after the result of the
examination has been declared/the selection process has been competed.”
6.
Instead of complying with the impugned directions, the petitioners still did not
feel satisfied and filed the instant writ petitions invoking the provisions of Article
226/227 of the Constitution of India, challenging the impugned orders, Annexure P-10
(subject matter of CWP No.9721 of 2010) and Annexure P-8 (subject matter of CWP
No.10304 of 2010). That is how, I am seized of the matter.
7.
Having heard the learned counsel for the parties, having gone through the record
with their valuable assistance and after bestowal of thoughts over the entire matter to my
mind, there is no merit in the instant writ petitions in this context.
8.
However, ex-facie, the main cosmetic contentions of the learned counsel for the
petitioners that the informations sought by the private respondents fall within the
exemption clause of Section 8(e)(j) of the Act the same cannot be supplied and since the
SIC has no jurisdiction to direct it (HPSC) to formulate the policy, so, the impugned
orders deserve to be quashed, are neither tenable, nor the observations of the Hon’ble
Apex Court in cases Maharashtra State Board of Secondary and Higher Secondary
Education and another Versus Paritosh Bhupesh Kumarsheth etc., AIR 1984 Supreme
Court 1543 and President Board of Secondary Education, Orissa and another Versus
D.Suvankar and another, (2007) 1 Supreme Court Cases 603, are at all applicable to the
facts of the present case, under the RTI Act.
9.
Sequelly, in Paritosh Bhupesh Kumarsheth’s case (supra) Maharashtra State
Board formulated Regulation 104(3) with regard to the valuation of answer sheets. While
interpreting the validity of the Regulation, it was observed that Regulation 104(3) cannot
held to be invalid on the ground of violation of the rules of natural justice. The process
of evaluation of answer papers or of subsequent verification of marks under clause (3) of
Regulation 104 does not attract the principles of natural justice since no decision making
process which brings about adverse evil consequences to the examinees is involved. The
principle of natural justice cannot be extended beyond reasonable and rational limits and
cannot be carried to such absurd lengths as to make it necessary that candidates who have
taken a public examination should be allowed to participate in the process of evaluation
of their performances or to verify the correctness of the evaluation made by the
examiners by themselves conducting an inspection of the answer books and determining
whether there has been a proper and fair valuation of the answers by the examiners.
10.
Likewise, in D.Suvankar and another’s case (supra), in the High School
Certificate Examination 2004, conducted by the appellant-Board, the respondent was
declared to have passed in first division securing 654 marks out of 750. When the
respondent a representation, the answer scripts were verified and it was found that the
marks awarded in one paper were wrongly shown as 35, though therespondent had really
secured 65 marks. It was pointed out that the mistake occurred due to wrong entry made
in the computer. The error was rectified and a fresh mark sheet was issued. The
appellant-board constituted a committee pursuant to the direction given in Bismaya
Mohanty case, (1996) 1 OLR 134. Therein, the High Court had directed that the answer
sheet of the students, who had secured more than particular number of marks were to be
re-examined by the committee of three examiners to avoid the possibility of injustice on
account of marginal variation in marks.
11.
On the peculiar facts and in the circumstances of that case, it was ruled as under:“It has also to be ensured that the examiners who make the
evaluation of answer papers are really equipped for the job. The
paramount consideration in such cases is the ability of the examiner. The
Board has bounden duty to select such persons as examiners who have the
capacity, capability to make evaluation. Otherwise, the very purpose of
evaluation of answer papers would be frustrated. Nothing should be left to
show even an apprehension about lack of fair assessment. It is true that
evaluation of two persons cannot be equal on golden scales, but wide
variation would affect credibility of the system of evaluation. If for the
same answer one candidate gets higher marks than another that would be
arbitrary. One thing which cannot be lost sight of is the marginal
difference of marks which decide the placement of candidates in the merit
list. Care should be taken to see that the examiners who have been
appointed for a particular subject belong to the same faculty. The
evaluation should be done by an examiner who is well equipped in the
subject. That would rule out the chance of variation or improper
evaluation.
Board authorities should ensure that anomalous situations as
pointed out above do not occur. Additional steps should be taken for
assessing the capacity of a teacher before he is appointed as an examiner.
For this purpose, the Board may constitute a body of experts to interview
the persons who intend to be appointed as examiners. This process is
certainly time-consuming but it would further the ends for which the
examinations are held. The Chief Examiner is supposed to act as a safety
valve in the matter of proper assessment.
The scope for interference in matters of evaluation of answer
papers is very limited. For compelling reasons and apparent infirmity in
evaluation, the court steps in.”
12.
Possibly, no one can dispute with regard to the aforesaid observations, but the
same would not come to the rescue of the petitioners, rather support the case of the
private respondents in the instant controversy, under the RTI Act.
13.
What is not disputed here is that the petitioner has already supplied the copies of
question papers to the respondents and have only challenged the supply of information in
regard to Model Key Code in the first case and other impugned directions in the second
case.
14.
At the very outset, the basic purpose, aims and objects of the Act, have to be k